Вы находитесь на странице: 1из 164

5.

(a)
(b)

Candidates should attempt any FIVE questions.


All questions carry equal marks.

Let V and U be vector spaces over the field K and let V be of finite dimension. Let T: V. ----+
U be a linear Map.
Prove that
dim V = dim R(T) + dim N(T)
Let S={(x,y,z)/x+y+z=O}, x, y, z being real. Prove that Sis a subspace of 1J3 . Find a basis of

(c)

S.
VerifY which of the following are linear transformations:

(iv)
2.

(a)

(a)

(c)

7.

(a)

r(~)=(~ ~ ~}rG)=(~ ~ ~)
Find

For what values ofY) do the following equations


x+y+z=1

(b)

.e

(b)

r(;}

x+2y+4z=7J

(b)
(c)

4.

(a)

(b)

(c)

~ ~]

Transform the following to the diagonal forms and give the transformation employed:
2
2
x +2y, 8x -4xy+5/
Prove that the characteristic roots of a Hermitian matrix are all real and a characteristic root
of a skew-Hermitian matrix is either zero or a pure imaginary number.
If y=eax cos bx, prove that
2 2
yr2ayl+(a +b )y=O
and hence expand e 2xcos bx in powers of x.
Deduce the expansion of eax and cos bx.
If x = r sin 8 cos ~' y = r sin 8 sin ~' z = r cos8
then prove that

Discuss the nature of the conic


2
16x -24xy+9/-104x-172y+ 144=0
in details.
2
2
A straight line, always parallel to the plane of yz, passed through the curves x +/=a , z=O,
2
and x =ax, y=O; prove that the equation of the surface generated is
2 2)
x 4y 2= (x 2-az)2( a-x
Tangent planes are drawn to the ellipsoid
x2 y2 2 2
-+-+-=1
a2 b2 c2
through the point (a,~,y). Prove that the perpendiculars to them from the origin generate the
cone
2 2 2
2
22
(ax,~y,yz) = a x +b /+c z
Show that the locus of the foot of the perpendicular from the centre to the plane through the
extremities of three conjugate semi-diameters of the ellipsoid
x2 y2 2 2
-+-+-=1
a2 b2 c2
a2x2 + b~2 + c2z2 = 3(x2+y2+z2)
Define an osculating plane and derive its equation in vector form If the tangent and binormal
at a point P of the curve make angles 8, ~respectively with the fixed direction, show that

State Cayley-Hamilton theorem and use it to calculate the inverse of the matrix---+ [

IS

(a)

3.

:;t

x+4y+10z=ry 2
Have a solution? Solve them completely in each case.
Prove that a necessary and sufficient condition of a real quadratic form X' AX to be positive
definite is that the leading principal minors of A are all positive.

(c)

If ax 2+2hxy+by 2+2gx+2fy+c=O
represents two intersecting straight lines, show that the square of the distance of the point of
intersection of the straight lines from the origin is

c (a+ b)- !2- g2 (


2
ab - h
ab-h

(b)

LetT: M2,1----+ M2,3 be a linear transformation defined by (with usual notations)

-1,q > -1]

ra

(iii)

[P >

]Jlr.

----'------'---....,- 2

ra

(ii)

6.

:13 ----+13 2 defined by T(x) = (2x,-x)


2 3
T: 13 ----+ 13 defined by T (X, y) = ( xy, y, X)
T: 13 2----+ 13 3 defined by T (X, y) = (X+ y, y, X)
2
T: 13----+ 13 defined by T ( x) = (1, -1)

(i)

1(113)1(2/3)=

ce

(b)

r\1 +xY (1-x)q dx,


and prove that

xa
m

(a)

'(

xa
m

1.

.c
om

---.

(c)

ce

Maximum Marks: 300

.e

Time Allowed: 3 hours

2
2
2 2 2 2 2
2
dx +d/+dz = dr +r d8 +r sin 8 d~
Find the dimensions of the rectangular parallelepiped inscribed m the ellipsoid
x2 y2
2
2++.;.- = 1 that has greatest volume.
2
a
b
c
Prove that the volume enclosed by the cylinders
x 2+y 2=2ax, z 2 = 2ax is 128a3/15
Find the centre of gravity of the volume formed by revolving the area bounded by the
2
parabolas /=4ax and x =4by about the x-axis.
Evaluate the following integral in terms of Gamma function :

.c
om

(c)

8.

(a)
(b)
(c)

( :::: )(

~:)

= - (:

where k and 1 are respectively curvature and torsion of the curve at P.


By eliminating the constants a, b obtain the differential equation of which xy=aex+be-x+x2 is
a solution.
Find the orthogonal trajectories of the family of the semi-cubical parabolas ay 2=x3 , where a is
a variable parameter.
Show that
(4x+3y+ 1)dx+(3x+2y+ 1)dy=O
represents hyperbolas having the following lines as asymptotes
x+ =0 2x+ +1=0

Solve : (D 3-1 )y=xex+cos 2x


2 2
2
Solve : (x D +xD-4)y=x

(a)

If ] ( x, y, z) = (y + z

J].dx, where C consists of

calculate

(iii)

the curve

If

a and

b are constant vectors, show that

(i)

div{xx(axx)} =-2x.a

(ii)

div {(a X x) X ( b X

112

x)

A(i)}

xa
m

~---s{(_K_J
"g ax
gii

where A(i) are physical components of A and use it to derive expressiOn of divA m
cylindrical polar coordinates.
Two equal rods, each of weight wl and length l, are hinged together and placed astride a
smooth horizontal cylindrical peg of radius r. Then the lower ends are tied together by a
string and the rods are left at the same inclination ~ to the horizontal. Find the tension in the
string and if the string is slack, show that~ satisfies the equation
1
3
tan +tan=2r
Define central axis for a system of forces acting on a rigid body. A force Facts along the axis
of x and another force nF along a generator of the cylinder x 2+y 2=a2. Show that the central
axis lies on the cylinder.

(b)

.e

(a)

X)} = 2a. (b X)- 2b. (a

Obtain the formula

divA=

11.

x+ ui + u 2 j + u 2 k, u from 0 to 1.

ce

(c)

the line segment from (0,0,0) to (1,1,1)


the three line segments AB, BC and CD, where A, B, C and D are respectively the
points (0,0,0), (1,0,0), (1,1,0) and (1,1,1)

ra

(b)

(i)
(ii)

.c
om

)Z + ( z 2 + x 2)] + ( x 2 + y 2) k, then

ce

ra

w
12.

(a)

3;r ( a + 4b ) + 32ab

4(3b;r+4a)
5

A particle is moving with central acceleration !J.(r -c\) being projected from an apse at a

2
distance c with a velocity )( ; }'. Show that its path is the curve

(b)

x4+y4=c4
A particle is projected with a velocity whose horizontal and vertical components are
respectively u and v from a given point in a medium whose resistance per unit mass is k times

A semicircular area of radius a is immersed vertically with its diameter horizontal at a depth
b. If the circumference be below the centre, prove that the depth of centre of pressure is

n2 (nx-z)2 +(1+n2r y2 =n4a2

(c)

A particle slides down the arc of a smooth vertical circle of radius a being slightly displaced
from rest at the highest point of the circle. Find the point where it will strike the horizontal
plane through the lowest point of the circle.

xa
m

10.

(c)
(d)

(c)

dx

.e

(b)

the speed. Obtain the equation of the path and prove that if k is small, the horizontal range is
approximately
2uv 8uv 2 k
g
3g

(a)

9.

Solve the following differential equation:


y(l +xy )dx+x(l-xy )dy=O
Find the curves for which the portion of y-axis cut off between the origin and the tangent
varies as the cube of the abscissa of the point of contact.
Solve the following differential equation:
(D 2+4)y=sin2x, given that when x=O, then y=O and dy = 2.

.c
om

(d)

f (x) = .J1- x2
I

when x is rational

= 1-x when xis irrational


and show that f(x) is NOT Riemann integrable in [0,1].
20
(b)

Maximum Marks: 300

Candidates should attempt any jive Questions.


ALL Questions carry equal marks.

(b)

that

~[ x]

<X +1>

(a)

.c
om

(c)

6.

(a)

2xyz-4zx-2yz+x 2 + i +z 2 -2x-4y-4z

(b)

20

If
U = 1+nx _ 1+(n+1)x

nenx

(n+1)e(n+l)x'

0<x<1

Prove that

-L,U

(a)

=L.-u

dx
n
dx n
Is the series uniformly convergent in ]0,1 [? JustifY your clain.
Find the upper and lower Riemann integral for the function defined in the interval [0, 1] as
follows:

r-1

z2n-l

(2n -1)!

f(x,y,z)

Examine

20

Find the region of convergence of the series whose n-th term is


( -1

20

for extreme values.

(c)

ce

lie between the circle lzl = 1 and lzl=2.

20

then show that the resulting metric space is NOT complete.

(b)

20

z7 -5z3+ 12=0

dt]

Prove that all the roots of

20

If we metrize the space of functions continuous on [a,b] by taking.

J: !x(t)- y(t)/

7r

Jo R 2 -2Rrcos(B+)+r 2

Find the degree of splitting field x -3x +x -3 over Q, the field of rational numbers.

p(x,y) =

2
2
(R -r )f(Rei)

f (rei(})= 11 217 r

<X +X+4>

20
If u=e-x(x siny- ycosy), find v such that f(z)=u+iv is analytic. Also find f(z) explicitly as a
function of z.
20
Let f(z) be analytic inside and on the circle C defined by lzi=R and let z=eri8 be any point in
side C. Prove that

ra

(a)

(b)

are isomorphic fields each having 121 elements.

.e

(c)

F[ x]
2

and

Evaluate

.e

(a)

ce

(c)

5.

ra

(b)

If H is a cyclic normal subgroup of a group G, then show that every subgroup of H is normal
in G.
20
Show that no group of order 30 is simple.
20
If p is the smallest prime factor of the order of a finite group G, prove that any subgroup of
index p is normal.
20
IfR is a unique factorization domain, then prove that any fER[x] is an irreducible element of
R[x], if and only if either f is an irreducible element of R or f is an irreducible polynomial in
R[x}.
20
Prove that x 2+ 1 and x 2+x+4 are irreducible over F, the field of integers modulo 11. Prove also

xa
m

(a)

20
(c)

.c
om

SECTION- A
1.

Discuss the convergence or divergence of


oo
xfl dx
a>fJ>O
0 1 + xa sin 2 x'

xa
m

Time Allowed: 3 hours

20

Expand
1
- (z+1)(z+3)

f (z) - --:-----:---:------:---:---in a Laurent series valid for


(i)

lzl>3

(ii)

l<lzl<3

(iii)

lzl<1
20

(c)

By integrating along a suitable contour evaluate

L =qkqk-~(1-q K)

dx
Ios cosmx
+1
x2

show that the generalized acceleration is zero.

20

20
(c)

(2x 2-/+z2-2yz-zx-xy )p+(x2+2/+z2-yz-2zx-xy )q=x2+/+2z2-yz-zx-2xy

20
Find the complete integral of

(c)

Use Charpit's method to solve


px + qy = z ~1 + pq

(a)

Find the surface passing through the parabolas


z=O, y 2=4ax
z=1, /-4ax

xr+2p=O

Solve:

xa
m

(b)

ra

and satisfying the differential equation

20

10.

(a)

20

(b)

a uniform circular disc of radius a and mass m rolls down a rough inclined plane without
sliding. Show that the centre of the disc moves with constant acceleration 2/3 g sin a and the
coefficient of friction !J. > 1/3 tan a, where a is the inclination of the plane.

(a)

Show that the variable ellipsoid

(b)

(ii)

holonomic or nonholonomic

(iii)

conservative or non-conservative

[I]

a horizontal cylinder of radius a rolling inside a perfectly rough hollow horizontal


cylinder of radius b>a.

[II]

a particle constrained to move along a line under the influence of a force which is
inversely proportional to the square of its distance from a fixed point and a damping
force proportional to the square of the instantaneous speed.

(c)

a particle moving on a very long frictionless wire which rotates with constant angular
speed about a horizontal axis.
10

20

scleronomic or rheonomic

Find the lines of flow in the 2-dimensional fluid motion given by


~+i'l' = -1/2n(x+iy) 2e2int

(i)

When the Lagrangian function has the form

30

is a possible form for the boundary surface of a liquid motion at any timet.

20

Classify each of the following dynamical systems according as they are

[III]

(b)

11.

30

.e

.e

20

(a)

9.

Solve:

30

A uniform rod oflength 2a which has one end attached to a fixed point by a light inextensible
string of length 5a/12 is performing small oscillations in a vertical plane about its position of
equilibrium Find the position at any time and show that the periods of its principal
oscillations are

r+s-6t=ycosx

(c)

~a {cos(B-75)+2cosX(B+75)}+costant

VerifY that 8=25 defines a position of equilibrium and investigate its stability.

ce

8.

V=-

20

ce

(y-x)(qy-px) = (p-q) 2

.c
om

(b)

The ends of a uniform rod AB of length 2a cos 15 and weight W are constrained to slide on a
smooth circular wire of radius a fixed with its plane vertical. The end A is connected by an
elastic string of natural length a and modulus of elasticity W/2 to the highest point of the wire.
If 8 is the angle which the perpendicular bisector of the rod makes with the downward
vertical, show that the potential energy V is given by

.c
om

Solve:

ra

(a)

xa
m

7.

A source of strength m and a vortex of strength k are placed at the origin of the 2-dimensional
motion of unbounded liquid. Prove that the pressure at infinity exceeds that pressure at
distance r from the origin by

1
--

(m2 +k2)
r2

20
12.

(a)

Compute to 4 decimal placed by using Newton-Raphson method, the real root of


2

x + 4 sin x = 0

20

(b)

Solve by Runge-Kutta method

x1+x2::::;4.
x1-x2::::;2.

dy =X+ y
dx
with the initial conditions x 0 =0, y 0 =1 correct up to 4 decimal places, by evaluating up to
second increment ofy. (Take h=0.1)

20

20

y:

The standard deviations of two sets containing n1 and n2 are cr1 and cr2 respectively being
measured from their respective means m 1 and m2. If the two sets are grouped together as one
set ofn 1 + n 2 members, show that the standard deviation cr of this set measured from its mean
is given by

(a)

Fit the curve y=aebx to the following data (e being napierian base 2. 71828):
0

(c)

20

10, 6, 16, 17, 13, 12, 8, 14, 15, 9


For another random sample of 12 horses fed on diet B, the increase in the same period were
7, 13, 22, 15, 12, 14, 18, 8, 21, 23, 10, 17 kg
Test whether the diets A and B differ significantly as regard the effect on increase in weight.
You may use the fact that 5% value oft for 20 degrees of freedom is 2. 09.
20

15.

(a)

(b)

For a random sample of 10 horses fed on diet A, the increase in weight in kilograms in a
certain period were

Maximize

10

12 17

15

B 15 13

18 11

13

14

20

10

D 13 19 7 5 12
The present allocation is as follows :
AtoE90;
AtoF10;
BtoF150;
CtoF10
C toG 50;
C to J 120;
D to H 210; D to J 70
(i)
Check if this allocation is optimum If not, find an optimum schedule.
(ii)
If in the above problem the transportation cost from A to G is reduce to 10, what will
be the new optimum schedule?
20
Minimize z = Yl + Y2 + .... + Yn subject to YlY2 .. Yn = d and Yj20 for all j.
20
Determine x1, x2, X3 so as to maximize
2
2
z=-x1 -xl-x3 +4xl+6x2
subject to the constraints
x1+x2::::;2
2xl+3x2::::;12
Xl, X2Z2
20
At what average rate must a clerk at a super-market work in order to insure a probability of
0.90 that the customer will not have to wait longer than 12 minutes? It is assumed that there
is only one counter to which customers arrive in a Poisson fashion at an average rate of 15
per hour. The length of service by the clerk has an exponential distribution.
20
There are 5 jobs, each of which must go through machines A, B and C in the order ABC.
Processing times are given in the following table:
Job
Processing
times (hours)

x( I n1

xi I n

w
w

(c)

(a)

.e

16.

20

5.012 10
31.62
2
2
Let x 1 , x 2 be independently distributed variates having chi-square distributions with n 1, n 2
degrees of freedom respectively. Derive the distribution of

A 8

.e

x:
y:

(c)

20

Prove that if A, B and C are random events in a sample space and if A, B, C are pair wise
independent and A is independent of BuC, then A, B and C are mutually independent.

ce

20
2
Each coefficient in the equation Ax +Bx+C=O is determined by throwing an ordinary die.
Find the probability that the equation will have real roots.

(c)

(b)

From

ra

xa
m

.c
om

14.

(b)

ce

(a)

x:

xa
m

13.

Fit the natural cubic spline for the data

ra

(c)

the following table gives the cost for transporting material from supply points A,B,C,D to
demand points E, F, G, H, J:
To

.c
om

(b)

1
8
5
4
2
10
6
9
3
6
2
8
4
7
3
6
5
11
4
5
Determine a sequence for five jobs that will minimise the elapsed time.
20

4.

(a)

Prove that f(x) = x 2 sin

(b)

but its derivative is not continuous there.


If f(x), ~(x), 'l'(x) have derivatives when a:s;x::::;b, show that there is a value c of x lying
between a and b such that

..!_, x:;tO and f(x) = 0 for x = 0 is continuous and differentiable at x = 0


X

Maximum Marks: 300

f(a) (a) lf/(a)


f(b) (b) lf/(b)
f(c) (c) !f/(c)

(a)

Prove that the inverse of (


1

xa
m

A[ c- 1BA- 1

~ ~) is

1 1

1 1 1

.e

1 1 1 1
If A be an orthogonal matrix with the property that-1 is not an eigenvalue, then show that a is
expressible as (I-S) (S+srs 1 for some suitable skew-symmetric matrix S.
Show that any two eigenvectors corresponding to two distinct eigen values of
(i)
Hermitian matrix
(ii)
Unitary matrix are orthogonal.
A matrix B or order n x n is of the form leA where A, is a scalar and A has unit elements
everywhere except in the diagonal which has elements !J.. Find A, and !J. so that B may be
orthogonal.
Find the rank of the matrix
1
;

(b)
(c)

(c)

A=~~

ls

~3 ~J
3
11 j

(a)

8.

(b)

x2

(c)

9.

ca-g 2

2h
ab-h 2
Find the equation of the director circle of the conic l/r = 1 + e cos 8 and also obtain the
asymptotes of the above conic.
A line makes angles a, ~' y, 8 with the diagonals of a cube. Prove that
cos 2a + cos 2 ~ + cos 2y + cos 28 = 4/3
Prove that the centres of the spheres which touch the lines y = mx, z = c; y = -mx, z = -c lie
upon the conicoid mxy + cz(l + m 2) = 0.
Find the locus of the point of intersection of perpendicular generator of a hyperboloid of one
sheet.
A curve is drawn on a parabolic cylinder so as to cut all the generators at the same angle.
Find its curvature and torsion.
Determine the curvature for which the radius of curvature is proportional to the slope of the
tangent.
Show that the system of confocal conics
+- - = 1 is self-orthogonal.
2
a +A b +A
Solve {y(l+1/x)+cosy} dx+ {x+logx-xsiny}dy=O
-

(b)

If ax 2+2hxy+b/+2gx+2fy+c=O represents a pair of lines, prove that the area of the triangle
formed by their bisectors and axis of x is

(a)

(a)

(a)

7.

3.

Show that the volume common to the sphere x 2+y 2+z2 = a2 and the cylinder x 2+y 2=ax is
2a 3
9(3;z--4).

(c)
1 0

fooo x 2 ne -x dx = ; : [ 1.3.5 ... ( 2n -1)].

(c)

(b)

(c)

Define Gamma function and prove that

(a-b) 2 +4h 2

c- 1

where A, Care non-singular matrices and hence find the inverse of

(b)

6.

{(1,1), (1,-1)}?

(b)

~ (a > 0) and deduce that

rnl(n+~)= 2./f_, f2n

ra

2.

Prove that fooo e -ax dx =

.c
om

Show that the setS = {(1,0,0), (1,1,0), (1,1,1), (O,l,O)}spans the vector space R\R) but it is
not a basis set.
Define rank and nullity of a linear transformation T. IfV be a finite dimensional vector space
and T a linear operator on v such that rank T 2 = rank T, then prove that the null space ofT =
the null space of T 2 and the intersection of the range space and null space to T is the zero
subspace ofV.
If the matrix of a linear operator T on R 2 relative to the standard basis {(1,0), (0,1)} is

G ~),what is the matrix ofT relative to the basis B

(a)

5.

ce

(c)

Find the triangle of maximum area which can be inscribed in a circle.

xa
m

(b)

(c)

(a)

ce

1.

'(

=O .. f

.e

---.

.c
om

Candidates should attempt any FIVE questions.


All questions carry equal marks.

ra

Time Allowed: 3 hours

yddx y- 2(dyJ
=y
dx
2

(a)

Solve

(b)

Solve d ; + cv6 y =a cos cvt and discuss the nature of solution as cv


dt

(c)

Solve

(a)

Prove that the angular velocity of rotation at any point is equal to one half of the curl of the
velocity vector V.

by reducing it to canonical form


Determine the following form as definite, semi-definite or indefinite:
2x 12 + 2x22 + 3x/- 4x2x3- 4x3x1 + 2x1x2
10.

(n

)ye-x

+D2 +1

12

dt

cos( x.fi I 2).

Jfs 11x FiidS

where S is the upper half surface of the unit sphere x +l+i=1 and
I

F = zF + x} + yk .
Show that

(a)

The end links of a uniform chain slide along a fixed rough horizontal rod. Prove that the ratio
of the maximum span to the length of the chain is

_P_

axq

is not a tensor even though Ar is a covariant tensor or rank one.

Time Allowed: 3 hours

Maximum Marks: 300

plog[l+7']

a-fJ

A particle moves under a force


4
2 2 5
m!J.{3au -2(a -b )u }, a>b

(a)

z[ .J=3]={a+Hb:a,bEZ}

is not a unique factorization domain.


(b)

(c)

a+b

. show that its orbit is r =

(a)

2
2
{(1+ I z11 )(1+ I z2l )}

112

Justify your claim

20

(b)

Show that x = 0 is a point of non-uniform convergence of the series whose nth term is
n2 xe-n2x2 -(n-1 )2 X e-(n-1)2X2.

uv
.
v ( tan -1 -u tanh-1 -u ) where VIS. the termm
. al ve1ocity.
.
af ter a time1
"\IU2+V2
g
V
V

I Z1 + Z2 I

A particle is projected upwards with a velocity u in a medium whose resistance varies as the
square of the velocity. Prove that it will return to the point of projection with velocity

v=

20
Construct the addition and multiplication table for
2
z3 [x] /<x + 1>
where Z 3 is the set of integers modulo 3 and <x2+ 1> is the ideal generated by (x 2+ 1) in Z 3[x].
Let Q be the set of rational number and Q(2 112, 2 113 ) the smallest extension field of Q
. . 21/2, 21/3 . pm d th e b as1s
. 10r
c
Q(21/2 , 21/3) over Q .
contammg
Does the set C of all complex numbers form a metric space under

d ( 2 b 2 2) -

.e

a+ b cos8.

(c)

If G is a cyclic group of order n and p divides n, then prove that there is a homomorphism of
G onto a cyclic group of order p. what is the Kernel ofhomorphism?
Show that a group or order 56 cannot be simple.
20
Suppose the H, K are normal subgroups of a finite group G with H a normal subgroup of K.
If P = K!H, S = GIH, then prove that the quotient groups S/P and G/K are isomorphic.
20
If Z is the set of integers then show that

.e

and is projected form an apse at a distance a+ b with velocity

(c)

(b)

(c)

(b)

(a)

ra

(a)

xa
m

12.

SECTION- A
1.

xa
m

where !J. is the coefficient of friction.


A solid hemisphere is supported by a string fixed to a point on its rim and to point on a
smooth vertical wall with which the curved surface of the sphere is in contact. If 8 and ~ are
the inclinations of the string and the plane base of the hemisphere to the vertical, prove that
3
tan=-+tane.
8
A semi circular lamina is completely immersed in water with its plant vertical, so that the
extremity A of its bounding diameter is in the surface and the diameter makes with the
surface and angle a. Prove that if E be the centre of pressure and ~ the angle between AE and
the diameter,
"' 3;r + 16tana
tan'f'=----16+15;rtana
A point executes simple harmonic motion such that in two of its positions, the velocities are u
and v and the corresponding accelerations are a and ~ Show that the distance between the
..
. v2 -u2
positiOns IS
.

ce

(c)

.c
om

(b)

Candidates should attempt any jive Questions.


ALL Questions carry equal marks.

.c
om

11.

aA

(c)

ce

Evaluate

ra

(b)

(a)

20

Find all the maxima and minima of


f(x,y) = x 3+y 3-63(x+y )+ 12xy.
20
Examine for Riemann integrability over [0,2] of the function defined in [0,2] by
2

f (x) = {

x + x , for rational values of x


2

x + x , for irrational values of x


20
(b)

Prove that

Joo sm x dx
0

converges and conditionally converges.


20

Iff x+dxdydz
y+x+1

SECTION- B

In the finite z-plane, show that the function


1

f(z) =secz
has infinitely many isolated singularities in a finite intervals which includes 0.

20

(c)

Prove that (by applying Cauchy Integral formula or otherwise)


2n
1.3.5 ... (2n-1)
cos ()d() =
2;r
o
2.4.6 ... 2n
where n = 1, 2, 3 ..........

(a)

If cis the curve y = x 3-3x2+4x-1 joining the points (1, 1) and (2,3) find the value of

.c
om

Find the orthogonal trajectories of the family of curves in the xy-plane defined by
e-x(x sin y-y cos y)=a
where a is real constant.

rff

ce

6.

(b)

oo

(b)

Prove that

I (z

(c)

Evaluate

20

(c)

20
10.

(a)

20

a2

(b)

(c)

(b)

af

a +K

where 2a is the breadth of the door and K its radius of

(a)

11.

30
A solid homogeneous sphere is resting on the top of another fixed sphere and rolls down it.
Write down the equations of motion and find the friction. When does the upper sphere leave
the lower sphere if
(i)
both the spheres are smooth
(ii)
the upper sphere is sufficiently rough so as not to slip.
30
Show that

-2xyz

(b)

(x2- i)z
2'V=

U=

(a)

8.

(c)

d()

gyration about a vertical axis through G, the centre of mass.

.e

(b)

rr/2

fo -&8

with an angular velocity

o x6 +1
by choosing an appropriate contour.

(a)

+ K2 Jl/2

2af

20

2
Find the surface whose tangent planes cut off an intercept of constant length R from the axis
ofz.
20
Solve (x3+3x/)p + (y 3+3x 2y)q=2(x2+/)z.
20
Find the integral surface of the partial differential equation (x-y)p+(y-x-z)q=z.
thought the circle z =1, x 2+/=l.
20
Using Charpit's method find the complete integral of2xz-px2 -2qxy+pq=O.
20
Solve r-s+2q-z=x~ 2 .
20
Find the general solution of
x 2r-/t+xp-yq =log x.
20

20

The door of a railway carriage has its hinges, supposed smooth, towards the engine, which
starts with an acceleration f. Prove that the door closes in time.
[

foo dx

7.

10
Find
(i)
the Lagrangian
(ii)
the equations of motion
for the following system:
A particle is constrained to move in a plain under the influence of an attraction towards to
origin proportional to the distance from it and also of a force perpendicular to the radius
vector inversely proportional to the distance of the particle from the origin in anticlockwise
direction.
30
A heavy uniform rod rotating in a vertical plane falls and strikes a smooth inelastic horizontal
plane. Find the impulse,

) converges absolutely for lzl::::;l.

n n +1

xa
m

n=l

ra

L(12z -4iz}tz

(b)

Consider the two dynamical systems:


A sphere rolling down from the top of a fixed sphere.
A cylinder rolling without slipping down a rough inclined plane.
(i)
State whether (I) is rheonomic, holonomic and conservative and justify your claim
(ii)
Give reasons for (II) to be classified as scleronomic, holonomic. Is it conservative?

.c
om

5.

(a)
(I)
(II)

9.

ce

20
(a)

ra

over the volume bounded by the coordinate planes and the plane x + y + z = 1.

xa
m

Evaluate

.e

(c)

( x2 + y2)

2'W=

( x2 + y2)

y
2

x +y

are the velocity components of a possible liquid motion. Is this motion irrotational?
Steam is rushing from a boiler through a conical pipe, the diameters of the ends of which are
D and d; if v and v be the corresponding velocities of the steam and if the motion be
supposed to be that of divergence from the vertex of the cone, prove that
~= D 22 2 -v 2 )!2K
v d
where K is the pressure divided by the density and supposed constant.

e(v

20
(c)

Prove that for liquid circulating irrotatinally in part of the plane between two non-intersecting
circles, the curves of constant velocity are Cassini's ovals.

20

Find correct to 3 decimal places the two positive roots of 2ex -3x2=2.5644.
20
3

J-3 x 4dx

Simpson's rule by taking seven equidistant ordinates.

20

Compare it with the value obtained by using the trapezoidal rule and with exact value.

(c)
20

(c)

Solve dy/dx = xy
for x=l.4 by Runge-kutta method, initially x=1, y=2 (Take h=0.2).

(a)

The intelligence quotient of 480 student is as follows :

Frequency f

70

74

9
28

86

45

90

66

94

85

98

72

102

54
38

110

27

114

18

122

(b)

I
A

.e
p

20

X: 1 2

y: 2

10

11

11

10

(a)

An event A is known to be independent of the event B, BuC and BnC. Show that it is also
independent of C.

If the probability that an individual suffers a bad reaction from injection of a given serum is
0.001, determine the probability that out of2000 individuals

11

28

14

26

38

19

18

15

Minimize

w
16.

IV

17

13

20

26

z=

24 10
2
(yl) + (y2i + (y3i

Y1 + Y2 + Y3 215,
Yl, Y2, Y3 2 0,

by dynamic programming.
20

Use Kuhn-Tucker conditions to


f(x) = (x1) 2 + (x2) 2 + (x3) 2
Minimize
subject to

20
(b)

20

III

26

(c)

II

Task B

Subject to

Fit a second degree parabola to the following data taking x as the independent variable:

D 19

If the correlation coefficient of zero order in a set of P variates were equal to p, show that
every partial correlation of the s-th order is

w
(a)

A Departmental Head has four subordinates and four tasks are to be performed. The
subordinates differ in efficiency and the tasks differ in their intrinsic difficulty. His estimates
of the times each man would take to perform each task is given in the effectiveness matrix
below. How should the tasks be allocated one to one man, so as to minimize the total man
hours?

20

Man

w
w
14.

3xl-x2+2x3::::;7

-4xl+3x2+8x3::::;10

1+sp

(c)

x1 - 3x2 + 2x3

-2xl+4x2::::;12

Find the moment coefficient of skewness for the intelligence quotient. It is skewed to the left
or right?
20
(b)

xo

Subject to

126

0.01.

Use Simplex method to solve:

11

118

(ii)

Minimize

xa
m

106

(a)

0.05

ra

16

82

15.

ra

78

(i)

ce

Class marks X

.c
om

20
13.

A machine has produced, in the past, washers having a thickness of 0.050 inches. To
determine whether the machine is in proper working order, a sample of 10 washers is chosen
for which the mean thickness is 0.053 inches and the standard deviation is 0.003 inches. Test
the hypothesis that the machine is in proper working order using a level of significance of

.c
om

Evaluate approximately

exactly 3 will suffer a bad reaction


more than 2 will suffer a bad reaction.

ce

(b)

(i)
(ii)

xa
m

(a)

.e

12.

2x1 + x2- 5::::; 0


X1 + X3- 2::::; 0
1-xl::::;o
2-x2::::;o
-x3::::;o

20
(b)

On an average 96 patients per 24-hour day require the service of an emergency clinic. Also
on average, a patient requires 10 minutes of active attention. Assume that the facility can
handle only one emergency at a time. Suppose that it costs the clinic Rs. 100 per patient
treated to obtain an average servicing time of 10 minutes and that each minute of decrease in
this average time would cost Rs. 10 per patient treated. How much would have to budgeted

Time Allowed: 3 hours

Maximum Marks: 300

by the clinic to decrease the average size of the queue from 1~ patients to ~ patient?

Candidates should attempt any FIVE questions.


All questions carry equal marks.

20

1200

1400

1800

6
7

ce

1500
750

(c)

375

2300

200

2800

200

3400

200

4000

200

xa
m

(b)

3000

[r:B]=~~ ~ ~1l

l-1 -1

2.

(a)

20

.e

If A = [aij] is an n x n matrix such that


aii = n, aij = r if i:;t:j, show that
[A-(n-r) I] [A-(n-r+nr) I]=O.
Hence find the inverse of the n x n matrix B = [bij]
Where bii = 1, bij = p when i:;t:j and
1
p:;t:1, p:;t:--.
1-n

20

.e

0
find the matrix ofT with respect to a basis
B1 = {(0,1,-1), (1,-1, 1), (-1, 1, 0)}.

At what age the two wheeler should be replaced? (Note that the money value is constant in
time)

20
Prove that the eigen vectors corresponding to distinct eigen values of a square matrix are
linearly independent.
20
Determine the eigenvalues and eigenvectors of the matrix

(b)

(c)

.c
om

1000

Resale (Salvage) price

Show that f 1(t) = 1, f 2(t) = t-2, f3 (t) = (t-2) form a basis ofP 3, the space of polynomials with
2
degree ::::; 2. Express 3t - 5t+4 as a linear combination of f 1, f 2 , h
20
If T : V 4 (R) ----+ V 3 (R) is a linear transformation defined by
T(a,b,c,d) = (a-b+c+d, a+2c-d, a+b+3c-3d)
For a, b, dER, then verify that
Rank T +Nullity T = dim V 4 (R) .
20
If T is an operator on R 3 whose basis is
B = {(1,0,0), (0,1,0), (0,0,1)} such that

ce

(a)

ra

Running Costs

1.

xa
m

Year

.c
om

A transport manager finds from his past records that the costs per year of running a two-wheeler
whose purchase cost is Rs. 6000/- are given below:

ra

(c)

3.

(a)

A~l~ ~ ~j
20
Show that a matrix congruent to a skew-symmetric matrix is skew-symmetric. Use the result
to prove that the determinant of skew-symmetric matrix of even order is the square of a
rational function of its elements.
20

20

Find the rank of the matrix


0
c -b a'

(a)

a b'
0 c'

20

.c
om

Reduce the following symmetric matrix to a diagonal form and interpret the result in terms of
quadratic forms:

(a)

~(b -a )
1

for

x2

a3

3x

f(x)= -b - - - 2

for

a<x:::=;b
x >b

ra

for

O<x:::=;a

(c)

A variable plane is at a constant distance p from the origin 0 and meets the axes in A, B and

C. Show that the locus of the centroid of the tetrahedron OABC is

7.

(a)

x2

If a

and~

y2

sectwn 2 + - 2
a
b
(b)

(y-/3)2

xa
m

Prove that of all rectangular parallelopipeds of the same volume, the cube has the least
surface.

Iff

~(log2-%).

20

(b)

. -1 - - Show that 1f
Q ay ax

dx

[ap

20

8Ql is a function of x only, say, f(x), then F(x) __

20
eft(x)cix 1s an

9.

20

(c)

Find the family of curves whose tangent from an angle !!..._ with the hyperbola xy=c.
4

(a)

Transform the differential equation

dxdydz
(x+ y+z+1) 3
taken over the volume bounded by the co-ordinate planes and the plane x+y+z=1, is

Solve : dy + xsin 2y = x 3 cos 2 y

sin;ra

Prove that the value of

20

integrating factor ofP dx + Q dy = 0.

_Jr_( 0 <a < 1)


20

z __

a+b

(a)

20

(b)

(z-r)2

Find f(8) so that the curve x =a cos 8, y =a sin 8, z = f(8) determines a plane curve.

Show by means of beta function that


--,-dx
___
f (z-x)l-a
(x-tt

20

.e

20

(c)

(a)

lie between the least and greatest values of a, b, c prove that

.e

3.

2c = 1, z = 0 .

(x-a2)

Where K = 1/2 (b-c)(c-a)(a-b).


(c)

22

-'--------'- +

8.

Planes are drawn through a fixed point (a, ~' y) so that their sections of the paraboloid
2
ax +b/=2z are rectangular hyperbolas. Prove that they touch the cone.

20

f(a) f(b) f(c)


f(a) f'(a) f"(fJ)
(a) (b) (c) =K (a) '(a) "(/3)
lf/(a) lf/(b) !f/(c)
lf/(a) lf/'(a) lf/"(fJ)

20

Find the equations to the generators of hyperboloid, through any point of the principal elliptic

Prove that f(x) and f'(x) are continuous but f'(x) is discontinuous.
(b)

~+~+~ = ~
X

ce

4.

1J

3
f(x) is defined as follows :

If the normals at the points a, ~' y, 8 on the conic 1/r = 1+e cos8 meet at (p, ~ ), prove that
a+~+y+8-2~=odd multiple of n radians.

ce

A=~~ ~ ~ l
1

(b)

.c
om

20

l-1

x2

P(x1, Yl) to 2 + = 1, prove that


a
b2
2
2
/c 1 cos ~ + lc 2 sin ~ = 0 where /c 1, /c 2, are the parameters of two confocals to the ellipse
through P.

-c' 0
where aa'+bb'+cc'=O and a,b,c are all positive integers.
(c)

If 2~ be the angle between the tangents from

ra

-c 0
b -a
-a' -b'

6.

xa
m

(b)

(b)

20

d 2y

dy

dx

dx

- - cosx+-sinx-2ycos
2

x=2cos 5 x

into one having z as independent variable where z = sin x and solve it.

20
2

If

d ~ + g ( x-a) = 0
~

(a, band g being positive constants) and x =a' and dx = 0 when t=O,
~

20
(c)

The sphere x 2+l+i=a2 is pierced by the cylinder (x 2+l) 2 = a\x 2-l). Prove by the cylinder
(x 2+l) 2=a2(x 2-l). Prove that the volume of the sphere that lies inside the cylinder is
3

8a
3

[~+~
= 4.J2l
4 3
3

show that x = a+(a'-a)cosK

20

(c)

Solve (D 2-4D+4)y = 8x2 e2x sin 2x


d
where D=-.
dx

(c)

times the velocity. Show that the greatest height attained by the particle is

(a)

~[ A-log(l +A) J

g
where V is the terminal velocity of the particle and 'AV is the initial vertical velocity.

20
10.

A particle of mass m is projected vertically under gravity, the resistance of the air being mk

Show that rnr is an irrotational vector for any value ofn, but is solenoidal only ifn = -3.

20

20
(b)

If

F=yt +(x-2xz)]-xyk,evaluate

(c)

Prove that {

(a)

Show that the length of an endless chain which will hang over a circular pulley of radius a so
as to be in contact with two-thirds of the circumference of the pulley is

= _j_(log
axk

A smooth rod passes through a smooth ring at the focus of an ellipse whose major axis is
horizontal and rests with its lower end on the quadrant of the curve which is farthest removed
from the focus. Find its position of equilibrium and show that its length must at least be

xa
m

.J1 + 8e J where 2a is the major axis and e, the eccentricity.

xa
m

3
( : +:

(c)

20

ra

a{ log( :+FJ) + 4;}


(b)

20

ce

zk

ra

fi).

20

ce

11.

.c
om

Where S is the surface of the sphere


x 2+/+z2=a2 above the xy plane.

.c
om

ffs( 11 xP).ndS

20

The height of a balloon is calculated from the barometric pressure reading (p) on the
assumption that the pressure of the ai varies as the density. Show that if the pressure actually
varies as the nth power of the density, there will be an error

.e

.e

h"l n~l {~-(~f}-log ~ J

u2

v2

w2

~ (b-c)(c-a)(a-b)= a

timeTisgivenby

20
(b)

A gun is firing from the sea-level out to sea. It is mounted in a battery h meters high up and

fired at the same elevation a. Show that the range is increased by -1 [( 1 + 2 2 ~h2 )
2
u sm a

20

If in a simple Harmonic Motion, the velocities at distances a, b, c from a fixed point on the
straight line which is not the centre of force, be u, v, w respectively, show that the periodic

(a)

12.

in the calculated height where ho is the height of the homogeneous atmosphere and Po is the
pressure at the surface of the earth.

112

-1

of

itself, u being the velocity of projectile.

20

4.

(b)

.c
om

(c)

ce

20

Show that the additive group of integers modulo 4 is isomorphic to the multiplicative group
of the non-zero elements of integers modulo 5. State the two isomorphisms.

(a)

Find all the units of the integral domain of Gaussian integers.

(b)

Prove or disprove the statement : The polynomial ring I [x] over the ring of integers is a
principal ideal ring.

(c)

If R is an integral domain (not necessarily a unique factorization domain) and F is its field of

5.

(c)

Test of uniform convergence in the region lzl::::;1 the series

ra

20

20

20

I cos nz

(a)

.e

6.

(z -1)

20

"Te,

n
Find Laurent series for
e2z
(i)
about z = 1,
3
n=l

(ii)

the topology induced by e? JustifY your

20
Examine the
(i)
absolute convergence
(ii)
uniform convergence
Of the series
(1-x)+x(l-x)+x\1-x)+ ...
in [-c, 1] where O<c<1.

(z -3) 2

about z = 3.
20

(b)

Find the residues off(z) = ez cosec2 z at all its poles in the finite plane.

(c)

By means of contour integration, evaluate

w
(c)

induced by d coarser then

20

20

e(f,g)= f~lf(x)-g(x)dxl.
'Td

How many zeros does the polynomial


p(z) = z4 + 2z3 + 3z + 4
possess in (i) the first quadrant, (ii) the fourth quadrant.

20

Let C[O,l] denote the collection of all real continuous functions defined on I = [0, 1]. Suppose
d and e are metrics on C [0,1] defined by
d(f,g) =sup {lfx)-g(x)l: xEI}

Is the topology
answer.

(b)

20

If dis a metric on a nonempty set X, then is the function e defined by


e(a,b) =min (1, d(a,b)
for a,b E X, also a metric on X? Substantiate your claim

(b)

Suppose that z is the position vector of a particle moving on the ellipse C :


z = a cos cot+ ib sin cot.
where a, b, co are positive constants, a> b and tis the time. Determine where
(i)
the velocity has the greatest magnitude.
(ii)
the acceleration has the least magnitude.

.e

(a)

(a)

20

3.

By means of the substitution


x+y+z=u, y+z=uv, z = uvw

taken over the volume bounded by x=O, y=O, z=O, x+y+z= 1.

quotients, then show that any element f(x) in F(x) is of the form f ( x) = fo ( x) where f 0 (x) E
R[x], aER.

Jr2

evaluate JfJ(x+ y+zf xyzdxdydz

20

(c)

xa
m

2.

J0 f(x)dx=-.
6

If G is a group such that (abt = anbn for three consecutive integers n for all a,b in G, then
prove that G is abelian.
Can a group of order 42 be simple? JustifY your claim

20
1
Let the function fbe defined on [0,1] by the condition f(x) = 2 rx when - -<x<.!.,r >0.
r +1
r
Show that f is Riemann integrable in [0, 1] and

.c
om

Maximum Marks: 300

SECTION- A

(b)

2 ,p > 1.
nP +nqx
is uniformly convergent for all values ofx and can be differentiated term by term if
q<3p-2.

Candidates should attempt any jive Questions.


ALL Questions carry equal marks.

(a)

Prove that
S(x)=I

Time Allowed: 3 hours

1.

(a)

ce

ra

'I

xa
m

i:,\ : ;'

20

Joo (loge U )
o

u2 +1

du.
20

7.

(a)

Find the differential equation of the family of all cones with vertex at (2, -3, 1).

(b)

Find the integral surface of

20
2
X

20

p +y q+Z

az

az

0, p = 8x 'q = 8y

which passes through the hyperbola

xy = x+y, z =1.

11.

(a)

The particle velocity for a fluid motion referred to rectangular axes is given by

20
Obtain a computer solution ofpq = xrn yn z2l.

(a)

Use Charpit's method to solve


16 p 2z 2 + 9q2z 2 + 4z2 - 4 = 0.
Interpret geometrically the complete solution and mention the singular solution.

(b)

Solve (D 2 + 3 DD' + 2 D' 2)z = x+y, by expanding the particular integral in ascending powers
ofD, as well as in ascending powers ofD'.

(c)

Find a surface satisfYing (D + DD')z = 0


2
and touching the elliptic paraboloid z = 4x + /
along its section by the plane y = 2x + 1.

where A, a are constants. Show that this is a possible motion of an incompressible fluid under
no body forces in an infinite fixed rigid tube
-a ::::; x ::::; a, 0 ::::; z ::::; 2a.

SECTION- B

xa
m

4m(a -b )r
4

(a)

A uniform rod AB held at an inclination a to the vertical with one end A in contact with a
rough horizontal table. If released, then prove that the rod will commence to slide at once if
the coefficient of the friction !J. is less than
3sinacosa
1+3cos 2 a

30

.e

12.

b, q

20

Find the positive root of the equation


x2

x3

ex =1+x+-+-e 03 x

correct to five decimal places.

(b)

30

Show also that the velocity at (r, 8) is

A small light ring is threaded on a fixed thin horizontal wire. One end of a uniform rod, of
mass m and length 2a is freely attached to the ring. The coefficient of friction between the
ring and the wire is !J.. The system is released from rest when the rod is horizontal and in the
vertical plane containing the wire. If the ring slips on the wire when the rod has turned
through an angle a, then prove that

ra

ce

ra

due to a source of strength m at the point r = a, 8 = 0 and an equal sink at the point r
0. Use the method of images to show that the stream function is

20

20

20

Between the fixed boundaries () = !!..._ and () = _!!..._, there is a two dimensional liquid motion

Twelve equal uniform rods are smoothly joined at their ends so as to form a cubical
framework, which is suspended from a point by a string tied to one comer and kept in shape
by a light string occupying the position of a vertical diagonal. Suppose that the string
supporting the framework is cut, so that it falls and strikes a smooth inelastic horizontal
plane. Find the impulsive reaction of the plane.

w
w

1+t'1+t'1+t .

20

!J-(10 tan a+ 1) = 9 tan a.

(b)

-X- -y- -Z-

20

(a)

(c)

20

Determine the streamlines and the path lines of the particles when the velocity field is given
by

20

10.

(b)

(c)

20

Two mass points of mass m 1 and m 2 are connected by a string passing through a hole in
smooth table so that m 1 rests on the table surface and m 2 hangs suspended. Assuming m 2
moves only in a vertical line, what are the generalized coordinates of the system ? Write
down Lagrange's equations of motion and obtain a first integral of the equations of motion.

xa
m

(b)

Also find the pressure associated with this velocity field.

What is D' Alembert's principle ? An inextensible string of negligible mass hanging over a
smooth page at A connects the mass m 1 on a frictionless inclined plane of angle 8 to another
mass m 2. UseD' Alembert's principle to prove that the masses will be in equilibrium if m 2 =
m1 sin 8.

.e

(a)

9.

.c
om

20

.c
om

8.

~X
~z
. ~X . ~ZJ
A cos-cos-, 0, Asm-sm2a
2a
2a
2a

20

ce

(c)

Fit the following four points by the cubic splines.


0

xi

Yi

11

y"3

0.

Use the end conditions

y"o

Hence compute

(i)

y (1.5)

(ii)

y'(2)

20

(c)

Find the derivative off(x) at x = 0.4 from the following table:

X 1 =grade in Statistics course


X 2 = score on Mathematics test

0.1

0.2

0.3

0.4

y=f(x)

1.10517

1.22140

1.34986

1.49182

X 3 = score on English test

Heights of 100 students at a college are as follows :


units~

the mean of

Number o[students

60-62
63-65

.c
om

.c
om

the standard deviation X1 = S1 = 10, S2 = 5, S3 = 3, S4 = 6;

18

66-68

the linear correlation coefficient between X 1 and X 2 = r 12 = 0.90, r 13 = 0.75, r 14 = 0.80, r 23 =


0.70, r24 = 0,70, r34 = 0.85.

42

69-71

Find the least square regression equation of X 1 and X 2, X 3 and X 4. What is the predicted
grade in Statistics of a student who scores 30 in mathematics, 18 in English and 32 in general
knowledge?

27

72-74

ce

[1 unit= 2.5 em]

14.

(a)

The contents ofurns 1,2,3 are as follows :

1 white, 2 black, 3 red balls


2 white, 1 black, 1 red balls

20

4 white, 5 black, 3 red balls

The frequency distributions of the final grades of 100 students in Mathematics and Physics
are shown in the following table:

xa
m

(b)

ra

Find the moment coefficient of skewness. Also obtain its value when Sheppard's corrections
for grouping are used.

One urn is chosen at random and two balls drawn. They happen to be white and red. What is
the probability that they came from urn 2 or 3?

Mathematics grades
90-99

10

fy

16

70-79

10

24

40-49

fx

15

25

(c)

21

21

20

10

100

15.

For this data, compute the standard error of estimate Sx, v and their covariance Sxv.
20
(c)

the Poisson approximation to the binomial distribution.


20

Show that in a 2 x 2 contingency table wherein the frequencies are

2
Chi-square (x ) calculated from independent frequencies is

12
23

the binomial distribution

50-59

Total

(i)

20

Ten percent of the tools produced in a certain manufacturing process tum out to the defective.
Find the probability that in a sample of 10 tools chosen at random, exactly two will be
defective by using

(ii)

w
w

80-89

(b)

Total

70-79

80-89

60-69

60-69

.e

.e

90-99

50-59

40-49

20

ce

Heig_ht (jn

the following calculations were obtained :

ra

(a)

xa
m

13.

X 4 = score on General Knowledge test

A college entrance examination consisted of 3 tests in Mathematics, English and General


Knowledge. To test the ability of the examination to predict performance in a Statistics
course, data concerning a sample of 200 students were gathered and analyzed. Letting

(a)

(a+b+c+d)(ad-bc)

X2 =~--~--~~~~~~
(a +b)( c+d)(b +d)( a +c)

Maximise

z = 3xl + 2x2 + 5x3 (by Simplex method)

Subject to

X1 + 2X2 + X3 ::::; 430

20

3xl + 2x3::::; 460


x1 + 4x2::::; 420
20

(c)

Consider the following data :

A person is considering to purchase a machine for his own factory. Relevant data about
alternative machines are as follows:
Machine A

MachineD

Machine C

Present investment(Rs.)

10,000

12,000

15,000

2,000

1,500

1,200
10

Destinations
2

Capacities

10

Total annual cost(Rs.)

15

Life (years)

10

10

Salvage Value (Rs.)

500

1,000

Demands

20

15

30

.c
om
40

ce

The cost of shipment from third source to the third destination is not known. How many units
should be transported from the sources to the destinations so that the total cost of transporting
all the units to their destinations is a minimum?

(c)

(i)

Single payment present worth factor@ 12% for 10 years= 0.322.

(ii)

Annual series present worth factor@ 12% for 10 years= 5.650.

20

Divide a quantity b into n parts so as to maximize their product. Let fn(b) denote the value.
Show that f1(b) = b, fn(b) =max {zfn-l(b-z)}. O::::;z::::;b

ra

20

xa
m

Apply Wolfe's method for solving the quadratic programming problem


Maximise

xa
m

(a)

ra

Hence find fn(b) and the division that maximises it.

16.

Zx = 4xl + 6xr2(xl) 2-2xlxr2(x2) 2


subject to

x1+2x2::::;2

.e

.e

160

130

175

190

200

135

120

130

160

175

140

110

155

170

185

50

50

80

80

110

54

34

70

80

105

w
w

20

A car hire company has one car at each of five depots a, b, c, d, e. A customer requires a car
in each town namely A, B, C, D, E. Distance (in kilometers between depots (origins) and
towns (destinations) are given in the following distance matrix:

(b)

1,200

As an adviser to the buyer, you have been asked to select the best machine, considering 12
per cent annual rate of return. You are given that

ce

Sources

.c
om

(b)

How should cars be assigned to customers so as to minimize the distance travelled?


20

(c)

(a)

+iJ2
[4 9

Maximum Marks: 300

(b)

(c)
3.

(a)
(b)

l3 -6 -4J

Iaii I> I Iail I

(b)

(c)

7.

(a)

(b)

where S is the focus of the conic.


Through a point P(x', y', z') a plane is drawn at right angles to OP to meet the coordinate
5

2x'y'z'

, where r is the measure of

OP.
Two spheres of radii r 1 and r2 cut orthogonally. Prove that the area of the common circle is
;rr2r2
1 2

(b)

Show that a plane through one member of the /c-system and one member of ~J.-system is
tangent plane to the hyperboloid at the point of intersection of the two generators.
Prove that the parallels through the origin to the binormals of the helix
x=acos8, y=asin8, z = k8
. upon th e ng
. h t cone a2(x 2+y 2) = k2z2.
11e
Determine a family of curves for which the ratio of they-intercept of the tangent to the radius
vector is a constant.
Solve

8.

(c)

j'Fi

1+x
prove that g'(x) = l+[g(x)] 3.
Taking the nth derivative of (xn) 2 in two different ways, show that
2
2
2
2
n (n-1) (n-2)
(2n)!
n 2 n 2 (n-1)
1+ 2 +
2 2 +
2 2 2
+ ... to ( n + 1) terms = --2
1
1 .2
1 .2 .3
(n!)

.!_ = 1 + e cos(), meets the

r2
+r2
1
2

IlaiJI,i = 1,2, .... ,n

f'(x) =-1-3'

axes in A, B, C. Prove that the area of the triangle ABC is

(a)

w
w

(a)

l ( 1 + 3e + e

----7---------,--'-

(1+e2-e4) '

otherwise prove that the eigen-values of A lie in the discs

4.

(~) n ~ 1

If the normal at one of the extremities of latus rectum of the conic

SQ =

for every i = 1, 2, .... n. Show that A is a non-singular matrix. Hence or

in the complex plane.


If g is the inverse off and

r ( n ~ 1J=

Two conjugate semi-diameters of the ellipse


x2 y2
2+= 1 cut the circle x 2+y 2=r2 at P and Q.
a
b2
Show that the locus of the middle point of PQ is

j'Fi

I A -aii 1::::;

J. .

ce

A=~ ~1 ~ ~ l is diagonalizable and hence determine A

Let A and B be matrices of order n. Prove that if (I -AB) is invertible, then (1-BA) is also
invertible and (I-BAr 1 =I+ B(I-ABr 1 A
Show that AB and BA have precisely the same characteristic values.
If a and b are complex numbers such that lbl = 1 and H is a Hermitian matrix, show that the
eigen-values of ai + bH lie on a straight line in the complex plane.
Let A be a symmetric matrix. Show that A is positive definite if and only if its eigen-values
are all positive.
Let A and B be square matrices of order n. Show that AB-BA can never be equal to unit
matrix.
Let A = [aij]; i, j = 1, 2, ... , n and

(c)

Show that

~J1 ( ~

curve again at Q, show that

xa
m

(a)

(a)

1(

.J2 for all b21. Does f attain its minimum? If so what is its value?

a2 {(x2 + y2r -r2x2 }+b2 {( x2 + i f -r2i} = 0

.e

2.

6.

ra

(c)

equal to ~1 + b 2
Show that

xa
m

(c)

Let f(x), x 21 be such that the area bounded by the curve y = f(x) and the lines x = 1, x=b is

(b)

(b)

1(

ce

(a)

Let T be the linear operator in R3 defined by


T(x1, x2, x3) = (3xl + X3, -2x1 + x2, -x1 + 2x2 + 4x3).
What is the matrix ofT in the standard ordered basis for R3? What is a basis of range space of
T and a basis of null space ofT ?
Let A be a square matrix of order n. Prove that AX= b has a solution if and only if bERn is
orthogonal to all solutions Y of the system ATY= 0.
Define a similar matrix. Prove that the characteristic equation of two similar matrices is the
same. Let 1, 2, 3 be the eigen-values of a matrix. Write down such a matrix. Is such a matrix
unique?

ra

1.

.c
om

Candidates should attempt any FIVE questions.


All questions carry equal marks.

.__y2
4
9

.e

Time Allowed: 3 hours

.c
om

5.

Let f(x,y), which possesses continuous partial derivatives of second order, be a homogeneous
function of x andy of degree n. Prove that
2
x fxx + 2xyfxy + /fyy = n(n-1 )f
Find the area bounded by the curve

(b)

(c)

9.

(a)

Test whether the equation


(x+y) 2 dx- (/-2xy-x 2) dy=O
is exact and hence solve it.
Solve

11.

(a)
(b)

o.

In the vector field u(x), let there exist a surface on which u=O. Show that, at an arbitrary point
of this surface, curl u is tangential to the surface or vanishes.
Prove that for the common catenary the radius of curvature at any point of the curve is equal
to the length of the normal intercepted between the curve and the directrix.
Two uniform rods AB and AC, smoothly jointed at A, are in equilibrium in a vertical plane.
The ends B and C rest on a smooth horizontal plane and the middle points of AB and AC are

ra

connected by a string. Show that the tension of the string is (

12.

(a)

1.

total weight of the rods and B and C are the inclinations to the horizontal of the rods AB and
AC.
A semi-ellipse bounded by its minor axis is just immersed in a liquid the density of which
varies as the depth. If the minor axis be in the surface, find the eccentricity in order that the
focus may be the centre of pressure.
Two bodies, of masses M and M', are attached to the lower end of an elastic string whose
upper end is fixed and hang at rest; M' falls off. Show that the distance of M from the upper
end of the string at time t is

.e

a+b+cco{

(a)

(b)

W
) , where W is the
tanB+tanC

xa
m

(c)

SECTION- A

ce

(c)

Candidates should attempt any jive Questions.


ALL Questions carry equal marks.

(c)
2.

(a)

(b)

(c)

Jf~}

from an apse at a distance c with velocity

8 2
;,

Jand is projected

~
. Prove that the orbit is
c

2
r =ccos( : }

;r c2

If t be the time in which a projectile reaches a point P in its path and t' the time from P till it
reaches the horizontal plane through the point of projection, show that the height of P above
the horizontal plane is _.!_ gtt' and the maximum height is .!. g (t + t ')
2
8

(a)

(b)

a(x)

--s~
(c)

3.

(c)

and that it will arrive at the origin after a time

20
Let G be a group of order pn, where p is a prime number and n> 0. Let H be a proper
subgroup of G and N(H) = { x E G : x- 1 h x E H \/ h EH}. Prove that N(H) -::F H.
20
Show that a group of order 112 is not simple.
20
Let R be a ring with identity. Suppose there is an element a of R which has more than one
right inverse. Prove that a has infinitely many right inverses.
20
Let F be a field and let p(x) be an irreducible polynomial over F. Let< p(x) > be the ideal
generated by p(x). Prove that< p(x) >is a maximal ideal.
20
Let F be a field of characteristic p-::F-0. Let F(x) be the polynomial ring. Suppose
f(x) = ao + a1x + ... + anxn
is an element ofF[x]. Define
f'(x) = a1 + 2a2x + ... + n anxn- 1
Iff'(x) = 0, prove that there exists g(x) E F[x] such that f(x) = g(xP).
Let K and F be nonempty disjoint closed subsets of I R2. If K is bounded, show that there
exists 8>0 such that d(x, y) 28 for xEK and yEF' where d(x,y) is the usual distance between
x andy.
20
Let f be a continuous real function on I R such that f maps open interval onto open intervals.
Prove that f is monotonic.
20
Let enzO for all positive integers n such that is convergent. Suppose {sn} is a sequence of
distinct points in (a, b). For x E [a,b], define

mp[:, +

A particle of mass m moves under a central attractive force

Let G be a finite set closed under an associative binary operation such that
ab = ac implies b = c
and
ba = ca implies b = c
for all a, b, c E G. Prove that G is a group.

where a is the unstretched length of the string, and b and c the distances by which it would be
stretched when supporting M and M' respectively.

(b)

.c
om

Isana ds

Maximum Marks: 300

ce

(b)

Time Allowed: 3 hours

ra

(a)

Find the solution of the equation


y" + 4y = 8 cos 2x,
give that y = 0 andy'= 2 when x = 0.
Consider a physical entity that is specified by twenty-seven numbers Aijk in a given
coordinate system In the transition to another coordinate system of this kind, let Ajk Bjk
transform as a vector for any choice of the anti-symmetric tensor Bjk Prove that the quantities
Ajk - Akj are the components of a tensor of third order. Is Ajk the components of a tensor?
Give reasons for your answer.
Let the region V be bounded by the smooth surface S and let n denote outward drawn unit
normal vector at a point on S. If~ is harmonic in V, show that

~- ~

xa
m

10.

-1

.e

(c)

Determine all real valued solutions ofthe equation


.
.
dy
y"'-1y"+y'-1y = 0, y' = dx

.c
om

(b)

= ~Cn.

{n: X> Sn}


Prove that a is an increasing function. Iff a continuous real function on [a, b], show that
b

Jfda= 2:Cnf(sn)
a

Suppose f maps an open ball U c I Rn into I Rrn and f is differentiable on U. Suppose there
exists a real number M > 0 such that II f'(x) II::::; M for all x E U. Prove that
lf(b)- f(a) I::::; M I b-al
for all a, b E U.

(a)

8.

az az)

f x,y,z, ( ax' ay

Suppose a is a real number not equal to nn for any integer n. Prove that

azazaz
3
- - - = z x1x2x3
axl ax2 ax3

JJ

e-(x +2xycosa+i)dxdy =

(a)

f (z) =

(b)

Find the Taylor series expansion of the function

(c)

radius of convergence of the obtained series.


Let C be the circle lzl=2 described counter clockwise. Evaluate the integral
cosh;rz

Let a 2 0. Evaluate the integral


00

I
0

cos ax dx
x 2 +1

20

SECTION- B
(a)

9.

around z = 0. Find also the

xa
m

(a)

+1)

z +9

20
(b)

20

.e

Suppose a power series

(c)

20

inextensible string of length

00

anzn converges at a point zo:;tO. Let z1 be such that lz1l< lzol and

n=O

z1:;tO. Show that the series converges uniformly in the disc {z: lzl::::; lz1l}.

(b)

aw

aw

aw

(y+z+w)-+(z+x+w)-+(x+ y+w)-= x+ y+z


ax
ay
az

20
(c)

Obtain the differential equation of the surfaces which are the envelopes of a one-parameter
family of planes.

20

(a)

4_!_

ems, is performing small oscillations in a vertical plane


6
about its position of equilibrium. Find its position at any time and the periods of principal
oscillations.
20
A carriage is placed on an inclined plane making an angle a with the horizon and rolls down
without slipping between the wheels and the plane. The floor of the carriage is parallel to the
plane and a perfectly rough ball is placed freely on it. Show that the acceleration of the
carriage down the plane is
14M +4m'+14m
.
------gsma
14M +4m'+21m
where M is the mass of the carriage excluding the wheels, m the sum of the masses of the
wheels which are uniform circular discs and M' that of the ball which is a homogeneous solid
sphere (the friction between the wheels and the axes is neglected). Show that for the motion
to be possible, the coefficient of friction between the wheels and the plane must exceed the
constant
7(M +m)+2M'
--'---------'----tan a
14M +21m+4M'
30

10.

20

In the context of a partial differential equation of the first order in three independent
variables, define and illustrate the terms:
(i)
the complete integral
(ii)
the singular integral
Find the general integral of

(a)

7.

ar

20

----=Qk,(k=1,2, ... ,n)


dt aqk aqk
correspond to a non-conservative but sceleronomic and holonomic system with n degrees of
freedom, where q, qk, Qk are respectively the generalized coordinates, the generalized
velocities and the generalized forces.
20
A solid uniform sphere has a light rod rigidly attached to it which passes through the centre.
The rod is so joined to a fixed vertical axis that the angle 8 between the rod and the axis may
alter but the rod must tum with the axis. If the vertical axis be forced to revolve constantly
2
with uniform angular velocity, show that 8 is of the form
2
n (cos 8- cos ~)(cos a- cos 8)
where n, a, ~' are certain constants.
20
A uniform rod of length 20 ems which has one end attached to a fixed point by a light

.e

Let f be analytic in the entire complex plane. Suppose that there exists a constant A> 0 such
that I f(z) I ::::; A I z I for all z. Prove that there exists a complex number a such that f(z) = az for
all z.

(c)

How do you characterize


(i)
the simplest dynamical system?
(ii)
the most general dynamical system?
Show that the equations of motion

d aL

with the aid of residues.


(b)

Solve
(Dx 3 - 7Dx Dy 2 - 6Dy3 ) z =sin (x + 2y) + e3x+y

20

ce

fdz z(z2

6.

20
(c)

Let u(x, y) = 3x2y + 2x2 - y 3 - 2/. Prove that u is a harmonic function. Find a harmonic
function v such that u + iv is an analytic function of z.

ra

5.

~
2sma

0 0

.c
om

20
2

Solve

.c
om

(c)

(b)

ce

Find and classify the extreme values of the function


f(x,y) = x 2 + y 2 + x + y + xy

=0
20

20
(b)

Explain in detail the Charpit' s method of solving the nonlinear partial differential equation

ra

(a)

xa
m

4.

~tan a , show that the sphere will cease to ascend at the end of a time
7

5V +2an
5gsina
where a is the inclination of the plane.

(c)
30

(a)

Determine the restrictions on f 1, f 2 , f 3 if

x2

y2

22

.c
om

11.

h (t)2+ !2 (t)-2 + h (t)2 = 1

a
b
c
is a possible boundary surface of a liquid.

Fidelity
20

Low

If a, b, c, d, e, fare arbitrary constants, what type of fluid motion does the velocity
(a + by - cz, d - bx + ez, f + ex - ey)
represent?

Selectivity

20
If the fluid fill the region of spaces on the positive side ofx-axis, which is a rigid boundary
and ifthere be a source +mat the point (0, a) and an equal sink at (0, b) and if the pressure on
the negative side of the boundary be the same as the pressure of the fluid at infinity, show
that the resultant pressure on the boundary is
2

(a -b )
;rpm 2 --'----.,------~
ab(a+b)

ce

(c)

(a)

(b)

20
Find the positive root of logex =cos x nearest to five places of decimal by Newton-Raphson
method.
20
3.4
3
Find the value of
f (x) dx from the following data using Simpson's Sth rule for the

(b)

13.

(a)

High

13

15

Total

52

88

Total

32

50

18

112

28

50

190

No. of degrees of freedom

Chi-square value

3
4
5
6
7

11.345
13.277
13.086
16.812
18.475
20

.e

For the Gamma distribution given by the density function


1
xa-le-x!f3 forx>O,a>0,/]>0
f(x)= fJa~
{
0
otherwise
20

find the moment coefficient of skewness.

20
= y-

12
61

2.4
11.023
3.4
29.964

For the differential equation


:

6
33

High

x , y ( 0) = 1

starting values are given as


y(0.2) = 1.2186, y(0.4) = 1.4682 and y(0.6) = 1.7379.
Using Milne's predictor corrector method advance the solution to x = 0.8 and compare it with
the analytical solution. (Carry four decimals).
20
A and B play a game of dice. A wins if he throws 11 with 3 dice before B throws 7 with 2
dice. B wins is he throws 7 before A throws 11. A starts the game and they throw alternately.
What are the odds against A winning the game ultimately?
20

20
State and prove Chebyshev's lemma for a discrete distribution and deduce from it the weak
law of large numbers.
20
Solve the following linear programming problem:

w
w
w

(c)

2.2
9.025
3.2
24.533

(c)

.e

1.6

interval (1.6, 2.2) and ..!:..rd, rule for (2.2, 3.4) :


3
X
1.6
1.8
2.0
f(x)
4.953
6.050
7.389
X
2.6
2.8
3.0
f(x)
13.464
16.445
20.086

Low
Average

xa
m

(a)

xa
m

where pis the density of the fluid.


12.

Average

Apply the chi-square test at significance level of 0.01 and show that there is indeed a
relationship between fidelity and selectivity.
Thev al ues of ch.1-square at 1o/to 1eve1or vanous
.
degrees offiree dom are g1ven b eo
1 w:

ra

(b)

14.

One bag contains three identical cards marked 1, 2, 3 and another contains two cards marked
1, 2. Two cards are randomly chosen one from each bag and the numbers observed. Denote
the minimum of the two by X and the sum of the two by Y. Find the joint probability
distribution of the random variable pair (X, Y). Find also the two marginal distributions and
the conditional distribution of X when Y::::; 4.
20
IfX1 = Y1+Y2, X2 = Y2+Y3 and X3=Y3+Y1, where Y1, Y2, Y3 are uncorrelated variates with
mean zero and standard deviation unity, find the multiple correlation of X 1 on X 2 and X 3 and
partial correlation ofX1 and X2.
20
Tests of fidelity and selectivity of 190 radio receivers produced the results shown in the
following table:

ce

than

(b)

.c
om

A sphere of radius a is projected up an inclined plane with velocity V and angular velocity n
in the sense which would cause it to roll up; if V> a n and the coefficient of friction greater

ra

(b)

15.

(a)

Maximize:
subject to

15

2x1 + x2+ 5x3

= 20

X1 + 2X2 + X3 + X4

= 10

20

Solve the transportation problem below for minimizing the cost

1
2
3
4

Warehouse
Requirement

9
7
6
6

12
3
5
8

9
7
9
11

6
7
11
2

9
5
3
2

10
5
11
10

5
6
2
9

22

Candidates should attempt any FIVE questions.


All questions carry equal marks.

11

---.

1.

(b)
(c)

2.

(a)

(b)
(c)

20

100
500
700
900
1300
1600

1600
1600
1600
2000
2400
2400

500
700
500
1000
1500
1600

3.

(a)

0 1

.e
w
w

0
1
2
3
4
5
6

Resale value
(Rs.)

l1 1 1J
lo o oJ
3
Let V= R and T: V----+ V be the linear map defined by T(x, y, z) = (x + z, -2x + y, -x +2y+z).
What is the matrix ofT with respect to the basis (1, 0, 1), (-1, 1, 1) and (0, 1, 1)? Using this
matrix, write down the matrix ofT with respect to the basis (0, 1, 2), (-1, 1, 1) and (0, 1, 1).
Let V and W be finite dimensional vector spaces such that dim V 2 dim W. Show that there is
always a linear map ofV onto. W.
Solve
x+y-2z=1
2x-7z=3
x+ y-z=5
by using Cramer's rule.
Find the inverse of the matrix
0 1 0 0

20000
14000
12000
10000
6000
3000
800

(b)
(c)

Determine the optimum policy for replacement of the above machine.


20

0 1

Loss due to
break-downs
(Rs.)

Salary of
Maintenance
Staff
(Rs.)

Cost of Spares
(Rs.)

R , let W1 be the space generated by (1, 1, 0,-1), (2, 6,0) and (-2, -3, -3, 1) and let W2 be the
space generated by(-1,-2,-2,2), (4;6,4,-6) and (1,3,4,-3). Find a basis for the space W 1 + W 2.
Let V be a finite dimensional vector space and v E V, v -::F 0. Show that there exists a linear
functional f on V such that f(v) -::F 0.
3
3
Let V = R and v 1, v 2, v 3 be a basis ofR . LetT: V----+ V be a linear transformation such that
T(v1)=v1+v2+v3, 1 ::::; i ::::; 3. By writing the matrix ofT with respect to another basis, show that
the matrix

~ ~ ~ ~lis similar to ~ ~ ~ ~ l.

F or a mach"me th e o11 owmg data are avm"1 a bl e.

Year

(a)

ce

5
12
10
6
Determine a sequence for the jobs that will minimise the elapsed time. Compute the total idle
times for the machines in this period.
20
Using dynamic programming technique solve the problem below:
. . .
2
2
2
M mnmze
z=x1 +x2 +x3
subject to
x1+x2+x3 2 30
x120,x220,x320.
20
If P n represents the probability of finding n in the long run in a queuing system with Poisson
arrivals having parameter A and exponential service times with parameter !J., show that
APn-1- (A+!J.)Pn + !J. Pn+l = 0 for n > 0
and
-A Po+ !J. P1 = 0
Solve these difference equations and obtain Pn in terms of p = ~.
Jl

(c)

'(

ra

xa
m

.e

ra

(b)

xa
m

(a)

Maximum Marks: 300

20
There are five jobs each of which must go through two machines A and Bin the order A, B.
Processing times are given below:

Job
Time for A (in hours) :
Time for A (in hours):

16

Time Allowed: 3 hours

.c
om

(c)

Availability

.c
om

Store

Costs

ce

(b)

by computing

its characteristic polynomial.


Let A and B be n x n matrices such that AB = BA. Show that A and B have a common
characteristic vector.
Reduce to canonical form the orthogonal matrix

xx -xx -xxj

X % %

4.

(a)

Find the asymptotes of the curve


2
2
2
4
4
4(x + y )- 17x / - 4x(4/- x ) + 2(x - 2) = 0 and show that they pass through the points of
intersection of the curve with the ellipse x 2 + 4y 2 = 4.

00

(a)
(b)
(c)

(a)

7.

Find the equation of the common tangent to the parabolas /=4ax and x 2 = 4by.
If the normal at any point 't 1' of a. rectangular hyperbola xy = c2 meets the curve again at the
. ' h ' , prove that t1 3h =- 1.
pomt
A variable plane is at a constant distance p from the origin and meets the axes in A, B and C.
Through A, B, C the planes are drawn parallel to the coordinate planes. Show that the locus
of their point of intersection is given by x-2 + y-2 + z-2 = p-2.
Find the equation of the sphere which passes through the points (1, 0, 0), (0, 1, 0), (0, 0, 1)
and has the smallest possible radius.
x2 y2
2
The generators through a point P on the hyperboloid 2 + - 2 - ;.- = 1 meet the principal
a
b
c
elliptic section in two points such that the eccentric angle of one is double that of the other.
Show that P lies on the curve

.e

(b)

a ( 1- 3t

w
8.

(a)
(b)

x2 (y- px) = yp2,p


(c)

ysin2xdx-

(1

(a)
(b)

(c)

=:

+ y 2 + cos 2 x )dy = 0

x+logx

State and prove 'Quotient law' of tensors.


(i)

and r =I r I' show that.


r X gradj (r) = 0

(ii)

div(rnr)=(n+3)rn

If

xr + y} + zk

VerifY Gauss' divergence theorem for

F=xyF +z 2 }+2yzk

11.

(a)

(b)

(c)

on the tetrahedron
x=y=z=O,x+y+z=1
A body of weight W is placed on a rough inclined plane whose inclination to the horizon is a
greater than the angle of friction 'A. The body is supported by a force acting in a vertical plane
through the line of greatest slope and makes an angle 8 with the inclined plane. Find the
limits between which the force must lie.
A body consisting of a cone and a hemisphere on the same base rests on a rough horizontal
table, the hemisphere being in contact with the table. Show that the greatest height of the
times the radius of the sphere.
cone, so that the equilibrium may be stable, is
A hollow cone without weight, closed and filled with a liquid, is suspended from a point in
the rim of its base. If~ be the angle which the direction of the resultant pressure makes with
the vertical, then show that

.J3

12.

(a)

bt ( 3- t )
x=
,y =
, z = ct .
1+t 2
1+t 2
A curve is drawn on a right circular cone, semi-vertical angle a, so as to cut all the generators
at the same angle ~ Show that its projection on a plane at right angles to the axis is an
equiangular spiral. Find expressions for its curvature and torsion.
Find the curves for which the sum of the reciprocals of the radius vector and polar subtangent is constant.
Solve:
)

(c)

10.

ra

6.

The area cut off from the parabola y 2 = 4ax by the chord joining the vertex to an end of the
latus rectum is rotated through four right angles about the chord. Find the volume of the solid
so formed.

Solve:
2
d 3y
dy
2 d y
x 3 --+3x
--+x-+
y
3
dx
dx 2
dx

xa
m

(c)

ce

ff-e dxdy

(c)

.c
om

-y

Solve:

ce

0000

dx

(b)

ra

au
au
au
x-+y-+z-=0
ax
ay
az
Evaluate

Find the value ofy when x = n/2, if it is given that y = 3 and dy = 0, when x =0.

xa
m

y Z X

dy
dx

"' 28 cot a + cot a


cot'f'=-----48
a being the semi-vertical angle of the cone.
One end of a light elastic string of natural length a and modulus 2 mg is attached to a fixed
point 0 and the other to a particle of mass m. The particle is allowed to fall from the position
of rest at 0. Find the greatest extension of the string and show that the particle will reach 0
again after a time

.e

(~, y ,.:.) , prove that

dx

(b)

If u = f

d 2y

- - +2-+10y+37sin3x=O
2

(a)

5.

(a)

9.

(c)

Show that any continuous function defined for all real x and satisfying the equation f (x) = f
(2x + 1) for all x must be a constant function.
Show that the maximum and minimum of the radii vectors of the sections of the surface
x2 y2
2
2
( x2 + y2 +z2) =-+-+.:.__
a2 b2 c2
by the plane
AX+ !J.Y + vz = 0
are given by the equation
a2 A2
b2 Jl2
c2v2
-----::~ +
+
= 0
2
2
2
2
1-a r
1-b r
1-c 2 r 2

.c
om

(b)

(b)

(c)

( ;r

+ 2-tan- 2

)I:

A stone is thrown at an angle a with the horizon from a point in an inclined plane whose
inclination to the horizon is ~' the trajectory lying in the vertical plane containing the line of
greatest slope. Show that if 8 be the elevation of that point bf the path which is most distant
from the inclined plane, then
2 tan 8 = tan a + tan ~
A particle moves under gravity on a vertical circle, sliding down the convex side of smooth
circular arc. If its initial velocity is that due to a fall to the starting point from a height h
2
above the centre; show that it will fly off the circle when at a height h above the centre.
3

(c)

Evaluate

~~

I I sin xsin

-l

(sin xsin y )dxdy

0 0

6.

(b)

(b)
(c)

for all f, gin F. Verify that dis a metric for F.


Prove that a compact set in a metric space is a closed set.
Let C[a, b] denote the set of all functions f on [a, b] which have continuous derivatives at all
points ofl=[a, b]. For f, g, E C[a, b] define
d(f,g) = f(a)-g(b)l + sup{lf'(x)-g'(x)l, xEI}.
Show that the space (C[a, b], d) is complete.
A function fis defined in the interval (a, b) as follows:
f(x) = q- 2 ,
when x=pq- 1,
f(x) = q- 3,
when x=(pq- 1) 112
where p, q are relatively prime integers; f(x) = 0, for all other values ofx.
Is fRiemann integrable? Justify your answer.
Test for uniform convergence, the series

4.

(a)

(b)

00

n )2n-1)

2
1+x2n

loge ( x +

1)

2
O

7.

(a)

(b)
(c)

8.

(a)

+1

dx

(b)

(c)

f(z) __ sinvfz
z z.

Is it a removable

About the singularity z = -2, find the laurent expansion of


1
(z-3)sin-z+2
Specify the region of convergence and the nature of singularity at z=-2.
(i)
Find the differential equation of all spheres of radius A having their centre in xyplane.
(ii)
Form differential equation by eliminating f and g from z=f(x2-y)+g(x2+y).
Solve: z\p 2+q 2+ 1) = C2
Find the integral surface of the equation
(x- y)/p + (y- x)x 2q = (x2 + /)z
passing through the curve xz = a3 , y = 0
Apply Charpit's method to find the complete integral of
z = px + ay + p2 +q 2.
Solve:
2
2
8 z 8 z
+= cosmxcosny.
2
ax
ay 2
Find a surface passing through the lines z = x = 0 and z - 1 = x - y = 0 satisfying

xa
m

(c)

d(f,g)= jlf(x)-g(x)ldx

singularity?
Prove that every polynomial equation
ao+a1z+a2i+ ... +anzn=O, an:;tO, n21 has exactly n roots.
By using residue theorem, evaluate

(c)
(a)

Show that z =0 is not a branch point for the function

ooi

3.

(a)
(b)

xa
m

2.

.e

(c)

(a)

.c
om

(c)

ra

(b)

Let R be the set of real numbers and


G={(a,b) I a,b E R, a:;to}.
* G x G ----+ G is defined
by (a, b)* (c, d)= (ac, be+ d).
Show that (G, *)is a group. Is it abelian?
Is (H, *)a subgroup of (G, *),when H = {(1, b) I bER}?
Let fbe a homomorphism of a group G onto a group G' with kernel H. For each subgroup K'
of G' define K by
K={xEG I f(x)EK'}.
Prove that
(i)
K' is isomorphic to K I H.
(ii)
Gl K is isomorphic to G' I K'.
Prove that a normal subgroup H of a group G is maximal, if and only if the quotient group
G/H is simple.
In a ring R, prove that cancellation laws hold, if and only ifR has no zero divisors.
If S is an ideal of a ring R and T any subring or R, then prove that S is an ideal of
S+T={s+t I sES, tET}.
Prove that the polynomial x 2 + x + 4 is irreducible over the field of integers modulo 11.
Let F be the set of all real valued bounded continuous functions defined on the closed interval
[0, 1]. Let d be a mapping ofF x F into R, the set of real numbers, defined by

ce

(a)

(b)

.e

SECTION- A
1.

.c
om

Candidates should attempt any jive Questions.


ALL Questions carry equal marks.

Sketch the ellipse C described in the complex plane by


Z=AcosAt + iBsinAt, A>B,
where tis a real variable and A, B, A are positive constants. If C is the trajectory of a particle
with z(t) as the position vector of the particle at timet, identify with justification
(i)
the two positions where the acceleration is maximum, and
(ii)
the two positions where the velocity is minimum
Evaluate
. 1-cosz
1lm ----,------cz~O sin ( z 2)

ce

(a)

Maximum Marks: 300

ra

5.

Time Allowed: 3 hours

a2 z - 4 a2 z + 4 a2 z =
axay
ax 2
ay 2

o.

1:

SECTION- B

1.36

0.58

0.34

0.20

Applying an appropriate formula interpolate for the value of i when t = 1.6.


A uniform rod OA of length 2a free to tum about its end 0 revolves with uniform angular
velocity co about the vertical OZ through 0 and is inclined at a constant angle a to OZ. Show

U2 = -X1 sin8 + X2 cos8

find the coefficient of correlation p between U 1 and U 2 and prove that 0 ::::; p 2 r. Deduce that

XhX
1

ce

ra

3.J3 + 6J.1
4(1-2j.1 2 ).

Show that the motion specified by q =

(c)

14.

(a)

is a possible form for an incompressible


x2+i
fluid. Determine the stream lines. Show that the motion is irrotational and find the velocity
potential.

~ + () J= a

sin (

~ - () J,

where a is the radius of the quadrant.


12.

(a)
(b)

Describe Newton-Raphson method for finding the solutions of the equation f(x)
that the method has a quadratic convergence.

0 and show

The following are the measurements t made on a curve recorded by the oscillograph
representing a change of current i due to a change in the conditions of an electric current:
t:

1.2

2.0

2.5.

3.0

are not correlated random variables even ifX 1 and X 2 are correlated.

x:

y:

10

22

38

(b)

Among the three hundred employees of a company 240 are union members while the others
are not. If eight of the employees are to be chosen to serve on a committee which administers
the pension fund, find the probability that five of them will be union members while the
others are not, using
(i)

hyper-geometric distribution and

binomial approximation.

Genetic theory states that children having one parent of blood type M and the other of blood
type N will always be of the three types M, MN or N and that the proportions of these three
types will be on the average as 1 : 2 : 1. A report states that out of 300 children having one M
parent and one N parent, 30% were found to be of type M, 45% of type MN and the
remaining of type N. Test the validity of the genetic theory using chi-square test at 5% level
of significance. The following extract from chi-square tables may be used.

(c)

w
w

r sin (

XhX

Ifx be one of the first hundred natural numbers chosen at random, find the probability that

(ii)

In a region bounded by a fixed quadrantal arc and its radii, deduce the motion due to a source
and an equal sink situated at the end of one of the bounding radii. Show that the stream line
leaving either end at an angle n/6 with the radius is

and

100
x+->50.

A sphere is at rest in an infinite mass of homogeneous liquid of density p. the pressure at


infinity being m. Show that, if the radius R of the sphere varies in any manner, the pressure
at the surface of the sphere at any time is

Find the stream function of two-dimensional motion due to two equal sources and an equal
sink situated midway between them

For the data given below fit a straight line and a parabola of the second order using the
principle ofleast squares and determine which of these curves is a better fit.

-yi +xj

(c)

For a statistical distribution the mean, variance, y 1 and ~ 2 are respectively 10, 16, + 1 and 4.
Find the first four moments about the origin.
X 1 and X 2 are random variables with means !J- 1, !J-2 equal variances cr 2 and positive correlation
r. If for some given 8
u1 = x1 cos8 + x2 sin8

.e

(b)

(b)

A homogeneous sphere rolls down an imperfectly rough fixed sphere, starting from rest at the
highest point. If the sphere separates when the line joining their centres makes an angle 30
with the vertical, show that the coefficient of friction !J. satisfies the following equation:

xa
m

(a)

(a)

A perfectly rough circular hoop of diameter 24 em rolls on a horizontal floor with velocity V
em/sec towards an inelastic step of height 4 em, the plane of the hoop being vertical and
perpendicular to the edge of the step. Prove that hoop can mount the step without losing

efl7rl3

11.

13.

A cylinder of mass m, radius Rand moment of inertia 1 about its geometrical axis rolls down
a hill without slipping under the action of gravity. If the velocity of the centre of mass of the
cylinder is initially v0 , fine the velocity after the cylinder has dropped through a vertical
distance h.

contact at any stage if 2.4.[2i > V > 2.4.ji.


(b)

for x = 0.3 given that y= 0 and z =1 when x= 0, using Runge-Kutta method of order four.

.c
om

(a)

Six equal uniform rods form a regular hexagon loosely jointed at the angular points, and rests
on a smooth table, a blow is given perpendicular to one of them at its middle point. Show that
the opposite rod begins to move with one- tenth of the velocity of the rod that is struck.

dz
-=-xy
dx

ce

10.

dy
-=xz+1,
dx

J.

ra

(c)

g
4acv 2

xa
m

(b)

Solve the system of differential equations

.e

that the value of a is either zero or cos- 1 (

(c)

.c
om

(a)

9.

Number of
Degree of
freedom

.01

.02

.05

.10

.90

.95

.98

.99

6.635

5.412

3.841

2.706

.0158

.00393

.000628

.000157

9.210

7.824

5.991

4.605

.211

.103

.0404

.0201

11.341

9.837

7.815

6.251

.584

.352

.185

.115

13.277

11.668

9.488

7.779

1.064

.711

.429

.297

Probability of a deviation greater than chi-square

15.

(a)

(b)

Solve that linear programming problem:

Determine the maximum value of


z=P1P2 ... Pn

Maximize

subject to the constraints

::::;4

subject to
X2

::::;

2:CiPi::::; x, 0::::; Pi::::; 1(i = 1,2, .... ,n)

3xl + 2x2 ::::; 18

i=l

(assume that Ci > x for all i)

24

45

80

43

D
E
F

VI

60

13

21

29

74

52

65

50

30

93

39

47

35

76

44

29

51

41

34

38

13

59

24

27

27

58

55

33

19

30

Market

M2
M3

Available at plant

P2

P3

P4

Market

21

16

17

18

25

13

11

14

23

13

32

27

18

41

19

10

12

15

43

Machine

M1

Machine

M2

Machine

M3

.e
w

Required at

Pl
M1

11

*******

Plant

Find also the idle times for the -three machines.

A company has four plants P1, P2, P3, P4 from which it supplies to three markets M1, M2, M3.
Determine the optimal transportation plan using Modi method from the following data giving
the plant to market shifting costs, quantities available at each plant and quantities required at
each market:

w
w

12

ce

11

IV

ra

III

xa
m

II

ra

xa
m

(a)

Solve the assignment problem represented by the following for minimisation of costs. Find
also alternate solutions if any.

16.

Job

.e

(c)

ce

A tax consulting firm has four service stations (counters) in its office to receive people who
have problems and complaints about their income, wealth etc. The number of arrivals
averages 80 persons in an eight hour service day. Each tax adviser spends an irregular amount
of time serving the arrivals which have been found to have an exponential distribution. The
average service time is 20 minutes. Calculate the average number of people waiting to be
serviced, average time a person spends in the system and the average waiting time for a
person. What is the expected number of idle tax advisers at any specified time:

Determine the optimal sequence of jobs that minimizes the total elapsed time required to
complete the following jobs and find the total elapsed time. The jobs are to be processed on
three machines M1, M2, M3 in the same order M1, M2, M3 and processing times are as below:

.c
om

.c
om

If the cost coefficient of x 1 is kept fixed, find the range for the cost coefficient of x 2 without
affecting the optimal solution.

(c)

(c)

5.

Maximum Marks: 300

(b)

Let a square matrix A of order n be such that each of its diagonal elements is !J. and each of its
off diagonal elements is 1. If B =leA is orthogonal, determine the values of 'A and !J..

(b)

Show that

(c)

diagonal. Hence determine A 25 .


Let A = [aij] be a square matrix of order n such that [aij] ::::; M \/ i, j = 1, 2, ... n. Let 'A be an
eigen-value of A Show that I'AI::::; nM.

7.

4.

(a)

(b)

Determine the curvature of the circular helix

(a)
(b)

(c)
8.

(b)
(c)

9.

Find an invertible matrix P which reduces


Q(x,y,z) = 2xy + 2yz + 2zx to its canonical form
Suppose
f(x) = 17x 12 - 124x9 + 16x3 - 129x2 + x- 1.
Determine ~(f- 1 ) at x = -1 if it exists.
dx
Prove that the volume of the greatest parallelopiped that can be inscribed in the ellipsoid

x2 y 2 z2
. 8abc
-+-+
- = 1 1S - a2
b2 c 2

3J3

(a)

r(t)=(acost)t +(asint)}+(bt)k

(a)

and an

;r: J.

Find the reflection of the plane x + y + z - 1= 0 in plane 3x + 4z + 1 = 0


Show that the point of intersection of three mutually perpendicular tangent planes to the
2
2
2
.
.d X + -y + Z = 1 1.1es on th e sp h ere x 2+ y 2 + z2 = a2 + b2 + c2.
e111ps01
2a
2
2
b
c
2
2
Find the equation of the spheres which pass through the circle x + / + z - 4x- y + 3z + 12 =
0, 2x + 3y- 7z = 10 and touch the plane x-2y + 2z = 1.
Solve the initial value problem
dy
X
-= 2
3'y(0)=0
dx X y+ y
Solve
2
2
(x -/+3x-y)dx + (x -/ +x-3y)dy = 0.
Assume that a spherical rain drop evaporates at a rate proportional to its surface area. If its
radius originally is 3 mm, and one hour later has been reduced to 2 mm. find an expression
for the radius of the rain drop at any time.
Solve

l~ ~: ~~j

.c
om

(c)

ce

A circle passing through the focus of conic section whose latus rectum is 21 meets the conic
in four points whose distances from the focus are YI, Y2, y3 and y4Prove that
1
1
1
1 2
-+-+-+-=-.
Y1 Y2 Y3 Y4 l

(b)

d 4y
d 3y
d 2 y dy
- - + 6 - - + 11--6- = 20e - 2 x sin x
dx 4
dx 3
dx 2 dx
Make use of the transformation y(x) = u (x) sec x to obtain the solution of

(c)

Solve

(c)

(b)

is diagoalisable over R and find a matrix P such that P- 1 AP is

.e

A=~ ~1 ~ ~l
l2 2 3J

Define a positive definite matrix. Show that a positive definite matrix is always non-singular.
Prove that its converse does not hold.
Find the characteristic roots and their corresponding vectors for the matrix

(b)

Let P be a point on an ellipse with its centre at the point C. Let CD and CP be two conjugate
diameters If the normal at P cuts CD in F, show that CD.PF is a constant and the locus ofF is

equation of the normal plane at the point ( 0, a,

(a)

(a)

a2 b2 [a2 -b2]2
x2 y2
2+= 2
2 where 2a +b2 = 1 equation of the given ellipse.
X
y2
X +y

3.

Show that in .[;; n + _!_ = J;


;r where n > 0 and .[;; denotes gamma function.
2 2 2n- 1

ra

(a)

(c)

B = [3 -1] C = [ 1 -5]
22'
-40

xa
m

2.

6.

Let V be the vector space of polynomials over R. Find a basis and dimension of the subspace
W of V spanned by the polynomials
v1= t 3 - 2t2 + 4t + 1, V2 = 2t3 - 3t2 + 9t- 1, V3 = t 3 + 6t- 5, V4 = 2t3 - 5t2 + 7t +5.
VerifY that the transformation defined by
T(x1, X2) = (x1 + X2, X1 - X2, X2)
2
3
is a linear transformation from R into R . Find its range, null space and nullity.
Let V be the vector space of 2 x 2 matrices over R. Determine whether the matrices A, B, C
E V are dependent where

A= [1 2]
31'

xa
m

(c)

(b)

oosmx

f --dx. Hence evaluate It.

ce

(a)

ra

1.

.c
om

Assume suitable data if considered and indicate the same clearly.


Candidates should attempt any jive questions.
All questions carry equal marks.

Show how the change of order in the integral fooo fooo e -xy sin x dx dy leads to the evaluation of

.e

Time Allowed: 3 hours

(a)

Show that the asymptotes of the curve


2
2
3
2
3 2
(x -/) (/-4x ) + 6x - 5x y- 3x/ + zy - x + 3xy- 1 = 0
cut the curve again in eight points which lie on a circle of radius 1.
An area bounded by a quadrant of a circle of radius a and the tangents at its extremities
revolves about one of the tangents. Find the volume so generated.

y"- 2y' tan x + 5y = 0, y'(O) = 0,. y'(O) =


2

.J6

2
2
(1+2x) d ;' -6(1+2x)dy +16y=8(1+2x)
dx
dx
y(O) = 0, y'(O) = 2.

Prove that if

(b)

aB X c + fJC X A+ rAX jj. For given determine a,~,y.


VerifY Gauss theorem for F = 4xt- 2y 2 } + z 2 k taken over the region bounded by x 2 + / = 4,

(c)

A cylinder of wood (specific gravity


oil (specific gravity

(a)

xa
m

units if g is assumed to be of 32 units.


A particle moves under a force m!J. {3au4
apse at a distance a+b with velocity

1.

2(a2-

2.J2
2

2.

b2)u5 }, a> band is projected from an

~ a+b
Jl. . Find the orbit.

A particle is projected along the inner side of a smooth circle of radius a, the velocity at the
lowest point being u. Show that if 2ag < u2 < Sag, the particle will leave the circle before
arriving at the highest point. What is the nature of the path after the particle leaves the circle?

3.

(a)

Show that a necessary and sufficient condition for a subset H of a group G to be a subgroup is
HR 1 =H.

(b)

Show that the order of each subgroup of a finite group is a divisor of the order of the group.

(c)

In a group G, the commutator (a., b),a, b E G is the element aba- 1 b- 1 and the smallest
subgroup containing all commutators is called the commutator subgroup of G. Show that a
quotient group GIH is abelian if and only ifH contains the commutator subgroup of G.

(a)

If x 2=x for all x in a ring R, show that R is commutative. Give an example to show that the
converse is not true.

(b)

Show that an ideal S of the ring of integers Z is a maximal ideal if and only if S is generated
by a prime integer.

(c)

Show that in an integral domain every prime element is irreducible. Give an example to show
that the converse is not true.

(a)

Show that the set C of all complex numbers is a metric space with respect to the metric d,
defined by

.e

(c)

SECTION- A

much of the wood is above the liquid. Also find to what additional depth much oil be added
so to cover the cylinder.
A shell bursts on contact with the ground and pieces from it fly in all directions with all
velocities upto 80 units. Show that a man 100 units away is in danger for a time of

(b)

All questions carry equal marks.

with its axis vertical in water and

~ ). The length of the solid in contact with the oil is a ( < ~ J.Find how

ra

12.

~)of height h, floats

Assume suitable data if considered and indicate the same clearly.


Candidates should attempt any jive questions.

ce

(b)

d ( zb z 2 ) =

for all

w
4.

z1, z2 E

I Z1- z2l

---;::::==================
2
2

(1+ I

z1l

)(1+ I

z2l )

C.

f is a mapping of a metric space X into another metric spaceY. Show that f is continuous if
and only if

(b)

w
w

Maximum Marks: 300

ra

(a)

Time Allowed: 3 hours

.e

11.

z = 0 and z = 3.
Prove that the decomposition of a tensor into a symmetric and an anti-symmetric part is
unique. Further show that the contracted product, Sij Tij of a tensor Tij with a symmetric
tensor Sij is independent ofthe anti-symmetric part ofTij
A heavy uniform chain rests on a rough cycloid whose axis vertical and vertex upwards, one
end of the chain being at the vertex and the other at a cusp. If the equilibrium is limiting,
show that (1 + !J-2) e!J.x/ 2 = 3.
A solid frustum of a paraboloid of revolution of height hand latus rectum 4a, rests with its
vertex on the vertex of another parboloid (inverted) of revolution whose latus rectum is 4b.
.
.
.f
3ab
Sh ow th at th e eqw.l.b
1 num IS state 1 h < - - .
a+b

.c
om

(c)

- ~ r ~

xa
m

in the form F

can be put

.c
om

A, B and C are three given non-coplanar vectors, then any vector F

(a)

ce

10.

Xn ----+ Xo = f(xn) ----+ f(xo)

(c)

Show that a non-empty set Pin Rn each of whose points is a limit-point is uncountable.

(a)

Show that

Iff xyzdxdydz =a 2b26 c

(b)

x2
2
z2
where domain Dis given by 2 + y 2 + ::::; 1
2
a
b
c

Ifu = sin- 1 [(x2 + /) 115 ], prove that


2

8 u
8 u
2 8 x
2
2
x2 -+2xy--+
y -=-tanu(2tan
u-3)
2
2
ax

ay.ax

ay

25

Suppose f is Riemann-Stieltjes integrable with respect to a on [a, b], m ::::; f ::::; M, ~ is


continuous on [m, M] and h(x) = ~[f(x)] on [a, b]. Prove that his Riemann-Stieltjes integrable
with respect to a on [a, b].

(a)

Prove that u =ex (x cosy- y sin y) is harmonic and find the analytic function whose real part
IS U.

A pulley system is given as shown, in the diagram Discuss the motion of the system using
Lagrange's method when the pulley wheels have negligible masses and moments of inertia
and their wheels are frictionless.

(a)

A solid homogeneous sphere is rolling on the inside of a fixed hollow sphere, the two centres
being always in the same vertical plane. Show that the smaller sphere will make complete
34
revolution if, when it is in its lowest position, the pressure on it is greater then
times its
7
own weight.

(b)

Three equal uniform rods AB, BC, CD are smoothly joined at Band C and the ends A and D
are fastened to smooth fixed points whose distance apart is equal to the length of the rod. The
frame being at rest is in the form of the square. A blow J is given perpendicular to AB at its

rhdz
..
Eva1uate 'f
- - wh ere C .
IS unit
eire1e.

A2

z-a

(z-a)

1
f ()
z = -+

de= 0.

2 + ..... +

An

.
f(z)
find the residue at a for - - where AI, A2, ...

(z-ar

z-b

6gm

10.

An, a and b are constants. What is the residue at infinity?

Find the Laurent series for the function ei/z in 0 < lzl < oo. Deduce that

iff

-1 exp( cosB).cos(sinB -nB)de = -1,


;r o
n!
2

(b)

Integrating e-z along a suitable rectangular contour show that


oo

J;

(a)

Find the function f(z) analytic with m the unit circle, which takes the values
a - cos + i sin
0::::; {1::::; 2;r On the Circle.
a 2 -2acose +1

11.

(b)

w
9.

Apply

Jacobi's

method

(c)
to

find

complete

integral

az
az
az
- p - - p - - and z is a function of xi, x2,
P1 - axl ' 2 - ax2 ' 3 - ax3

of

(~-~)
2g A

below

the pipe where sis the delivery per second.

3 3

D/)z = x y

Here

X3.

bounded

12.

SECTION- B

(a)

Using Lagrange's equations obtain the differential equations for planetary motion.

(b)

A circular, cylinder of radius 3 em and weight W whose center of gravity, G, is at a distance


1 em from the axis, rolls on a horizontal plane. If motion be just started from the position of
unstable equilibrium, show that the normal reaction of the plane when G, is in its lowest
position is W(2 + k 2)/(l + k2), where 2k is the radius of gyration about an axis passing
through G.

Using the method of images prove that if there be a source m at the point zo in a fluid

(c)

A stream in a horizontal pipe, after passing a contraction in the pipe, at which its crosssectional area is A, is delivered at the atmospheric pressure at a place where the crosssectional area is B. Show that if a side tube is connected with the pipe at the former place,

Solve : (Dx

2
+q )=1

Use Charpit's method to find complete integral of z\p

(b)

x2

Show that 2 tan 2 t + - 2 cot 2 t = 1


a
b

water will be sucked up through it into pipe from a reservoir at a depth

(a)

8.

Solve (y+z)p + (z+x)q = x+y


2

40m

is a possible form for the bounding surface of a liquid, and find an expression for the normal
velocity.

2
Find the equation of surfaces satisfYing 4yzp + q + 2y = 0 and passing through/+ z = 1,

.e

(a)

x+z=2
(c)

]!__ where m is

the mass of each rod. Find also the blows at the joints B and C.

Find the differential equation of all surfaces of revolution having z-axis as the axis of
rotation.
Form the differential equation by eliminating a and b from z = (x 2 +a) (y 2 +b)

(i)
(ii)

(b)

middle point and in the plane of the square. Show that the energy set up is

xa
m

7.

b2

ra

fo e-x cos2bxdx=-e2
(c)

( n = 0,1, 2, ...... )

.e

(a)

ce

6.

ce

If

1+2cose
5+4cose

ra

27r

fo

.c
om

cz+2

Deduce that

(c)

(c)

xa
m

(b)

.c
om

5.

(c)

(a)

by

the

lines

and

1r
() =-,
3

th e

.
so1utwn

IS

usual

notations,

+ ilf.l = -m log [ ( z 3 - z~ )( z 3 - z~3 ) where zo = xo + i Yo and zo = xo - i yo.

Apply that fourth order Runge-Kutta method to find a value of y correct to four places of
decimals at x = 0.2, when

y'=: =x+y,y(0)=1
(b)

Show that the iteration formula for finding the reciprocal ofN is Xn+l

Xn (2- Nxn), n

0, 1 ...

Obtain the cubic spline approximation for the function given in the tabular form below:

f(x):

33

244

Let the variable X have the distribution P(X = 0) = P(X=2)=p, P(X=1)=1-2p, for O::::;p::::;._!_ For
2
what value ofp is the variance of X a minimum?

(b)

If A

are

any

P(A)-P~A)nB)
1-P B

two

events

and

the

probability

P(B)#1,

prove

that

and hence deduce that P(AnB) 2 P(A) + P(B)- 1.

-1

0.1

Total

the value ofT?


Job
:
Machine A:
Machine B:

Total

0.1

0.2

0.2

0.2

0.6

0.1

0.1

0.2

0.2

0.4

0.4

Give that Y = 0, what is the conditional probability distribution of X?

name the distribution

(X -m1 )
(Y -m2 )

obtained.

Describe a quadratic programming problem and outline a method of solving it.


2
2
2
Ul + U2 + U3

(b)

Minimize

(c)

Consider a modified form of matching biased coins game problem The matching player is

U1, U2, U3, 20.

paid eight rupees if the two coins turn both heads and one rupees if the coins turn both tails.
The non-matching player as paid three rupees when the two coins do not match. Given the
choice of being the matching or non- matching players, which one would you choose and

If X and Y are independent normal variates with means m 1, m 2 and variances o}, cr 22

what would be your strategy?

J3

J4

1
5

3
6

8
3

5
2

.e

Find E(X), E(Y) and E(XY). Are X andY independent?

Subject to

Jl

*******

0.2

respectively, determine the probability distribution Z =

in the order A, B. The processing timings (in hours) for the jobs are given below. Determine
the sequence for performing the jobs that would minimize the total elapsed timeT. What is

.e

(i)
(ii)

In a factory, there are six jobs to perform and each should go through two machines A and B

xa
m

-1

(a)

(c)

Let X and Y be two random variables taking three values -1, 0, 1, and having the joint
probability distribution given below:

15.

subject to the constraints

.c
om

The random variables Y 1, Y 2, ... , Yn, have equal expectations and finite variances. Is the
weak law of large numbers applicable to this sequence if all the covariances are negative?
If X is a Poisson variable with parameter 'A and x 2 is a chi-square variate with 2k degrees of
freedom, prove that for a positive integer k, P[X::::; k- 1) = P[x 2 > 2/c].

(c)

Use simplex method to solve the following Linear Programming Problem:

ce

and

.c
om

(b)

(b)

ce

(a)

State the Transportation problem in general terms and explain the problem of degeneracy.

ra

(c)

(a)

Maximize

(a)

P(A! B)=

14.

16.

Mo = 0, M3 = 0.

xa
m

13.

Is

J6

6
2

3
10

ra

and

X:

(c)

X p-1

oo

0 (

(b)

Iff

6.

LetT : R ----+ R be defined by T(x 1,x2 ,x3 ) = (x2 ,x3 ,-cx 1-bxrax3 ) where a, b, c are fixed real
numbers. Show that Tis a linear transformation ofR3 and that
A + aA + bA + ci =0,

Where A is the matrix ofT with respect to standard basis of R

(a)

(b)

1fT is a complex matrix or order 2x2 such that tr T = tr T 2 = 0, then show that T2 = 0.

(c)

Prove that a necessary and sufficient condition for a n x n real matrix A to be similar to a
diagonal matrix is that the set of characteristic vectors of A includes a set of n linearly
independent vectors.

(a)

Let A be an mxn matrix. Then show that the sum of the rank and nullity of A is n.

ra

If A and B are two matrices of order 2 x 2 such that A is skew Hermitian and AB = B, then
show that B =0.

(b)

Find all real 2 x 2 matrices A whose characteristic roots are real and which satisfY AA' = I.

(c)

Reduce to diagonal matrix by rational congruent transformation the symmetric matrix

(c)
7.

(a)

(b)

(a)

-3

-6

Find the co-ordinates the point of intersection of the generators

.e

Hence show that the locus of the points of intersection of perpendicular generators is the
curve of intersection of the surface with the plane 2z + (a2- b2) =0.

Find the asymptotes of the curve


(2x-3y+ 1)2(x+y)-8x+2y-9=0

(c)

ns 3 =54 (n+ 1)2v 2 .

J J

y"-

Let P =(x, y, z) lie on the ellipsoid

A thin closed rectangular box is to have one edge n times the length of another edge and the
volume of the box is given to be v. Prove that the least surfaces is given by

Show that

-1

x2 y2
of the surface 2 - = 2z .
a
b2

::" (x" y") ~+" -(~ y"-'x+(;


5.

-1

~- y -2..1 = 0 =~- y -~and~+ y -2J1 = 0 =~- y -~


ab
ab..1
ab
abJl

1j

Ifx + y = 1, Prove that

0.

Find the smallest sphere (i.e., the sphere of smallest radius) which touches the lines

(c)

Prove that a curve x(s) is a generalized helix if and only if it satisfies the identity x".x"' x xiv

and show that they intersect the curve again in three points which lie on a straight line.

(b)

which subtends a constant

.e
w

(a)

x-5 = y-2 = z-5 and x+4 = y+5 = z-4

4.

!_ = 1+ e cos()

Show that the plane ax+ by+ cz+ d = 0 divides the join of P1 = (x1, y1, z1), P2 =(x2, y2, z2) in
ax1+by1+cz1+d
the ratio . Hence show that the planes U = ax+by+cz+d= 0 = a'x +
ax2 +by2 +cz2 +d
b'y+c'z + d' = v, u+/cv =0 and u- lev= 0 divide any transversal harmonically.
=

A=~~ ~ ~ ~
l-1

Find the locus of the pole of a chord of the conic


angle 2 a at the focus.

(b)

(a)

xa
m

3.

ffa'be: _;}

.c
om

If V is a finite dimensional vector space over R and iff and g are two linear transformations
from V toR such that f(v) = 0 implies g(v) = 0, then prove that g = Affor some 'A in R.

ce

(b)

The ellipse b2x 2 + a2l = a2b2 is divided into two parts by the line x =..!.a , and the smaller
2
part is rotated through for right angles about this line. Prove that the volume generated is

find a basis of R 4

Given two linearly independent vectors (1, 0,


which includes these two vectors.

(c)

2.

(a)

ra

1, 0) and (0, -1, 1, 0) of R

(c)

ce

1.

'(

dxdydz

~( 1 _ x2 _ y2 _ 2 2)

Integral being extended over all positive values ofx,y,z for which the expression is real.

.c
om

---.

dx=B(p,q)

Show that

Maximum Marks: 300


Candidates should attempt any FIVE questions.
All questions carry equal marks.

)p+q

xa
m

Time Allowed: 3 hours

1+x

x2 y2 2 2
2+-2 +2=1.
a
b
c

If the length of the normal chord through P is equal to 4 PG, where G is the intersection of
the normal with the z-plane, then show that P lies on the cone

x2
y2
2
-(ac 2 -a 2 )+-(ac 2 -b 2 )+~ = 0
a6
b6
c4

x +... +(-I)" x"]

2 2

8.

(a)

Solve the differential equation:


dy
2
xy--=y3e-x
dx

(a)

If r1 and r2 are the vectors joining the fixed points A(x1, y1 ,z1) A(x2, y2, z2) respectively to a
variable point P(x, y, z) then find the values of grad (r1 . r2) and (r 1 x r2).
Show that (ax b) x c =ax (b x c) if either b=O (or any other vector is 0) or cis collinear with
a orb is orthogonal to a and c (both).

11.

fi)

(c)

Prove that {

(a)

A heavy elastic string, whose natural length is 2na, is places round a smooth cone whose axis
is vertical and whose semi-vertical angle is a. If W be the weight and 'A the modulus of
elasticity of the string, prove that it will be in equilibrium when in b the form of a circle

zk

(c)

a(1+~cota).
2;rA,

Show how to cut out of a uniform cylinder a cone, whose base coincides with that of a
cylinder, so that the centre of gravity of the remaining solid may coincide with the vertex of
the cone.
One end of an inextensible string is fixed to a point 0 and to the other end is tied a particle of
mass m. The particle is projected from its position of equilibrium vertically below 0 with a
horizontal velocity so as to carry it right round the circle. Prove that the sum of the tensions at
the ends of a diameter is constant.
Two particles of masses m 1 and m2 moving in coplanar parabolas round the sun, collide at
right angles and coalesce when their common distance from the sun is R. Show that the
subsequent path of the combined particles is an ellipse of major axis (m1 + m2)2 R/2m 1m2.

(a)

axk

.e

(b)

= _j_(log

xa
m

whoseradiusis

12.

.c
om

ce

(b)

containing two liquids of densities cr 1 and cr 2 respectively. Show that the plane of separation
of the two liquids cut off from the axis of the cone a fraction
113
f2- O"2 ]
of its length.

(c)

(]"1 -

(J" 2

A cone floats with its axis horizontal in a liquid of density double its own. Find the pressure
on its base and prove that if 8 be the inclination to the vertical of the resultant thrust on the
curved surface and a the semi vertical angle of the cone,
1
then () = tan - [ ; tan a] .

Maximum Marks: 300


Candidates should attempt any jive Questions.
ALL Questions carry equal marks.

SECTION- A
1.

(a)

Prove that if a group has only four elements then it must be abelian.

(b)

If H and K are subgroups of a group G then show that HK is a subgroup of G if and only if
HK=KH.

(c)

Show that every group of order 15 has a normal subgroup of order 5.

(a)

Let (R, +, .) be a system satisfYing all the axioms for a ring with unity with the possible
exception of a+ b = b + a. Prove that (R, + , .) is a ring.

(b)

If p is prime then prove that Zr is a field. Discuss the case when p is not a prime number.

(c)

Let D be a principal domain. Show that every element that its neither zero nor a unit in D is a
product of irreducibles.

(a)

Let X be a metric space and E c X.. Show that


(i)

interior of E is the largest open set contained in E.

(ii)

boundary ofE =(closure of E) n (closure ofX- E).

(b)

Let (X, d) and (Y, e) be metric spaces with X compact and f: X ----+ Y be continuous. Show
that f is uniformly continuous.

(c)

Show that the function f(x,y) = 2x - 3x y +/has (0, 0) as the only critical point but the
function neither has a minima nor a maxima at (0, 0).

(a)

Test the convergence of the integral

00

A right circular cone of density {},floats just immersed with its vertex downwards in a vessel

(b)

Time Allowed: 3 hours

.c
om

x 3 d y +2x 2 d y +2y=10(x+_!_)
dx 3
dx 2
x

ce

(c)

ra

d 2y
dy
+ 2 - + y = xsin x
Solve the differential equation: - dx 2
dx

e (x + 9)

xa
m

(b)

10.

-ax

(b)

.e

d 2y
dy
Solve the differential equation: - -2- 5 - + 6y =
dx
dx

(a)

ra

9.

represents a family of hyperbolas having as asymptotes the lines x+y=O, 2x+y+ 1=0.
Solve the differential equation: y = 3px + 4p 2.

smxdx,a2

-X

00

(c)

Show that the equation


(4x+ 3y+ 1)dx+ (3x+2y+ 1)dy = 0

(b)

Test the series

n=l ( n

+ x2 )

for uniform convergence.

(c)

Let f(x) = x and g(x) = x . Does

(a)

Show that the function

I fdg exists ? If it exists then find its value.


0

- x
(z) -

(1+i)-i(l-i)
X

j(O)

+y 2

z:;tO

is continuous and C- R conditions are satisfied at z=O, but f'(z) does not exist at z =0.

z+2

) about the singularity z = -2. Specify the region

SECTION- B

of convergence and the nature of singularity at z = -2.


(c)

9.

=-1
2 1<

n=O

e2xcos(}d(}

Prove that all roots ofz7 - 5z3 + 12 = 0 lie between the circles lzl=1 and lzl=2.
By integrating round a suitable contour show that
00

_ _!!_ e
f xsmmxdx
x +a
4b
4

b
smm ,

-mb.

Using residue theorem evaluate


d()

27r

(c)

A rod of length 2a is suspended by a string of length l attached to one end; if the string and
rod revolve about the vertical with uniform angular velocity co, and their inclinations to the
vertical be a and~ respectively, show that

(ii)

(a)

Show also that when OP is first horizontal, the acceleration of a is


(b)

(c)

Find the integral surface of the linear partial differential equation :

11.

(a)

+z) ~: - y ( x 2 + z) ~; = y ( x 2 - y 2 ) z

q=

Find a real function V(x, y),which reduces to zero when y = 0 and satisfies the equation

a2 v a2 v

Apply Jacobi's method to find a complete integral of the equation


2

az
az 2 + ( -az
2 -xlx3
+ 3 -x3
axl

ax2

ax2

( x2 + y2) ( x2 +

i)

y
2

x +y

Steam is rushing from a boiler through a conical pipe, the diameters of the ends of which are
D and d; if V and v be the corresponding velocities of the steam, and if the motion be
supposed to be that of divergence from the vertex of the cone, prove that
V

(DJ2 e

-= _

v2-v2
2k

where k is the pressure divided by the density, and supposed constant.

--+--=-4;r(x
+y)
ax 2 ay 2
(c)

2 2
(X - Y ) z

2xyz

q given by:

w
(b)

(b)

_l -

Show that a fluid of constant density can have a velocity

Determine if the fluid motion is irrotational.

Use Charpit's method to find a complete integral of

(a)

3m

5a+9x J .....!:.!_J 5a-9x J


10ma 2 '5ma 2 '10ma 2

through the straight line x + y = 0, z = 1.

8.

49

Three equal uniform rods AB, BC, CD each of mass m and length 2a, are at rest in a straight
line smoothly jointed at Band C. A blow J is given to the meddle rod at a distance x from its
2
centre cr in a direction perpendicular to it; show that the initial velocity of cr is J , and that

.e
2

x (y

~g .

the initial angular velocities of the rods are :

Form the differential equation by eliminating a, band c from


Z = a(x+y) + b(x-y) + abt +c.

au
au
au
Solve: x-+y-+w-=xyz
ax
ay
az

A particle of mass m is fixed to a point P of the rim of a uniform circular disc of centre cr,
mass m and radius a. The disc is held, with its plane vertical its lowest point in contact with a
perfectly rough horizontal table and with OP inclined at 60 to the upward vertical and is then
released. If the subsequent motion continues in the same vertical plane, show that, when OP
makes and angle 8 with the upward vertical
a(7 + 4 cos 8)8 2 = 2g (1-2 cos 8).

Find the differential equation of the set of all right circular cones whose axes whose
axes coincide with the Z-axis.

(b)

3gtan,B
31 sin a + 4a sin ,B

.e

(i)

10.

(a)

xa
m

{ 3 - 2 cos() + sin()
7.

Using Lagrannge equations, obtain the differential equations of motion of a free particle in
spherical polar coordinates.

ai =

ra

where b = .J2
(c)

(b)

ce

(b)

.c
om

f[XnJ
(a)

Two particles in a plane are connected by a rod of constant length and are constrained to
move in such a manner that the velocity of the middle of the rod is in the direction of the rod.
Write down the equations of the constraints. Is the system holonomic or non-holonomic ?
Give reason for your answer.

By using the integral representation of f'(O), prove that

where C is any closed contour surrounding, the origin. Hence show that

6.

(a)

.c
om

)(

ce

z+1

ra

Find the Laurent expansion of (

xa
m

(b)

J az

-=

ax3

(c)

Between two fixed boundaries () = !!._ and () = _!!._ there is a two - dimensional liquid motion

due to a source of strength mat the point (r = a, 8


= 0). Show that the stream function is

0), and an equal sink at the point (r = b, 8

(b)

Solve the unbalanced assignment problem in minimization


where
12 10 15

Evaluate

Jdx by Simpson's rule with 4 strips.

10 3

Determine the error by error by direct integration.

14 10

13

13 12

By the fourth- order Runge- Kutta method, tabulate the solution of the differential equation

12 11

13 10

dy

xy+1

dx

10y 2 +4'

y(O)=O

(c)

-3

-2

(a)

A die is tossed. Let X denote twice the number appearing andY denote 1 or 3, depending on
whether an odd or an even number appears. Find the distribution, expectation and variance of
X, Y andX+Y.

If X is N(3,16), find P(4 :s;X:s;8), P(O:s;X:s;5) and P(-2:s;x:s;1).


If X is N(25,36), find the constant C such that P( IX- 25 I :s; C)= 0.9544.

l ,Jz;
1

_W2

.e

= 0.5987, 0.6915, 0. 7734,0. 894~, 0. 9772

11

11

10

A certain stimulus administered to each of 12 patients resulted in the following increases of


blood pressures :5,2, 8, -1,3,0,6,-2, 1,5,0,4.
Can it be calculated that the stimulus will be, in general, accompanied by an increase in blood
pressure, given that for 11 degrees of freedom the value t.os is 2.201?
Prove that a basic feasible solution to a linear programming problem must corresponds to an
extremes point of the set of all, feasible solutions.

Processing Time (in hours)


Book
Printing
Binding
Finishing
1
50
60
90
2
100
70
110
3
90
30
70
4
70
40
80
5
60
50
10
Determine the order in which books should be processed in order to minimize the total time
required to process the books. Find the minimum time.

10

A bookbinder processes the manuscripts of five books through the three stages of operation
viz., printing, binding and finishing. The time required to perform the printing, binding and
finishing operations are given below :-

Y1 Y2 y3 subject to the

ra

for z- 0.25, 0.5, 0. 75, 1.25, 2 respectively.

(a)

(c)

Fit a second degree parabola to the following data taking x as the independent variable:-

(c)

Yl, Y2, Y3 2 = 0.
A bank has two tellers working on savings accounts. The first teller handles withdrawals
only. The second teller handles depositors only. It has been found that the service time
distributions of both deposits and withdrawals are exponential with a mean service time of 3
minutes per customer. Depositors and withdrawers are found to arrive in a poisson fashion
throughout the day with mean arrival rate of 16 and 14 per hour. What would be the effect on
the average waiting time for depositors and withdrawers if each teller could handle both
withdrawals and deposits ? What would be the effect if this could only be complished by
increasing the service time to 3.5 minutes?

.e

xa
m

(i)

w
w

(ii)

Given

(b)

~1 ;

ra

If x andy are correlated variables and Sx = Sy, then find bx, x+y and bx+y, x and hence show that
rx,x+y =

(a)

(b)

Use dynamic programming to find the maximum value of Z


constraints: Y1 + Y2 + Y3 = 5,

ce

Let X 1 and X 2 have independent Gamma distributions with parameters a, 8 and ~' 8
respectively. Let Y1 = X1/(X1+ X2) and Y2 = X1 + X2. Find p.d.f., g(y1, y2) ofY 1 and Y2. Show
that Y 1 has a Beta p.d.f. with parameters a and~

(a)

xa
m

ce

Use Regula- Falsi method to show that the real root ofx logw x- 1.2 = 0 lies between 3 and
2.740646.

(c)

15.

(c)

(b)

14.

2
6

A= -6 10

16.

Solve the mx2 game :

in [0, .4] with step length .1 correct to give five places of decimals.

13.

16 12

11

(b)

[ci1 ] =

22 18 8

25 15

.c
om

(a)

.c
om

12.

10 18

(c)

Show that

fJ xm-lyn- dxdy over the positive quadrant of the ellipse ;

+;

= 1 is

anbn r(ml2)1(nl2)

4 r(;+%+ 2)

Maximum Marks: 300

Diagonalize the matrix

(b)
(c)
(a)

f (x) = _x_
1+1 xl

(c)

4.

(a)

(b)

and

.c
om

112

(a)

+a 2 y =sec( ax)
dx 2
If a, b, c are the position vectors of A, B, C prove that

d2y

7.

(c)

B=[~i ~lProvethatA2n=B 2m=1

axb +b XC +ex a
is a vector perpendicular to the plane ABC.
(b)
(c)

is differentiable.

Find three asymptotes of the curve


x 3 + 2x2y- 4x/- 8y 3 - 4x + 8y- 10 = 0.
Also find the intercept of one asymptote between the other two.
Find the dimensions of a right circular cone of minimum volume which can be circumscribed
about a sphere of radius a.
Iff is Riemann integral over every interval of finite length and f(x + y) = f(x) + f(y) for every
pair of real numbers x andy, show that
f(x) = c x where c = f(l).
Show that the area bounded by cissoid
.
. 3;ra 2
sin 3 t
x = a sin 2 t, y =a-- and Its asymptote IS - cost
4

(b)

A=[~ ~1 ]

1- x - y
x2 +

d y
d y
dy
X
Solve ---3--+4--2y=e
+cosx.
dx 3
dx 2
dx
By the method of variation of parameters solve the differential equation

(b)

when m and n are positive integers.


If A is a skew symmetric matrix of order n, prove that (I- A) (I + Ar 1 is orthogonal.
Test for the positive definiteness of the quadratic form
2x2 + / + 2z2 + 2xy - 2zx.
Determine the set of all points where the function

3.

Testforcongruencyofthematrices

= [

Evaluate F=V(x3 +i+z 3 -3xyz),find VxF

(a)

(a)

xdx + ydy
xdy- ydy

.e

2.

xa
m

A=l~ ~ !J

6.

xa
m

B1 = { ( 1,1) , (1, -1)}

~ ~], then what is the matrix ofT with respect to the ordered basis

(c)

a2 b2
show that the tangents at P and D meet on the ellipse x 2 I a2 +/I b2 = 2.
Find the equation of the cylinder whose generators touch the sphere x 2 + / + i = 9 and are
perpendicular to the plane x- y- 3z = 5.
Calculate the curvature and torsion at the point u of the curve given by the parametric
equations
y = 3au2
x = a(3u-u3)
2
z = a(3u + u ).
Solve the differential equation

8.

Evaluate fc(e-x sinydx+e-x cosydy)

(b)

(by Green's theorem), where C is the rectangle whose vertices are


(0, 0), (n, 0), (n, nl2) and (0, n./2).
If X, Y, Z are the components of a contravariant vector in rectangular Cartesian coordinates
x, y, z in a three dimensional space, show that the components of the vector in cylindrical
coordinates r, 8, z are
. () -X . () y
()
Xcos () +Ysm ,-sm +-cos ,Z.
r
r
Show that the inner product of two tensors AP and Btqs is a tensor of rank three.

(c)

Show that V = gzf [

B = {( 1, 0), ( 0, 1)} is [

(c)

(b)

(a)

(b)

IfP and Dare the ends of a pair of semi-conjugate diameters of the ellipse

x2 y2
-+-=1

Let V be the vector space of functions from R to R(real number). Show that f, g, h in V are
linearly independent where
f(t) = e2t, g(t) = t 2 and h(t) = t.
If the matrix of a linear transformation T on V 2(R) with respect to the basis

ce

(a)

(a)

ra

1.

5.

.e

SECTION-A

SECTION- B

ce

.c
om

Candidates should attempt FIVE Questions,


Selecting at least one from each of the Sections A, Band C.
All questions carry equal marks.

ra

Time Allowed: 3 hours

2
2

where

a 1.
1

ax ax

a~ {~}J

ax

lj

is a scalar function of coordinates x.

SECTION- C
(a)

A perfectly rough plane is inclined at an angle a to the horizon. Show that the least
eccentricity of the ellipse which can rest on the plane is
.
]1/2
2 sma
[ 1 +sin a

(b)

A string of length a forms the shorter diagonal of a rhombus formed of four uniform rods,
each of length b and weight W, are hinged together. If one of the rods be supported in a
horizontal position, prove that the tension of the string is

Time Allowed: 3 hours


Candidates should attempt any jive Questions.

A uniform chain, of length .e and weight W, hangs between two fixed points at the same level,

SECTION- A

A particle moves with a central acceleration !J.(r

(a)
(b)

12.

(c)

(b)

A sequence {Sn} is defined by the recursion formula Sn+ 1 =

(c)

converge? If so, find lim Sn.


Test for convergence the integral
1

then deduce law of composition of velocities from this equation.


(i)
Define relativistic energy and momentum and establish
2
22
2 4
E =pc +moe
with usual notation.
(ii)
Two lumps of clay each of rest mass mo collide head- on with velocity 315 c, and stick
together. What is the mass of the composite lump?

.j3S;, S1 = 1. Does this sequence

JxP (logl/ x)q dx.


0

3.

(a)

Find the shortest distance from the origin to the hyperbola


x 2 + 8xy + 7/ = 225, z = 0.

(b)

x- y 3 dxdy does not exist over R = [0, 1; 0, 1]


(x+ y)
VerifY the Gauss divergence theorem for
F = 4x ex- 2y 2 ey + z2 ez
taken over the region bounded by
x 2 + y 2 = 4, z = 0 and z = 3.
where, ex, ey, ez are unit vectors along x-, y- and z-directions respectively.
Examine the nature of the function

(c)

4.

Show that the double integral

fJ
R

.e

A particle of mass m projected vertically under gravity, the resistance of air being mk
2
(velocity). Show that the greatest height attained by the particle is V I g [A,- log (1 + /c)]
where V is the terminal velocity of the particle and A,V is the initial vertical velocity.
An ellipse is just immersed in water (touching water surface) with its major axis vertical.
Show that if the centre of pressure coincides with the focus the eccentricity of ellipse is V4.
Two solids are each weighed in succession in three homogeneous liquids of different
densities. If the weights of the one are w1, w2, W3 and those of the other are W1, W2 and W3,
prove that
WI(W2- W3) + W2(W3- Wl) + W3 (Wl- W2) = 0.
Masses m and m' of two gases, in which the ratio of the pressure to the density (pip) are
respectively k and k', are mixed at the same temperature. Prove that the ratio of the pressure
. . h
d . mk++m'k'
to th e d ens1ty m t e compoun s IS - - - m+m'
Derive the Lorentz transformation equations.
Ifu and v are two velocities in the same direction and Vis their resultant velocity given by
tanh- 1VIc= tanh- 1~-+ tanh- 1~'

(c)

is a homomorphism of G into G with kernel K, then show that K is a normal subgroup


of G.
Ifp is a prime number and pa lo(G), then prove that G has a subgroup of order pa.
Let R be a commutative ring with unit element whose only ideals are (o) and R itself. Show
that R is a field.
Let A be a subset of the metric space (M, p ). If (A, p) is compact, then show that A is a
closed subset of (M, p)

Determine its path.

(b)

c\), being projected from an apse at

(a)

If~

(a)

2.

(a)

11.

(b)
(c)

.e

(c)

(a)

distance c with a velocity J2f.l I 3c

1.

xa
m

ce

ra

(b)

xa
m

(a)

K W +_!__W'+_!_W.
21
4K
8K
If in a simple harmonic motion u, v, w be the velocities at a distance a, b, c from a fixed point
on the straight line (which is not the centre of force), show that the period Tis given by the
equation
u2 v2 w2
4;r 2
~(b-c)(c-a)(a-b)= a
b
c
1 1
1

ce

and weight W is attached at the middle point. If K be the sag in the middle, prove that the pull
on either point of support is

10.

Maximum Marks: 300

ALL Questions carry equal marks.

b ( 4b2 - a2 t2 .
(c)

~- ~

.c
om

2W(2b -a

-1

ra

.c
om

9.

(b)

f (z) =

(x+iy)
4

+y

10

'z

:;t

j(O) = 0
in a region including the origin and hence show that Cauchy-Riemann equations are satisfied
at the origin but f(z) is not analytic there.
For the function
-1
f(z) = z 2 - 3z+ 2
find the Laurent series for the domain
(i)
1 < lzl < 2,
(ii)
lzl > 2.

Show further that

mx = _ av + .Y[aB _aA]-z[aA _

ax

p!(z)dz=O

(b)

(c)

+4

sma,

a> o)

The function f(z) has a double pole at z = 0 with. residue 2, a simple pole at z = 1 with
residue 2, is analytic at all other finite points of the plane and is bounded as lzl -+oo. If f(2) =
5 and f(-1) = 2, find f(z).
What kind of singularities the following functions have?
1
(i)
-at z = 2;ri
1-e 2
1
- - - - atz=;r/4
(ii)
smz-cosz
cot ;rz
(iii)
--~ at z = a and z=oo
2

(c)

(a)

9.

2
{v(l :k) }(l+~k+ s!')
2

(b)

ra

(z-a)

xa
m

In case (iii) above what happens when a is an integer (including a= 0) ?

(b)

(c)

8 z

-+
-=
e
2
2

(a)

-x

cosy
8t
which----+ 0 as x -+oo an has the value cosy when x = 0.
One end of a string (x = 0) is fixed, and the point x =a is made to oscillate, so that at timet
the displacement is g(t). Show that the displacement u(x, t) of the point x at timet is given by
u(x, t) = f(ct- x)- f(ct + x).
where f is a function satisfying the relation

+2a)

10.

(a)

J(t)-

gC:a J.

A particle of given mass m moves in space with the Lagrangian

L = ~m(x 2 + y2 +z 2 )-v +xA+ yB+zC


where V, A, B, Care given functions ofx, y, z. Show that the equations of motion are

/i = a2(x2- /

+ /cxy)

Obtain the Simpson's rule for the integral

ax

J(t
8.

8 z

(x2 +

Where A, is a variable parameter.

I=jJ(x)dx

(a)

7.

Two sources each of strength mare placed at points (-a, 0) and (a, 0) and a sink of strength
2m is placed at the origin. Show that the stream lines are the curves

and show that this rule is exact for polynomials of degree n ::::; 3. In general show) that the
error of approximation for Simpson's rule is given by

(c)

(c)

(b)

Verify that the differential equation


(y 2 + yz)dx + (xz + z2)dy + (y 2 - xy)dz = 0
is integrable and find its primitive.
Find the surface which intersects the surfaces of the system
z(x + y) = c(3z + 1), c =a constant,
orthogonally and which passes through the circle x 2 + = 1, z = 1.
Find the characteristics of the equation pq = z, and determine the integral surface which
passes through the parabola x= 0, = z.
Use Charpit's method to find a complete integral of
p 2 + q2 - 2px -2qy + 1 = 0.
Find the solution of the equation

.e

(a)

(Assume that k is so small that the flow remains appreciably one-dimensional throughout).
A spherical globule of gas initially of radius R, and at pressure Po expands in an infinite mass
of water of density p in which the pressure at infinity is zero. The gas is initially at rest and
its pressure p and volume v are governed by the equation pv 413 = constant. Prove that the gas
112
28
2
doubles its radius in time
15
P0

Ro ( PJ

SECTION- B

6.

ce

-00

xsmaxdx_Jr -a.

ra

00

(b)

l-

.c
om

(a)

ce

5.

transforms the circle x 2 +


4x = 0 into the straight line 4u+3 =0, where w = u+ iv.
Using Residue theorem show that

xa
m

2z+3
z-4

W=--

and two similar equations for y and z. Find also the Hamiltonian H in terms of generalized
momenta.
A wheel consists of a thin rim of mass M and n evenly placed spokes each of mass m, which
may be considered as thin rods terminating at the centre of the wheel. If the wheel is rolling
with linear velocity v, express its kinetic energy in terms of M, m, n, v. With what
acceleration will it roll down a rough inclined plane of inclination a ?
Find the moment of inertia of a solid hemisphere about a diameter of its plane base.
A solid hemisphere is held with its base against a smooth vertical wall and its lowest point on
a smooth floor. The hemisphere is released. Find the initial reactions on the wall and the
floor.
Derive the equation of continuity for a fluid in which there are no sources or sinks, liquid
flows through a pipe whose surface is the surface of revolution of the curve y = a+ kx 28 /a
about the x-axis (-a::::; x::::; a). If the liquid enters at the end x =-a of the pipe with velocity V,
show that the time taken by a liquid particle to traverse the entire length of the pipe from x=-a
to x =+a is

.e

(c)

ac]

az ax

.c
om

where C is any closed contour enclosing the points z = and z = 2.


Show that the transformation

ax ay

R=-

b-a )
2880

jiv(7J)

1J E ( 0, 2)
1

Apply this rule to the integral

J1+x
dx and show that IRI::::; 0.008333.
0

(b)

Using fourth order classical Runge-Kutta method for the initial value problem
du I dt =- 2t u2, u(O) = 1,
with h = 0.2 on the interval [0, 1], calculate u(0.4) correct to six places of decimal.

In an examination students are awarded letter grades according to the following scheme:

80 2 Score < 90

70 2 Score< 80

60 2 Score < 70

Score> 60

256

For each score print a message identifying the letter grade. For grade Fan additional message
"This is a failing case" may be mentioned. Prepare the flowchart for this scheme and write
the program in BASIC.

SECTION- C
(a)

13.

Suppose that the universal set U is given by U = {xi 0 ::::; x ::::; 2}. Let the sets A and B be
defined as follows:

ce

11.

(b)

A printing machine can print n "letters", say a 1, a 2 , ..... , an. It is operated by electrical
impulses, each letter being produced by a different impulse. Assume that there exists a
constant probability p of printing the correct letter and also assume independence. One of the
n impulses, chosen at random, was fed into the machine twice and both times the letter a 1
was printed. Compute the probability that the impulse chosen was meant to print a 1.

(c)

If two independent variates, X andY, have Poisson distributions with means m 1 and m2 , find
the distribution of the sum X+ Y.

(a)

A certain cubical die was thrown 9000 times, and a 5 or a 6 was obtained 3,240 times. Find
the standard, error of the proportion of successes in 9000 throws; and deduce that the
probability of throwing a 5 or a 6 in a single trial almost certainly lies between 0.345 and
0.375. Can we say the die is unbiased?

(b)

Suppose that X has distribution N(3, 4). Find a number C such that

(c)

A physical quantity is measured many times for accuracy. Each measurement is subject to a
random error. It is judged reasonable to assume that it is uniformly distributed between -1 and
+ 1 in a conveniently chosen unit. How many measurements should be taken in order that the
probability will exceed 0.95 that the average will differ from the true value by at most 0.2?

(a)

A police department has the following minimal daily requirements for police officers during
its six shift periods:

P(X> C) = 2 P(X :s; C).

AUB

(iii)

AnB

(iv)

AnB

14.

xa
m

(ii)

ra

AUB

xa
m

Time of Day

22

f(x) = 2x,

6 a.m.- 10 a.m.

55

elsewhere.

.e

0,

0<x< 1

Let H(x) = 3X + 1. Obtain the pdf ofY = H(X).

elsewhere

Find P(B) where B ={X+ Y 2 1}.

(a)

Eight coins were tossed together, and the number x of heads resulting was observed. The
operation was performed 256 times: and the frequencies that were obtained for the different
values of x are shown in the following table. Calculate measures of central tendency and
mean deviation about mean:X

1
2
3
4
5

f
9
26
59
72
52

88

110

6 p.m.- 10 p.m.

44

10 p.m.- 2 a.m.

33

x + x; ,

=0

10 a.m.- 2 p.m.
2 p.m. - 6 p.m.

An officer must start at the beginning of a 4-hour shift and stay on duty for two consecutive
shifts (an 8-hour tour). Any one starting during period 6 stays on duty during period 1 of the
next day. The objective of the police department is to always have on duty the minimal
number required in a period but to do so with the least number of officers. Develop the
corresponding linear programming model.

x,

Suppose that the two-dimensional continuous random variable (X, Y) has joint pdf given by

f ( y) =

12.

Minimal Number Required

2 a.m.- 6 a.m.

(c)

Period

Suppose that X has pdf

.e

(b)

(i)

ra

A={xl1/2<x::::;l} andB= {xl1/4::::;x<3/2}.


Describe the following sets:

7
1

.c
om

Score 2 90

29

6
7
8

Letter Grade

ce

Score Range

.c
om

(c)

(b)

Show that a problem in the theory of games can be expressed as a linear programming
problem

(c)

Respond True or False to the following, justify your answer in case of False:
(i)

If the number of primal variables is much smaller than the number of constraints, it is
more efficient to obtain the solution of the primal by solving its dual.

(ii)

When the primal problem is non-optimal, the dual problem is automatically


infeasible.

An unrestricted primal variable will have the effect of yielding an equality dual
constraints.

(iv)

If the solution space is unbounded, the objective value always will be unbounded.

(v)

The selection of the entering variable from among the current non-basic variables as
the one with the most negative objective coefficient guarantees the most increase in
the objective value in the next iteration.

I_-=========

In the simplex method, the feasibility condition for the maximization and
minimization problems are different.

Tim e.AI/()Wet/; J (t(mr,v

(vii)

A simplex iteration (basic solution) may not necessarily coincide with a feasible
extreme point of the solution space.

Candidates should attempt Question Nos. and 5 which arc compulsory. and any three of the remainin&
questi.ons selecling al least one question from each sectiofl.

(viii)

If the leaving variable does not correspond to the minimum ratio, at least one basic
variable will definitely become negative in the next iteration.

Develop mathematical model of a balanced transportation problem Prove that it always has a
feasible solution.

(b)

Find the optimal assignment for the given assignment costs:


Machine
2

14

10

15

Artemp{ any five of lhe foiiO\\Ing;


Ill)
Let V be a "ector space overR an

(b)

12

(e}

16

(d)

(e)

*******

tf)

Define addilion in T component wise and scalar mulliplicatlon by a compte~ number a + ifJ.
by ( +IP)(x.y) =(a.(- py.Px+ay)V.P I< how that T s a veclor space over C
Show thai if A is a characteristic root of n nonsingular malrix A chen }..' 1 is a clmrameristic
root of A' 1
2
2
Use the Mean value theorem 10 prove that- < log 1.4 < - .
7

Show thai

JJ r"'-v'"' 'tlxd1- ,..!.,.:,,,ml


r 1rm
4
r(i + m+'l)
ctrcle r +y~ : r l.

for all positi ve 1ulues

of~

and v lying inside the


.

Find lhe equalions of 1he pranes bisecting me angles bel ween U1e planes 2x- y - 2: - 3= 0
and 3.H 4y+ I = 0 and specify U1e one 111uch bisect$ 01e acute angle.
Find U1e equation to the common conjugate diamete-rs of the conics x'-.. -tw +C.y'; I and

zr 1- G..:y+9y' = L

(a)

Pto1 e !hal n. real symmetric matrix A is ]lOSiln'e definite if and on I~ 1f A = BB' foqome onesiugulut m!ltri.x B. Show also that

2,

A=[~ ~ ~)
3 7

IJ

.e

Give the economic interpretation of duality in linear programming.

w
w

13

MaxillllUII Mark~-: J(JII

Let 'f' : {(.t, y) f x.y ~: v}

xa
m
3

(c)

I.

Job

""'===========:=J

PAPER-I
SECTION-A

ra

MATHEMATICS

(a)

ce

15.

I C.S-J\l nins 211M

.e
xa
m
ra
ce
.c
o

(vi)

.c
om

(iii)

Is posilive definite and find the matrix B such lha1 A = eat_~Jere B' srands for the transpose
of B.
Prm'e ~1n1 a system AX =B or a non homogeneous equations in a unknowns has a unique
solution pro1acled 011~coefficeni matr" is not singular.

te)

Prove thai two similar matrices have the same characteristi<' roots. Is Its converse true?
JustHv your claim.
Reduce the equation
' +/+='-2.w-2y=+2:,r+.x:- y -2z+l\: (t inlo canonical from and determine the
nature or the quadnc.

(b)

td)

ccr~trc

of gmvity of the positivi." octont of the cllipsoicl :: 1 {, 1

6,
demityv:rries as xyz.

solve

f (x)={O. x~ irrmlonal

d' J'

dx'

t
x 1.r rntmrurl
Show thot fis nQt RiCilUinn- integJoble on [n, b)
(c)

lb)

Show that

(U)

2 3

F(n.b) = J:{inx (ax'

ill atu.iniruum.

1 b.t)}' dx

d"'l x
\JfTl
Find consl:mts a and h for whl<:h

ce
.c
o

4.

-,I ) = a t -- b,.n+ c.,. b...


111
Prove the re(luction formuloe;
a,., 1 : : 't ''*- Kb,.
b_1 "' b' - K11. r~

c:_

rb.,

_.: y' ,:" = 4. .,1 2y - : = 2and thepoinl(l .-1, 1).


A varillhlc stroigbtlinc alway inttsect. the linos x c c.y =0;; = c.= ~ 0= = c. .!' = 0 ,

(d)

Find the cquotion to lL IDeUS.


Show that the locu. of mid-points of cho rds o f the co no

7.

t>J~ ""' = 2Jj: + 2g;.-+ 2h.')' = 0


the

x _ _J' _ _=
1 m n

IS

the

fm + an)~~ o

r,:

(c)

!>rove ihat n cenlrnl ti>rcd tnoticm is motjun in phan~ and \be are.. I velocity o f a particle U.

(d)

A lt#pc:-mid.>l pint" hovinglts poral lel sides of length x Md y, {X. > y) ot a distance z apart. is
immer$ed vertically in wnter into x ~tide ujlpemost (horizontal) M a depth d below tl1e water
surlhc:e. Find the totallhrust ()n the s urface.
In what direction floon the point (1. I. l ) i.~ the directional derivative of f~.,)y::'a
m.aximuon? Compute its magnitude.
(i)
Show lbat Ute vovariont dcrivotives of Ute fundamental metric tensors.
. I o.
g, .g " , t:,' vnrus

(e)

(l)

oonsIJ! u~

x::k and S is thesutfnce ofthe porallelepiped bounded by

x = Qy ~z - Q K ~y = la~z - ~

1
If sij ami Yli '" " lwo m<1.ric tensono dcGncd al point .ond { } and
fj

Ha~ an integral which ~~ a polynomial in x. Deduce the genef31 solution.


It 1' is the tenslOn :It 1ny Jll)lnl P of' 3 cotenary Md '1'6 that ~t the lowest point ~. then shc>W that
1''- = W1 where\\' is tlte weight of(b" arc CP of U>o Catenary.

its centn:. show that, if straight frictionless airless tunnel be m01de &om one p<1int of Ute
c:w'lh ' surface to any point, a ttin would traverse the tunnel in slightly less than three
quarter of Oll hour, As$Ume the ""rth. to be oJtomogcneOUS sphere of' rodiu~ 6400 km.
A stn~ II bead is projected with any velocity alo.ng the smooth circulor wire under the action of
for~ varying inversely as tbe liiUl power of Ute dist>nce from a centre of forcesitu.1ted on
llu>e!rcumf~'l'Cnec. Prove tlun Ute pressure on lli<> wire is conslllnt.
A conicol buoy I meter long, and of boliC diameter 1.2 meter, tlool in water witlt i\s op<l.'<
downwards. l) etermin e u,e minimum weight of the buoy. for st~blc equilibrium.
Suo\\ Uoat
(i)
(A I B). {13 C} (C . A) ; 2A.D c
(ii)
\ (A B) = {B.\7),\ B(V ,Aj - (A.V)B ' A(v.B)
Evaluate
n 1/Jt,
Where F :. 2.tyr y=' l

acnON - B

a,,

hypcrll<llns with a

Jf/r

plone

w
tlx'

(a)

Q(

A telephonic wire wdghing 0.04 Jb p~ foot ha a borizonlll l span of 150 fe..t and b'ilg of J.5
feeL Find tl1e l<:ngth oftl1o 1vino nod also fm(( maximum ton. ion.
Asuming that U1e ea rth otlracLs points inside it wlth 11 force which varies as the distance from

('.hrisl.ofl'e l ymbols Qf U1e second kind. then prove that {

~ l. fJ
yf I!I

fl "'" t.hccOIIil>IJOOtlinj!
l!i
is mi.' ed tensCJr of the

type -~-

.e

(gl

line

( c~,,

.e

to

8.

(d )

pomllel

(a)
(b)

AUempt any five of the follow!ng:


(o)
Show tl>e
tPy 4.~-~ 8y - 0
3--:-

(b)

(dl

common axis and Inngent at the vertex.


Solve

Jv

ra

(c)

xa
m

F ind lhc ~quotioa of the sphor~ lhrough lbo circle

(ul o hm 1 gt~) .<( hl o bm fn)y

5.

Solve the differential equation y = x - 1ap - ap1 find the$ingular so lutioo and interpret ii
Show Utat (4.H3y.,. l )dx .... (3.H '2)'~ l)dy= O ttj)l'l:s\lnts n (~,nily

(c)

(b)

11-r I

.~e.Y sin 2x.

x - - Y=(x-1) - -:x- 1
clx
d,..!
by tl1e metl10~ of varlntion of paramcl.elli.

Whcte symbols bnve Utoir usunlmeaniugs.

Drawn

l)y=

(e)

1ru,~ m1h <,lerivtiw of r with respect lo ~ ~s given by

C111 , 1 -

- 4x dx - (4."'

geometricnUy.

log~
I -I .....-l )
-d " ( -) = (- 1) - 11! ( l<l~x- 1 --

(d)

I)

Let

{b)

Sbow tbot simultaneity i~ rcl,,tive in special n:btivly theory.


d' l'
ely
Re<fuc~ - - - P- + Qv- R where P, Q. R are nmction~ nf x, tcJ !he nonnnl fom1, Hence
d-r'
clx .

(ii)

~ - 1 . lf the

ce
.c
o

Find the

ra

(n)

xa
m

3.

Estoblis h the .lormulll E = mt;', tb"symbols have their usual mo:uung.

I C.S..1\lnins 200<1
Suppose f

~ twice differentinblc real vnlucd function in (0, "' ) nnd Mi), M 1 nnd M; lite
le"!;l upper bounds of j{(x)l,lf'\x)l and IJ"(.<)I tt'8pec.tively in (0, <7J). Prove for

<i)

MATHEMATICS

===:::::=::====:::'.l

['(x) =

MIL>illuull Mark~: J (JII

(d)

=f(.s)ds

(s - =)

o Utewi>c, derive nn intcgr>ll,p1<l$cntalion far f(zJ iff(z) is rutlilytic on :md

Let n he a nxed posicive rnteger and let Z., ba the ring of Integers modulo n Let.
G={liEZ,tl l a ~o !Wd a is rela\ivi ty prrme to o) . Show that 0 is a gr0 up t.urder
(a)

multiplication defin.ed in
Hence. or otherwise, show that n' 1' 1"'a (mod n ) ftlr all integers
a relatively prime ton where <!>(n) denotes !he number of positive rntegers that are less than n
and are relative!) prime to n.
l-et M be a subgroup and N a nomu'\1 subgroup of o sroup 0 . Show that MN is a subgroup or
G and MNIN is isomorphic to M /(Mn N).
Gi\en that the lenns of a sequence 1a.,} are such thai each II!<- k ~ 3. is the arithmetic mean of
its two ]mmediately preceding terms. Show that ~le sequence converges. Also lind the limil
or the $ec)uence.
Delenuine the lues of x for which I he infinite product

Evo luale

( i)

(b)

21; - 3>:, '1' 3 ..;, ~ ll7


4x1 + x, + 2,., = 283.
L<:l

(i)

Ia)
(b)
(c)

"

by means of a truth table.

Let I' ba a Jlnite field. Show that the charaot~ristic of I' must he a prime integer p and the
number of elements in f' must: be p'" f-ot some positive integer m.
Let F be a field and F[x] denote tlre set of all polynomials defured over F. If f(>t) is rur
irreducible polynomial in F(x]. show !hat the ideal generated by f(~) in P[x] is ma.im!ll and
F[xJ/(f(x)J is a field.
Show that ru1y finite commutative ring wrrh no zero dlvrsors must be a field.

r,

1(.<-1
(''-1
)'1
)'
-)' ,1
_
_
-(-1
- -). l(.t-1
(.1x!-l)... 2x
3.t
4 x
+- - 5.>:

IIC'IION

A ttempt nny live ofthefoUowing:


(o)
Solve:
JX1 = X-. J'' ::J

(b)

Prowtlul if :<: t xi

-><: = 1 when z O. thosolutiou ofUtec~uotion

(S-x1 ) p, +(S -.~;) p, t- (S -X, ) p, = S-:


C~n

etiiTeCI

to five

0 ()]

.e
5.

actu:ol error,

.8 = A1 ond C = 0 I I)
4 9
0 0 1
.j,

(p+q)'= p',q'

7j

;t= -

Prove De Morgan's Theorem

11 1md

Write 3 BASIC prognnn thai compul~ tho inVcr.Jc of A. detcrminont of A ond the
product of llu> mntrix nnd it invcl'!lc.
Write a bssicprogmm to evnJuate Ute fonn:uli

(ii)

2.

$
A~ 3
[l

x, ( J- 1 -i'l,~, )

insid~ of C.

Solve the fuilowing SY$lctn oflineor equ~tioM, u.King GausH~Iiminauon metb~


..... . 6.t, 3 ..... = 6

w
w

(ii)

l+-~

pJc;:;s orde.oimuls.

(ii)

converges absolutely. Find its value whenever it converges.


Show !hat any for given points of the complex plane can be carried by a bilinear
transformation to positions I, - I, k and - k where the valueofk depends Onlhe &-iven points.
(i)
Using Newlon-Rirrhson's method, show that 1he iteration fom\Uia for finding the
reciprocal of tlre p1 rool oFN is

:r,.,

fonncd by Ute
tmd

d\ , by subdividing tbe inlet-val (0, 1) into 6 equol pnrts and using

Simr;on s o ne-th.ird rule, Hence lind Ihe value of

f10+ X~ )

If)

surf~ce

J!- :1 S ii',:=O

z. v:u-ies inid.. uf C, i< difliorertt.iable under the integtal sign. Find Uu:. derivutivc. Hence of

xa
m

(c)

cylinder ;<'+ l=tt1 , 0 <.r~b and the circulor dise~~


.'t-' + y -' "S a-..
' -z =- b .
S1qlpooe f(s) ~~continuous on 11 circle('_ Show thnt

z..

(b)

JJ. (x'O:ldr 1 x'ydr dt+X:d1:cly)

ce
.c
o

(h )

Allempt any fave of !he following;

(a)

Ev-.lualc

By transfonning in1o triple integral where S is the clo..,d

.e
xa
m
ra
ce
.c
o

I.

(ii)

Candidates should attempt Quesuon Nos. I and 5 11 hich arc compulsory. and any three of !he remafnin&
questions seleccin!l al least one queslion fmm eaph seclion.

PAPER U
SECTION A.

2~1 [f(.H2h)- f(x)]- lif"(u)

11 or some u .; ( .<.:t~ 2/t ). Hcncc sho11' thai ;\/t' !. ~.\10'\1(1

ra

Time:;ll/()Wetk J (t(mr,v

coch x " (), ~ > 0 that

he given in lhe form

s'{lx. -:)' + (.~,-=)' +(>;, -zfl' =(;>.:,+x,->;, - 3:(


W harc

o: .1 :

S = XI .a-:<.>+: and p i = -

a.,

l.l,3.

Pmd the ruomenl ofineni~ of

:n1 elliptic nrea ~bout o line CP ln~lined at 0 to the major ax!.


a tangent parolleltc> C P. where C is the centre oflhc ellipse.
Dclennlne tb<> ~ircam lin"" nnd the t>alh titles Ql' the particle when the eQmpOnt11ts of the
veii)City focld ore gi1'en \ly

(b)

I' ( A) = ~/Jiul P(B )=1 .

4
Shu\V tlull

3
4
.
;; PA
( .~ B ) ;.-5
And -->
~8
( ii)
ff X is a N(O. I). lind tlie prQbability dcnity function of[ X J.
Two plu~ers each take Olil one or two mntt:hcs nod guess how many matches lhc 11Pil0n cnl
has tlten. lf nne of lbe players guaS>Cll etJrtoctly. Oten the Jost.-r hns lo l,tny him "" many
1'\lpee>< as lhe sum <>f the number held by bo01 plyen~; Qth~vise tb~ pay ment is Z<'fO. Write
dt\wn lhe pay-off matri~ ~nd ohwin the optimal slratcgi<=i' nfboth plnyc;n; AL<IJ l'ind th~ v~ lu~
of the game.

(h)

(~ )

Solve by Ch rpit' method the equation


;l.<(x 1) 2!'4\)' lt/J'(y \ )- 2Ju:: - "2qy:>l

( ii)

Solvll:
( D' - DD '- 2D")~ = 2x~3_1>~ ~'r.

w
8.

(a )

tv

20

2
J
"

7
16

12
15
J7
12

16
6
12

1;

(x'. ,.')'

nrc tho velocity components q(-a poss ible liqu id motJOn_


E.~antine this for irrolOIIiQIJll l motion.
An explosion 1n a fnetory manufucluring explosives can occur be.:ause of
(i}
lo.OO.ge of olcclriciry.
(ii)
defooh> in machinr.ry.
(iii) <:Melessness of workers or
( iv)
sabotage.
Titu prol)ability lhnl !!tore is n leakage of clcctricily is 0.20, lite m~chittery ~ defective
is 0.30. the wor'-.-ers ~re cMeles5 1< (\,4 0, there i ~botage is 0. 10. 1'he engineers feel !hat an
explosion can occur wiUt probability
(i)
0.25 be<:uusc of l""kbge uf "lcctri~iry,

("' - l'')=

;.sr " (x'-l )"w

-2..w~

Whe re " i&lhe length of Oto plonk.


Do fine irrotalionalnnd rotational flows giVing on ex.1mple for each.
Show !hal

""" (.r'-

ur

17

21
14

24
10

z'- 0

A lightly slretclled string with tixed end points x = 0, x = I is initially al rest in equilibrium
posilion.lC it is set vibtatin~ by giving ""cb poinl x of it ~ velocity kx{/ x), obt:tiu t time. l
tho displ>octnent y nl a dist.nnoco x tr(>tll Ute end x 0.
A plank of 01an, m. L< initially ~I rest along a line of 8.<e:~l'est lope of~ $tnoolh plane inclined
at nn angle rt to the horiT.On. and a mon or mas.< M' , slal'lins from lhe upper end walks down
Ule plank so UtaliLdoc not move. Show Utal llte get to Ute. other end in time
2A/'II
\ (M + AI ')g~in (.<

(b )

II

28
11

15
16

26
13

ra

(i)

.e

7,

ra

()

xa
m

6.

Solve Ulc follo" 'ing ossignmenl prohlem for tho given ""~ignmlllll costs:
P erson

Job

ce
.c
o

P(,II. B) :!:-

(f)

(e)

xa
m

te)

hi
2r
3 1
Also tate tJ1e c.Jnd ition tor which the stream lines are identic.1l with the path lines,
(i )
L.el A and B be the two events such that

u = - .11 = --.,cmd lF-=- - .

.e

(d)

0.20 because <>f <lcti:cts in machinery,


0.50 hec.~use ofcar<:lessne~s of workers and
(iv)
0. 75 beenusc of ~ahottse.
Dcteomine the most likely coUI!c or e:-.'jliMion.
1'wo un.biased coirc< are tos~ed once ( independently) ond the number X ofheod,; thllt hm>ed
up is note(!, A nwnber is selected at random from X. X ~ I aJHI X + 2- trY i the number
~elected, lind the joint distribution of X and Y, Also obroio the e).(pe.::tation of J\.' Y.
(ii)
(iii)

<~nd about

ce
.c
o

(c)

C.S.-J\l nins 200 1

(<I)

9 ~ 11(~ ~ xi: ~ .... x;) -1-'i ' -'~ xJ

_f

- -===========:=!

(15)

2x( ,v- 3)' : 3y(x- J)'


And which touchc.qlt~ xaxis ullbe urigin ~ntl pMscs tlorough tho point ( I. I)

Candidates shoL~d anempl QuesLion Nos. and 5 11 hlch arc compulsory, and any three of the remaining.
ques!ions selecli.ng at least one queslton from each section.

(b)

( .:

(e)

(c)

(d)

(a)

(d)

5.

()

(IS)

Prove that
Sx' r Sy' 8:1 1 8y: , S.:x 2.\:l' 12.t- l 2y 6: 0
( '5)

lfTP. TQ. and T'P'.T'Q' aU lie on conic.

( IS)

SI!CTION B

t\IJempt any five of Ute following:


A continuous func.tion y(t) sn(j,jfie.s the difft>rential equt.ion
l +e,.,
O~ t < l
d)f dt =
.
.
{ 2 + 2t - 3f, [ >, 1< 5

Hence, delennme A!<>


( 15)

When is a square matrili: A said co be congruen t to a square macrix B'? Prove !hat e1ery matrix

(b)

congruent to a skew-symmetric matr:L\ IS skew-symmetric.

( 15)
Delennine an orthogonal matnx P such lhm P-l AP is a dmgoJJal

- - - + - = 1

t )

w
w

(c)

liS)

Reptesenls a cylinder whose cross-section is an ellipse of ecc<."ntrlcity 11../2

4" =A> + A' - 1

tel

z:

c'"

( 12)

Show lhat for every integer n ~ 3

(b)

)..:

li - fl

.~

+ /+ 8X- 2(Jy + 15 = II

(IS)

Find the lt)Cus of equal co1li ug.1te diamc:t~rs of the eWpsQid


-

(12)

( IS)
tl1e cnrdiotd r = a( l - co B) about the

(h)
( 12)

A=r: :: ~ J
()

revoh~ng

Find lht c:<fU3tion or the circ.fe cm:umscribing ""~ triangle t\umc:d by the point
( o.O,U ).( O.b.O ).( O.O.c) Obtain also the coord tll3tes oftl1e centre ofthe circle.

Find the shonesa distance betw~en the axis of z and the Line
a.x + b~ _.. cz + d = 0. a'x + b1y + c'.1. + d' = 0
Jf

Find tlte volume of the solicl generated by

( 15\

(U)

2.

Evaluate

illi(i;llline.

represents a hyperbola. Fmd U1e coordinates ofns centre and ~1e length ofns real seani-a.xes.
(I)

x ~ \),y ~ O. = ~ U,..t: y - :; ,; t

Show U1n1 the equalion


x~ 5xy

b1:i t c1

Over the region dt:tinetl by

( 12)
If). is 11 churJcteri stic rooc of a non-songular mairi.~ A tl!en prove chat IAIO. as a dmracceristic
rool of Adj. A
( 12)
Lei f be defmed on ffi by selling f(;~) ; x, iJ x is rational, and f(x) = 1-~ if xis irracional.
Sho11 U1ai fis continuous a1 x = 112. bul ls continuous at Cl'el) other poinL
( 12)

(d)

J -;

(c)

+ y' +3 ); - tl i

JJf(~ + y - ; ~ I)' d,<dydz

Anemp! 311) fovc of lhe follolllllg:


(a\
Show ~t.~l che ,ector (I , 0, - I ). IV. -3. 2) and (I. 2. I) form a basis for the \ector space R'' (Rl.
(b)

Ry the plane f.y + my 112 = 0

SECTION-A

Find tbc maximum and minimum radii vectors of the section of lite surlllce

(15)

.e
xa
m
ra
ce
.c
o

PAPER-I

Find the equation oflhe cubic cuiVe which h the m~ nsyrnptot<"S '"

3.

ce
.c
o

il'flu:illllllll Marl/~: 3011

Time ; lllawed: 3/t (lllr>

Inn variable is positNcscmidcfinit.c.

ra

MATHEMATICS

xa
m

I.e========

Show lltat the real quadr;~tic from

.e

matn~<.

where

I I' y(O) = -e, lind y(2),


(12)

Solve:
d' r
x' --:;--

ch.'

A:r: : =~J

dr
_,._.
_ _ 3y - X'. log .\
dx
(12)

(o)

- 4 -1 - 8

( IS)

Flnd the l~w of force to Ute pole " 'lten the path of o particle i$ the c.trdioid r = a( I - cos 6)
and prove that ifF be the lbree at the ups<: nnd u the veloQit"y there, then :'It/ - 4oF

( 12)

The middle points of the opposite side. of~ joinood quadrilateral nre c<)nnected by light rod'
of lengths. 1,1' 1fT. T' be tit<> tettsions is these rods, prove tltal

a ,~o

c11rl -

- -t- - = 0
I I'

A soHd r\gbt

ci:r~.ular ctme with sc:miVc.t1iofil n&Jc e1. is just imm.,cJ in " li4uitl whh

g.;n.,..!lng Hnu on the suda~~- u 0 be tllu


on the curved sur13c-o. pro\'c tbnt

en

or the a:re IJ! the

Fiudtltedir<'Ctionaldeivativeof f =x',.-:' >long .r - e' . y ~ l + 2sin l.~ -" l - oosl ol / = 0

(c)

Show thotlhe vector tield defined by


F = 21<.''=J' + ;,.!:.' j ~ 3x1 y:.:k
is irrototionoL Find olso lbe sc:~lor u such the F ~ grt~d u.

of the v~rt.ic..;l wiUt Ute NSult.ont UmtsL

3~in' a) U>n0 . 3smacoa

find the length

3r

: - , - - ,(

(b)

( 15)

(t

in~liMtioo

( 15)

(l2)

(c)

a.r)
r r
\Vhere a is a consl3nt vector,
r3

lwi.<tetl curve r = (31, 31' ' 21' ) from the point l "'

!121
the

( 15)

n to

rll'
d.;

(b)

v( los v )'

1'

- - ~log.
X

J' -

.'

.,

Find the general sohll lun

ur

a,tp' - (2x-b)p - t'-" ll.tl 0


Solve:
( IJ' .

(12)

{15)

(15)

ra

(~ )

ce
.c
o

Svlve:

t)' y = 24xcns .<

ra

(a)

ce
.c
o

point I = I. Find olso the unit tangent t. uniluonnl nand Ute unit binom ial b nt l - J.
1),

(1 5)

A comet descdhing a rrabob lln<ler inven;e S<tnre litw bout the sun. when nearest to it
sudd<:uly breaks up. witlt out gin or loss of kinetic eueoogy. into two equal pootions. ooe of
which describes A oirclc. Prove Utot tit~ other will dc5crib" a hyperbolo ofi:<lccntridty 2.

(b)

A pankle a mass M is at rest and begins to move under tile acHon of a c~nstant force I' In
li~o.>d dircctioo.lf ""~uokrs the r.o~istonce of sl>cnm of fin., dust moving in the opposite
dir..x:tion with n>lodty V. which dcposilll rontter on it ot 3 colliltJJnt oat" p. Show Utot tloe ouas
of the particle will be m when 1t hu tnwelled a distance

.e

(a)

F ~,y.
(15)

OA. OB und OC' are edges of

cub~

of side n. and 00' AA'. BB' ond CC" are its diogonals.
'\long 08'. O'A. BC ond C'A' nc.tlo rces cqult~ P. 2P, 31' and 4P rcspectil<>ly. RcJucc the

system to a for<'<> at 0

togeth~.,.

" 'iUo a couple.


()5)

(d)

A rigbt circulur ~;yl inder Ooating with 1!.< nxis hori'tonlol untl in tht: suo:face is displac-ed on tht."
vt:t'fiCJ t pl:mt: lbrvugh the aKi.i.. Oiscw; iL~ $1llbmty of c-qullihrium ,
(IS)

~-

(o )

JJ

(c)

Whet~ k =

log:;

( 15)

w
w

;{m-M{It

xa
m

( 15)

.e

xa
m

Given Ot~t y = Dy = 0 1r = 0 ntl 0 1 !' = 12 whatx = II.

1.

(tl)

I C.S.E-1\Inin~

2001

''

MATHEMATICS

(n )

===============::::J

~ JI ./.when x= ~JJ ' q


Whore p, q are relatively prime integers.
f(x)~ o

MILcimmu Marli~: 3011

Time ; Wowed; 3 ltou r,<

A fun~-tion fis defined in U1e lntervnl ( ~.b) ns follow~


f( .<) = ll q',when .< = p l q

foratl other valuesof x.

fs f R.ictnann intcgra'bfo'? Justify your 1lnsWer.

(101

the rcl"Uon ::.,.x "'* ,by 1, cz


be + .-a) -,. (o +b- -..c)

I C)

4.

Il l !

(ft)

Evaluate

JJJ (a,,~- b,v' + c:') clxd)~b taken U1rougb out the region .~' + y' + ='

(ii)

( 12)

(b)

( 12)

r"

converges wlifonnly Re z 2 1- c 1vhere s > o

I.

( I 2)

q(=) = ~) = converges for Re z > I and

IS arbiltruy

,,,

smal L

( 12)

Compuce all basic f-easible solutions of the linear programming problem


MttY z = 2x; + 3.r0 +2x,
.rulyec/ tQ 2.r, + 3x1 - x, = 8
.r, - 2., + 6x, + 6x, = -3

(a)

LecN be a llOrmal subgroup of a group G. Show d1at GIN is abelian if and only if for all
.L J' eG..\:vz y l EN
(20)

(b)

lfR is a commulative nng with w1it elemenl and M is an1deal of R. tl1en show thai M is a
ma, imal ideal ofR if and only 1f RIM is a field.

(c)

Prove chat evel}' Cinile exlension of a field is an algebraic e.'tension. Given an example
sho11 that the converge is not lrue,

( 12)

Dete rmine the minimum transpo rtatio n cost.


(15)

w
w

.r, ,:c:,, x, '"' 0,


And hence indicate the optimal solution

2,

(lSI
A manufacturer has ~islributioo o~-nt"'~ at DeU1i. Kolknlll and (1,cnnai. Tiu:"c ""ntOJ'lo
have avnilnbk 30. 50. and 7() tmits of bls producl His four retnil outlds rcquir~; Un>
following number of un it.. : A. 30; 13. 2!~ C. 60; D, 40:
1 he transponaliun oost per unil in n1pees he.tweeu each ceutre nml outlet is giveu in
u.~ follO\<mg l;lbl c:
Distribution
Rclail outlets
Centres
J\
B
c D
7
Delhi
JO
3
6
Kolknta
I
6
7
3
4
Chen113i
7
5
3

(ii}

(I)

X,--~.~ 0

( 12)

'

Prove thai lhe Riemann ~Ia function .; de6ned by

(1 5)

xa
m

.e

(e)

(1 5)

(.Ising duality or otherwise sohc the liner programming prohlem


l\linimize
1Sx, + 1 2.~>
Subjeelln
2-; - 2.r1 ~ - 3
3., 2.~; ~ 3

(i)

- - dt exisls

''..

>. R'.

"J:exp( cosO)cos(sin 0 - nO)dO = t / 1!


Show that J"__!_.,. dx =..;
1 x'
v2

if.and only ifm < n +I


(d )

t-

Pinel l.he Laurent series fm- the function e"' in IJ z '"'' l Ising this exponslon. ~how

(i)

'' sinl!l ~t

'oL
L (l,+U, + ,. .. +I1,
1f L I 0 = I men prme mal I .
' '
n

Land n, b. c are positive! consl3.nt satisfying lltc condition 2(ab

that!

Prove rhac lhc polrnomial I + X+ x' +....+ x' '' . where p is a prime number. is icT~duci ble over
lhe tield of' rational numbers.
Show chat J

three \'llrbbles ru'e connected by

ce
.c
o

(b)

~.e

(25)

(c)

Anempt any fil'e of the following:


LeeK be a foeld and G be a finice subgroup of d1e m\JI Ilplicalll'e group of nont.ero elemencs of
G, Show !hal G is a cyclic group.

ia)

Show tbal Ll ~ -:cy + yz ' zx has a maximum value when

ra

.e
xa
m
ra
ce
.c
o

PAPER - U
SECTION A

I.

Cl)

Candidates shoL~d anempl Question Nos. and 5 11 luch arc compulsory, and any chree of ihc remainin,R
ques!ions selecling at least one queslton from each section.

5.

SECTION - a

Attempt any five oft he following:


(a )
Find thecnmpleteinlegral offhe partial diHe renlial equa tion
'2p-g , 3..- y ~ sx- 9- x- + y
,

_,.

._ (

'

')

(12)

(l(l)
10

(20)

(h)

Find the general integral oftheoquatino


{my(.l' -1- y )Cl= I a< - ~b:( X+ y) -

tc'}

n='} <3= cy (Ll- my ):.


(12)

Show that lhe trun~U~tion error associated with line:ll' interpolation of f(x). using ordinol.es at
and x 1 with "" -s x ~ x, is not lflrger in onagnilodo then 1/SM1(x1 - -to)1 where
M, - max
.~ fn \\; ~ ,, ~ .:c ~ '1

Xu

(3())

(b)

If' ( Jl

(i)

s:

u~nce show lhut if f(x )- }; e-~ dl.


Thc (11JIIC3linn

CIT<>!'

C<)tl'Cl!ponding to ijnear inttll)('Jzotion <>f n'() in 'V "

(ii)

s .(j C.1nnot

2 2nl'

(121

Civcu AJJ'1 A '.B-C


Show lhnt Jl.C '1- A ' .C ~ B .
E.xpre;~ tbe ore of tho triangle h>ving s ide!! of lengtlo! 6.Ji.12.6..fi. units

( i)

(iii

ce
.c
o

numbt1' s~stcm.

r-

,.

Show thnl tl1e velocity distrjbution in axiJJI flow of viscous incompressible j]uid along u pipe
of n.n.nul::~r cross--sectlon. rndii fl < r'!. is given by

dp{ r - r,

11!1 = .Jp d:

(12)

xa
m

_, . y(l}
8; Ce
/II

De~

p( r) ~A ( B' ce-' )', <)II) = B(.-1 +De" )'

rf - r,' loQ ( -' ) ~ .

log( r,

.e
w

( 10)

!-inlve the eq11a!ic>n


...

~~;

...1

...

r fr-;;

(.1~

r.x- . <Y'

a~

\' - ,- ,.

...

('~

)'
fly
hy re(luclng it to the cqu:ltion with '-'OnKt{lnf ~oefficients..
- -x--y- ~~~

(2())

osing Gous. Seidel itcr:oti1 e method ond the starting solution .'<j = ' = 'J = (), clct~nn in" the
solution of the follow in~ system of equ;otion in two iteration.
10.'1 -,\1 - Xl : 0
x, + 10"! +x, : 12

x1 - .t, lOx, = 10
Compare the opprox.im:.lc solution with the exact ulution .

x(x>+ y' +=') =r',i.

(ii)

(30)

Write clown the system of e<tnaloons for obta ining the geneml equat ion of S1trl~ces
ortloogonal to tlte family given by

(i)

.e

Whort A, B, C, 0 und ao-e Mhilr3ry con.sL:mL Ht:t~ct li11d tho vallJcs oi' ohtst nrbitrory
coo~U.nls il'tbe intogrl surtooc. pas$t>Slhrougb the littt z = (), ' = y.

'

(12)

11(t ) =E - (AC rBD) .:-

7.

r,) - r1

_L

Prove thallbr the equation


1. ~ px- ltY - 1- p(Jx)'~ = 0
the clumn:Lcristic strip nrc giv<>11 b)
.t(/} -

(b)

JLheu
. dc1em1 .mc Wa parameter k~ !IUt;Jl lhat il .ts -a posst'blc m otton..
.

Hcnoc rind its' elooi~ polontinl.


(a)

(b)

u1 bitwy

If the velocity distribution of an incompressible Huid at the point (x. y, z) is given by

I -J.v=
3y= lc' _, r'
.- - - , .

6.

Find the equation of motion lbr a particle of mo~s m which is constr~ined to move on the
surface ()I' cone of sem i-vertica l angle and whic.h is s ubjcctt:d to a gravitaJional l'orce.

ra

(I)

(3)

(6 6 : 12)
l)etepnine the mlomenl of inertial of a uniform hemisphere "bout it~ ;oxi$ <lf ymmetry and
about an axis pe~pendku lar 10 tile l\Xis of .Bymmetry and through centre of U\e bas~.

ra

(e)

( IS l

~.

xa
m

(d)

( 15)
Find 1l1e volues of U1e tl\()vlued variables A. B. C lllltl D by solving tile set of
situultaneous cqulioM
A'+A . B = O
A.B = A.C
A. B A. C' ,. C. D = c . D

<Xceed(x,Jix,)' .

Write a computer program in BASIC to evall"Ue the polynomiol


Oor" -+ a,.x,t~~l + l~.xp-..2 + .... + " ... 1.'l+ ft,.
for values of x = 1.2(!l.2)2.0.

ce
.c
o

(o)

(30)

(30)

C.S.E-Mnins 2002

3.

MATHEMATICS

fl - d

Show~uti ~ ;; sm h - sou ' a ~ ~

1- h'

Con.~idc:.-

(.:)

iftbe roots orU1c CCJIIalion


1?. - y)l - ();.~ c;.. - w)1 = 0
in ;, ~rex. y.1.. $how thai

-1.

(3)

I l l)

(b)

j Je ("')J,,Q' = :r /-I

....

(c)

( 11)

(1 2)

ci'(b'-c')"' b' (c'-a' )!l c' (ll' - o')r


.

(d)

~ , - 3~:z + 5x~- 5x.a -2x = 0

- =0
(15)

,\ vMiahle plane p~lll llel to lhe plane

.:!+L

=-=0 meet> the <o-ordia~ate 3Xes of A. Band C. Show Ihat thtl circle ABC lies on tlte

v=(!!..+
!!.J.. =x(=--t~
\ AJ'l~
b)=v
cb
ac l
btl

5.

2x 1 + ' - 2x,+ 3>u - -h s= 17


(l5)

COU1C

corres.pondin~

Sohe tbe following systew of Lmenr eqwtlloos


Xt - 2x:t-3x,+ 4J4 = -l

r~ ; ~]

- "

a b c

(I S)

U~e Cayley-

J'

(c)

(d)

(J2)
b + e)

Hamilton U1eorem to find the i"'erse of the follo111ng IRUlri:-::

SECnON-B

Altempt :my five of the following:

Let A be a real 3 3 symmetnc matrix woth eogen 1alues CJ. U !llld 5. If the
etgen 1 ectors are (2. 0. I). (2. I. J) and (I . o. -2). then Jiod U1e matrix A_

(IS)

Consider rect.-11\gular pariiUelepipcd with cd~ .\, b. .:. Otl!J!.i.n the sbortcsl uisuu,ce ""'" een
one of ils diagonAb ~nd nn wgc wbith do~ not intL'1'8cct lhil diogonat
( IS)
Sb0\9 lh~t the t'eoo orth~ Sl~ nonnal !lnrwn f i'Qm ;ny point (u.. p, y) U) ~.e ellipsoid

---~--~-~~

(15)

(b)

Tangool~ are dt:"Vn Iron\ im)' pMnl (In 11\C clliJISd .\~
~ ~ I h i the Qirc lc .i'- V - r'. Show
t>'
b

tlJ
b1 c lie (In the O<)n~

~-~y-~ z - O nnd x +y+z=a

~.

( 15)
FiJ:,d the centre of .sr;~vity of the region bottndcd by tl1e cu"'Q (~111}:>'3 ~ (yibf"- I ~~nd both
oxes m ibe Hrst qudront. the density bemsn p = kxy. where k ~~ constnnL
tl5)

x" ~y'-~1 - 1

Find the co-ordinates of the ceoue of the sphere inscribed io the tetrahedron fom1ed by the
piMes

Let r II' -+ R' be a linear mapping gil en by T(a, b. c, d. e) = (b d, d + e. b. 2d +


Obtain bases lorn~ omll spnc~ and range SJlnce

2(" - ")(1>- I' )( w- u)


(x J')(J' =)(=- .r)

that the cl10rds of conbct ar;: ~gents to tle ellipse thr.' +b'.l" = r'

for H< a < b < l/

mpresents a hyperbola. Obtain its eccentrici t) and foci.

(n)

vl

il(.r. y. : )
r3(11, .,, ,.)
(d)

(IS)

Show tha1 the equation


9x' - 16y'- l &x - 32y - JSI ~ o

2,

the ~ct of tri;ngles h;ving given hase and a giv<m verte.~ nAie. Shuw that the
triiln~tle ltoving the mo,<imum area will be isoscel~.

ce
.c
o

I-ts'

(f)

f 'IS contmuot$

p ~ucb thal (')


t
:at _x

ra

h-(1

(h)

11\111

( 12)
A squarQ mauix. A is non. singular If and only if the constant h!nu in Its ch(IJ'acteristicpolynomial LS diJferem .('rom ;~ro.
( 12)

Show thm

l.!fln d..
llton o n

.t= U

xa
m

.e
xa
m
ra
ce
.c
o

Attempt an) li1e of the followmg:


(u)
Show that the mapping T' fl' -.. R' 11here T(n, b. c) = (u- b. b- c. D+c) is lin~(IJ' and
singular.

(d)

(15)

PAPER-I
.S ECTION-A

(c)

md (li) r i dill"ert:nliable lil .~ -'= ().

Candidates should ijttempl Question Nos. nnd 5 which are compulsmy. and any three of the remaimng
quesuous selecting ru leas~ one quesuon from each section.

(b)

'
Lt.il r( .\' ) - .r' ~inl /.~, -~- O Oblitin

Maxlm lwt Marks: 3(10

Tlu w l'liltJII'ed : 3 lltlitrS

I,

()

.e

(~)

Solve:.~~ +3y= (!''

thl

Find the value of i. Cdr whieh aU ~vlulion of


:r> -d'y
. +.3 t-dy - -'J' = II
d:r d~
1e11d to zero as JC. ..... ""

( 12)

( 15)

( 12)

A p3rtiek of mnss Jtt is act..d upon by

fd)

di>lnce-a frOID the Qngm, show th;n(he time f~l<en by it tQ reach 1!1eorigiJI is 7t 4,
( 12)
Obllltn U1<: l:lfU>lioo of tho cun>e ln whiub a slrutg hangs moder ~rravity from hi<> Jixed points
(not lying in a venic>aiiJne). whcniJne mas. density ot cnch of it~ poinls varie$ as the rodil.lll
of curvature oft11c cmve.

(c)

( 12)
UaU' uf the ol.litlSe is vortic:tUy immer.cd in wuter 11 ill! minor Mti:l j u.st in the sud'oce. Futd the
position of centre of pressure.

:01

ll

grod f).,; as forf : 2x ' y .-t 2zovers x' +

Hence

r-t r

Find tbc voluo oreons taui l. uch lbllltho foiiOW1111! dim..'runlia l <'tfUaliiiD becomes "''"CL
( U l" + 3l

)"Y' d <+ (3.<' " .k' )=0.


y- 4
- -.t -"--

dx

I c)

-- y-(1

J' )'

LISJng the meiliod of vanatlon of paramcoter~. rtrul Uoe sobli!On or cJ;c' - 2

( 15)

(15)

ra

")'

ra

Sulve-

lit

d.. - >' = :re ~ID -~

xa
m

7,

xa
m

with y(O) - 0 :ond (dy/dxJ.,o = 0..

( 1s)

(d)

Sohc: (D I) (01-20+2)y - r!' whoro D

( U)

.-\ beavyrHi~lc ~f mns> rn , tides una sm011~1~ of C)'ChJill in medium 11host n:sistlln~e
itt m,v2 2C. v being U1Q vducity of tlo~ parl.ick an~ c buing lhc dislllne<> ohbt: ~tarting point
from the ' erte.'l. If the axis ;., V''rlie<l l und vert e.~ upwards, find the velocity uf the pfirtick ut

did:.:.
(IS)

A jmticlc J~ctibcs curve \oith '"""""' velocity and il~ angular velocity ubom gi1en

its tlisl>nce trom ().Show th;u the curve is an equiangpbrspiro[.

w
w
w

(d)

ft.

(15)

(a)

Find the curv3lln'c ldiwdoe !q)>ce curve:


lC : a cQ'< 0. y : o $in II. <~ = o H t:m c~

(b)

Show that c:url/ c,.r/v): grod(d"i'J -'i"V

(c)

(15)
l.et D be a do~ed and hounded region hnving bqnndllry S. l~nrlher, let f be s;::olo, lionction
having second
ord~r
pat tial
ddivalives
dilfifle!;
on
il.
:Sl1ow
lhL

(IS)

( 15)

Five wcigJHIC!!s ruds of eqWll lengths rc jointed togctlter s<J il~ to f<mn 11 rhombllll ABrl)
with dinsoul BD. l fa wcigbl W be attaclted to C nnd Ute ~ystctn be SUfoJiended from a p<.linl
t\. how Otaltbc Uu-ust .6D is oqW11 LO WI .fi.
(15)
.\ solid cylinder noots in " liquid with iil> a:<is verticol. Lct a be the. rolio of lbe opedlic
!!111Vil)' or the cylinder tQ l)ulf ()f lhe liquid, J"nol'lo"th>l lite \:(J\lilibnwn ~~ ~lllnle if the rliU() of
the radiu. of the b~sc 10 the height is sre.1ter th>n ~2<T( 1- a) .

II$

polnt Ovarie mwo>et)

tc)

( I3J

.e

.e

theeu$p.
(b )

= 4.

( 15)

Further. for this value of A, fllve u,c:: eqttnt1on,


(b)

uvnlunle

ce
.c
o

(D )

ce
.c
o

6..

if be the unit V.:ClOJr long tho vetll)f r(r).Shnw Uoal R

Llll

olhcrwillc

Find the v;~luu of constants a~ b 411d c such thot the maximum voluc of dlrcctlonnl deriv:~tivc
off= ax-y1 + byz + ='x' at ( I, -L I) is in the direction parallel toy-axis ond has magnitude 6.

(12)

(I)

or

( 15)
(d)

res1

.
Jf(!'

II([ grrl(/ f ).li dl ~ II J[l grt~d f l' t fV' f ]dF.

fore~ m[ .~+ ~ }towurd$ lb~ origin. If iltnm from

(<>)

(b)

MATHEMATICS

td)

l'lnd aUthe positive values of a for 11lucb the senes

compte~ numb~rs

i (f.-) =x (; ) for n= I. 2.3...._. l1Jen show that flt.l = g(Z) i'or each.1.on

\~)

!tt I S., "

wiU1
llll

(20)

(i)

Show tb~t el'ery ftnlle m1egral dorualu js a fielc\.

(il)

Lel F be a field wuh q elements Lcl .E be a 11m1e extension of degree n Ol'er F.. Show
thm E has q etemen1s.

( lll)

5.

(15)

(IS)

XJ~X22 2.

()5)

z.

y
7

!I
6

'J

711

SO

6()

so

!lSI

8ECnONa
( 12)

(b)

Find the solution of the eqtraLion l: = JJ2 (11' ,.. ,1:) ,.. (p- xl(q - y),

(c)

Find~ rtol rO<>I <lflhc oquntion ltx) = xJ 2x S - 0 b.)' tho meUtod ~ffl.~c f'OIIitiuu.

( '12)
( 12)

Test w1Lfonn coo1ergence of the series


sinnx
where p> O.

(i)

C"mwcrt ( 100.85)1,, into i~~ binul') oquivoloOitl

(ii)

1\<lu ltiply the l>intt1')' oucnbm Cl I L1.0 l): ~nd ( I 10 I. I l)l ond ~heck witlt il4 dccintnl

(4)

I --,""-\

Allempl any fiveoflbefollowing:


(a)
Find h o complete mtegrals of tho partial diff<rrontial equotion ,.,: - y~cf ~. ll

( I ll)

3.

(~-3

5x l I 2X2 ::, 10,

8
1\

( I U)

(c)

:- 1

- (::- 1)"
( : - I) 3{;, ~

2 11)
50
~tl
VJJld the nJmimllm Mstribultvn cos~

( lO)

Prove thal io the nog Fl-~ 1 orpolynon~al 01er a field 1', the ideal I = IP(Jc)lts ma.~ mal if and
only ifU1e polynonual p(x) is irredUclbleol'~r F'

(b)

( ll)

-1

A compnny h:t$ 3 f:.ctories A. U and C whfcb ~upply untts 1.0 warehouses :'\. Y and Z. Every monlh
the capoeities- of the fuctorios per monlli aro 60. 70 n.ud 80 unlts at . \_ B nod C tOilpccli\r~ly. The>
r"'Juiromonis of X. ' nod 7.. per month arc 50. SO ond ~0 respectively. Thc necossory dot.1 in terms of
unit transpot1ation oosts in mpees. facl01:y copacities and warebouse re<joirements are given below;

Pro1e that ngroup of order 42 h;lS a normal subgroup of order7

(ii)

2,

.l.x1 8~2 S: 11

(!'

t l2)

I lsing Simple)( method m:iximi?.e


Z. ~ 5x 1 I 3l!:
SuhjL't!l to

Show thai a group orp~ is abelian, where pis a prime number:

(i)

(12)

Using Simple.'< method


Maximize 45x 1 r SOx~
Sui:lject Ia
Sx1 t 20x1 ~ 400,

(i)

'

( 12)

x,_ ~1~ 0.

(a)

Sbow tllaL w hen 0 ~ IZ - I ~ 2. ihou !unction (\X ) - ..,(--)"'-~) bas tbe Lam'llnl

(ii)

to,,+ 15x,S 4511,

2,

(i)

suriC3 expnnsion in po"orof!zl) 'u;

f (an)' converges.

Stlpp\lse thnJ f Mel g are two analytit functfcms on the s~t (J' of nil

integr:~ls.

115)

4.

Prove that the integral J...-'e '<i~ is c<Jnvergent if and only tf m > 0.

"" Ill

(I)

p k(2o - R)
wharet k is a constant.
find the m:JSs of the hemisphere, by che method of multiple

ra

(c)

.e
xa
m
ra
ce
.c
o

Allempt :Ul)' li1~ of the foliO\I ong:


(a)
Show that a gro!J.P of order 35 is cychc.

112)
{b)
Show that polynomial 2.Sx4 ~ 9x~ ~ 3x+ 3 is irreducible over the field of riuionnl numbers
( ll)

(25)
depending cmlhe ditanc.: R.rrom Ihe centre

xa
m

PAPER - II
SECTION A

('1

ce
.c
o

Candidates should anempt Questi"on Nos. I and 5 which are compulsmy. and any three of the remaimng
quesuous selecting al iens~ one ques11on from e:~ch section.

A ~olid hem1sphertt H of radius hallensfiy


3ml is given by:

M aximum Marks: 3(10

(c)

I,

.e

nnw Alll]ll'ed: 3 lwnn:

()bbln the mnxrin~ nnd minim of


x' + -+?.' - yz - xz - xy
HUbjcctlu the condition
S.: I
I z' 2:l( ' 2'j I 61. I 9 ; 0.

1jUi ~lel\l,

rt

(2U)

(c)

Find the ntOmeJtt of u1ertia 11fo eiteular wir'<> about

(i)

o d1runotcr. itnd

(ii)

a line Utr<tuj~li the ttrltrc and JlL-rp~'ntli<:uwr to it~ plllnt:..

Show thac u

(4)

(f)

Sho\1 Ut,llhc velocity poten tial <r ~ 1'2 b(x'


determine the stTcMn Jines,

(Ul
y~ 2~1 03li$ftcd t he LnplftC/lt: equation. and

- 2.\j>::
( .(' - .r' )=
y
(.~ t l ( ,. (x'. y')""' '"' + /

ora )lOasible liquid motion. Is tbis motion irrlgatiooa I?


Prove tbat; ("V' ~)'iT'v1 " c( vr. Vvr)

are the w locity comJlonenls

(ii)

C!r

(12)

(15)

Oll ,;",,
Solve - = - . b <.x <I .1 0
t">r

(l,,.-

ce
.c
o

Find the ~har3eteristic 1trlp Of the .:tluation


-~1' yq - pq_ 0
ond then lind the equation of the intcgrnl llllrfnce through the cune 7 = x/'2, T =0.

u(!). l) = u(l. t) ~ 0
U(l<. 0) - K{fX). (I 'S ,~ 5o I.

t21J)

(30)

(lil

( Ill)

Giv<m <ly cl~ ~ y-'< where )'<0) =2,. usin~ the Runge-Kutlll r11urtlt nrder metlmd, fmd
y(O, tlaod y(O. 2J;, Cnmpt" the opproximote 50lulioo. witJt il.ll ~ttsoJutJiJn.

=1.2214),

xa
m

(c01 ~ Ll0517.e

(~)

ra

find Ihe cubtepolynQmi:rl which l:l.ke. U1e followmg vulues:


\'(0) = t. y(l) = 0. yt2) = I an~ } t 3) = 10.
Hence. M nllterw/.sc. ubt.rin yc4).

(n)

ra

7.

dltncnsionl motion of a viscous Jluid..

xa
m

(c)

(2ll)

A toacllcr cunaucts lllc:ft: tcs~~ 'fEST I. TEST 2. and PINAL for 50 marl.:.~
ecb. Out of the marks soort!d in Ute two tests_ TEST 1 und TEST 2 he l3ke~ lhc:.
beU.erono nnd 11dds to U1c mntks .cored in ~'L\IAL. so lhot lhc total mad< cored wiU
be for n mnxtmum J{)(),
The Jen.:r grad~ wiU ~e :tS~ignod dep.:nding oo llte marl:.s scored a~ p~r ll1e
( i)

(a)

{2())

Dra11 a now chart to e.~amioe wheUJer a !liven number is a prime.


(10)

A thin mrcubr disc or 01035 M and rnd.lus C30 tum feolv about .1 thiu axis OA, wlllcb i,
perp.,dlcular to IJ plnne and passes through point() orlts d11:umierence. 'nte-o.xi5 OA l~
corupellod lo move in a horizontal pl;~ne with ongular velocity w about its end A.
Show \hat the incllJ1ntion 0 II) lhe \'ot1itul of the tlldJ<t~ of tlte di<e tlrouglt 0 \s ll0> 1tgllm 2)

unless w::...!ifa o.ud llum 0 i~ zct\>.

.e
w

(ii)

40 - 49 : l)
50 - 59 . c
60 - 74 : B
75 - lOO ; A
Pot' o:lloh student data eonsistin~ of name. scores in TEST I. TI'S r 2. and
FINAL llr<> given. Writ<> :1 program in IlASJC "'ltieh will pcint out uu, tlllmllS. totaL
lllilrk scored 3nd gtadc oblllin~d foroll20 stud<mts in a clnn.

.e

followin!! nonn:
o- 39 , e

s.

o(x,y )

Wber<; )" is the kiucmatle ''isCO!;iry of the .Ouid md IV il! the Stream [\m~tion for 111 <r

(b)

fr.unc Urc p>Jttal diffel'cniiaL equolibn by eliminating lhu arbilraJy couslnnlll :1 and b from
log (oz. I ) x + ;>y + b,

ce
.c
o

(3)

C.S.E-Mnins 200J

MATHEMAT=IC=8==------~f

11me ; 11/mvetl: 3 fi.QIIrs

.e
xa
m
ra
ce
.c
o

SECTION A

PAPER - I
Attempt IDt) fo.veoftlte folloMug.
Let S oo any non emply subset of a vecl<.>r space V over the tield F

t (lj

Show Lhnt U1e set tu,a1tt up 1 ..

+a,a,. : o 1 .al'''"''' fl11 ~ P.a,.a: .. "

e-.'-1. 1 e

If . 1

z]

- z :< - 1 then lind a diagonol matrt" D :rnd ;tmatri.x B sucl1that .A=BDB ' 'Vhent
'

- 1 3

s d"'1ote the tranposeofB.


liS)

Maximum Mar ks: 3()0

Candidates should auempl Quesuon Nos L nnd 5 11 hicit nre compulsory. nttd ally tluee of tbe remnillittg.
questions selecting at least uoe question from eucb $ettion

I.

(c)

(d)

3.

(a)

Reduce the quadmtic form give1t belo" to canonical fom1 nnd fmd llil rnnk and s igtmtwe:
~; I ay +- 9:<' I u 12yx I 6zx *'' )' 2xu 6zo.
l 15)
1
A 1-ectangular box. open at the 1011. is lo hnve :1 volume nf 4m u.<inJ! L.1granl!e's methnd uf
multipliors, .lind U1c dintcru.ions tJf ili<> bax so Uml the mattial of a given type n>quired tu
com1rucl It may be least,
( 15)

(b)

Test the convergence of the inlegrall

f'.,!"(1"".... ,

( i)

,r, IS the subspace

generated by S.

:,1, then lind the matri>< represented~

I d)

2At. ION+ l-IN- M - 3A" -<- 15A' 2 1A' -+ <)N + A- I

(cf

(dJ

( 12)

Let f be~ real function deli ned ns follows:


fl,, }=x. -t :S >. < 1
1\x-l)=x. V " 1 R.
Show tlt.nt f is disconunuous at every odd integer.

()

( h)

( 12)

For all real numbers x.l'lx) 1s g11en as:


e'+osin.L
.V< II
f (x ) =

{~> ( x- t r +.r-2, .r~ n

(c)

Find q!lue a nnd b for which r is diJrerentinblc ol ax= \1.


" < ()
e .. osln :r,

(d)

r( -~J :{b(x- 1)--;-;r- 2

x:<!tl

Filld 1alues of a und b for which f din'erentmbleat x=t)

(I')

(at

( 12)

Fmd the equallon of the two straJgbt lines through the jl()tnl (I. J. l{ Jhat mlersect the line x-4
-lly -4,) = 2lz-l) al an angle or 60"
ll2)
Pr01'C that lhe eigen 1ectors corresponding to distincL e~gen values or a square matnx are
linear!) independent,

(b)

( L5)

x)~(<r-/ )

!15)
A .'lphcre of oonst:mt radius r pscs through Ute origin 0 nntl cut< llto 1:0-llrilinalc 4l\~$ at A,
ll nnd C. Find the locll~ of the fool of ll1e- porpondicular from 0 tu the plane ABC.
( I 5)
l'ind tl1e equ~Lilln!i 01' lhe lin~ Of inlll!St<:Lion Oi'thc plane X-+ 7y- 57 ond lht C()IIC3~y- 14Z.'(- 30xy = o.
(15)

Find the equation of the lineof\horiesl di.~ance hetween the lin~:


)' - 1, = I, X-() lind :< - z= I, y = 0 ~ the inleiSt:eliOn of IW() planes.
(IS)

ai!CTION B

Ahem pi nY five out of the follvwing;


()
Shnil lhal lhc Clrlhcrg<mn l lraj~IC>ry ul' n sy~lc:m ot' lirl!nl clllpR<>~ i,qclf orlhll!!onal,
(~ )

(e)

(12)

rA

$vlve: ~-;:- _ylogy : >;w


tu

(IS)

"sphere vtweiS)Il \V and rudiu.~" u~-. w]thln. lixcdsphc:rital shell ot...dius b. A partjele oJ'
weight 1\ .is fixed 10 tbe "l' l"'t end of Ute vot1i.:<.U dillmcter. Ptove IIIAL I!Je <>quilibrium t~
. W b - la
st31leaf-~
l
-lf

(!2)

If H Is a Hermttian matnx. then show that A=(H+ttr 1 H-il) ts a Ullitary matrix. Also sho"
thnt Cl'err rrnit!lly matrix c.an be. expressed in this form. provided I is not an eigenvalue of A

. ~ru

( 15)
Find U1e volume goncroled by reoolvin& H1e a rco, bounded by t.ho curves (x~ .,. 4a')y ~ g,J, 2y
= x. :tnd x = U. obnut the y-axi.
(15)
Find tho volume of tit" l<;trahcdronformtd by tltc fouT plllltes k 1 roy IlL " p, lx + my 0.
my I 112. " 0, Md 112. I lX = 0

ordinate axes at A. B and C. Find the locus of the centre pf the sphere passing through the
origin and the point A, B 3lld C

S.

.1\ l'ariable plane remains at a conslllnt distnnce unlt) from the poi11l {I, 0. Ol nnd C\JIS the co-

(e)

( 12)

(15)

-~J.:..'::;.~r~~;;;,=;co

ra

21

f'f:

xa
m

l I

Evaluate the 1111"!\llll

.e

tf 1=[;, :

( 12)

(b)

ce
.c
o

(d)

"

(12)

particlc desoribco the curve

~ o(l < Q()S h 0) ( cosll ~~ 2)


unller a fon:e F lo tl1e pole. Shc1w thnt lle low crt' foroe i F ~ l1r1

( 12)

C.S.ll-M nin,2003
(c:)

(f)

MATHEMATICS
----~=====

=====--

17me Allowed: 3 l1m1rs

(15)

Cand1da1es should altempl Quesllon Nos. and S ,,~l]clJ are compulsory. and any lh ree of the remmmn&
questionsselectmg al Jeasl one quesuon fmm each secuon.

(a)

(b)

Solle the difl'erential equ"tion (p;<1 - y1) (JI" -')' ) = (p 't I)'. where p = dv/dx. by r~ducing it
Lu Clairuut's form using ui tablo: gubstituliou .

(c)

Snl\'c: ( I - -c1 )y'1

l11c diffc:rct>li:tl equation

~Y b y' - 6) ~ x scc1 g

hy vMiution nf par.unelcmo.

(c)

If ~ particle

Answer any five of !he following:


lf H is~ subgroup of a group G sJ 'eH for e 1CI) xe G, then pro1e lh$1 H is a normal
subgroup of G.
(J :!)
{b)
Show lhal the ring Z[ij = (a+b1)!aeZ,be'Z,r= ._r-j) of Gaussian integers IS a Euclidenn
domnin.
(~)

(c)

(15)

slides down n smooth cycloid. s wrting 11-om a point whO!Ic anmnl di.stanoe fro m
ihe venex is b, prove that Its speed at any lime t os 2-<b! l s in(2l\T1'}. where I' is the time of
curuplete oscillation uf tle porticle.

ra

(b)

I,

( IS)
An elastic string of oan1ral length a + b. where a b, 3nd modulus of ela~lioity )., hus a
p:u>ticlc uf moss on all4ched to it al a d i!itanco a ftum un~> end wbk h is f't"<ed to " poinl A of a
sn1ooU1 ho rizonl!>l plana. T ho o ther end of the string 15 IL.cd to a point B Ill thoi lhe string is
JU$t unrtretchad. rrlhe p~rticle b<> held a and then released. ti nd the periodic lim0 nnd the
dlsbnce in wht~h the particle wall oscillate to ~nd fro.

Id)

(e)

(J5)

(1)

J'

uu, noot.

(d)

r\n elliM

~ ~-: I

.e

(i)
(ii)
(iii)

Lc:1 llte position vector nf : prticle mol ing em a plane curve be r(t), where t is the time. J'incl

(o)

( l SI
l'ind the radii ofcurvoturc tmd torsiOn at pOint of inter$edion of the ~uo1'nc~ ~'-y= .:', y = x
tan ht!Jc).

0 5)

all basic solutions


all bas1c feasiblesolmions
a feasible solutions whicl11s 1101 a baste feasi ble solutions
Let R b~ lite ring of all rcal-1alued continuous functions on lhe closed interval 10. I I
Lei M=V(x)e Rf/( 1/3)=1)1 Show thut M 1s a muxtmal1deal of R_

<l=

(i)

~~~

a~

(a)

(ii)

(15)

( 12)

Determine all the bilinear rr~nsfom1atioos which transform the unll circle 1: 1,-1 into the uml
ctrcle I'" IS I
( 12)
For the lbllo11ing system of equations

O~tennine'

( 15)

Prove that Jd""tity VA ; 2{A.VJ,I ~ 1A (v I) where V l ~ }~ [-i!...

( 12)

If a continuous function of:- sa11slies the funchonnl equalion ITx + y) = f(x) + I()) lhen show
lli!U /'(xtu..- where 11 is ~ conslnnt

2xr- x, 3x,= 4

thQconJponcnl.$ of tl.$ uccxlerutlo11 olntg the raclil and r~nsvorse direction, ,

(b)

x,+cs +~ = 3

os itl:nrierHcd vc:rtk olly in 11 fluid with llli semi-osis- of length

lumzonlaL If iLi o<mtl't> he >t n d"ffth h. lind the depth uf the centre of pn:ssurc.

(o)

( 12)

Lei n be a posrlil'll real number and (x,1) n sequence of rational numbers such rhat lim >, = U

Show !hill hm n' = I

( 15)
.\ ladder on a horizontal floor le:uiJ ogoin.sto vertical \vnU. 1'he ooofficicnts of Friction of lhe
flcw r ;mel tho woll with the lachler ilfi>JI nne! respc:.:tivel~ 'I f mon, whMe c1 eight is o ticiie.
thot nf the ladder. wont tc1 clil1lh up Ue ladder. lind the minimum so fe aos le ofthe ladder with

xa
m

(o)

PAPER - II
SECTION A

.e
xa
m
ra
ce
.c
o

~olvt

' y : ~in 2[lllJ! ( I - x)l

ce
.c
o

(d)

1 ( 1 < :t)y'

(15)

7.

Ma.\'illilllll Mt~rks: JOfl

(12)

6.

Slww lh>t if.o'. h' and e' ore the r<.-eiptoc~bl to tbe nonooplunilr vccton~ll, h un<l e, then any
vwtor r ny be el\pret~sitd R r : tra')n (r b')h + (n:'jc
l12)
?nn'f! lhalthe dlverg.:nce of vector field is invariant w.r. 10 ~o-ordinate tmnsfomations.

~1a1 Mu

N IS an 1deal of R 1f :llld only 1f


(W)

Sho" lh~t Q( ,fi. i) is asplimng field for s.~- 3x +x'-J where Q ts the ueld of rattonal
numbers.

(b)

(li)

( IS)

I I ill

Let M and N be two 1deals of a nng R. Sho"


eiIher ~1s:; N or Nc M

( 15)
.Pro1e Ol!U x'+x+41s irreducible over F !he lield of integers module l l and pro"e

funher that (

,"H ) is a field hnl'ing 121 elementS

.~- +.r+~

( 15)
Cc)

Let R be a uruque factonzation domam (UFD).Ihen pro1e thai Rlxl ts also UFO.
( 10)

x> " y'

+ 1.< = c L. c'/27,

(c.)

(20)

(e)

The axe. 11f twt> equal eylioders intctllect at right angle~ . If he th eir
\'Oiumeenmmnn 10 the cylinden< b) the metl"~l qf m~ltiple integr:tls.
Show Ulllt

J-.X

U~rough U1e pole and J1"1'1l<fldkular to tbe initinllb1e is

then liuu tile

Sin ;r

Cf)

is t11 ergcrtL

rr ~(:

lc

.J.

(e real) shwing tltat the vpper hnlf oftbe

(il

ad(J,

-SlD t}

1., :. O)

ra

nr v

xa
m

11 Ill

..!

{j

JJ

(' I I

1 9 11
g 17 .:!

{II)

1\ hero

C)

I IS)

SI!CTION.

(dj

(II)

(l2)

(12)

wlwJ.t s.~-s zl

/l{x-3/)

1/wn 1ls .<'!>3/

h ts ~ comltnnL

(30)

l"md the rositive rooLOtthe equotmn


2e . ; 1 __l_
.t+ 2 ~+ I
Oslng Newton-R.1pbson mdtbod corre.zL to four decimal pl~~es. Also ;h0\1 tbllt tllil lollowlfli!
!>Chv'lllc for C'n:or of so:cond ordert

(b)

Find tb.. multipU<:J~lion of the following bin:lfy uumben~:


ll.OOL 1. and 101.1
(6)

(301
Draw~ flow chan and write a pro,gumme BASlt' fot .Simpsans 113"1 nllefot lJII\}grntion
I

J.-."[
~tt correct al Ill"',

8.

e" d<: employing lhtee poiols

Go:tu.ian qundrnl11rq follnul>. fwding the rcqi1Ucd woights and rcsidu.,;. lle live dec.lmol
plaollli for computation,
021
(i)
Convert the foUbwlng binary numbor into octal nnd bexa dc-<:imals ystc:m:
101 110010.10010
(li)

(IS)

ilr

Show U>al tlte differenhal e<Juo lion of aU cones whk h hve tltetrvertex ~llhe ongin arepx q)- z. Verif) that Y" zx ~ ><y 0 is surf.lcc satisfying the a hove cquatiotL
Evaluate

""""II" .~ ~ I

/i.3J-1.-)
l

(c)

find lhc &ingulnr <olutillfl of lito equnticm. if it e.x.ists.

.~.. , =~X (I+~1

=x ~ ) ~ cos(2.-. ~.3y)

(a)

Findtbegeuerol solutfou of i}':+J ll'.: +2.:

(b)

( 15)

d<<lJ

( 12)

Find Ihe deOc:etion u( 't.. t) (J[ il vibmting $Iring, :Ill'<I~hod he!IV con fi.x"d .(lllinL (0. II) and (.'/,
0 ), cvrr<:sp<luiliug to zero iniLi l velocity atld tollowi11g inili~l deflection.

1\X) -

p' - q' 2pq - 241) 2.xy ~ 0

!!:..
I

Answer any tiVe of the foiiDl\"ing:


(a)

SlliVe the tquntion

"'

.e

5.

(ij)

ll~ing Clmrpti' metbt~cl Also

( 15)

I \I
6 II
E 8 )II II
13
Indicate olenrly the rule you npply lo arrive nl the complete assignment,

1) -1

SQ Ivc::

(15)

F'ind the ()Jltimal $Olution for the assignment problem with Ute following cost matrix:
1

( i)

(IS)

An animal feed. .:omp.'lny mi1St(lroduce 200 !.. of :r .mixture comlsbns of ingtedi~uts


X, and X. d3ily. X cooll Rs. 3 per k.g and X: cosu ~. S per kj!. No more tltan 80 kg
tlf X1 <;an he tted, nnd at le<ll!t 60 kg of X, mu.t he: a.~ed. Formulntc linear
programming model tif th~ prohlom and us~ Simple!( ~Mthod to dotermin~ Ute
ingredients und x_to b.: ll$.:11 to minimize 1!()$1~.

x,

(iil

j1

(a)

....
n,_ ,.~-., -~ -+,
... ' <'
.,,
....
- , -- ,- = ..nt+~
ar i!y ax 0

.e

fh)

fc.r.

tl \'l>)ocity distdbut.to11 u

tl.t .w : ,,n w ltcrcr dtnotcs II "' tl'I.'IIJtnceuumJ.O.'U~.


'-"

.v=-,
r

line.

: lc
W-plane Cllm>O"ptmds to the intcrioc of Ute .semicircJc lymg to tbe right of imaginary
axil! in the 7.1'lnna.

J;ing lhe ru.d.hod ofcontol1t integrotion II) t>rove thai

pn',

Sbo\\ tl1<11 it is :\ possible motion and dcll,'t'n>ine ll1e u1fncc wltich is odhogonol to st=UI

Disom lhc trmfi>rmotion

ce
.c
o

(i)

11

FOJ' n in~>()Jnl>ressible homogcfiOU.< Uuid at. the point (s. y, z}

' ' ; - -d I '


g1Venvyu
(20)

Ca)

352
lOS

(\2)

(20)

s ':'.,
~

radi11~.

(b)

A $Uiid body of dcnity p i.1 in I!Jo shape of the solid lllm1cd by the rcwlution o.f thu c:mlinitl
n( I tws tl} al>oul the in ilia I Una. Show thai its moment of inertin about the s l:nlighl line

t =

ce
.c
o

Shnw that the ml!Ximum \','J]ue nf x'y'z' S\lbje<:t tn the condition


Interpret the re>ull

ra

(o)

xa
m

3.

(:10)

A line c1reu lar lube, rndnJS c.. ties on o f mootlJ horiZOntal p.lone. and CtJnlaim lwo equal
p:u-lk:lcs connected by an elasuc string in the lube. Ute> nnlurnllenl!Ut of whtcb is equ.1llo half
U1c ci'n;,umfercncc.. 'D1o particles ure in cunlllct and Jiutcnod I0<1llor. I!Ju string being
sl~ched round the tube.
Jf tho l)n.i de bect1me di.un11ec~ provo lh>t the velocity <>f the mbe wht:n th~ SU111J! h(l.
rcgiucd it~ .natura( length ~

L~~~Am~~l'
Whon M, m urc tho mois"' ol' Ute lttbc anJ ead1 p3rtic.le MpcCtiVc(y. anil J. is tho
cltisticit).

moilulu~

uf

(30)

.e

xa
m

Max. Marks: 300

ce
.c
o

INSTRUC TIONS
Each question is printed both in Hindi and in English.
Answers must be written in the medium specified in the
Admission Certificate is.med to you, which must be stated clearly
on the cover of the answer-book in the space provided for the
purpose. No marks will be given for the answers written in a medium other thantltat spec(fied in the Admission Certificate.
Candidates should attempt Questions I and 5 which are
compulsory, and any three of tile remaining questions selecting at
least one question from each Section.
Assume suitable data if considered necessmy and indicate the
same clearly.

ra

ra

ce
.c
o

(15)

Time allowed: 3 hours

.e
xa
m

or

(PAPER -I)

All questions carry equal marks.

115)
An Infia.ito moss of fluid is acted upon by force ur l 'l per unit mnss directed to the
ortgm. lfinitia lly d1e fluid is at rest and there ~ a cavity in the fot'l\1 a sphore r = o
in it~ s how that the cavity wil.l ~ li.lled up aner an mteov l of the ~2/5ft)112 c51~.

C.S.E. (MAIN)
MATH EMATI CS-200 4

SECTION 'A'
Q. 1. Attempt any five of the following:
(a) Let S be space generated by the vectors {(0, 2, 6), (3, 1, 6),
(4,- 2, - 2)}. What is the dim ens ion of the space S ? Find a basis for
S.
12
3
(b) Show that f: IR ---7 IR is a linear transformation, wheref(x,
y, z) = 3x + y- z. What is the dimension of the kernel? Find a basis
for the kernel.
12
(c) Prove that the function/defined on [0, 4] by f(x) = [x], greatest

(ii)

f\\ o SQUTC<lil. eaeh of su-englh mare pl~t-ed at the p<Jints (-a, 0) and (a. 0) 1111d n ink ot'
su-en{IUt 2m i;; placed t Ute ori{lin. Sho1\ that Ute slream lin<:s are Ute curvell (xl + y1i
'" (x'-y'+->.'ty) wbcre '1. j$ 4 nt-inhle parAmeter.
1hw'
Also ~how ll111t tltc Ouid speed al -nny point ~ - - . wltcre r,. r, and tJ arc
'l r!'1
respe.;tively the dis_tances of U1e point !rom the sources artd sink.

(i)

(b )

(c) Fin d the characteristic polynomia

(d) Shaw that : x - - < log 1 + x) < x - - - - , x > 0.


2(1 +x)
2

12

ra

ce

.c
o

(e) Prove that the locus of the foot of the perpend icular drawn
2
3
2
from the vertex on a tangent to the parabola y = 4 ax is (x + a) y + x
12
= 0.
2
2
(f) Find the equations of the tangent planes to the sphere x + y
+ z 2 - 4x + 2y- 6z + 5 = 0, which are parallel to the plane 2x + y- z
12
=4.
3

which is represen ted by the matrix

Find a basis for its image.

1 3

0
-2

1 is one-to-one.

-1

A.

(A - x) 3

+( A- y) 3

be u1 v, w. Pro ve tha t

8(u ,v,w ) =- (y -z) (z- x) (x -y ).


2
8(x ,y, z)
(u - v)( v- w) (w - u)

15

x + 3z = 5

x2

+y 2

dx=

na b

[4+ (a- +b )(a -2 +b-?


-)] ,
4
2

wh en the integral is taken rou nd the


elli pse x? + Y = 1 and p is the
a- b 2
length of three per pen dic ula r from
the cen tre to the tangent.
15
(d) Ifth e fun cti onf is def ine d by

-2x + 5y-z =o

xy .
15

15

(b) Pro ve tha t an equ atio n oft he for


m x 11 = o:, where ne! N and a
> 0 is a real number, has a pos itiv e
root.
I5
(c) Pro vet hat :

(b) Verify whether the fo !lowing system of equatio ns is


consistent :

-x+ 4y+ z=4.

+( A- z) 3 = 0

.e
xa
m

Q. 2. (a) Show that the linear transformation from IR to IR

2
X

He nce find A- 1 and A6 .


15
(d) Define a positive defmite quadratic
form. Re duc e the quadratic
form x~ + xj + 2x x + 2x x to can
oni cal form. Is this qua dra tic
1 2
2 3
form pos itiv e definite ?
15
Q. 3. (a) Let the roots of the equ atio
n in

.c
o

12

~J

ce

l of the ma trix A. ( _\-

ra

Jf(x)dx = 6.

.e
xa
m

[0, 4] is integrable on [0, 4] and that

integer< x, x

f(x ,y) =

x-? + y 2 '

'

(x, y)

* (0,0)

(x, y) = (0,0)

ch mee~s the line s y == mx,


(b) Pro ve tha t the locu s of a line whi
2
2
2
2
2
is c2 m 2 (cy - mzx ) + c (yz z == c and the circ le x + y == a , z == 0
15
2 2
2 2
cmx ) 2 == a2 m (z - c ) .
of pai rs of tang ent plan es
(c) Pro ve that the lines of inte rsec tion
2
g per pen dic ula r gen erat ors lie
2
. to ax 2 + by + cz == 0 whi ch tou ch alon
2
2
15
2 (b + c)x2 + b2 (c+ a)y + c (a+ b)z == 0.
a
e
con
on the

(f) Sh ow tha t if A and B are


irro tati ona l, the n A
sol eno ida l.
Q. 6. (a) So lve : (04 - 40 2 - 5) y =
e" (x +c os x).

l .

dx

12

12
2
- x 2 y) dy == 0.
(b) Sol ve : y (xy + 2x );1) dx + x (xy
!era tion des crib es dist anc es
(c) A poi nt mo vin g wit h uni form acc e
rva ls of tim e t 1 and t2 sec ond s.
s 1 and s 2 met res in suc ces sive inte
12
sl' s 2, t 1 and t'l'
Exp ress the acc eler atio n in term s of
gravity. Its cto ss-s ecti on at
(d) A non uni form stri ng han gs und er
the tens ion at tha t poi nt. Pro ve
any poi nt is inv erse ly pro por tion al to

.c
o

ce

(l- x

d2

d
dx

4x i-( J+ x 2 )y==x.

15

Q. 7. (a) Pro ve that the vel oci ty


req uir ed to pro jec t a par ticl e
from a hei ght h tc fall at a hor izo
nta l dis tan ce a from a poi nt of

dy

ng diff ere ntia l equ atio n


(a) Fin d the sol utio n of the foll owi

-+ yco sx = -sm 2x .

(d) Sol ve the following differential


equ ati on:

dy-

SECTION 'B'
ng :
Q. 5. Att em pt any five of the foll owi

15

dx

2
pro jec tion , is at least equ al to ) g[ 2
a +h

clz2 .

12

ra

.e
xa
m

tha t the per pen dicu lars to them


thro ugh the poi nt (a, ~' y). pro ve
2 2
2
(ax + ~y + yz) == a x + b2 y1 +
thro ugh the orig in gen erat e the con e
15

15

(b) A car of mass 750 kg is run nin


g up a hill of l in 30 at a ste ady
spe ed of 36 km/hr; the friction is
equ al to the we igh t of 40 kg. Fin
d
the wo rk don e in 1 second.
15
(c) A uni fon n bar AB weights 12
N and res ts with one part, ,AC
of len gth 8 m, on a hor izo nta l
tab le and the rem ain ing par t CB
pro jec tin g ove r the edge of the
table. If the bar is on the poi nt
of
ove rba lan cin g when a weight of 5
N is pla ced on it at a poi nt 2m from

ra

12
- .
X B IS

dy
(b) Re duc e the equ atio n (px - y) (py
+ x) = 2p, wh ere p = dx to
Cla ira ut's equ atio n and hen ce sol
ve it.
15
d2
(c) So lve : (x+ 2) {. (2 x+ 5)dy
-+ 2y =( x+ l)e x.
15
dy

.e
xa
m

.c
o

ce

y- z+ -, == 1
+
,
id
pso
elli
the
to
wn
dra
(d) Tan gen t pla nes are
a- b2 c
X

tha t the cur ve in which the stri ng han


gs is an arc of a par abo la with its
axis ver tica l.
12
(e) A circ ula r are a of rad ius a is imm
ers ed wit h its pla ne vertical,
and its cen tre at a dep th c. Fin d the
pos itio n of its cen tre of pressure.

tial der ivat ives at (0, 0) but it is


the n sho w tha tfpo sse sse s bot h the par
15
not con tinu ous ther eat.
poi nts of the cho rds of the
Q. 4. (a) Fin d the locu s of the mid dle
2
2 - y 2 == a2 whi ch tou ch the par abo lay == 4ax .
x
ola
erb
hyp
r
rect ang ula
15

fin d the length of AB.


B,
m
fro
ng
hu
is
N
7
of
ht
A an d a we ig

C.S. E. (MA IN)

15
x
lume, floats with its ve rte
vo
d
an
t
igh
we
en
giv
of
,
(d ) A cone
the
the cone in co nta ct with
of
ce
rfa
su
the
t
tha
e
ov
downwards. Pr

Time allow ed: 3 hours

wh ere

ffi

ce

= 1T + kB .

15

ra

dB-_ -d N _x_
d - = ffi x B,
= ffi x T ,- = ffi N an m

.e
xa
m

(b ) Pr ov e the identity

A )+ A
v(A- B)= (B V)A +( A V )B + B x( V x

(V X B) l5

(c) De riv e the identity

~)dV = Jf
s

15

,..

? "'

= (2x - y )i - yz - j -

')

,...

y~ zk ,

surface of the sphere


where S is the up pe r ha lf
.is its boundary.

SECT ION 'A'

Q. 1. Answe r any jive of the following :


(a) Ifp is a prime numbe r ofthe form 4n + I, n being a natural
number, then show that congr uence x 2 =- I mod pis solvable. 12

ce S.
un de d by the clo se d surfa
bo
e
lum
vo
the
is
V
ere
wh
for
(d ) VerifY St ok es ' theorem
-

(b) Let G be a group such that of all a, b,

x2

y2

All questions carry equal marks .

($ \7 \lf - \jfV<j>).ndS,

Each questi on is printe d both in Hindi and in English.


Answ ers must be writte n in the mediu m specif ied in the
Admis sion Certificate issued to you, which must be stated clearly
on the cover of the answe r-boo k in the space provid ed for the
purpose. No marks 1<:ill be given for the answers written in a medium other than that specif ied in the Admis sion Certificate.
Candi dates shoul d attem pt Quest ions 1 and 5 which are
compulsory, and any three of the remai ning questions selecting at
least one question from each Section.
Assum e suitable data if consid ered necessary and indicate the
same clearly.

\lf - \lf\7

JfJ ($\7

INST RUCT IONS

dT

.c
o

en in
rret fo nn ul ae ca n be writt
Se
eten
Fr
the
t
tha
ow
Sh
Q. 8. (a)
the form

Max. Mark s: 300

.c
o

15

ce

~).

ra

(PAP ER-I I)

.e
xa
m

1
l angle is 2 tan ca
rti
ve
its
en
wh
st
lea
is
liquid

MA THE MA TIC S-20 04

+ z2 =

l and C
15

ODD

(i) ab = ba (ii)(O (a), 0 (b))= I then show that 0 (a b)= 0 (a) 0


(b).
(
12
(c) Show that the functi onf(x ) define d as:

is a hom omo rphi sm from I+ (Gro up of all integers with


addi tion) onto
E unde r mult iplic ation .
I0

<x <- ,n= =O , 1, 2, .....


f(x )= =- ,
1
2"
2". 2"+
f(O ) == 0

ra

12

0.

(e) Fin d the ima ge oft he line y

4
z
=
w
ng
2+
x und er the ma ppi
1

ses
nts wh ere this tra nsf orm atio n cea
and dra w the sam e. Fin d the poi
12
to be con for ma l.
linear pro gra mm ing pro ble m:
(f) Us e Sim ple x me tho d to sol ve the
z = 3x 1 + 2x 2,
Ma x.
sub jec t to x 1 + x 2 < 4
x -x 2 < 2

(b) Find the volu me boun ded by the para bolo id x 2 + y 1


= az, the
cylin der x 2 + y 2 = 2 0v and the plan e z = 0.
20
(c) Let
J (x) for ever y X in [a, b] and. fand g are both
boun ded and Riem ann integ rable on [a, b]. At a poin t
c E [a, b], lct.f
and g be cont inuo us and. f(c) > g (c) then prov e that

rr.< .:_

(I

Jf(x) dY > Jg(x) dr and hence show that

xl' x 2 > 0.

.e
xa
m

.e
xa
m

ra

x wh en x is irra tion al
.
.
.f( x) == {
-x wh en x Is ratiOnal
is con tin uou s onl y at x

.c
o

ce

R by :

15

(d) Defi ne irreducible elem ent and prim e elem ent in an


integral
domain D with units. Prove that ever y prime elem ent in D
is irreducible
and conv erse of this is not (in gene ral) true.
15
Q. 3. (a) If (x. y, z) be the lengths of perp endi cula rs draw
n from
any inter ior poin t P of a trian gle ABC on the sides BC,
CA and AB
respectively, then find the mini mum value ofx2 + y + 1
z , the sides of
the trian gle ABC bein g a, b, c.
20

011

(c) The resid ue class ring (m) is a field iffm is a prim


e integer.

12

ce

ine d
(d) Sho w tha t the fun cti onf (x) def

.c
o

of
has an inf init e num ber of poi nts
is inte gra ble in (0, 1], alth oug h it
dis con tinu ity. Sho w tha t

(b) Prov e that if the canc ellat ion law hold s for a ring
R then
a (:;t: 0) E R is not a zero divis or and conversely.

~.

12

4
fou r roo ts ofx - 1 = 0 fon ns
the
of
E
set
the
t
tha
ifY
Ver
(a)
2.
Q.
= i"
ve tha t a tra nsf orm atio n T, T (n)
a mu ltip lica tive gro up. Als o pro

I
b

1
--<
2

1
x' cos5x dx <2
2
2+x

their intrinsic difficulty . The estimate of time (in hours), each


technician would take to perform a task is given below. How should

Q. 4. (a) If all zeroe s of a polyn omial P (z) lie in a half plane then

the tasks be allotted, one to a technician , so as to minimize the total


work hours?
15

ra

10

14

00

10

16

14

00

14

D 24

12

00

10

E 2

16

00

B 12

26

17

11

13

38

28

26

19

18

15

19

26

24

10

.e
xa
m

00

IV

SECTION 'B'

Q. 5. Attempt any jive of the following :


(a) Find the integral surface of the following partial differenti al
equation :
)

III

x (y- + z) p- y (x-? + z) q

II

.,

(x-- y-) z

12

(b) Find the complete integral of the partial differenti al equation


(p 2 + ci) x = pz and deduce the solution which passes through the
curve X= 0, = 4y.
12

.e
xa
m
c

.c
o

15

(c) A travel ling salesm an has to visit 5 cities. He wishe s to start


g
fiom a partic ular city, visit each city once and them return to his stat1in
point. Cost of going from one city to anoth er is given below :

~
A

ce

.
de 0
cos- 38
<p< l
'
7
1-2p cos2 8+p-

ce

2n

.c
o

(b) Using conto ur integr ation evalu ate

.
show that zeroe s of the deriva tive P' (z) also lie in the same half plane
15

ra

20

You are requir ed to find the least cost route.


(d) A depar tmen t has 4 techn ician s and 4 tasks are to be
in
perfo rmed. The techn icians differ in effici ency and tasks differ

(c) The velocity of a particle at distance S from a point on its


path is given by the following table :

S (meters)

V(m/sec)

47

10

58

20

64

30

65

40

61

50

52

60

38

con stan t, in ma gni tud e and dire ctio


n. Do the vel oci ty com pon ent s
sati sfy the Lap lac e equ atio n ? Justify
.
12
Q. 6. (a) Sol ve the par tial diff ere ntia l equ
atio n

ciz

w
w

.c
o

(e) A particle of mass m moves under the influence of gravity on


x2 +y 2 =

az which is

assumed frictionless. Obtain the equation of motion. Show that it will


describe a horizontal circle in the plane.:-= h, provided that it is given
CJ)

= .j2g /a.

15

ce

(ii) In a 4-bit representation, what is the value of Ill! in signed


integer form, unsigned integer form, signed I 's complement form and
signed 2's complement form?
6

an angular velocity whose magnitude is

= y-l )e X .

12

(f) In an incompressible fluid, the vorticity at every point is

ra

.e
xa
m

12

(d) (i) If (AB, CD) 16 = (x) 2 = (v) 8 = (.:-) 10 then find x, y and z. 6

the inner surface of the paraboloid of revolution

oy

.e
xa
m

3 rule. Compare the result with Simpson's 's rule.

4x -y + 8z= 26
5x + 2y -z= 6

x - 1Oy + 2z =

13

upt o thre e iter atio ns.

02Z

(b) A uni for m stri ng of len gth !, hel


d tigh tly bet wee n x = 0 and x
= 1 wit h no init ial dis pla cem
ent , is stru ck at x = a, 0 < a < !,
with
vel oci ty v . Fin d the dis pla cem ent
of the stri ng at any tim et> 0. 30
0
(c) Usi ng Ch arp it's me tho d, find
the com ple te solu tion of the
partial diff ere ntia l equ atio n p 2x + c/y
= z.
15
Q. 7. (a) Ho w ma ny pos itiv e and neg
ativ e roo ts of the equ atio n
e'- 5 sin x = 0 exi st ? Find the sma
lles t pos itiv e roo t cor rec t to 3
dec ima ls, usi ng Ne wto n-R aph son
me tho d.
15
(b) Usi ng Gau ss-S ied el iter ativ e me
tho d, find the sol utio n of the
foll ow ing sys tem :

(c) In a cer tain exa m, can did ate s hav

ra

ce

.c
o

axay

Estimate the time taken to travel the first 60 meters usin2


Simpson's

15

e to tak e 2 pap ers und er par t


A and 2 pap ers und er par t B. A can
did ate has to obt ain min imu m of
40% in eac h pap er und er par t A,
with an ave rag e of 50% , tog eth er
with a min imu m of3 5% in eac h pap
er und er par t B, wit h an ave rag e
of4 0% . For a com ple te PASS, an ove
rall min imu m of 50% is required.
Write a BA SIC pro gra m to dec lare
the res ult of I 00 can did ate s. 15

(d) Write a BASIC program for solving the differential equation

MA TH EM AT IC S- 2005

+ y2 ,y(O) = 0.1

PAPER-I

.e
xa
m

ra

30

DOD

ce
.c
o

(b) The space between two infinitely long coaxial cylinders of


radii a and b (b >a) respectively is filled by a homogeneous fluid, of
density p. The inner cylinder is suddenly moved with velocitY v
perpendicular to this axis, the outer being kept fixed. Show that the
resulting impulsive r-ressure on a length I of inner cylinder is,

ra

30

.e
xa
m

e, <jJ ).

ce

Write these equations in spherical coordinates (r,

Max imum MarkS : 300

INSTRUCfiONS
Each question is print ed both in Hind i and in English.
Answers must be written in the medi um speci fied in the
Admission Certificate issue d to you, which must be state d clear
ly on the
cover of the answer-book in the space prov ided for the
purpose.
No marks will be given for the answers written in a medi
um other
than that speci fied in the Admission Certificate.
Candidates shou ld attempt Questions 1 and 5 which are
compulsory, and any three of the rema ining ques tions selec
ting at
least one question from each Section.
Assu me suita ble data if cons idere d necessary and indic
ate the
same clearly.
The numb er of marks carri ed by each question is indic
ated at
the end of the question.
SEC fiON 'A'
Q. 1. Atte mpt any five ofth e following :
(a) Find the values of k for whic h the vectors

.c
o

Q. 8. (a) Derive the Hamilton equations of motion from the


principle ofleast action and obtain the same for a particle of mass m
moving in a force field of potential V.

Time allowed : 3 hour s

(I, I, I, 1),
(I, 3, -2, k), (2, 2k -2, -k -2, 3k- 1) and (3, k + 2, -3,
2k + 1) are
linearly independent in R 4
12
(b) Let V be the vector space of polynomials in x of degre
e <n
over R. Prove that the set {I, x, x 2, ... , x"} is a basis for V.
Extend this
basis so that it beco mes a ba~is for the set of all polyn
omia ls in x.
12
(c) Show that the function given below is not conti nuou
s at
theo rigin :
12

to gety (x), for 0.2 <x < 5 at an equal interval of0.7, by Runge-Kutta
15
fourth order method.

dy = x2
dx

'

C.S.E. (MAIN)

2 0 -1
5 I 0

. {0 if xy =0
o
r .:
1. l>XY*'
=
y)
f(x,

(d) Let f: R 2 ~ R be defined as

(e) If nonna1s at the points of an ellipse whose eccentric angles


~. y and meet in a point, then show that
are
12
sin(~+ y) +sin (y+ a)+ sin (a+ p) = 0
(t) A square ABCD having each diagonal AC and BD oflength
2a, is folded along the diagonal AC so that the planes DAC and
BAC are at right angle. Find the shortest distance between AB and

a,

.e
xa
m

ra

ce

oc

.c
o

~~

ce

.c
o

f(O, 0)= 0
Prove that fx and fY exist at (0, 0), but f is not differentiable at

8(x, y, z)

B(u, v, w)
(b) Evaluate

Q. 2. (a) LetT be a linear transfonn ation on Rl, whose ma3


trix relative to the standard basis of R is

3 3

Find the matrix ofT relative to the basis.


~ ={(1, 1, 1), (1, 1, 0), (0, 1, 1)}
(b) Find the inverse of the matrix given below using elementary
15
row operations only :

m-1

+x

n-1

(c) Evaluate

(1+ x)m+n

fJ

15

in terms of Beta function.

w
w

1 2

15

zdV, where Vis the volume bounded below

by the cone x2 + i = z2 and above by the sphere x2 + y2 + z2 = I,


lying on the pr,sitive side of the y-axis.
15
(d) Find the x-coordinate of the centre of gravity of the solid
lying inside the cylinder x2 + y2 = 2ax, between the plane z = 0 and
the paraboloid x2 + y2 = az.
Q.- 4. (a) A plane is drawn through the line x + y =I, z = 0 to

1 -1

ra

v(x2 +l)

(c) lfS is a skew-Hermitian matrix, then show that A= (I+ S)


(1- S)-1 is a unitary matrix. Also show that every unitary matrix can
be expressed in the above form provided -l is not an eigenvalue of
A
15
(d) Reduce the quadratic form
6x 12 + 3 x/+ 3 x/-4x 1 ~ -2x2 x3 + 4x3 x1
to the sum of squares. Also find the corresponding linear transformation, index and signature.
15
Q. 3. (a) Ifu = x + y + z, uv = y + z and uvw = z, then find

.e
xa
m

f(x,y)=

.
,(x,y):t:(O ,O)

xy

with the plane x + y + z = 5. Show that two

1
(e) Show that the volume of the tetrahedron ABCD is 6
--+
~
~
(AB x AC). AD. Hence, find the volume of the tetrahedron with

ce

ra

where p

.e
xa
m

dy , by reducing it to Clairaut's form


by using suitable
dx
substitution.
15
(c) Solve the differential equ atio n
(sin x- x cos x) y" - x sin xy' + y sin x
= 0 given tha t y =si n x
is a solution of this equation.
15
(d) Solve the differential equation
x2 v" - 2xy ' + 2y = x loa x x > 0
C>
'
by variation of parameters.
15
Q. 7. (a) A particle is projected alon
g the inner side of a
smooth vertical circle of radius a so that
its velocity at the lowest
point is u. Show that if 2ag < u 2 < Sag
, the particle will leave the
circle before arriving at the highest poi
nt and will describe a pa=

xy dy =~(x -/-x /-1)


dx
(c) A body of mass (m 1 + m 2) moving in a straight line is split
into two parts of masses m 1 and m 2 by an internal explosion which
generates kinetic energy E. If after the explosion, the two parts
12
move in the same line as before, find their relative velocity.
(d) If a numbe r of concurrent forces be represented in magnitude and direction by the sides of a closed polygon, taken in order,
12
then show that these forces are in equilibrium.

1
[(x + 1)4 D3 + 2 (x + IY D2 - (x + 1)2 D
+ (x +I) ] Y = x +
1
(b) Solve the differential equation
(x2 + y2)(1 + p)2 - 2 (x + y)( l + p)( x
+ yp) + (x + yp)2 = 0

(b) Solve:

(f) Prove tha t the curl of a vector fiel


d is independent of the
choice of coordinates.
12
Q. 6. (a) Solve the differential equatio
n:
15

.e
xa
m

ra

ce

.c
o

such planes can be drawn. Find their equations and the angle be15
tween them.
(b) Show that the locus of the centres of spheres of a co-axial
15
system is a straight line.
(c) Obtain the equation of a right circular cyclinder on the circle
through the points (a, 0, 0), (0, b, 0) and (0, 0, c) as the guiding
15
curve.
(d) Reduce the following equation to canonical form and deter15
mine which surface is represented by it :
2x2 - 7y 2 + 2z2 - 10 yz - Szx - 10 xy + 6x + 12y - 6z + 2 = 0
SECTI ON'B'
Q. 5. Attemp t any five of the followi ng:
(a) Find the orthogonal trajectory of a system of co-axial circles
12
x2 + y2 + 2gx + c = 0, where g is the parameter.

12

~)

.c
o

make an angle sin-' (

vertices (2, 2, 2), (2, 0, 0), (0, 2, 0) and


(0, 0, 2).
D

3
2
2(u - 2ga )
g3a2
rab ola who se latus rectum is
27

x =a (3u - u3)
y = 3 au 2

15

z= a (3u + u3)
(d) Evaluate

f.(

15

(x 3 dy dz + x2 y dz dx + x 2 z dx dy)

ce
ra
.e
xa
m

.e
xa
m

ra

ce

.c
o

by Gauss divergence theorem, where Sis the surface


of the cylinder
x2 + y2 = a2 bounded by z = 0 and z = b.
15

.c
o

ng are con stra ined to


(b) Tw o particles con nec ted by a fine stri
l plane. The axis of the
mo ve in a fine cyc loid al tub e in a ver tica
ve that the tens ion in the
cyc loid is vertical wit h vertex upw ard s. Pro
15
.
string is con stan t thro ugh out the mot ion
, of length a eac h, are
(c) Tw o equ al uni fon n rods AB and AC
rica lly ove r two smo oth
freely join ed at A, and are plac ed sym met
ance c apa rt (3c < 2a). A
peg s on the same hor izon tal level at a dist
from the join t A. In the
wei ght equal to that of a rod, is sus pen ded
ion of either rod wit' h the
position of equilibrium, find the incl inat
15
k.
hor izon tal by the principle of virtual wor
bre adth 2b is com (d) A rect ang ular lamina of leng th 2a and
fluid, so that its cen tre is at
plet ely imm erse d in a vertical plan e, in a
a. wit h the hor izon tal. Fin d
a dep th h and the side 2a mak es an angle
15
the pos itio n of the centre of pressure.
ular heli x is
Q. 8. (a) The parametric equ atio n of a circ
a cos ui + a sin uj + cu k.
ter. Find the u11it tang ent
whe re c is a constant and u is a par ame
d from u = 0.
at the poi nt u and the arc length mea sure
vec tor

grad~)+ grad( k.grad ~) = 0

k is the uni t vec tor in the


whe re r is the distance from the orig in and
15
direction OZ.
the spa ce cur ve
(c) Fin d the curvature and the tors ion of

. cur l (k x

15

,
dt
length.
arc
the
is
s
re
whe
,
find
o
Als
ds
(b) Sho w that

u~.

Q. 3. (a) Iff' and g' exis t for eve ry x E


[a, b] and if g' (x)
does not van ish any whe re in (a, b), sho w that
there exists c in (a, b)
such that

g(b )- g( c)

8(x,y ,z)
z
1
1
::l(
= , eva uate uu,v, w) .
+
+
a+f.. b+A. c+A.

15
(ii) If f : G ~ G' is an isomorphism, prove that the
15
order of a E G is equal to the order off (a).
(b) Prove that any polynomial ring F [x] over a field F is a

30

ra

(~_.?xpandf(z)= (z+ l)(z + ) inL aure nt's


seri esw hich
3

is valid for

(i) I <I z I< 3


(ii) I z I > 3 and
(iii) lzi <L
~
(b) Use simplex met hod to solv e the foll owi
ng :
30
Maximize z = 5x + 2x
1
2
sub ject to 6x + x > 6
1
2
4x 1 + 3x 2 > 12
X + 2X > 4
1
2
and
x 1, x 2 > 0.
SECTION'B'
Q. 5. Answer any jive ofth e following :
(a) For mul ate part ial differential equation
for surfaces who se
tang ent plan es form a tetra hed ron of con stan
t volu me with the coord inat epla nes .
12
(b) Find the part icul ar integral of
x (y- z) p + y (z- x) q = z (x- y)

.e
xa
m

JIGi and I K I> JIGi. Prove that H n K i= {e}.

Q. 4.

ra

.e
xa
m

ln (x + y + z); dx dy dz. .

(f) Put the following program in standard form:


Minimize z = 25x 1 + 30 X 2
subject to 4x 1 + 7x 2 > l
8x 1 + 5x2 > 3
6x 1 + 9x 2 > -2
12
and hence obtain an initial feasible solution.
Q. 2. (a) (i) Let Hand K be two subgroups of a finite group

G such that I HI>

fa e-t e- dt is an improper integral which con-

verges for n > 0.

The integral being extended over all positive values of x, y, z


12
such that x + y + z < 1.
(e) Iff (z) = u + i v is an analytic function of the compl ex
variable z and u- v =ex (cosy - sin y), determine f (z) in terms of

(b) Sho w that

JfJ

ce

(d) Evaluate

.c
o

g'(c )

00

(c) If u, v, w are the roots of the equati on m A and

f'(c )

ce

in~g~.

f(c) -f(a )

.c
o

SECf iON'A '


Q. l. Answ er any five of the follow ing:
(a) If M and N are normal subgroups of a group G such that M
n N = {e}, show that every eleme nt of M commutes with every
12
element ofN.
(b) Show that (1 + i) is a prime element in the ring R of Gaussian

with h = 0.2

.c
o

Hence obtain an approximate value of 1t. Justify the use of a par12


ticular quadrature formula.
(d) Find the hexadecimal equivalent of (41819) 10 and decimal
equivalent of(111011.1 0) 2
(e) A rectangular plate swings in a vertical plane about one of
its comers. If its period is one second, find the length of its diago-

ce

.e
xa
m

ax

a<p

u=J..l

,v=J..lay ,w=J..l

(M+ M') g sin a

whe re a is the length of the plank.


30
(b) State the con diti ons und er whi ch Eul
er's equations of motion can be integrated. Sho w that

B<p

az

--8<p+- Iq 2 +V + fdp
-= F( t)

.e
xa
m

a<p

ra

(f) Prove that the necessary and sufficient condition for vortex
lines and stream lines to be at right angles to each other is that

2M 'a

at

30

Q. 7. (a) Find the unique polynomial P (x) of degree 2 or


Jess such that P (1) = 1, P (3) = 27, P (4) = 64. Using the Lagrange
interpolation formula and the Newton's divided difference formula,
30
evaluate P (1.5).

a
a
ax and D' = ay

where D =

whe re the sym bols hav e thei r usual mea


ning .

12 where J..l and <pare functions ofx, y, z and t.


Q. 6. (a) The ends A and B of a rod 20 em long have the
temperature at 30 C and at 80 C until steady state prevails. The
temperature of the ends are changed to 40 C and 60 C respec30
tively. Find the temperature distribution in the rod at timet.
(b) Obtain the general solution of
(D- 3D'- 2f z = 2 e2x sin (y + 3x).

~.

o 1+x

.c
o

and Simpson's rules to numerically integrate

dx

ce

(b) Dra w a flow chart and also write a prog


ram in BASIC to find
one real roo t of the non line ar equ atio n
x = cp (x) by the fixed poi nt
iteration met hod . Illustrate it to find one
real root, correct upto fou r
places of dec ima ls, ofx 3 -2x - 5 = 0.
30
Q. 8. (a) A plan k of mas s M, is initially
at rest alon g a line
of grea test slop e of a smo oth plan e incl
ined at an angle a to the
hori zon , and a man of mass M' star ting
from the upp er end wal ks
dow n the plan k so that it doe s not mov
e. Sho w that he gets to the
othe r end in tim e

ra

12
which represents a surface passing through x = y = z.
(c) Use appropriate quadrature formulae out of the Trapezoidal

30

C.S.E. (MAIN)

MATHEMATICS

2006

.e
xa
m

ra

ce

hence evaluate the inte gral

J: x6~(1-x2) dx

2 = 1 intercepts a
2
(e) A pair of tangents to the conic ax + by
that the locus of thei r
constant distance 2 k on the y-axis. Prove
poin t of intersection is the conic.
2
12
ax2 (ax2 + by2 - 1) = bk (a:x2- 1?
nce betw een the
(:t) She w that the leng th of the shortes~ dista
2
ellipse x 2 sin2 a + y
line z = x tan a, y = 0 and any tang entt o the
12
= a 2, z = 0 is constant.
2
2
Q. 2. (a) If T : IR -7 IR is defined by
T (x, y) = (2x - 3y, x + y)
compute the mat rix ofT relative to the basis
15
$8= {(1, 2), (2, 3)}
rank of the mat rix
(b) Using elementary row operations, find the

SECTION'A'

12

(c) Find a and b so that f' (2) exists, where


1

f (x) == /X/

if

/x/> 2

a+ bx 2 , if JxJ< 2

12

-1

.e

15

1 -2 -3 -2
1
2
0 1

equations
(c) Investigate for wha t values of 'A and J..l the
x+ y+ z=6
x+2 y+3 z=1 0
X + 2y + AZ = J..l
hav e(i) no soiution;
(ii) a unique solution;
(iii) infinitely man y solutions.

[~ ~]

3 -2
0

Q. 1. Atte mpt any five ofth e following:


(a) Let V be the vector space of all 2 x 2 matrices over
the field
F. Prove that V has dime nsio n 4 by exhi bitin g a basi
s for V
12
(b) State Caylay-Hamilton theo rem and usin g it,
find the inverse of

12

om

.c
o

Each question is prin ted both in Hindi and in Engl


ish.
Answ ers mus t be written in the med ium spec ified in
the Admis'sion Certificate issu ed to you, which mus t be state
d clea rly on the
cove r of the answ er-b ook in the spac e prov ided for
the purpose.
No mar ks will be give n for the answers written in
a med ium othe r
than that spec ified in the Adm issio n Certificate.
Candidates shou ld attem pt Questions 1 and 5 whic
h are com pulsory, and any thre e of the remaining question
s sele ctin g at
leas t one question from each Section.
Assu me suita ble data if cons ider ed nece ssm y and
indicate the
same clearly.
The num ber ofmar ks carr ied by each question is indi
cate d at
the end of the question.

and

.c

INSTRUCTIONS

xm (1- xn)P dxi n terms of Gam ma function

ce

Maximum Marks : 300

s:

ra

Time allo wed : 3 hou rs

(d) Exp ress

xa
m

PAPER-I

15

(d) Find the quadratic form q (x, y) corresponding to the symmetric matrix

.e
xa
m

show that
xz

82

z+ 2

8x2

82

z+yz -.,.8 z
0

xy 8x8y

15

chords of a conic - = 1 + e cos


r

e, prove that

a2 p + bz m2 + cz n2 = 3 pz
SECTION'B'
Attempt any five of the following :

15

1t

(a) Find the family of curves whose tangen ts form an angle

4
12

15

xz yz zz
-+-+~=1
~
az bz cz
(a) If PSP' and QSQ' are the two perpendicular focal
1

prove that

Q. 5.

f"'J~~J e -y dy dx
Jo x y
and hence evaluate it.
(d) Find the volume of the uniform ellipsoid

ce

y2 zz
- + - + - =1
a2 b2 c2

xz

(c) Change the crder of integration in

Q. 4.

15
b c
(d) If the plane /x + my + nz = p passes throug h the extremities
of three conjugate semidiameters of the ellipsoid

a~/

ra

(~)

-+-+ -=0

.e
xa
m

+g

15
is constant.
(b) Find the equatio n of the sphere which touches the plane 3x
+ 2y- z + 2 "' 0 at the point (1, -2, 1) and cuts orthogonally the
sphere
15
x 2 + y2 + z 2 - 4 x + 6y + 4 = 0
(c) Show that the plane ax+ by+ cz = 0 cuts the cone xy + yz
+ zx = 0 in perpendicular lines, if

ra

z=xf(~)

15

ce

{b)If

SQ SQ'

<.

SP SP'

with the hyperbolas xy = c, c > 0.


(b) Solve the differential equatio n

x~O

.c
o

. a sin2 x x b log cos x


1lffi
4

Is this quadratic form positive definite ? Justify your answer.


15
Q; 3. (a) Find the values of a and b such that

.c
o

A= (~3 ~)

---+---

xy +e

__1) dx-x ydy=O


x3

12

(c) A particle is free to move on a smooth vertica l circula r wire


of radius a. It is projected horizon tally from the lowest point with
velocity 2

.jia . Show that the reaction between the particle and

the wire is zero after is time

(D 2 - 2D + 2) y = e" tan x, D = dx by the method of

12

force m ( x + ::) towards the origin. If it starts from rest at a

(d) The middle points of opposite sides of a jointed quadrilateral are connected by light rods of lengths /, /'. If T, T' be the
tensions in these rods, prove that

15
variation of parameters.
Q. 7. (a) A particle, whose mass is m, is acted upon by a

15

15

.c
o

15

ce

ra

~tan- 1 (~)

15

+4;}

15

(c) Show that the length of an endless chain, which will hang
over a circular pulley of radius c so as to be in contact with twothird of the circumference of the pulley is

ctog (:+.13)

(d) A uniform rod of length 2a, can turn freely about one end,
which is fixed at a height h (<2 a) above the surface of the liquid. If
the densities of the rod and liquid be p and cr , show that the rod
can rest either in a vertical position or inclined at an angle 8 to the
vertical such that

dy
p=dx
(c) Solve the differential equation

4.

(b) If u and V are the velocity of projection and the terminal


velocity respectively of a particle rising vertically agains t a resistance varying as the square of the velocity, prove that the time
taken by the particle to reach the highest point is

-1
dy
(l+y2)+ (x-e-ta n Y)-= 0
15
dx
(b) Solve the equation
xz pz + yp (2x + y) + yz = 0
using the substitution y = u and xy = v and find its singular solution, where

1t

distance a, show that it will arrive at origin in time

.e
xa
m

.e
xa
m

ra

ce

.c
o

12
I
/'
(e) Find the depth of the centre of pressure of a triangu lar
lamina with a vertex in the surface of the liquid and other two vertices at ~:epths b and c from the surface.
12
(f) Find the values of constants a, b and c so that the directional derivative of the function.
f= axy2 + byz + cz 2 x3
at the point (1, 2, -1) has maxim um magnitude 64 in the direction
parallel to z-axis.
12
Q. 6. (a) Solve :

T T'
-+- =0

Jflog( J5 +J6)

h~

cos 8 = 2 a~~

15

SECT ION'A !
Q. 1. Answer any five of the following:
(a) L~t S be the set of all real numbers except -1. Define * on S

(a)If A=2t+k B=t+]+k, C=4t-3]-7k,de-

termine a vector R satisfying the vector equations.

.c
o

sine d8
k
---=-sin~ d ~
't

15

2A

ce

(d) Verify Stokes' theorem for the function

F =x i-xyj
""' integrated round the square in the plane z = 0 and bounded by the
15
lines x = 0, y = 0, x =a andy= a, a> 0.

.e
xa
m

ra

A.

ce
.c
o

and <jl respectively with a constant unit vector a , prove that

dx

xl/2(1 -x)l/2

.e
xa
m

b make angles e

12

0
(d) Prove that the function f defined by

when xis rational


1,
f(x) = {
-1, when x is irrational

12
is nowhere continuous.
(e) Determine all bilinear transformations which map the half
U
plane Im (z) > 0 into the unit circle I w I< 1.
(f) Given the programme
Maximize u = 5x + 2y
subject to x + 3y < 12
3x- 4y < 9
7x + 8y <20

t and binormal

a* b =a +b + ab
Is (S, *) a group ?
Find the solution of the equation
12
2 * x * 3 = 7 inS.
(b) If G is a group of real numbers under addition and N is the
subgroup of G consisting of integers, prove that GIN is isomorphic
to the group H of all complex numbers of absolute value 1 under
12
multiplication.
(c) Exami ne the convergence of

(c) If the unit tangent vector

by

RxB=CxB and RA =0
15
(b) Prove that rn r is an irrotational vector for any value of n,
15
but is solenoidal only if n + 3 = 0.

ra

Q. 8.

:t:

(c) Evaluate

J
0

cos 28
l-2a cos 8+a 2 dS ,-l< a< l

20

x2 y2 z2
-+ -+ -= 1
a2 b2 c2
(a) Wit h the aid of residues, evaluate
1t

by the Simpson's rule

(x, y) > (0, 0)

Q. 4.

12

(0, 0)

(i) is continuous at (0, 0).


(ii) possesses parti al derivatives
f (0, 0) and f (0, 0).
(c) Find the volume olth e ellipsoid

2 sin (3x + 2y)

ax3

12

.e
xa
m

()

(x, y)

30

f(x, y)=

x3 +2y3
x2+ y2'

15

.c
o

ce

ra

.e
xa
m

s, s

circles I z I = 1 and I z I = 2.
(c) Use the simple x metho d to solve the proble m
Maximize u= 2x + 3y
subject to -2x + 3y < 2
3x+2 y<5
X, y > 0
SECT ION'B '
Q. 5. Answ er any five of the following :
(a) Solve:
3
2
px (z- 2y2) = (z- qy) (z- y - 2x )
(b) Solve:

ce

J2

.c
o

J2 ]

Z[
= {a +
b I a, b E Z}
is a Euclidean domain.
30
Q. 3. (a) A twice differentiable function fis such
that f (a)=
f (b)= 0 and f (c)> 0 for a< c <b. Prove that there
is at least one
value
a < < b for which f" @ < 0.
20
(b) Show that the fJnc tion given by

3
7
(b) Prove that all the roots of z - 5z + 12 = 0 lie betwe en the

ra

y> 0
Write its dual in the standard form.
12
Q. 2. (a) (i) Let 0 (G)= 108. Show that there exist
s a normal
subgroup or order 27 or 9.
(ii) Let G be the set of all those ordered pairs (a, b)
of
real numbers for whic h a :t: 0 and define in G, an oper
ation as
follows:
(a, b) (c, d)= (ac, be+ d)
Exam ine whether G is a group w.r. t. the operation
. If it is a
group, is G abelian ?
10
(b) Show that
X,

15

+ 4f (x 3) + ..... + 2 f (~n- 2) + 4f (~n-l) + f (~n)]

with
12
2n = 10, Ax= 0.1, Xo = 0, x 1 = 0.1, ... , x 10 = 1.0
(i) Given the numbe r 59.625 in decimal system. Write
(d)
6
its binary equivalent.
(ii) Given the number 3898 in decimal system. Write its
6
equivalent in system base 8.
(e) Given points A (0, 0) and B (x0, yo) not in the same vertical,

it is req uire d to find a cur ve in the xy pla ne join ing A to B so tha t


a par ticl e star ting fro m rest will trav
ers e fro m A-t o B alo ng this
curve without fric tion in the shortest pos
sible time. lfy = y (x) is the
req uire d cur ve find the fun ctio n f (x, y,
z) suc h tha t the equ atio n of
mo tion can be wri tten as

Fin d the deflection of the stri ng at any


ins tan t of time.
30
(b) Fin d the sur fac e pas sin g thro ugh
2
the par abo las z = 0, y =
4ax and z = 1, y2 = - 4ax and satisfying
the equ atio n
X

fiz

fJz

+2 -

ax

8x 2
(c) Solve the equ atio n

15

prove that there exists a unique polynomial of degree


< n with given values ck at the point ak, k = 1, 2, ... n.
30
(b) Draw a programme outline and a flow chart and also write a
programme in BASIC to enable solving the following system of 3
linear equatfuns in 3 unknowns xl' ~ and x3 :
C*X=D
with
3
3
.
.. )3 . 1 X= (x)
D = (d)11~
30.
C = (cIJ~J~'
1
1
JJ~'
Q. 8. (a) A particle of mass m is constrained to move on the
surface of a cylinder. The patticle is subjected to a force directed
towards the origin and proportional to the distance of the particle
from the origin. Construct the Hamiltonian and Hamilton's equations of motion.
30
(b) Liquid is contained between two parallel planes, the free
surface is a circular cylinder of radius a whose axis is perpendicular
to the planes. All the liquid within a concentric circular cylinder of
radius b is sudderily annihilated; prove that if P be the pressure at
the outer surface, the initial pressure at any point on the liquid,
distant r from the centre is

ce

ra

.e
xa
m

2 < x < !.

H~nce

x
I
u(x 0) =- O< x< '
I '
2

P(cxk)
Q'(cxk) (x-cxk)

.c
o

ce

.e
xa
m

ra

p {0, 0, 0} =p and g= -g i
0
12
2
Q. 6. (a) The deflection of a vib rati
ng stri ng of len gth I, is
governed by the par tial differential equ
atio n ~ = C 2 uxx. The end s
of the stri ng are fixe d at x = 0 and I. The
init ial velocity is zero. The
init ial dis pla cem ent is giv en by

(I- x),

{:t

12

(f) A steady inv isci d incompressible flow


has a velocity field
u= fx v= -fy w= O
'
wh ere f is a constant. Der ive' an exp res
sio n for the pre ssu re field p
{x, y, z} ifth e pre ssu re

P(x) "
Q(x)-

.c
o

dx
dt = f (x, y (x), y' (x)).

=l

by Charpit's method.
15
Q. 7. (a) IfQ is a polynomial with simple roots al' a 2, ... an
and if P is a polynomial of degree < n, show that

log r-logb
p log a-1ogb

30

.,

(Dp) (x): P1 + 2p2x + 3p3x2


2
3
where p(x) =Po+ p1x + p2x + p3x +
Is D a linear transformation on X? If it is, then construct the matrix representation for D with
respect to the ordered basis {1, x, x2, x 3} for X.

Time Allowed: 3 hours

Maximum Marks: 300


(c)

Candidates should attempt Question Nos. 1 and 5 which are compulsory, and any three of the remaining
questions selecting at least one question from each section.

(20)
Reduce the quadratic form q(x, y, z): = x 2 + 2l-4xz + 4yz + 7z2 to canonical form Is q
positive definite?

(a)

(b)

(b)

LetS be the vector space of all polynomials p(x), with real coefficients, of degree less than or
equal to two considered over the real field IR, such that p(O) = 0 and p(l) = 0. Determine a
basis for S and hence its dimension.

(c)

(d)

A figure bounded by one arch of a cycloid

(x

(12)

4.

(b)

A line with direction ratios 2, 7, -5 is drawn to intersect the lines

(12)

.::_= y-1 = z-2 and

2.

(a)

Find the equation of the sphere inscribed in the tetrahedron whose faces are x = 0, y = 0, z = 0
and 2x + 3y + 6z = 6.
(12)

Let W be the set of all 3 x 3 symmetric matrices over R. Does it form a subs pace of the
vector space of the 3 x 3 matrices over IR? In case it does, construct a basis for this space and
determine its dimension.

Consider the vector space


X: = {p(x) p(x) is a polynomial of degree less than or equal to 3 with real coefficients}, over
the real field R, Define the map D : X ----+ X by

x-ll
3

= y-5
-1

=!._
1

(15)

(c)

Show that the plane 2x- y + 2z = 0 cuts the cone xy + yz + zx = 0 in perpendicular lines.

(15)

(12)

(15)

Find the coordinates of the points of intersection and the length intercepted on it.

(12)

(20)
(b)

(d)

.e

w
w

(f)

Show that e-x xn is bounded on [0, oo) for all positive integral values of n. Using this result
show that

(a)

(-n, n)? If it is continuous, then it is differentiable on ((-n, n)?

Find the focus of the point which moves so that its distance from the plane x + y - z = 1 is
twice its distance from the line x = - y = z.

(15)

(25)
2
2
2
2
2
Show that the spheres x + y + z - x + z - 2 = 0 and 3x + 3y - 8x - 1Oy + 8z + 14 = 0 cut
orthogonally. Find the centre and radius of their common circle.

x =a (t- sin t), y =a (1- cost), t E [0, 2n], and the x-axis is revolved about the x-axis. Find
the volume of the solid of revolution.

(e)

w; a is a constant.

(20)

ra

ra

xa
m

Determine a basic for the Null space ofT. What is the dimension of the Range space ofT?

Let f(x) (x

Prove that if z = \lf(y + ax) + \Jf(Y - ax) then

for any twice differentiable cp and

T(xl, X2, X3) = (2xl + X2 + X3, X1 + X2, X1 + X3, 3xl + X2 + 2x3)


For each (x1, x 2, x 3) E R 3.

(c)

Find a rectangular parallelopiped of greatest volume for a given total surface area S, using
Lagrange's method of multipliers.

(12)

LetT be the linear transformation from R 3 to R 4 defined by

.c
om

Attempt any five of the following:

ce

1.

(a)

ce

Section-A

3.

xa
m

'(

.e

---.

.c
om

(20)

Show that the feet of the normals from the point P(a, ~' y), ~
4z lie on the sphere
2

2fl(x + y +z

)-(a

-::F

0 on the paraboloid x +

+ P)y-2,8(2+y)z = 0

(15)

A particle is performing simple harmonic motion of period T about a centre 0. It passes


through a point P (OP = p) with velocity v in the direction OP. Show that the time which
elapses before it returns toP is
T
1 vT
-tan
-7[
27rp

ce

& show that the beam is unstable in this position.

ra

liquid. Prove that the pressure on the curved surface is

(a)

(b)

SECTION- A
1.

Answer any Five of the following:


If in a group G, a
order of b.

(15)
(b)

Let R

!]

= [:

Find the curvature and torsion at any point of the curve x = a cos 2t, y

a sin 2t, z = 2a sin t.

(c)

(15)

x,y )

G, then find the

(12)

Z. Show that R is a ring under matrix addition and

(12)

(X, y) ;to ( 0, 0)
(X, y) = ( 0, 0)

0,

(12)

Using Lagrange's mean value theorem, show that

(12)

(d)

2a

(c)

that starts from the point (2a, 0, 0) and goes at first below the z-plane.
(15)

x +y

.e

.e

(x- a) + (y - a) + i

b for a, b

Ieos b- cos a I ::::; lb- al.

2
= 2a ,

~ ~]a, bE z}

is not continuous at (0, 0) but its partial derivatives fx and fy exist at (0, 0).

(15)

(15)

x + y

=
{

Determine (ydx + zdy = xdz) by using Stroke's theorem, where C is the curve defined by
2

where a, b, c d,

Show that the function given by

If ; =xi+ y} + zk, find the value(s) ofn in order than r"; may be (i) solenoidal, (ii) irrational.

e, e is the identity element of G aba-

Then show that A is a left ideal of R but not a right ideal of R.

(15)

For any constant vector ~, show that the vector represented by curl (~ x ;) is always parallel

(d)

to the vector ~ , ; being the position vector of a point (x, y, x) measured from the origin.

(c)

multiplication A= {[

w Jl9 where W is the weight of the

xa
m

liquid.

Candidates should attempt Question Nos. 1 and 5 which are compulsory, and any three of the remaining
questions selecting at least one question from each section.

(a)

A cone whose vertical angle is !!__, has its lowest generator horizontal and is filled with a
3

Maximum Marks: 300

.c
om

8.

(15)

A uniform beam of length 1 rests with its ends on two smooth planes which intersect in a
horizontal line. If the inclinations of the planes to the horizontal are a and ~ (~ > a), show
that the inclination 8 of the beam to the horizontal , in one of the equilibrium positions, is
given by
tane =-(cot a- cot /3)

(d)

ce

(c)

'I

Time Allowed: 3 hours

A particle attached to a fixed peg 0 by a string of length 1, is lifted up with the string
horizontal and then let go. Prove that when the string makes an angle 0 with the horizontal,
the resultant acceleration is g ~( 1 + 3 sin 2 e)

.c
om

(b)

i:,\ : ;'

ra

(a)

xa
m

7.

Prove that the function f defined by


2

f( z)= lzl 4
{

0,

z;toO

z=0

is not differentiable at z = 0.

(f)

Put the following in slack form and describe which of the variables are 0 at each of the
vertices of the constraint set and hence determine the vertices algebraically:
Maximize

u= 4x + 3y

SECTION B

subject to
x+y:::=;4

5.

-X+ y::::; 2

Answer any FIVE of the following:


(a)

(i)

Form a partial differential equation by eliminating the function ffrom:

x,y 20.

z = y +2/(;+logy

(12)

(c)

(c)

(d)

(20)
-

9/.

(20)
(e)

Convert :

(ii)

(1111 0.01) 2 into a number in the decimal system

(15)

Solve the following by Simplex method:

(6+6)

Find the equilibrium position and determine if the equilibrium is stable.

(f)

(15)

(12)

Consider a system with two degree of freedom for which

Evaluate (by using residue theorem)

Show that the transformation w = z2 conformal at point z = 1 + i by finding the images of the
lines y = x and x = 1 which interest at z = 1 + i.

(12)

5x - 7 = 0 lying between 2 and 3

46655 given to be in the decimal system into one in base 6,

.e

(20)

(i)

converge to 1.

.e

.c
om

Use the method of false position to find a real root of x 3


and correct to 3 places of decimals.

Rearrange the series

w
w

(b)

ce

ra

Find the volume of the solid in the octant bounded by the paraboloid z = 36- 4x

(6)

into one in polar coordinates and thereby show that the solution of the given equation
represents surfaces of revolution.

(30)

Transform the equation


YZx- XZy =

are relatively prime, but ay and ~y have no g. c. d. in R,

(a)

(b)

Given a positive real number a and any natural number n, prove that there exists one and only
one positive real numbers such that s,n =a.

~( -1 )n+l .-to
1
Ln=l
n

4.

(15)

6.

(a)

(12)

Show that ( :: cos 2 t + (~: sec 2 t = 1 is a possible form for the boundary surface of a liquid.

(c)

2H

xa
m

(b)

Solve
2zx- px2 - 2qyx + pq = 0.

ce

Show that in the ring

The element a = 3 and~ = 1+


where y = 7 (1 + 2H) .

(a)

(ii)

z [H] is an irreducible element, but not prime. Justify your answer.

R={a+bFia,bEZ},

3.

(15)

(b)

1+Hand

(6)

ra

(ii)

Prove that there exists no simple group of order 48.

xa
m

(i)

(a)

.c
om

2.

(12)

Solve

Uxx

+ Uyy = 0 in D

where D {(x, y) : 0 <x < a, 0 <y <b} is a rectangle in a plane with the boundary conditions:
u(x, 0) = 0, u(x, b)= 0, 0::::; x::::; a

Maximize
u=x+y
subject to
-X+ y::::; 1
x+2y:::=;4
x,y 20.

u (0, y) = g(y),

Ux

(a, y) = h(y), 0::::; y::::; b.


(30)

(30)

I C.S.E-Mnins 2008
II ~~~;

MATHEMATICS

.ll.x -.-.:v
by ,;hansing Ihe order of inlegntliun

! 20J
tel

(A)

4.

is invenible, Hence find {ndJ

(b)

Ob1:1iu Lite volume bounded by ll1o ~lliplic. p:orabo loide gil .:n bJ tltu equ:ttious
:,. = x' - 9y' and z ~ 18 :<1 9y~

(c )

Sliow ll10t ll1e ~n.wluping eylindi:t'> of l11e ellipsoid nx= ~ by1


p"'l'<"dJcubr to z <~.~k muct tho pln~ z 0 in parnboiOJi,

(c)

3.

(a)

Alt=pt :my f!VI' ofll>e fotlowing:


Sol\'e lle dill'<m:ntinl equation

ydx (" ..ll Jdy o.

..

( 1,2)
Tbe plane K- 2y+3z =II is rotated through u nght angle aboul tts hne of 11\lersecllon with the
plane ~ ~ 3y -4z.-5 = ()' fmd the equation of the plane tn tis new posttion.
( 12)
Find the equations (in symmetnc form) or tbe t1mgent liue to the sphere >.1 -t ) 1 + i + Sx - 7y
+ 2>.- 8 = 0 . .3x - 2y +4J. + 3 = I) at~u, pmnt (- 3, 5. 4).

~1e

Use th.e method of vnnatum of paramet= to find lbc generl $olu1Jon ol


- 4xy'+ 6y = -11' ~in )C.

(c)

Evaluate th!} double mtegral

(12)

( 12)

w 1s Ihe wctght of lhe pttrtlcle. r' the radius of ClllValme of the tube,

4~'

it.s IRill~ rectum and 'h ' 1lc 111ili RI Ve>1loal beighl of u,e porticlo abuV<: the \>CJ'tes of tl10

lube,

(20)

Let a be a non-smgular mnlrtx.


Sho\1 that If
I t A -t A1 -t .. . .... A" = O.
lhen A 1 = A"

(20)

~;nwotlt p:mtboli~ lUbe i plocc:d with vertex plano. A plllticlc <lidCII down the tube from
~~ under the inllu.:nc.; i>f gravity, Prole th:ol in ~nyposihon. Ihe =~rion cif thu tube is e<1u:l

(d)

cune gtven by the

(20)

(2(1)

with generators

.._:y.

to 2w ( h ; " } Whore

(12)

(2(1)
A snhere S has pomts (0, I. ()j , (3, - 5, 2) at opJ)ostte ends or a diameter, Find the-equahon of
~le sphere hm'iu~ ~le intersection Q( ~le sphereS " i lh the plane 511-2)' + 4z + 7 ; () 35 a T\lat
ctrcle

cz1

(b )

Show that B = {(1. o, 0.) ( 1, I, 0), ( I, I, I)) is a basis of R3 Lei T . RJ ~ R~ be a linerutransformation such thnt Tl 1. II, U) = ( 1. 0. 0). T(l , 1, U) = (I. I , I) IU1d Tf l. 1. I) = (I . I. U)
Fmd T(xs. z).

(e)

(f)

(12)

A trigbt uui lbnn beam ot' loogth ' 2h' ~Is in lim iting cqutlfurium. In conllU:t wilb n rougll
veniCD! woD of height ' b'. wllb one ood on~ rough borlzon!Jl) plnoe Rn<l 11itl1 U1e olher end
projectmg heyond the wnIL lf bolh the wnll and tho plane he equa lly rough. prove lltoJ r , the
angle of lriclion, is given by s in 21. = '>ina. ~ina.. a being the inclin>lion of the beom lo the
hocizur1.
(12)
Jlind !he coml,linl 3 nnd b <o !hot the mf nce .,.~ - by1. ~ ( + 2 )x will be ortltogon>llo the
urfilce 4x' y - zl = ~at the llOinl (l. - 1. 2)
(12)

Show Umt F = (h l '+ z' )i+.i j-.h z1k il a eonservntivo force field. Find the s<:alat
polcntllll tor

(20)

(b)

I.

E1slua(c (r 1'11x)' d.<

Determine Ute maximwu and minimum distances of the on,gio from


equsriou
3x! +-lxy + (ir"~ 14(1

(20)

SECTION.a
(>)

(b)

(201

Find lhe dinlotlSlon oftlt~;~ubspoco: ofR~ s pnnnod b.YIll,. Set


{(1,0.0,0). (0, 1,0.0), ( 1.2.0, i ), (0,0,0.1)) .
H;,nce find a b~siJ Iori he suh~p~ce

( 12)

Itt)

olh~ two,

(b)

Find the value of hm fl( l - .l')cot r.;r

( 12)

.e

If)

( 12)

LetS be 3 non-empty set nod let V denote the set of nil fWJctons from S mto R. ShC)II that V
is a vector space wiU1 [espect to ~1e rector addition (f + gl(x) = f(x) + g(x) snd scalar
11\ultiphcation (d) (x) = cf(x)

-y

!l.

(e)

3xy

(~)

87.>:

rind the equaliofl!l. ofth"

(c)

3 -

5y:L

(A)I

(b)

ce
.c
o

Allell)pt any FIVE. or the following:


Show th!ll the maLriK A is inl'er1ible if and only If the ~dj

t~)

xa
m

1.

.e
xa
m
ra
ce
.c
o

PAPER-I
SECTION-A

lf ~ ~L ~ -=. f"'lfOS011lQne uf a $CI Qf Uu:ee m.utu~nv ,....,eudlotilnt ~tcnornltlrs of the "'l11"

ra

Candidates should attempt Question Nos. I nnd S 11i1ich are compolsmy. and any three of the remainrng
quesuons seleeting ru leas~ one quesuon from each section.

Mnxi11111111 M 11rks: 3f/O

"/1me 1'11/tJII'ed: 3 lltlitrS

6.

(~)

F nd lhe wn1 b. douo itt muvingllll objeo.t in lhi. field from (I,

2.1) to (3, I, 4 ).

( 12)
1\ p3rttele I> moVel\ in n plan.: such that il ~~ acted on bl' two consmnl v<:lucltle$ u ond v

t't:>f>C<:Iively altm.S llte dircclilm 0X 3n<l ~lung lhc dircclion p11K'ttdicular lo OP. ll'ht(t: 0 is

some fixed pom1 .thai

io lh~

origjn . Show lhnt the pud1 ltnsvers~d hy P 1s. o come seCLion wilh

lllcu.~ ol 0 and 1:CcenlnciL)' urv.

(\;)

(15)

(d)

JJF.ii tiS where F: -l.<i - 2ylJ+ :ir nnd S the ~urface of lhe cylinder hounded

Using L<!plru:c tr.rnsfunn. soh" lh<> initial valuo pr'ilblom


l 3)" 2y - 4t ~
with y!OJ = I. y:(Ol =- t.

Evulu3tc

byx1 + y' - 4. ~ - 0 :andz - 3.

c''

(15)

Solve Ihe dltl'crenhnl equation


x'f "- JK1'- 9' = sinlin.'<J - L

!ISl
Ic)

A part:ic'le of m~ss m move under a fqree

7.

mfJ.Parl~

l(a 1 hl)u' l. u = .!. ~


r

'

ce
.c
o

ce
.c
o

(IS)

(I 5)

b. n. b, nnd

!LIO>()) being 8"""' consl.ant. tl is projected from nn apse at a distance - b with

velo~ity a-Jib

Sh0\1

h~l jiJ; 1rr~il ls given h} the equntkm r = a 1 ~ en~ 1). where Ito U) are 1he

plnne pol :or CO()rd inale;< orA p Orn I

ra
xa
m

ra

OS)
A ollclllying in 11 trigbt <mooU1horizoniJ!l wbc 6udd.mly b-oks into two pnrtiom of mas. e.<
m1 and"'" lfs be the dist.tnee bllweert lhe two ma.sse< inside the tube aft<>r tim<>" >hO'I thai
lhe walk dune by lhe expto.ion can he wrillen 01s "'I" I Ia
1
1 m1m.r t

xa
m

(b )

2m,+ m,'7

(h)

I'>Qve t)lut \7 j ( rl

dr

rdr

r ~ (.
:r~ I y 1 I ~)II>

f1 r) ~uch 'llllll \7"f1r)


'
Hcn~e hnd

=U

(12)
Sho11 tML roo Ihe space curve
X

the ounalure

(c-)

cl'
2 c!f
-,-~where

Evalunle

11 7.

~ l 1
3

:and lonuop Dre n .mc Jt lWery poi:nL

J,i,df alan& the cmve

(15)
ll!

+ y"' = 1. z = 1 !Tom CO. 1, 1) IP ( L. (), I) if

w
w

( )

(IS)
1

UVt.T.

.e

( 15)
A sol id right circular cone whoo buighl i b and rndius of whase base .is r; i phced un an
inclined plnne. It L< prevenl<>d from sliding. Jf the inclination nl' U1c plonc 0 (to Uu~ l10nzonul)
be gr~dunlly decre3sed. fiJld when the cone will tnpple ova. For~ con~ " hose emiwrtic;~l
angle is 301, detcm1ine the critical value of tl which when excc.::Jetl, the cone will topple

(d)

(15)

A ladder of wefghl tO kg. re>ts on 1 mooU1 hori7ontnJ ground le:ming agains1 ~mooth
,o,:rlicat walt ~I nu inclination lau 2 with U1c horizon aud is prev.:ntcd from 5lippl'n,l\ by a
Iring oltncbod ol ils lower cod, and to the junction of tho flour and the wall. A boy of weight
30 k.g, b~gin. tu ~end tl1e b dder, If the string can bear a ten< ion of 1U kg. wL, haw for along
I he ladder can I he boy ril;e with s.tety?

.e

(c)

I C'..S.-1\taluc\ ZOOS I

(ii) Ker ~is ~ llrlnndl KUbJlTf>Up llfG.


( IS)

l.,ot R l>e u ring "iU1 tltll l) , If the prl)duct ofhny two non Yt:rl) ~lt:m<:t~L~ i1 ~<}n lt:tcl 1rovc 111at
ab ~ 1 ::::) bn L
Wholhcr ~ hns th~ OLbi>\'C property '" JlOl C.'tplain. II z~ in inle!lf1ll domain?

(c)

Dtseuss U1t conviltgenee of tb~ ~eries

(15)

Moxiflllmt Murks: Jf/0

Caud>dates should auempl Ouesboo Nos l and 5 which are compulsory. and any ~uee or the relll!llmng
queslions selecting at Jettsl one question from each .section.

(c)

3.

Prove thai c:ve~y lnl~grol Dllmain can b" l:mbedded wa fi~ld.

(h)

(15)
Sho\v Ihat MY max imn l ideal in the commutntivu ring fl{x] of polynomials over field I' ;. the

( 12)

(c)

(15)
Let fbe eontJnuous function on [0.1). Using first Mcon V>lu" lltcorcm qn lntcgmtion, prove
that

(d)

(i) Po'Qvctlthl thesc\N

( 12)

(I) F or~ > 0, sI10w- < log ( 1 + xl. x.


I +~

H.ne }u{1 ~ " e N fu{r. 3~


'r
N

. 11

(6)

t-an,'t

Smte the the<:>rerns ~""use in "valuating nh()veintegrnl.

( I j)
Let l'tz) be nn entire run~tiilll Sloti~f)'iolg l'(.t) I :: k 11. 1~ lllr some positive con, tonl k unJ all z.
Show tlut l'rzl n'L2 for some cousl3nt tL.

(c)

Solv~lhefQUowlnglmn.."jlortalion

(15)

(1 2)

Let 0 and G be IWO groups and fe( $ 0 -.G be a homomorphism. For an)' element a. 1: 0
(i) prove U1a1 ()(>j>(a))/O(n),

problem :

O.:Stin~tions

o,
r,'

x, ,x,, .r,~ u

(9)

(b)

F.

2,r1 -t- 3x:-li ~ 5

(a)

log

+.,., - 3x S R

4.t; - X,+ .rJ = 2

2.

where C i& the cin::lol71 :

x,

II:

! ~(='+'2: ~'2)

w
w
w

=ai.r. -_11

tr

H.valujlle

x IT I)

Find 1he dual of the folio" lng It near proyamrnlng problem:


Max.. Z = 2.-ca- !:7 -r x,

such lbal

(3)

\"

_ ._ ~ " j (1 - l
s in x-'
'2 ~

.e

4.

\12)

.~

(hi

find 1be residue of


.>

(6)

(if) Provo that

e N}

lor all " e 0

cot =coth

{15)

A = {0.1 ). B = I0. I( !Ire equivnlern set,<;.

ofT also find Supr mum of'!' and grea1est uumber !>1'1'

I Cf : R-> R Js continuous and


I(~ +y) = f(xl = f(y).
for alii\,~ E n tllCll sho\\ lh!ll f(x) =

(15)

(n )

pnnoipo l ide~J genernted by on irreducibla polynomial,

Suppose lbat there is a positive even imeger n such Uml a = a for all theelemems a of some
nng R. show ihat a + a = Ofor al l a E rand a + b = !)= a= b for all a.b E R

Find denved set

(J)

L 11 /ln"1) i. equivalent to

21} ,,._.

(Sx 12-6())
Let Ru be tim $et of all real numbers c.~cept 1.er"' Define a binary operation on t:t,, 11J! . a b
; I a I b '' here I a I den01es absolute 1alue of a.
Does (Ro, ) fom1 a group' Examine

r=

(~)

c15)

Show tlutlluaeri""

(ii) Let

(d)

2 'l 6

ra

(b)

(d )

24

-I ( 1+ -I-. )

Answer any FIVE of the tollomny:


(a)

xa
m

.e
xa
m
ra
ce
.c
o

PAPER - II
SECTION A

x_ l3~ + 1 35x' ~ ........ x o.

ce
.c
o

Time A llmi'ed: 3 huurs

(h)

2
3

D, 0,
(

2 2
.3 5 .j
4 2 2
Demand 30 50 20

F,
I'

'

1), Av:oil:thili\Y
5
50

1),

1),

3
-1

I
I

.to

2
I

30

.w

~0

10

6()

by finding lhe inlti.l solution by MAtrill Mlnimn metbod.

f(:<)
1 1585
L2 S 17
I.B GO
" ''"' lruct the qunclmlic inte, olating polynomi:ol thnt lits the dMa

!30)

culclllte j (
(ii)

SECTION B

where D

Find tbc ~u\ullest po~itivc root ~>fcqualiOu J/c'- cusx= O u.,i<t&r<~gulllfalsi


it=llqm.

ra

g,

n~etbud. Do

(h)
( 12)

( 12)

Find the steody st.lre tcmperwte distribution in Utin t'dCI:tngul~r pbte bopndt<l by lite lines
x : 0. x = n. y ~ 0 and y = h The edge!! x ~ 0. 11 =' u and y = 0 are kept at tempernture zero
while: the edge y ~ I> l$ kept at IOO"C.
t~OJ

Find COOIJ>Icte ~d ingolnr integ.rnl!t of


2.~ - PI'" - 2<~'-'Y -'- pq = tl
using ('harpil'~ ooethod.
USl

7.

(a)

(i)

11tc Ji.lllnwing values of Ute function

"

10"

I~X)

20"

= tin x

cos x art ~ve11:

30

+- 7' 11teu $h,uw 'tltllt thc:-

).l

w
(b)

.e

.e

( 12)

w
w

(a)

xa
m

p bdng the tletu!il) <Yf the fluid.

(~)

+-

Sha\\ thar the hea vy end ol'che rod will

'*

iu NOR In NOR log.lc network

is;= ~ .:. tan (:~ ) r' ~ x

J.

(30)
Let we lluid till the region ~ " O(right h;tll' of 2d plnne), l.et A 'ource .lt <0- Ytl 3nd e<lul
~ink :u ({). Y:).l't > ~:. Lot tltc pressure llc sam~!!$ pressure lit inl'init) i.1:. P' Show thut Ot~
l""lUIIliJJ! pt'I:S<Ur'l: o.n tlu: 1Juund3ry ( )'>"'i~) i~
ll'p a' (y, - J': )' 12,y, (y, ~ v, ).

A cir~ular hoord ~ placed on moolh horironlal plane and a boy run.~ round the edge of it al
a uniform rate. What is the motion of the centre of Ute bD:ttd 'I Explain. What lt3Jlpens if the
liL15S of th~ beard and bb)' aroc:qunl ?

II'Utt: n:looity polenti l (>f~ tlu[d

to live decimal plnces hy Ne"1Qn

falt till Ote inclinolioo of iJIC> rod to the vcrtiOlll is co.4 ( .fi- 1)

{ 12)

Utree

((})

C\lrre<U

(30)
,\ uniform rod of mas~ ~m ~md length 21 has il' middle p oint fixed and rna~. m is attochcd
to un~ of its .:~remlty. The rocl when in n liorizonL'll position i$ $et rotating abt,ul 3 vertical
a><is lhrough its centre with an angular velocity

lie (Itt Ihesurfaces

. Q et\n.ronl
.
strcrun 1meo.
lC' +- y-" 1 z-' : c(K"' -" y')"
- 1, c bl:m[l

6,

(a)

Draw n 11!1\V cl1:1rt for ~qlvin& equation F(x) = tl


R:lphson mt!hl)d.

Represent

(.I+ 8 -c)( A- ii+c.:)(.-1- 8+~)

(f)

(12)

Sbtd tlte lll'inclplc of dlUIUty


(i)
in Boolailn algcbt'll and gi''~'ll lito dunLoflho Boolton L'Xprcssion5
(x~ r){f f)(f ~2) and X .Y=O
(ii)

(e)

( 15)

(b)

(II)

&:'

xa
m

(~)

.!!_ D ' il

the~ystm~;

- 1' 6J- = 32
with initllsl values (467. 762. 905). Carry out computations for two ilcratioos.

0.

general solution ol'thc pnr1iclc d.it:l'c:rentoal t:quation :


( l)' + V0 '- 6D'" ); =}'COS'X,

Apply t~uss-Seidel method to c.~lcultex, y, z. from


- X- y-r 6: = -12
6.r - y -::-: 11-33

ce
.c
o

~ind

I. y

ce
.c
o

and also ftnd th.:.pnrtlculor solution whloh P"'" '"" through Ibe llnos"

~ ).. Cumpare wilh exact value.

ra

AnsWI!!1' artyFIVEoftb~ thllol'lng:


()
Pind the s""ec:~l $olutio:n of Ute parlin I ,litlerentin .
.:qunlicm
(2xp-1) p + ( ;--2.ct' )'1 = 2(x- y: )

5.

1-fence

C. S. ( ~v1 ;\.I N)

EX~\1.,

2009

C-DTN-J-NUA

Sl. No .

Section-A
1.

MATHEMATICS

Attempt any fiue of the following


fa)

Paper-I
Th~ee

Time AllO\\-"ecl

H ours

I I Ma."Ximum
l.. -- -- --

Find a Hermitian and a skew-Hermitian


matrix each whose sum is the matrix

-....,

3
2 +3i

Marks :

---

_ ___J

~~]
:>t

INSTRUCTIONS
Each qu estion is printe d both tn
in English.
Answers must b e written zn
specified in the Admission Ce ~~~~ras
to you., which must be state
c space
cover of the answer bo
provided for the pu.rpo
arks U!ill be
en in a m e dium
yiven for the anst.uc
in the Adrnission
other than that
Certificate.
Candidates
and 5 tuh I
of the e m

;ltternpt Question Nos. 1


> :ompu lsonJ. and any three
g questions selecting at least
ti~ from each Section.

(b)

Prove that the set


(x 1, x 2 , x 3 , x 4 ) in R4

equations x 1 + x 2 +
and
2x 1 + 3x 2 - x 3 + x 4
a subspace
of lR 4 . What is
ension of this
subspace?

J1ssume suitable data if considered necessary


and indicate the S(.lme clearly.

~ned on [a, bJ,


a number l1

Symbols/ notations
curry
!heir
u s u.al
rneanings u.nless othenoise indicated.
1

prove that there exists


[a, bj, such that

fb f(t) dt ~ f(D}(b- a)
Ja
(c)

12

0. is the derivative of some function

ber of rnarks carriecl by each


1:s indicated at the en.c.l of the

1.

" is continuous on [1, 2J


has three zeroes in the
. Show that f '' has at least
1n the interval (1, 2).

12

at

{c)

(d)

its bases.

12

A line is dra\vn through a variable point


2

on the ellipse x -1- }!____


a2

b2

= 1,

z = 0 to meet

two fixed lines y = m1C: 7. == c and y = -mx,


z =~ -c. Find the locus of the line.
C-DTN--J-NUi\/ 44

12

{fj

Find the equation of the sphere having


its centre on the plane 4x- 5y- z = 3.
and passing through the circle
')

X~

(r:r.) ~ ~ ~ ~ fu"q-ll-tf4lt ~1-=-'l{~


~~. NHCfll
!

')

y ... + z... - 12 X

3y + 4z + 8

3x -~ 4y- Sz ~ 3

~ ~lv{(. -r.t

-1 l
3
I
1
2 +3i 2 !!
4
-i + 1
Si

2i

')

2.

(a)

Let
. /l'

. i'J

= {(2,

=0

= {(1, 1, 0), (1, 0, 1), (0, 1, 1)1


1, 1), (1, 2, 1), (-1, 1, 1)} be th

ordered bases of 1R 3 . Then find ~~'=~'


representing the linear trans~
3
')
T: R - ' R-- which trans fa
./J '. Use this n1atrix rep
to
find T(:\t where x = (2, , ).

20

(b)
12

( 1)
(1)

(-q)

(c)

m~ [c

~~
'

-m

. .::\c+

011~.

skcw-synunetric?
Justify your answer.

.L f (t} d t = I (11 }(b -- a}


{I i ~ '1

20

12

it

~'n: y

")
X-
')

a-

~:

11

+ :.:_ . .:. 1 z
b~)

tal it

= --mx,

12

f~;~o

7::

0 q

1:'srr-:1it

v-=F
}

z ~ -c <:n-1 IJ:k1f~

t:rft-T<ff

'~

C-DTN -J -- N U!\./ 44

-: JR 3 be a iincar t ransfor-

L((x 1 x~, x_ 3 ~ x.1 })

= mx. z = c
in AQ. ~~fi-.-ir

~ '~. I :;u ~1:'fl ~ f.ij~-;:rr~..f -;:f.T ;n rf -~:fl f:-Jtt. I

Ination defined by

P.Fi

Let L : ~~

20

=:

l2

[ P.T.O.

(x]

-r x4 -- xl - x2~ x3 - x2, x.1 -

xd

Then find th~~ rank 8nd nullity of L.


Also, detern1ine null space and range
space of L.
C--DT!\-J-NUA/ 44

20

(~)

" ' ~
..
"
'"' R R
~ 1!,B
Cf.T Afl!Cf.{OI
~ C:hl'-1l!,,
'-1fi2hl ~
4X

fllOI(id

5y -

3 t:f1:

Z =

if

~tt

(b)

=0

rtit~~m1

./1

,/)' = {(2,

1, 1), (1. 2, 1), {-1, 1, 1)}

m~m ~ iR

~1

~~ t:.,rcn V.~
l'i'qo1
"'
~

12

{(1, 1, 0), (1, 0, 1), (0, 1, 1)}


~ 3ii&J;f. ~ ~ ;:f))f~lJ

(R"

~.q i<H'-JI

t-fisr-..r.

mm ../J
' '

~ 3ll~ f?t~qo1

Cfil

T: 3.

T(x) ~1 3JT?f

(c)

20
;;.ny

-A 4 llh1 c$it
~.ci

tm

q"{

3fu

"(.fl~ L: lR

-> lR

if (x, y) = (0, 0)

A space probe in the shape


ellipsoid 4 x 2 +y 2 +4z:2 = 16 ente
e
ea rth's atmos phere a n d 1
st rfa ce
begins to h eat. After
ternperature at the
probe surface is

oint on the p robe

4.

-m 1

m i.r m

t he set V of all 3 x 3 real


.tric matrices fo rms a line ar
__, pace of the space of all 3 x 3 real
~1atrices. \Vha t is the din1ension of this
s ubspace? F ind al least o ne of the b ase s
for V.
20

__

~ ~

~ MIMCh
q ?.d it

6T? ;sffi ::rrr: ifi

~ ~ 'tp')rq:; ~-Yio{ 0 1

{a)

~~ 1

20
3

if (X, y ) ~ (0, 0)

9~

chl f;jj t.~ I


4

20
(3, 4)

~-fl4fild <f'r;fi

%81fuifl ~

~Tft:Jl:

:?.11&{~ .A ~ ~ cf)f~o,,
~ ("

+y

., ~ ~? 3Wi

20
(TJ)

Is f continuous at (0, 0}? Compute


p a rtial derivat ives off a t any point (x,
if exist.

-: .,

(X, y)

CfiT
.... ./.J' ~
~ ~
..

. IT

~f(l4k1 4lGQ,, :iit.f x = (2, 3, 1


(B) ~X =3 0-01 -Jfl\ y =
~ P(x, y) ~ wfm

fefi <1 <fl q f(:U ~<11 if Bl?


~ ~ li.t-1C1-f1 cnl -~
~ -q ?11-rr. ~ -

-) JR 3

~ IR be defined as

-l~ V ~y;y

+y 2 +z 2 -12x-3y+4z+8 =0

(en) ({lf;jjQ.

f : R2

\;IT ~

3x + 4y- Sz + 3

2.

Let

t,

Evaluatt:

iT

I=

JJx cl y cb; + dz dx + xz 2 cL'K: d y


s

= {X 3

+ X4

- X l - X 2'

X 3 -- X 2'

B -cffi~ it 1 (fq

L ~ ;:f.tP- ~fn ~T~(f[

<:F':'If-31~1 Ul~ -~) L

cht

CfT

1ft R~.rr{Ul

C-DTN--J-NUA/44

~I

~f'~ Hlifu

3fu:

X4 - X

tl

where S is the outer side of the pa rt of


~
~
~
I he ~Dher e x - + y . . . + z . . . ::: 1 1n the first
octant .
20

cit ~

'

~ ~

20
P.T.O.

C-DTN- J - NUA/44

.....

f : IR 2 -~ R RRF8fi:J<l iJ1 ~ ~

(G) ctlf:tl!\

{c)

Prove that the nonnals fron1 the point


")

~:

qftmftrn'

(u,

~XI.J_ ') , <TR

(x, y}

f(x, y) = I~ x- + y-

l
<P-n (0, O) ~

(x, y)

lie on the cone

= (0, 0)

~'

-?l'l::Ctqol-m

!J -

~~

')

-IJ~

--- :..: 0

Section-S
5.

Atietnpt ;:u1y

= 16 <it -

~ ~a:T ~'Q'UT -m;r -gvit


~
.

X- U

u-

+ - .

~l:-S1r1\Ji 4x 2 + y 2 + 4z 2

.)

I)

ct.

~ -~? %m ~ (x, y) ~

L{"

r) to the paraboloid - ) . "';" ..., = 2 z


a})-

m1

31ful<"'l
(Tf)

* (0, 0)

p.

X.:...

of

en

ccccn trici tv
. e under
cent r:=tl force directc

'

~ ~ T.{S 11~ ~
~~~g

fnrcc i~; trc.tnsfc -eel


Find the ecce
of the 111.:\V orbit in
hTn1s of t
:e1 ricity pf the original
nrbit.

I'Jf

T(x, y, z)

=-=

8x 2

6z + 600

~~~:,.
4.

(Cfi) fu;s.

20

e:

~'1'811~ ~ ~ 1 ~

it

"31=l1:v:rfu

q)f

:F,(

-~ CFn

~~ <:ntr -it-~ -n;:Fi 31T~ ~d cilf~lU

= JJ x dy clz -r dz dx + xz

20

cL~ d y ;;F,I !{~r>"ll <:F.=i

~'

~~

2
X :2 + y '2 + z

C-DTN-----J-NUA/44

1he

set

of

ntlf~o "~

u'lfi 3 x 3 q Jwf'qqo,
- ~ "{:rm 3 x3 r:l[f<1~q:) ~ ~ ~

._ V

of

(bj

1 ')
~

=1 ~

~;..fi

'qlTT q.;-t

Jt

~l ~

T~

it I

20

[ P.T.O.

t n the inkn;d J-1, lJ and detcrn1ine


whet her the set is lincelrly dependent on
i- L lj.

12

i\ unifonn rod AB is rnovabl'.' about a


hinge 81 A ~u:.d rests with one end in
contnct vvith a smooth Vt~rticd \\'::11~. If
1he rod is inclined at an angle of 30
with the horizontal, find tl.(~ reaction at
tile hinge 111 magnitude 8nd dirccti:.n1.

12

C--DTN-.J-NUi\/ 44

f\

(u:,

.-,
u.
I)
a "'"
- - + - - - +X-

y -~~

(t_

(d)

y)

- I) -

=0

Z -- '{

A shot fired with a velocity V at an


elevation u strikes a point P in a
horizontal plane th~ough the point of
projection. If the point P is receding
from the gun with velocity v, show that
the elevation must be changed to 8,
where

. 20

Sll1

(e;

r\

= Sln LU

2v . e
+ -Sln

Show that

where

12

U.J
1

- r-.. ct-:l

:%
'H

;H-~

i,hn

T1 <-l;n

(i)

~01

~, -<Y-1 ~

z-axts;
(ii.)

(1, --2, 1)

tn

the

66

12

6.

I3 ~-- ' I ")


-~

fiH-1--~ ::;lid

'
v ,)

..._) .-'\

..-'\.

far

II
~

1] ~H

., of circles in the xy-plane passing

th~ ugh (- 1, 1) and (1, 1).

-fJ}j l! ~){ f.t~~!J_lJT q:;~ ff, ~

equation of the

(a)

.l

'tfi:J cfld:

f.r~h

t,

3-T?.fQT ;; 1-:fi

~I

20

inverse Lapbcc transfonn of

12

F(s)

=In( s+~ '1


\S+;)i

('1)

1:.:-r. (,5,t"J41'"1 ~-~ ~- /\B, f--=i;~,_, i\ !~~ ff,-:;; -T. ~r;~~~


qf-ct-:.n8 % .~fn ~ ~ f{v.J ~~-;ti f'=Fh~f ~;Afc;..:n:
~HP

;r,

-~-f:.-d;

-ii

~ f.?.TI '1.1 -~f'~ ~s. f%1Pr3l


~~

m~~ 30" ;-f. .:r:i'Tf ~ 3:rf11"1(f

i1

'1P,41'1 .-~)r_ ~~~-n

C-DTN-.J--NUA/44

it
9

(c)

dy -

n' riT lt"i 1'17 ~fctf~

-:TT7l -of.if~ 1

Solve :

20
y 1 {x - y}

~~- 3.~~~2 --~'(2y- 4y]'

12

P.T.O.

C-DTN--.J-NUA/44

20

10

y(O) = 1

(''-1)

"Wr

m~ ~ 3 -;I d 1!< 1 a l:R zyft

iii

Tli rr"hft,

~~ f~ ~ ~ -B" Tj;J~,dt ~ ~~f<NJ B~

i1

~ P t:R rn1. :wfT ~ t ~R ~ F' ~

it en -::;n

~ W'-l ~
8

it

"{(.1

it, <fr

~Tfffi1 '::fiG11 ~j1Jq {'1:l q:;


. ?c.
stn
_o

==

. 0 .
s1n
.:..U.

<1rr

~~~~~ fu; 3';:lrli!lt

it d'
l

it

7.

(o)

One end of a light elas!ic string of


nat ural lc:ngth I ~~ nd modulus of
elasticity 2mg is att3ched to a fixed point
0 :1nd the other end to a particle of
mass m. The particle initi~llly held at
rest at 0 is let fall. Find the greatest
of the
the

(bj

/\ particle is

:r-1

'it~~

2v s1n
. nu

-t- -

12
(T-~)

6+6
--1'

2)

frurn t ht~ cusp of


cycloid down the

inverted
that the

t in1 c

,' v ..
I!

it

{ii)

(1,

-2,

1) o/

t.

--=-

71

2~ag /

radius of the generating


10

:sfu: (

--1 ~
1, 1} it it :r~1_:iJ ~ xy~-{~
-~ Cf. ~~ fllft41J_!JI !'fi1 ~ -:h~l

it
body,

20

the
'

:)( 2i - j : 2k) N
A

furccs
and

G12i :- 2j - k) r; :rc acting at points with


..

-.

..

,.

....

..

poslTwn vectors i - j, 2i -r 5k and 1 i - k

20

respectively. Reduce this syst('m to a


~

20
dy Y~ (x- y)
y(O)
~~ - ~lxy 1 _ x 2 y ~~+~;.;,,
C-DTN-J--NUA/44

11

=-=

..

together with

')

single forceR acting at the point (4i- ~~j)


~

coupk G whose axis

pas~c:s

.t h ro_ugh this point. Does the


point Hi +2j) lie on the central a.\:is?

I P.T.O.

C-DTN-J -l'iUi\/44

12

lS

7.

Ceo) Wf?!

<:1~1t l

3ftt

~ ~'-l i'1;1 Cfil


~311 % ~tt ~HI

~ ~I ~
~P)l

fum

2rng r:mft ~~
~ lJ:HI ~ f. ~~
. 0 TTI ~f~
:;:..-.)~111'1 m ;fi ~% ~ -1: UP..T

>r?ffi.'lJ(n 'TJlfq)

fun

it

0 t:P.: f~HI~Iq{-?--11

%I TTfu ~ fu.:r

(d)

radiu~ a so as to be in contact with


three-fourth of the circumference of the
pulley.

lt W ~ qi~Jf cnl

sm Cfil ::nfi.r:r..:m
8.

(n: + 2 - tan - l 2)

~fi

m if 0

11(!,

V ~ W'4 ~

f2L

~ ~

--+

F ;;.. {2 X - y

z) i

qTf

~-,r-1'.

J,

15

Find the \VOrk done in !":"lOVIng


particle
once
round
the
ellip,<'- - x"' . y-.,.. . . __: 1, z. == 0 under the field of
25 16
given by
')

TTJ: q~-':l ~HI

it ;fl--it

(a)

v9

(G) ~ ~ !Jf<1d1fiP1 ~ ~ q:;~:q

Find the length of an endless chain


which \Vill hang over a circular pulley of

T ~ToT

y,.. 4z) k

~I

20

(b)

'! .

10
....

,...

1%

q~

,..

....

j + 2k}

O(i.

~2-TI

T-1
,...

N~

i + 2)- k) N

.J

S(-2i - j

~-f:Pl CR

r%

r.,

r:p:

(c./

...

2k) N

.?,ffi

+ y J + z ~ k and S is the

value of

~~

taken over the upper portion of the


...,
surface x "- - y ~ - 2nx..,.. az = 0 3nd the
")

--+

~.q ~t (~~ ~g:-:(;1 ;..rc:--T R ;:r: 1JJn:r-:.R #.rf}ft!.~ :;i1


"

,.

-)

~ (tti + 2Jl lH ~1n-r G ~ m';l..f


fli~*-' 3-P,l ~ fil~ ;t. ~:ff...~

"-0\ ~~T

n ~VHt=tl i.T I '-f"'=ll

(4i + 2)) $.{1~ .wH 1l: ~~-Fi

%7

bounding curve lies in the plane z


when
- .
., .
..,

..

01,

JC1.-S,

=ju""
~'
I

15

t(z-

:=

0,

~- ~ I\ r.:

'-

'')
'> .
')
~x- -y~)J+(x

+!J

:2

-z )k
20

C-DTN-J-NUA/44

13

[ P.T.O.

C-DTN ,J -NUA/44

ll

(~)

itB HWi?l<i ~R cfi1 dkil~ ~ ~fm!,, iT


~ a ~ ~ rfm ftrGt 'n: ~ ;F-F;T{ ~ mm

~ ~ ~ cfit r.rft"F-1 ~ aR-~~~ ~ ~

8.

("'.fi)

= (2 x

1s

- y + z) i

+ (X + y -

IDU

ilm 1

C-DTN-J-NUA

3ffiJ ~ -~1:1

l + (3 X - 2 y + 4 z) k
x2

y2

25

16

B!1'1 -:.. r:rr-1

--~

.~-,i]~

it 2.1tfR <ftd111 -- + --

z=O~~sm~~TJ!fi~

-- ~

~ ~ Cfi1 ~ cflf~=g~l

20
].;{1t7.jlf

~;JIJ .:;n< -~
"

x ~ + y 3 ) + z~ k it 3fu
~T ~ i:,ll

(rr)

~ ,

;$

l[l!l-

RFf!:!

F2tR ~

g;f

20

- 2ax + az = 0 ~ "PP.1 ~rr 1 ~{ 1<:11.


~. Cffi ~ Bl1<ic:3

-rn!,

.1

CF,{ p:rf!

-- ~ ~
,, ~ X F)' dS

q;j

= (y 2 + z......, -

X"- )

')

z = 0 .q W.l(T l:4

tH

11R ~ rtlGilt~, ~
~

+(z2 +x2 -y2}_J.-r(x2 +y2 _ 2 2)1~

20

***
Note
C-DTN-J-NUA/44

15

JS-1000

Engtisl: veTs ion of the Instruct ions is


printed on the front cover of this
question paper.

~.

!J

t, (

S. (\.1;\IN) EXAM, 2009

I /

Serial No. [. . . _ _~_._J_ __

SE~ION-A

( C-DTN -J-NUB )

1.

Attempt any FIVE

(a)

(12x5=60)

If !R is the set of real numbers and IR i is the set


of positive real numbers, show that IR under

Paper-1 1

addition (IR, +) and IR ~ under multiplicatio

(IR +'
Maximum Marks :

Each question is printed both in


and in English.
Answers must be written in th
specified in the Admissio
3' h stated
issued to you, which
wer-book
an
cover
the
on
clearly
purpose.
he

prov
space
in the
answers
the
for
b
will
marks
No
that
than
other
a
in
written
.
Certificate
ission
~
i
specified
a/tempt Questions 1 and
s
r &
Candf
y, and any three
ompu/sor
5 wh
selecting at
questions
aining
o the
Section.
each
from
question
e
e t fttl
me suitable data if consider ed
"ecessary and indicate the same clearly.
Symbols and notations carry usual
meaning. unless otherwise indicated.

~ 41 ..-{1 (

<r

!1F7- !ffl'

ct

are isomorphic. Similarly if t1) is the

(b) Determine the number of hom

the additive group zl5 tot


(Z n is the cyclic group

(c)

ht sms from

ad

ve group ZIO'
n).
12
I

to prove that between


there will be a root of

2+10=12

+1

if

X<

+]

if

I~ X<

-~+1

if

2
X
')
....

2sx

What are the points of discontinuity off, if any ?

All questions carry equal marks.


CfiT

')

of rational numbers and <Q+ the set ofpositiv


nwnbers, are (<Q, +)and (<Q+, ) isomorp
your answer.

INSTRU CTIONS

rrr rrw

following :

MATHE MATIC S

(Time Allowed: Three Hours)

e'ZlFf ~ : 31'1~

~he

fflER~ ~

What are the points \\here f is not differentiable,


if any ? Justify yours answers.
12

~ 1

(Contd.)

.'

~-(.f)

I.

if if Fcf; .. t1

f'i~'1fc1~d

cfi

n- 1

~ <flf~t!,

~ 1R qlffl~ctl ~311 CFT ~z


cll~f1ctl ~m CfiT ~?:

if.

m..

: (12x):..:60)

_
( e) Let f(z) =_a.. .; . .o_+_a-=-J_z_+_._.__._+_a__:n.:.-._ t:.z_
'bn ~ 0 .
b 0 + b 1z + ....... + b"z"

3fR- lR + u., l\liCfl


fill IR tinT cF

Assume that the zeroes of the denominator are


simple. Show that the sum of the residues of f(z)

Gtllb~

3Ttft';r (lit + ) 31~ lR _ ~ cf1 ~nft4 ( IR+ , ) "


iffi g I ~ cF f!AI"1, ~ Q) qf{~~ ~3fl <fir
f>1' ~ (() tl"ii,Act) qf{~Q ~3TI Cf1T ~c.

at its poles is equal to an -I .


bn

(<Q. +) ~ ((j)_, ) '41 ~~J.:Ft;lfl ~ ? ~~~~

if

-q(R

~ 4l~Q I

7/
( '@}
. . ~ ~
~ ~15

eFT

(1\Qll

m<'FT

it ~ ~
'' ~- z
A uhuT c:F) R:1c~ I '

.......,.,..~--i"=Tm

m
f*

<fi
e~

paint from two raw materials M 1 and~,.,..,


data is as follows :

-=

"

1 ~'!:rt

CFT ~ ~C1

Cf)l

Exterior

daily

tilf~d ~ ~ ~

~ ~

ir

e~ sin x =

~ \ :fl i4 IM rtf] RrJ Q, I

2+10=12

Maximum

12

~ 7FT q)<r-f {

cos x

A paint factory produces both interior a

n CflT i1 J\1 1

~~)

ur)

(f)

availability

Raw Material M 1

24

Raw Material M2

Profit per ton

+1

+1

trfu:

s-fR

x< I

survey indicates that the daily demand


i~terior paint cannot exceed that of exterior
aint by more than 1 ton. The maximum daily
demand of interior paint is 2 tons. The factory
wants to determine the optimum product mix of
interior and exterior paint that maximizes daily
profits. Fonnulate the LP problem for this situation.
12
rna

IXj

--

rf

~fi id\1

~~

it ~
tn ffi (

<t1'1~

-t-

')

iF cFt::r ?r

I :S x<2

2 :S X

~ ~. ~

~ ~. ~ f 3ldCfl f1 ;:fm"

thl { if ot ?
"itT ~ . ~

~ ~{i ~ -qe1 if~ ~I

....

12

(Contd.)

(Contd.)

2.

(a) How many proper, non-zero ideals does the ring


Z 12 have ? Justify your answer. How many ideals
does the ring Z 12 $ .'12 have ? Why ?

2+3+4+6=15
(b) Show that the alternating group on four letters A
4
has no subgroup of order 6.

(c) Show that the series :


(~)

tr<:

CfiT

~ Cf1H ~Ci l11

3li<:iR't1 ~ ~
iS!

f.'1 ~ I4)

~.

ca

m"11

~ ~ M1

1rz1

f.'1 101 ff1 ~ (1

qi1

Jc-41~1 ~_..

3Prtl"R.
~

( ), +

G::

+ .......... +

converges.
Cf)-.:;i!l l=fTR M
C'fi "til l l1"TR M

(1,000

24

(d)

!R is a continuous function

c, d] for some real numbers


5

~)

~ ~ "tfffi

15

4
"'qffi

f<F 31 id f{ Cfl

~z

cfr ~ PI q;

ire ~ I z;:r if ..,-qf~l m efT ~q;<ft ~ I


~,rT"""""l'"t-T-::r- qz cFr ~ ~f.1q; 11f1T 2 c~ ~ I Cf>l (\~111
3i i ct~Cf1 31h <mrr qif tfi ~ ~!Sed~ Jc-q i G-ftr~
CfiT ~ uhuf Cf1 { =i I :q l~ct I ~ f<F f\?1 ti ~ ~ PJ Ch 1_1~
3lftrcF ~ 31-fucf; it ~ 1 W ft~ ~ ~ LP t1 ~ ftfl

....._""~~>Mrr, ~

---

15

12

3.

hat Z[X J is a unique factorization domain


a is not a principal ideal domain (Z is the ring
of integers). Is it possible to give an example of
principal ideal domain that is not a unique
factorization domain ? (Z [X] is the ring of
polynomials in the variable X with integer.) 15

(Contd.)

(Contd.)

2.

(ct1) ~ Z

12

~ \3~(1 . lh:-~

~ ~ ~

F<hii~

3lTGVf

"qaf

if

~ {l~cz

g ?

fcl;~Pi

3lTGVf

mit g

(b) How nu-tny elements does the quotient nng

1~ Z 12 EB Z 12 cfi

z(IX]
(X.l + l) have? Is

2+3 +4 +6=15

it an integral domain? Justify


IS

yours answers.

("&)

(c)

Show that :

(d)

15

a limit point.
4.

c-q)

G~,r~cz fcr; ~

m~

1~

IR ~

c 3Th:

ttmr

d~ c <

m,

%1-1

ct

(a)
show that :

~ ~

15

27t
- - + cos e + y sine =

.J;;_z_ B2- _ y

2-

30
3iffid)ll ~o 1"1 <.cnF1

:!)01\illqctl

(Z lj\ulf'fll.

lJhf
Cf)T

~,

1ffi1 CFT 3GI6{01


~~ tf<qq g~ uiT 3f~Jill ~~U1 1Zcis1 '9Tif ~ it ?
q;-r ~ ~I)

15

3x 1 + 5x 2

-+-

4x 3

'x + _..,.2_'
~x < 8
._,l
3x + 2x 2 + 4x 3 <
1
2x 2 + 5x 3 < 10.
X.~
I

7
....

ubject to :

---- ~) I Cfm ~ ~ ~ :!)01\Jtlqd)

(Z [X]~ ~ ~ X if (;l~q~y

ax<hnize : Z

(Contd .)

15~

30

0.
8

(Contd.)

(&)

(~;[::)

<rc'lll

<f; f<l>ao?f

m~

SECTION-B

? '1m

5.

~ "QCfi ~~f~a 1Jhl ~

Answer any FIVE of the following :

(12x5=60)

G~~it!\ I

(a) Show that the differential equation of all co


which have their vertex at the origin is px +
22

Verify that this equation is satisfied by

L -~-=L _E__.
+1
Ltl

Lt
x.....;.l

n4

n=l

ex:

+X

n=l

yz + zx + xy

n4

(b) (i)

= 0.

quation by

Form the partial diffe


eliminating the arbi

12

tion f given by :

6
(tl)
(ii)

asses through the curve :

dO

2n

~-------

a+()cos8+ysin0- ~a 2

-p 2 -y?
30

xy = x + y, z = 1.

The equation x 2 + ax + b = 0 has two real


roots a and

p. Show that the iterative method

given by :

2x

3x 2 s; 8,

-~

3x 1 + 2x.,
lv

"'-'2
X.

+ 5x. 3

<
.....:

4x 3
.

xk+ 1 = -(a xk + b )/xk, k = 0. 1, 2, ..... .

15,

is convergent near x = a, if I a

10 '

I > I P I

30

'2: 0.

(Contd.)

10

(Contd.)

(ii) Find the values of two valued boolean

1.......-.J

~- ~"'

5.

~ ~ fci;..-ift ~

f.1 J:1 f<1f{g(1

(CF) ~~nf~C( fcp ~

cf

"Jm

men ~I

$) Rrl C!> ~ ~ fll"il'f' {O 1 ~ yz + zx

(\Cf) (i)

men i. I
~

AB =AC
AB + AC + CD = CD

+
12

f(x 2 + y2 , z - xy)

cfi ID11

simultaneous equations :
A+ AB = 0

CFT, Ril1 ~ ~ \3<(Jfl4

tr<AT ~3TI

~ ~I 31C!Cfifl fl41'f'{OI px + qy =z

GRT ~ 1tSc:

variables A~ B, C, D by solving the following

<flfJI ~ : ( 12 X 5=60)

=0

t4 -cg

(d) (i)
....,...,.,:T'\...._...,_,,

Realize the following e


NAND gates only :

[m, 31i~ICfi

(ii)

he eotnplement of x.

al equivalent of (35732) 8 .

(ii)

6
~~{(11tfrl

~4)Cfi{OI x 2 + ax + b

aft-< p
Xk4 l

cfi

~~~f~C?>

=-{a

xk

= 0 cF <IT
~ :

11

I a I > I P 1.

~ted to

one tlat surface

or a cvlindcr
of the
..

e radius and of the same material. If the length

of the cYlinder he I and the total tnass be m. show

+ b)/xk' k = 0, 1, 2, ..... .

~ ~ ~"'1 U1ffP!\<'1Cf1 ~

~ ~, ~

~~t~Cfl ~

(e) The

=a, cfi P, CfiC:

(Contd.)

that the moment of inertia of the cornhination about


the axis of the cylinder is given by :
.(f

mr~

1 /(

1 r'1

l '- + }): r //I/ l' + -_).


3
12
....

12
(Contd.)

(f)

Two sources, each of strength mare placed at the


points (-a, 0), (a, 0) and a sink of strength 2 m is

A+ AB = 0
AB =AC
AB+AC+CD=CD

cF [m G) +l!<:tif<f;(l ~ ~
B, C, D if; "ll"A1 'lit 'ili'i q()f;! <:';, ~~
Cfl {(i I ~ X ~ ~ eFt I
+
6

CfiT

('q)

(i)

~~

at the origin. Show that the stream lines arc the


curves :

'"ffm

f~fl-'1

R1 f{q {1

IDiT

dj \jj Cf) q)l

Cfi1

where A is a variable parameter.

Show also that the fluid speed


2

is (2ma )/(r 1r r ), where r ,


2 3
1
distances of the points fro

~ ~

point
~r are the

urces and the

sink.

~..........~~I'T'T

6.

12

(a)

e their usual meanings.

( i i)

15

(b)
(

..,
..,
6
0
0
vhi:re D and D' represent -and-.

ax.

av"

15

A tightly stretched string has its ends fixed at


x == 0 and x = I. At time t = 0, the string is given
a shape defined by f(x) ~-: ~x(l - x)~ \~here Jl is a

ft :

I + -4 - r J ;'( I + 2r .
me, ( ~
._ 15 1 .
3.
13

12

(Contd.)

constant and then released. Find the displacement


of any point x of the string at time t > 0.
30

14

(Contd.)

mm eFt:

~) ~

~k~Cfi m ~

~ (-a~ 0), (a, 0)

Cf)f,

7.

(a)

~ neff ~3fT ~ ~ Wlluf 2 m <:fiT tzCfi ~ 3 <{lll=l


tfr

fmfi1 ~
'

(x- +

I G!llf~C!\ ~

") ...,
y...,.. t.., = a-(x""

um ~ZC41ct

a:sfi

Develop an algorithm for Regula-Falsi n1ethod to


find a root of f(x)

0 starting with two initial

iterates x 0 and x 1 to the root such that sign


(f(x 0 )) ~ sign (f(x 1)). Take n as the maximu

- y + /...xy)

number of iterations allo\ved and


prescribed error.
~

'41 C:!llf~C!\ f$ fcRft ~ ~ ~

it ~3IT <tt ~ ~
6.

(b) Using Lagrange interpolation formula, ca~-.....-..

of x and f(x) :

('fi) y 2r - x t = 0

cfi

~e:fUT iil<fli .,...~,. .,

3Tn: t cF

~ flll=Jid'f

~~I

f(x)

19

15
(c)

= (2x + xy -y) sin xy- cos xy


2

'"'

~...:~1h-- D'f.i~fqti ~ ~ ~~- ~~


ax (Jy

cF

m-'4

~ ~~ ~

* ftn

vith step size h = 0 2 from the initial

= 0 3ih:

30
15

o y( 12) using Runge-Kutta fourth

15

= I ~ ~ ~ I ~ t = 0 1R, ~ eFT
f(x} = ~x(/- x)
mr tift~
<ft \;fRfr ~'
~ ~ ~ f.i t1ct iCfi ~ 3lh: ~ ~ ~ q?r <ftffi
-mm \JfTdT ~I~ t>011r ~\ cF ~ ~ ~

15

'2)z

l=l\51\cll

t r values

value of f(3) fron1 the followin

12

(Contd.)

15

A perfectly rough sphere of mass m and radius

b~

rests on the lowest point of a fixed spherical cavity


of radius a. To the highest point of the n1ovable
sphere is attached a particle of n1ass m' and the
16

(Contd.)

7.

(CF) ~

{j ~ 3;1 {1~-rl) x

1f{

3fr\

x 1 "it ~ ~

~. f(x) = 0 CTlT ~ ~ l11<1C\l1 ~ ~ ~.

ttma

(~-cn,""tn) f&fu

f4un

* ~. ~ ct\" ~nrt~

system is disturbed. Show that the oscillations are


the same as those of a simple pendulum of length

<FT

(a- b)

7
4m'+-m
5
m+m'(

2-~)

(b) An infinite mass of fluid is acted on b

(&)
r
<fi1 q11 ~<:1 ~ en~~~
X

f(x)

14

~ 2 per unit mass directed to the ori

the fluid is at rest and there is


of the sphere r = C in it, s

:
6

19

be tilled up after

al of time (

15
(lT)

- 2

,.......,......... m

h = o2 ~ m -nT-~ :q1&t1'1fl fctfu

~ ~, y(l2) Cfi1 l1R liltitf ~~~I

CFr

15

~cJWl~H m ~ ~ \J"~ I b CFT ~ ~: \~{~<I Jflfl(fl,


~\J"~I a cf; ~ ~ lllC11Cfll{ q;)n ~ ~ ~..hil1 ~

"tiT

fZ<fT

~3fT ~ I 11fcl !JftM ri1<14i


17

cF

3 -cilct l"1 ~ Lf{


(Contd.)

r.c: .
30

3Tffi%

5~

18

~ cl ~ 11 m'

CN

~ Cf1UT

\iffiiT ~ I ~~If~ Q,

".1 S3!1

eFt fe C11 tll

Serial No. [...___ _____,)

fcfi ~1 C1 "1 crcft ~I ~ ~

C-DTN-J-Nui)

JlfUI~
~-~-11

.........'lllrt~M-

lflC1Cf

~!llf~~ ~ ~

30
"fJ
.:>

~ 311<:frt4Ch

ir fit

Jtf4Cff1

3/Jct>sl

q;r "fJlR

~++--'u........... am Jrtct>) R~ Jrit~ I


!](f)Cf)/'

.3i1T ticf>(Ftl

(fCfJ 3Pl[q-{

ct

9 ilfRa 3t4 ~~ UTGr

1 CflN TJllT

"R"'lfr TJ!l1J' $ 3f<f> "fm71

..

ir l
fr I

==::=:.:~- ~----=-======-:..:===:::====-:::..::.=~::-:-:======

---

lVote : English n:rsion qf'the Ins/ructions is prinled on the


.fiont co\er of this question paper.
19

20
....

CS {MAIN) EXAM, 2010


Sl. No.

127

1.

MATHEMATICS

Paper

Time Allowed

Sect:ion-A

C-DTN-K-NUA

Three Hours

Attempt any five of the follo.ring


(a}

j Maximum

Marks 300

If A 1 , A 2 , A 3 are the eigenvalues of the


matrix

26

-2

21
2

A= [

INSTRUCTIONS
show that

Each question is printed both in Hindi and


in English.
Answers must be written in the medium
specified in the Admission Certificate issued
to you, which must be stated clearly on the
cover of the answer-book in the space
provided for the purpose. No marks U>ill be
given for the answers written in a m.edium
other than that specified in the Admission
Certificate.
Candidates should attempt Question Nos. :1.
and S which are compulsory, and any three
of the rem.aining questions selecting at least
one question from each Section.
The number of m.arks carried by each
question is indicated at the end of the
question.
Assume
suitable
data
if
considered
necessary and indicate the same clearly.
.
Symbols/ notations
carry
their
usual
m.eanings, unless otherwise indicated.
Wl'i ~ :

3l~'lfl Cf>T
<J4T ~I

f%-<1

<-<'4HR W

"Sf'R-'ffl

f<it<Jl

JAr +
(b)

(c)

(d)

A~ + A~

< .JI949

What
is
the
null
space
differentiation transformation

12
of

the

where Pn is the space of all polynomials


of degree < n over the real numbers?
What is the null space of the second
derivative as a transformation of Pn?
What is the null space of the kth
derivative?

12

A twice-differentiable function f(x) is


such that f(aj = 0 = f(b) and f{c} > 0 for
a < c < b. Prove that there is at least one
point l;, _a< I;< b, for which f"{l;) < 0.

12

Does

the integral

If so, find its value.

'[lO 'R

C-DTN-K-NUA/46

f
1-1

JS

dx exist?
12

(e)

(f)

Show that the plane x + y - 2z = 3 cuts


the sphere x 2 + y 2 + z 2 - x + y = 2 1n a
circle of radius 1 and find the equation
of the sphere which has this circle as a
great circle.
Show that the function
f(x) = [x 2 ) + lx-

=[ ~ -2~ ~)
4

~ ~~fill<'h "JfR

%, <iT

.JAr +A.~

~11!<1{ ~

J:

12

< .J1949

2.

(a)

d
dx

- : Pn--? Pn

(Tf)

(ti)

f(x) i1;m ~ fif;


a< c < b ~ ~ f(a) = 0 = f[b} afu: f(c} > 0
~ ~ cfil~~ ~ <P<-~-<Pl l1;<'h ~ 1;,
a< I; < b, il;m ~ ~(1~ ~ f"(f,) < 0.
l1;<'h

<?I om:

<P-IT fil'll<'h<ii

31Cl<'h<'1;;fi<!

i1_m t, <fi !!fit <'hi

C-DTN-K-NUA/46

"JfR

m<r

Let

f<ii,;JtJI'l

cfil~v,: I

itffi

Find

transformation

the

12

un1que

T: lR 3 ~ lR 3

so

that M is the matrix of T with respect


to the basis

13 =

{v 1 = (1, 0, 0). v 2 = (1, 1, 0), v 3 = (1, 1, 1)}

of lR 3 and

12

13' = { z.v1 =

'6<'1;;;

J-1-v~
~ dx

f(x) dx, where f(x) is as above.

linear

~ ~ ~'lfe, ~ ~ Pn Cllffif<il<'h B&llaTI 'R "flO


< n ~ ~ <iiG<i'l ~ fil'l~ t, qm ~? Pn ~ l>'4id{UI
~ ~ ii fir.fl<~ 31 Cl<'h<:1"1 ~ ~ e'lm <P-IT ~? k ~
31 <I <'h<:1"1 <#ft.~ ~'lfe <P-IT ~?

ll

is Riemann integrable m the interval


[0, 2), where [a.) denotes the greatest
integer less than or equal to a.. Can you
give an example of a function that is not
Riemann integrable on [0, 2] ? Compute

28

+A~

12

of lR
(b)

12

~? ~

(c)

12

[ P.T.O.

(1, 0), w

(1, 1)}

Also tmd T(x, y, z).

20

Show
that
a
box
(rectangular
parallelopiped) of maximum volume V
with prescribed surface area is a cube.

20

5
Show that the plane 3x + 4y + 7z+- = 0
2
2
2
touches the paraboloid 3x + 4y = lOz
and find the point of contact.

20

C-DTN-K-NUA/46

(s)

R@~<;
'fcl> w;oc-1
x + y- 2z = 3
i'l<1if>
x 2 +y 2 +z 2 -x+y=2 <iT ~'i'<ll 1 i\; VI if

3.

{a)

if>li!:dl ~ ~ <ffi >fi<'Jif> "if>T f<<flif>(Of ~ <f>l~<;

~eii <rc:

('f)

VI 1l:'f>

~VI

ti'Rn t

12

(b)

R@;;<; Fl> 4i<'l"l


f(x)

= [x 2 ] + [x -11

'!?'''*'

Cf>Ji <n

<hlRl<;,
2.

(ct;) +rR

~ f(x)

Rl~<;

aqirtdtm ~ 1

M = (

31ilt<fl>t

'!.<'41d{uf T: lR 3 ~ lR 3 ~ chl~Q;, ~e!il

13' =

~JR3
~ =

{w1

= (1,

0), w

= (1,

Show that every sphere


circle

x 2 +y 2 -2ax+r 2 =0, z=O

20

4.

(a)

(i)

{v 1 = (1, 0, OJ, v 2 = {1, 1, 0), v 3 = {1, 1, 1)}

<iT oft

(ii)

20

R@;;<; Fl> f.'t<la 'lBt<l <it::~'li<'l crrJ ~ 311<ld"l


v CfiT 1l:if> 'i.<h <311 "l a 'li<'l <hl (1 q i a<"' <<t><'l > 1l:if>
<:A

tmn t

'*'

determine

Let T be a linear transformation


from a vector space V over reals into
V such that T - T 2 = I. Show that
Tis invertible.

20

3x + 4y + 7z +- = 0
2
2
2
4W<'l"l'>l 3x + 4y = 10z <iT ~ <h(dl ~ ~
~-~ ~ <filRi<;l

20

[ P.T.O.

C-DTN-K-NUA/46

In
the
n-space lRn,
whether or not the set

is linearly independent.

chl~l{l

.a

through the

cuts orthogonally every sphere through


the circle

1)}

am:m: i\; Bm T CfiT 3110<(_<:1 ~I T(x, y, z)

("&)

(c)

'tfulqo;

'fcl>

20

12

~I

20

Let D be the region determined by the


inequalities
X >0,
y > 0,
Z<8
and
z> x2 + y2. Compute

Jff 2x dx dy dz

(0, 2] if tl'"lH-El'llif><'l=ft'4 ~. ~ [a.J, a. ii


"eih 'll'l 'li''d'l
f.'tf4e <Mdl ~ 1 <f!IT 3lN
1l:if> i)jt 'li<'l'l CfiT 3<!1(<{01 ~ <i!if>" ~ ,.j't 'fcl> (0, 2) 'R
tl'lH 'llif><'l=fl<l ~ ~? J~ f(x) dx "if>T 4P:<h<'l"l
3HHI<'l

Let A and B ben x n matrices over reals.


Show that I - BA is invertible if I- AB is
invertible. Deduce that AB and BA have
the saJne eigenvalues.

{b)

If f(x, y) is a homogeneous function of


degree n in x and y, and has continuous
firstand
second-order
partial
derivatives, then show that
(i)

x af +yaf =
ax
ay

C-DTN-K-NUA/46

nf

20

3.

(<f>)

(<9)

f<l6 A 3fu: B <I lli<lfc! 6fi e@'lt3TI 'R n x n


3110<[15 ~I K<91~<; f<l6 I - BA ~qofl<l t, ~
I- AB OI:J>df>40fl<l t 1 f.'tl4'1 <t>lM!J> f<l6 AB 3fu:
BA $ ~lllH ~e.1f016fi "4R ~I

>iRT

f<l6
3fu: z >

-.:rR1

x 2

3'1flm<t>I3TI

+y

fff 2x dx dy dz

x > 0,

IDU

y > 0,

20

20

Z <

(ii)

(c)
I

20

"6fil qf{Cf>cl'l 4>\MIJ>I

Find
the
vertices
of
the
quadrilateral
formed
by
the
generators of the hyperboloid

(TI)

K1l~IJ>

x - +y 2 - z 2 =49
4

f<l6 V'!
x

skew
four

+y

2 -

tmn -s.:m ~

2ax + r

i\ci<t>

= 0,

passing through (1 0, 5, 1) and (14, 2, - 2).

z = 0

V'!

20

Section-B

x2 +z2 =r2, y=O

5.

Attempt any five of the following


(a)

Consider the differential equation

y'
{e 1 - e 2 ,e 2 - e 3
IJ>6fi"''l<1d' f<ld?l

,,en-l

-en,. en - e1 }

{i)

f<l6 r .. lli<IRI<t> @1011311 'R v:<t> fl R'!l ft4fe


v ~ v -ij v:<t> itm 'Utliifi ('<4id{01 i f<l6
T - T 2 = I. K11~1J> f<l6 T OI:J>df>4ofl<l t I
20

~ f(x, y),

x .3fu: y -it n ~

i 3fu:

~flit>
3'1 i ~ 16fi 3'1 q <t>cl"l ~ ,
<ficl'l

(i)

x iJf + y iJf
iJy

ax

C-DTN-K-NUA/46

"6fil

-if;

= nf

if
<P{x)
is
any
solution
ljl(x) = <P{x) e- a.x,
then 'lf(x)

and
a

lS

constant;
(ii)

v:<t> fl4"'11<1

"ffirn ~ ~ firdl<l ct?IQ


<IT f<l @I 'it!; f<l6

X>

where a. is a constant. Show that-

<rr "'~"iff I

fiiJ -.:rR1

(<9)

=X,

(b)

if a. < 0, then every solution tends to


zero as x ~ oo.

Show that the differential equation


(3y 2 - x) + 2y(y 2 - 3x)y' = 0

admits an integrating factor which 1s a


function of (x + y 2 ). Hence solve the
equation.

I P.T.O.

12

C-DTN-K-NUA/46

12

(c)

Find

f r for the curve

-;Jo

---+

-j>

---7

r (t} = a cost i + a sin t j + bt k

{d)

20

('T)

3ffi

{1 0, 5, 1}

(14, 2, - 2)

:X'>~{d

3l Rl q (q <"'1"1 "'

x2

if> -..m: "'+t-.1

IDU

+y 2 - z 2 =49

oR ~ fEi "111 d <'.'fl;q ~ if> "l!M

W0 <f>l ~II> I

If v 1 , v 2 , v 3 are the velocities at three


points A, B, C of the path of a
projectile, where the inclinations to the
horizon are a, a -13, a - 21> and if t 1 , t 2
are the times of describing the arcs AB,
BC respectively, prove that
1
l
2cos 13
v 3 t 1 = v 1 t 2 and + -- =
vl
v3
v2

12

12

20
(e)

Find the directional derivative of

f(x, y) = x2y3 + xy
at the point (2, 1) in the direction of a
unit vector which makes an angle of n/3
with the x-axis.
(/)

y' =ax, x>O


~

(i)

ex ~

i,

'R Fcl~ R <hl~ 11> l

R- q,(x) ~ ~ !rcl i 3ffi 'lf(xl


ci'l

(ii}

3'R!<:

'If( X) ~

3'R!{

Show that the vector field defined by the


vector function

R1 liN ffi--

---+

----7-

-;Jo

V = xyz (yzi + xzj + xyk)

= <P{x) e- a.x,

1s conservative.

~;

~ ex < 0' ci'l ~ !r<"'1 ~ "<f>l

6.

3ltt >lVI mor

~~m x~~-

12

(a)

12

Verify that

1
1
(Mx + Ny)d(loge(xy}) +- (Mx- Ny) d(loge(x}j
2
2
y

=Mdx+Ndy

("IS)

R@t;<{-% 3!Cl<h<?l (i<ft<t>(OI

Hence show that-

(3y 2 - x) + 2y(y 2 - 3x)y' = 0

"<tiT~ fll41<hcl"i !!OI<fi

zrtor t

'h<?l'l ~I 3f, fllfi<f>{UI

C-DTN-K-NUA/46

Cfil

12

~-%(X + y

!r<"'1 <f>l~!(l

{i)
)

"<tiT~
12

I P.T.O.

if

the

differential

equation

M dx + N dy = 0
1s homogeneous,
then (Mx + Ny) is an integrating
factor unless Mx + Ny = 0;
C-DTN-K-NUA/46

10

(ii)
--+

--+

-+

+ a sin t j + bt k

r {t) = a cost i

~ ~ K/-r mn

v 3 t1

(:s)

fdx y) y dx + f

f(x,

v3

Yl

= X

<ti't mn cbll'ilq; 1

R11~11> f<f;

'hcl"l

eRlll

-+

-+

Prove that
->

12

"itm t

->

div (f V) = f(div V) + (grad


where

7.

--+

(a)

12

(q;)
1

e<"'l!H cblM<~; f<f;

= Mdx+ Ndy

(b)

31<1: R11~"> ~

(i)

">fR

3l<l<t>cl

"lElld
'>!<if

t, "ffi (Mx

(ICf;

C-DTN-K-NUA/46

e>fl'f>(O!

+ Ny} l!;'fi

Mx + Ny
11

M dx

=0

+ N dy

=0

>A"ll<t>cl"' TJI<ti

t ;

Show that the set of solutions of


the homogeneous
linear
differential
equation

p(x)y = 0

A
particle
moves
with
a
central
acceleration !-l{r 5 - 9r), being projected
from an apse at a distance .J3 with
velocity 3-./(2!J.). Show that its path is
the curve x

[ P.T.O.

20

on an interval I = [a. b) forms a vector


subspace W of the real vector space of
continuous functions on I. What is the
dimension of W ?
20

(Mx + Ny}d(loge(xy}) +- (Mx- Ny) d(loge{xl)

->

f) V

f is a scalar function.

y'

6.

20

(c)

12

V = xyz (yzi + xzj + xyk)


IDU ~ R"!I l$ m~

= 0.

A particle slides down the arc of a


smooth cycloid whose axis is vertical
and vertex lowest. Prove that the time
occupied in falling down the first half of
the vertical height is equal to the time of
falling down the second half.
20

v2

--+

(x y) x dy = 0

(b)

+ xy

~ ~-3l<l'f><:1"1

factor unless Mx - Ny

2 cos (3

+-

equation

then (Mx- Ny)- 1 is an integrating

~ x-31~ il; ~ n {3 cm
~ {2, 1) 'R

eRlll <fit,
"'"li<H t, "Rlffi -q

v;<ii "ll::l'fi

("f)

_.....__

~~~ vl

differential

of the form

cbll'ilV>I

= v1 t 2

the

12

~ v 1 , v 2 , v 3 v;<ii >1~ ~ ~ ~ <IH ~an


A, B, c 'R WI ~. ~ la.iRt"' it 3lHRt4i a:,
a: - ~. a: - 2~ ~ 3fR ">fR t 1 , t 2 liii!tl: ...-rq AB,
BC~ f.\qful <fiG iJ; ~~.at~ cblMt!; ~

("f)

if

M dx + N dy = 0 is not exact but is

--+

C-DTN-K-NUA/46

+y

12

= 9.

20

{ii)

...-R:- 31<1'fi<:1 eifl'fi:co1


Pot '"'*<:1 <ft'fi "'l1ft ~ ~
f 1 (x y) y
it> ~

dx +

1'i ~. <fl

(c)

Use the divergence theorem to evaluate

If v-ndA

{Mx- Ny)Ny

x dy = 0

2 (x y)

!!OI'fi ~ "1<f <fCI> Mx -

(~)

M dx + N dy =

=0

--+

Vllhere V = x

~ etll'fi<:1"'1

-:r@

tI

-+

--+

-+

zi + y j - xz 2 k

and S

is

the boundary of the region bounded by


the paraboloid z = x 2 + y 2
and the
plane z =4y.

20

~ <f;OT ~ Rl<ti'l ~Sfi'>l -if;~~ 31'R -;fR <'t{<t><11


t ~e'fit 31~ ~m ~ 3ltt :m<f e01~ -;fR ~ 31'R
~I ~ chl~<:t ~ ""lit 3>~1{ -if; "SI'lfli 3llit <IC!>
-;fR ~ 1'i- WTT fl4<l, ~ 3llit <1C!> -;fR ~ l'i<.'{i'r <"14"1 if; 01{!01( %I

8.

(a}

20

Use
the
method
of undetermined
coefficients
to
find
the
particular
solution of

y" + y =sin x + {1 + x

)ex

and hence find its general solution.


->

->

->

div (f V) = f(div V) +(grad f) V


~

7.

(q;)

{b)

~ 31fu~t 4><:1"'1 %I

20

fu!1!<1.!; ~ ~ 3ictU<:1 I= [a. b] 'tR <"14"lld 'tf@'h


31Cl'fi<:1 eifl'fi:cot

y' + p(x)y = 0
-if;~ it> flt!O'iil'l, l"R

'riaa 'b<:'lofl -if;

e4~-$~eR~13qeq~

<m<il.~'fi eR'll

w<i1 Ol'lldl~l

tan <!> = - + tan


8

~ ~ ~ W! 3..j{2Jl.) -$ m~
~ .,.,-A "" ~ <f;OT it>rs{l'l ~(01
J.l(r 5 - 9r) it>~ ~<:1dl ~I R@l$1.!; ~ ilfl'fil ~

../3 ""

20

<rsfi x4 + Y4 = 9% I

13

Verify Green's theorem for


e-x sinydx+e-x cosydy

;r~

C-DTN-K-NUA/46

20

(c)

(-.s) ~

20

A solid hemisphere is supported by a


string fixed to a point on its rim and to a
point on a smooth vertical VITali Vllith
Vllhich
the
curved
surface
of the
hemisphere is in contact. If 6 and <!>are
the inclinations of the string and the
plane base of the hemisphere to the
vertical, prove by using the principle of
virtual Vllork that

w~~

~t?

20

20

[ P.T.O.

the path of integration being the


boundary of the square Vllhose vertices
are (0, 0), (n /2, 0}, (1t /2, :n /2) and (0, 1t /2).

C-DTN-K-NUA/46

14

20

-- - -

('T)

---------------------,

fJ V rt dA <FiT -.:rA f.l'f>l<"'<i if> ~ 3jqft{UI ~


s

"ffli V = x
2
z = x 2 + y

<FiT aq<ill <h!Fi!l{,

3i'tt S q{'l<"'<l"'
il qft<ilO&: ~ -<hJ
8.

(q;)

--+

zi + yj - xz
<NT <:14d<"'

z =

2-+

4y

qft'!q I ~I
2

y" + y =sin x + (1 + x

C-DTN-K-NUA

20

}ex <1>r f4iJq t:<1 ~

Cfi8 if; ~ 3l f.l to~ffi:d T' I jq; f4R:f <FiT


3i'tt m ~<:lib I Olllq<f; t:<"' ~ <h!Fill(l

34<ll'l

1'f?lffi

<hi fill(

, 300

20

(<9) ~ im ilatd, ~

1.fi (tT<"' 'R ~ ~ il ~


~ "i\ti il 3i'tt ~ Rl<b4l ~ <!l<m: 'R ~
~ it fW:f 'R
a1>t <FiT <~Sf> 'l!J q it ~, ~
g"lf ~I ~ 6 3i'tt $ "it{} ~ fl<11tl if> <:14d<"'
<>llt:~R if> ~ il 3li'IRI ~. -ffi <bl<'<l<l ~ if>
~0&:1"<1 <~>r aq<ill ~ ~ fuos: <hlfill!> -fu;

e <t

3
8

20

tan<j)= -+tan6

(TT)

"'l7f,

3i'tt
<1

31tdRm 3F<1 I<#; #I 4IUl7l

Fil~<h ~ (0, OJ, (n 12, OJ, (n 12, n I 2)


(0, 1t 1 2) ~. -<hi qf1:lqt <ffi ~>il<b<"'"l <FiT <N
~ e-x siny dx +e-x cosy dy if> ~

!fR-~

<FiT t'l('lll q"l

<fil fill!; I

m &41 3'iR 3iiMI <ir.if if f:Pff ~ 1


JINJ it> ;ffl"{ "Jtft 41t214 if IM& ,-f.{ >iir&~, f.5toq;i ajJ&
31Nii> uiht ~ it fil>211 'PH~. 3'iR w ~ <~>r ~
ajJ& ;ffl(-!.JRt<i> it> ![&-'PI <n: 31-fitrt F-if.fe ~ <n:
fil>211 'ffRT "11&~ 1 uJ111 ~ <n: 3&if&a ~ iii
u<'itq;

m ti&n
20

<n: qitf

:Wfi

~ ~

<!/tat~ I

51<'4<i>

***

<i1>

~ 311<r.l21<i>
3"1</i)

f.'lfife

"R1r!: f.'l21rt :Wfi m iii 3irt it ffr:l: 71<! if r


it, rtt

Note :
BS--900

d48'di 31i<i><if q;r "121'1 <liltat~

(fPff

<t)f}f~ 1

Udl<i> jtiii>rl U"'fMrl

15

;ffl"{

s 31f.'i<w4 if 1 'illcfJ u:toif if it uRJq;


&u.s it enrr-it-enrr ~ m "!j'l<i>< !<#;-iff rtr.r JIJ(;ff <i1>

;ffl"{

C-DTN-K-NUA/46

it A & 71<!

;ref! {qdl/1

3T"if if

U&'di

if,

31'""4211

f.'/ fife

gI

English version of the Instructions IS


printed on the front cover of this
question paper.

CS {MA\N) EXAM, 2010


Sl. No.

253

C-DTN-K-NUB

MATHEMATICS

Section 'A'

1. Attempt any five of the following :


(a)

ITime Allowed : Three Hours II Maximum Marks : 3001

Ullrt

~: at:t~~tif

C>r

'R~~I

F$r+::1

(j>qlifH

~ Y~trr-rpr i1> fil@il ~

(c)

Discuss the convergence of the sequence

{x,}

sin(';)

where x" = _

(d)

Define
X 11 +t

{x11 }

12

__:,_8__.:::__,;_

by x 1 = 5 and

.J4 +xn

for

11

(e)

12

> 1.

Show that the sequence converges to

Assume suitable data if considered necessary


and indicate the same clearly.

All questions carry equal marks.

Show that a cyclic group of order 6 is


isomorphic to the product of a cyclic group of
order 2 and a cyclic group of order 3. Can you
generalize this? Justify.
12

Candidates should attempt Questions 1 and


5 which are compulsory, and any three of the
remaining questions selecting at least one
question from each Section.

Symbols and notations carry usual meaning,


. unless otherwise indicated.

be the set of all real numbers

(b)

Answers must be written in the medium


specified in the Admission Certificate issued
to you, which must be stated clearlJ on tire
cover of the answer-book in the space
provided for the purpose. No marks will be
given for the answers written in a medium
other than that specified in the Admission
Certificate.

= lR- { -1}

omitting -1. Define the binary relation * on G


by a* b =a+ b + ab. Show (G, *) is a group
and it is abelian
12

Paper II

INSTRUCTIONS
Each question is printed both in Hindi and in
English.

Let G

(1 + .Ji7) .
2

Show that
u(x, y) = 2x- A3 + 3Ay2 is a harmonic function .

. Find a harmonic conjugate of u(x; y). Hence


find the analytic function f for which u(x, y) is
the real part.
'12
C~DTN-K-NUB

(Contd.}

,--

--

(f)

. .

(<i>) -I <fiT rn~<fi>:: ~ <H!M<fi fi<t><llaiT ~ e~;q

em

G. lR- { -1} <'i'IMq; I G 'R i1"

fiW * <iT

= a + b + ab GRT ~

*b

<til Mll; 1 Kl..'ll'till;


. (G. *) ~ tl'f~ ~aiR~ 3\liil<'i'l ~I
12
(<..<J) R<..ilr'tiq; fci> q,')Q 6 q;r ~ "''shl4 e'f~ q,'JQ 2 ~
~ "'1 l q e 'f\? aiR '*'I R: 3 ~ ~<fi"'1 sh1 q tl'f~ ~
a

2. '(a)

'JOI"''h<'f ~ <;f'<llCf,(Q ~. ~ I 9<IT 3!1'l 'tifl<fil

a:r 14chl<fi>::Ul

<n<:: fl<fid ~

(<I} 31'j5fi'"l {x,.}

cfu -~ -cfu

. (nn)

.~

? ai'l Ml:ll <ilctl'ti~

Xn

(<I) {x,.}

sm T
=
. 8 .

12

12

<fiT x 1 = 5 aiR

n"> 1 ~~x,+l

R<..ill "'~ fci>

=.J4+xn ~~chlMq; I

3l '.!*''"~

Let ( IR*,
zero reals
plicative
matrices.

)be the multiplicative group of nonand (GL (n, 1R), X) be the multigroup of 11 x n non-singUlitr real
Show that the _quotient group
GL (n, ~)/SL (n, 1R) and (IR~: ) are tsomorphic where

SL(n, 1R)={AEGL(n, IR)/detA=I}.

f44"'1'11 <lilM~,

'

Construct the dual of the primal problem: .

Maximize z = 2x 1 + X:z + x 3 , subject to the


constraints x 1 + x 2 + x 3 :;:.: 6, 3x 1 - 2x2 + 3x3-3,
-4xr + 3x2 - 6x3 = 1, and xl' x 2 , x 3 :;:.: 0.
.12

What is the centre of GL (n, 1R) ?

(b)

Let C ={/:I= [0, l]- IRj/ is continuous}.


Show C is a commutative ring with 1 under
pointwise addition and rnu ltiplication.
Determine whether - C is an integral domain.
Explain.
15

<fitl ~ I

12

(c)- Define the function

( s-) R~m,~ fci;

u(x, y) =

2x- .;~.3 +

<ma Pi q,

C-DTN-K-NUB
.

'

f(x)

3.\.y 2 ~ 9ti<w~1 'h<'i'i ~ I

B':f''il IDff <hiM<:( I am:


cf>JM~ M'9Cti ~ .u(x, y)

"q[lf

mer

t- 1

12
3

(Contd.)

2 . I
"f
x sm x 1 x

= 0,

u(x, y) q;r ~ gfi<w;l

"fcr.t(:jfqq; 'h<'l'i

15

if

X=

0
0

Find f'(x)._ Is f'(x) continuous at x Justify your answer.


C-DTN-K-NUB

0?

15

(Contd.)

'
('f) 3fRI *Ill~ I :
OlM<t>d>i'lif><:ur chlMl!; z = 2x 1 +x2 +x3

(d)

<fiT

Consider the series

~il;~JFr

x 1 + x 2 + x 3 ~ 6, 3x 1 - 2x2 + 3x3=3,
-4x 1 +3x2 -6x3 =l, aft<: x 1,x2 ,x3
Yffiflllt=<ll ~ ("i!'il chlM(( I
. .i

~o:~
12

"

z. <"'*'> m'i' <'ill'ilo:

(IR*. ) >Q<ii<R crrfdf4<t>

<hT <jOFll<ll<h fl"[l:i ~

aft<: (GL (n,

Find the values of x for which it is convergent


and also the sum function .
. Is the convergence uniform ? Justify your
answer.
15

e<i>lum

3. (a)

lR). X)

n x n a;]O&illOflzt Ci1Rtf4<t> 3lle>!f<~'i <fiT <JUI>ik'l<h

*l"[l:i ~ I IG<iU~((

fit 14">11'1 fl"[~

aft<: ( IR*,

GL (n, IR)/SL (n, IR)

) <j<.:!t<t>IU

(b)

Show that the quotient ring z [i]/(1 + 3i) is


isomorphic to the ring Z tlOZ where z (i]
denotes the ring of Gaussian integers.
15

(c)

Let f,,(x) = ~ on -l < x.;; 1 for 11 = 1, 2, ....


Find the limit function. Is the convergence
uniform '1 Justify your answer.
15

~~
SL(ri,lR)={AEGL{n, iR)/detA=l} ~I
GL (n, JR) <hT ~ C!<IT ~

() <'i'lMq

c = {f: I= [0,

K<ill~ ~T:

15 .

I]~ IRIJ fi<ld ~}

<i't7f aft<:

*I ~a iii li 14 Pt il zt cr <1<1 ~

3!Ur.r c ~ I
1 f.ruRur <fil Mq fit c

<J>U1'1 il;

~ '{ozfchlzt ;ria ~ I *l"'lH 1~2l I

('lf)

'<1"1 ~

j(x)

zrR X =

ma chlMl!; I <fliT X= 0 tR
3!G<I. -d'<l< q;r af1Gw01 ii!dt\ttct I
J'(x)

C-DTN-K-NUB

j'(x) fidd ~

15
(Contd.)

) (-x2-y2)/2

the surface Z = x 2 - y 2 e

4. (a)

2 . I
_c_
= x sm-.
"'!G
. X

= 0,

(d) Find the maxima, minima and saddle points of

15

4ilN1q

Consider the polynomial ring Q[x]. Show


= .x3- 2 is irreducible over Q. Let I be the
ideal in Q[x] generated by p(x). Then show
that Q [x]/1 is a field and that each element of
it is of the form a0 + a 1t + ai2 with a 0 , a 1 a2
in Q and I= x + 1.
15

p(ic)

15

(i) Evaluate the line integral Jf(z) dz where

c
f(z) = z2, cis the boundary of the triangle
with vertices A (0~ 0), B (1. 0), C (1, 2} in
that order.

(ii) Find the image of the finite vertical strip


R : x = 5 to x = 9, -1t ~ y ~ 1t of z-plane
under exponential function.
15
C-DTN-K-NUB

(Contd.)

() ~ 'L ( .
x

n=O

l+x 2

~ +wr

~ f4~1Rl!> 1

)"

f(z) =

affi::

(<a) K'ill'a~ fcr; 14"'111 Cl<'l<l

it <j<'lllct>l(l

~~

<if ~Rld Cf>(d I ~ I


(>r) <'llMt:; n = 1, 2,
~W=~

e1&~ 1 'h<'i'1 m
~ 'd'd'( q,r

() 'rB' Z =

affi::
4.

M'14> ~ ~ ~ ~

m ch\N1q;

~ 'fi<'i '1
m chi N1 q; 1 C!<IT ~
l!;<fi'+t 1'1 ~ ? ~ 'd* q,r ail Rl <""~ "l a 1'4l!> 1 15
(<:K) "lq<: <'h'l<l Q[x] +rR <"flMq; I K<.<ll'al!; Q 'R
.p(x)=x3-2 <>l<'1''(Uflq ~~ +wr <flNiq; I
p(x) it '>!Ad Q[x] it "J"I'>II<t<it ~LID fu@~ Fs
Q[x]/ I l:% l$r ~ affi:: Pcs '4'6"f>l >J~ct> <>t<l<l<i a 0 ,
a 1, a 2 Q ifaffi:: t=x+l4>'~ a 0 +a 1t+a2 t 2
<'!"=f q,r ~ I
15

3.

(b) Find the Laurent series of the function

x2

oo

( <:K)

;E

[iJt(l + 3i) <l<'ill Z/lOZ

(iJ ll&tflll fi<i'lllaiT 4;- <i<'ill


15

4->' ~

....

-I <x ~ 1 'R

.
cfil N1 c; t C~<IT ~ "l!>""<t>""'~"~~''1~ ~ ?

:mPl<'1

"l<ll"al!; l

(x2- yz) ~-x

q<'lliUt.f<ii"Sa?f

(i) ~ fl"' lct><'i

4;- ;afue, f.ifi:i1"6

<if m <h'lMt:;

Jf(z) dz

f(z) . z2, c

-y )I

15

lifl'm' A

<e.~ q~c:.').

where C,

z-'1.1+td<1

<fit qf.<fil<l

(Contd;)

<I<:!< oo

= .!._
J:>rcos (n- A. sin )d,
n o

n = 0, + 1, + 2, .....

with A. a given complex number and taking the


unit circle C given by z = ei~( -n "i tiJ""" n) as
contour in this region.
15
(c)

Determine an optimal transportation programme so that the transportation cost of


340 tons of a certain type of material from
three factories F 1 F 2 , F 3 to five warehouses
Wp W 2 , W 3, W 4 , W 5 is minimized. The five
warehouses must receive 40 tons, 50 tons,
70 tons, .90 tons and 90 tons respectively. The
availability of the material .at F 1, F 2 , F 3 is
100 tons, 120 tons, 120 tons respectively. The
transportation costs per ton from factories to
warehouses are given in the table below :

WI

Wz

w3

w4

FJ

Fz

F3

(0, 0), B (1, 0), C (1, 2),

R : X = 5 it X = 9 O<t>, -:n ~ y ~ 1t CfiT


gffif<ilkl m cfilf';]q; 1
15

C-DTN-K-NUB

for 0

<t>T +tT'1' Act> I R>l c; ~

* anftrr

.n

15

~Sfi+fif, ~~<fitqfw1'lr ~ 1
(ii) '''R"'Idichl 'fi<'l'1

exp[~( z- ~) J as L~~ z"

Ws

Use Vogel's approximation method to obtain


30
the initial basic feasible solution.
I

C-DTN-K-NUB

(Contd.)

'.
(~) 0 <

lzl <

f(z) =

~ ~ 'liM'i

oo

Section 'B
S. Attempt any five of the following :
(a) Solve the PDE

exp[~( z- ~) J cfiT <>fl>J i?<uft , 1:=5 z"

~~if m

ch'1Nil!;;;@ ~<ft ~ flfiil~;nj{;.q, A.


~ m'<l aiR z = e i</J ( -:n: ~ ~ n) if ful: +Jir C!!'f'i Cfl
<[U C CflT ~ &'t?r if qf<{<..<i I it; ~if ~ ~
C, _!_

n:

J"'0 cos (n4>- A. sin q'>)d,

n = 0, +I, + 2 ..... .

(D 2 -D')(D-2D')Z=e 2 x+y+xy.
(b)

it t:(fq <fl<;i+\1 W 1 W 2 , W 3 W 4 W 5
GCfl' i'l' ;;rf.f if q f<:<! \5'1 <WI C1 rq:_<1 C1 '4 i?t I ~
<fl<;l:fi if ~T: 40 C<f, 50 C<f, 70 C<f, 90 C<f aiR
90 C<1 1='fffi 4l:i'ii'11 -<ilf%1:!; I F~. F 2 , F 3 'R 1='fffi <fiT
Fp F 2 , F 3

91'4C11 Sh"f1iT:

'hifi!f.c41 it

.H+"'i

f!ROfl

100 C<l, 120 C<f, 120 R

fl<;li'f GCfl'

if <ft

cfiT 9fu

(i) Suppose a computer spends 60 per cent


of its time handling a particular type of
computation when running a given
program and its manufacturers make a
change that improves its performance on
that type of computation by a factor of
10. If the program takes 100 sec to
execute, what will its execution time be
after the change ?
(ii) If A67B=AB'+A'B, find the value of
xEElyEElz.
6+6

(e)

A unifoim lamina is boun~d by a parabolic


arc of latus rectum 4a and a double ordinate at
a distance b from the vertex.

t :
.

Wz

w3

w4

Ws

F
I

F2

F3

am:rrU ~fi1a '<F'f gn:r Cfl<:<l ~ ~


cfiT 'H 8r Cfl61 f4 fa CflT 'd'"! <ii. I ch1 M l1> I
C-DTN-K-NUB

lOxe-x -1=0
correct up to 6 decimal places by using
Newton-Raphsori method. Carry out compulations only for three iterations.
12

(d)

C<f qf<<~<Qrt <'ll'ld

wl

Find the surface satisfying the PDE


2
{ D - 2DD' + D' 2 )z = 0 and the conditions.

that bZ= y2 when x = 0 and aZ = x2 when


y == 0.
12
(c) Fincl the positive root of the equation

15

(or) ~ <:<<>c:a'"l qf<<it;rt !ll{ll'"l f.ialf<a ch'1Ml!; M'Hil


fit f4ifl1 ~ 9CflR it> 340 C<l 1='fffi Cf>T cft'1 <h<tc:k<iY

12

If b = ; ( 7 + 4-/7), show that two of the princi-

cf!l<'l
30

(Contd.)

pal ,axes at the end of a latus rectum are the


tangent and normal there.
12
C-DTN-K-NUB

10

(Contd.)

-------------------------------------

........ <

l!f

@06

(f)

In an incompressible fluid the vorticity at


every point is constant in magnitude and direction; show that the components of velocity
u, v, w are solutions of Laplace's equation.l2

6- (a)

Solve the following pru1ial differential equation


zp+ yq =X
x 0 (s) = s, y 0 (s) = l, zt!(s) = 2s
by the method of characteristics.
20

(b)

Reduce the following 2nd order partial


differential equation into canonical fortn and
find its general solution
ry
xu= +2x-uxy -ux = 0.
20

(c)

Solve. the following heat equation


u 1 - uxx = 0, 0 < x < 2, t > 0
u(O, t) = u(2, t) : 0, t > 0
u(x, 0) = x(2- x), 0 ~ x ~ 2.

s. M"""1RIRl<1 i'i- "it Pct;'ifl'

qj,. ~ '3'<1< ~IRtt:t,


31<lCfi<'l H'ilCfi<'ll (PDE)

(<il) arif..il<f>
(D 2

(<9}

D') (D- 2D')Z-

x+y

CfiT ~

MctilRIQ> 1

arif..il<f>

31<\Cfi<'l HtflCfi(Ot (PDE)

{D

- 2DD' + D'

+ xy
12

)z = 0

aft<: 9 RJ.,fUf Fn bZ = y2 ~ x = 0 aft<: aZ = x2 "!<if


y = o <it~ cti<<i ~ '!'SO <it mer 4i'l M Q; 1
12
(11') 4'&1-{q~H Mfil CfiT ;aq4l'l <f,(Ch esf!Cfi(UI
.

!Oxe-x -1=0

~ 6 G<~lil<'l<t ~ '<rq> ~.!'\U

(~)

wr

<it mer

4i'!RIQO 1 <i><l"'' ffi-r 'i"~<11Rl4'i <fq> ~ ~


4i1Ml!> 1

12
(i) l'fR 41RIQ> -Fcn ~ cti"'i.c:< ~ ~ f.m ~
si"Hlt"l <it ... <'11 (I ~ tJ>Cfi m)l;r 9 <iiI( ~
31MCfiWI <it Cf0l if arq;f fl"'l! CfiT 60 9Rll!Rr
~ *" Wild 1 ~ aft<: -.;e 4> f.1"' f<u ~ q f<4 J "l
<ll<d ~ fiime ~ ~ ~ ~ <R 1;eih
CfiTlf ?i 10 ~WIT<~ "lid I ~I <rR Sl!ll"'
HtS41~"l ~if 100 e&>6 i'Rrr ~. ffi" qf.<:Cld"l
~ <illG 1;e <til f.\ osq 1~"l e "''ll <~<IT ~ ?
(ii) ~ AeB=AB'+A'B, ffi"
X$ y $ z CfiT l'fR ffiO 4i'l f'>lt:t I
6+6

(s:} ~ t:t<iifl"''l"l q(!<'l ~ 4a ~ q'(Cl<'lf2i<ii 'Tfl'1


aft<: .l!Wf il"
b <R ~ f@:cti1R: il" qf<il'{a ~I

<tt

<rR b
~

;(7+4-/7) ~. mR<:.<~I'NFnl:%"~

rn< <R ~ arm if e- ..rr <:>=rl!f oo aft<: ~ <R

~~~
C-DTN-K-NUB

11

12
(Contd.)

20

Given the system of ~quations


2x+ 3y= l
2x+4y+ z=2
2y + 6z + Aw = 4
4z+Bw= C
State the solvability and uniqueness conditions for the system. Give the solution when it
exists.

20
(b) Find the value of the integral

1. (a)

J log 10 xdx
1

by using Simpson's t-rule correct up to


4 decimal places. Take 8 subintervals in your
computation.
20
C-DTN-K-NUB

(Contd.)

12

L__ _ _ _ _ _ _ _ _ _ _ _ _ _ _ _ _ _ _ _ -

(c)

('f) fct>e1 "lfi<fl<t<~ d<<1 if 'l"!PlW11 ~ ,f.il>s 'R


qf<liiur aiR furr if C(Cfl~lil'i ~. R<,ll\ii<J> f<h" im ~
"l<!Cfl u, v, w <11t<1H:i flli~ICfl(OI ~~~I
12

(i) Find the hexadecimal equivalent of the


decimal. number (587632) 10
(ii) For the given set of data points
(xl,f{x1 )), (x~,.{(x2)), ..... (x,.,f(:X,))

.<P+yq=x
x 0 (s) = s, y 0 (s) = 1, z 0 (s)

( <R)

= 2s <!iT~ <hlMC( I

Pt 1'41 Rt rna fuq,1R ari"f.il'f>


x U..x + 2x 2 uxy- u, = 0

6l <l C!l<'t

am:

(<R) Hl&4fl'1 $' Rll'51{ Hllli


5

J Jog!O X dx

Cjl"f

!t

20

m.
20

'344'11 IDU flliiCfl<'l

4 ~lli<1Cl ~' <fCjl" m~ JfFf

ID<r <hl 1\ii <J> 1 ~ aiT>!C!lct'i

<'111\iiq:
C-DTN-K-NUB

the

a +a'b + a 'b' c + a 'b' c 'd + ......


x'y'z + yz + xz

where x' represents the complement of x.

8. (a)

6l & dl<:m I 9 Rr iii

cit iiidi'$.C( 1 ~ <flMC( ~~~~I

{ii)

simplify

5+10+5

(C!l") e..0Cfl<ur HC!llll R<~r ~ ~:


2x + 3y= 1
2x +4y + z = 2
. 2y +. 6z + Alv = 4
H Cfll <l ~ ~ mu='"ll'rfl<l""'d'"l

( '1)

20

u 1 - uxx = 0, 0 < x < 2, t > 0


u(O, t) = u(2, t) = 0, t > 0
u(x, 0) = x(2 -x), 0 :% x:% 2.

4z+Bw=C

(iii) . Using Boolean algebra,


following expressions

<!iT fc!R;a ~if ~li1'1<1'1

aiR lii~<h "'li4C!l ~<!iT ID<r <hlMC( 1


(tr) f.il'i'IR!rna ~ ~41Cfl<or <!iT~ <fll\iio: :

7.

20
~lift Cil'<"l

<hll\iio:

'

. write an algorithm to find the value of


f(x) by using Lagrange's interpolation
formula:

if 8 ;aq i a <r<>i'i <!iT

A sphere of radius a and mass m rolls down a


rough plane inclined at an angle a to the
horizontal. If x be the distance of the point of
contact of the sphere from a fixed point on the
plane, find the acceleration by using
Hamilton's equations.
30

(b) When a pair of equal and opposite rectilinear


vortices are situated in a long circular cylinder
at equal distances from its axis, show that the
path of each vortex is given by the equation
.

(r2 sin 2 e-b 2 ) (r 2 -a2 )

=4a 2 b 2 2 sin 2 9.

being measured from the line through the


centre perpendicular to the joint of the
vortices.
30

20

I
13

(Contd.)

C-DTN-K-NUB

14
'

(i) ~1'4<1Cl ti~l (587632)10 4; ~~..~~~'11'1<1"4

(1f)

I C-DTN-K~NUB

m-ff 4>11'>1~ I

<rn IGI~ 4> ~ ~

(ii)

e lj"illl

{xl ,f(xl )), (xz,f(xz)), .. : .. (xn, f(x,.))


4> ~ <'1'!l i"' aid <f>!rrr- 'I:f 4> ;aq 41r it f( x)
q;r '4N m-ff <Mrt 4; ~ ~' ({<"40 Rut

R11Z<~C(

Hk1R1R<~<~

;;rQi" x' x 4>. '1'<Cfi <if n~+.M"'d"" Cfi<dl

t"

<h\or
1..-R x

6"1<~<1 <4<: ~ H<~<~ ~ it ~ 4> ~f ~ cfiT


. ~M<-C:<i emCfi(UI q;r ;aq4'lr Cfi<4> ~<OJ

.GUt-.
... m

mo 4il 1'>1 tJ; r


(<a) ~

"'I

30

aft<" e "'J:1 ~i.iil'"' 'tl M<'11 q;r ~ <;FT ~


<>fit 1fflll iii wr if t.e 4> 3\~W. it e '"l H <m
... '4<: ~a
R11"1~ Fcfi 9~Cfi '!lM<1 CfiT <w "6""'"~1"'<t>~<o~r

'1

m. m

e -,b ) (r - a
e
it W:rr sm t-. e "'rqr "'rar t- ~it ~"~<Oi <u<oft
~it~ Fcfi 'ilM<1'i iit ~ '4<: ~ t" 1
30
2

(r sin

C-DTN-K-NUB

15

3i'k aiiJ;;fl <il'fl if UC!T ~I


;yift ~ if ~ ;;nit "<llf'f;r<;

= 4a 2 b 2 r 2 sin 2

it>

dfH

d#l& ;;w1ih !Tct~r-q:;r if Pt><~l TfZIT ~.


3i'k w ~7:{l'[ q;r PIT! d#l<.<t dfl< ~t'd<i> it>
~ tR aifCha f.ifif& t"</1'1 'R Pt><~l '41'11
-qtff;r:; 1 srihr-q:;r tR aAf&d 1'1Th7:l7'r it> 31fdRm
arrl:f Pt>HI '1TC7:l7'r if ~ TT<.f dfH w <itt aiCf;

M+l<i>l

t" I

q;r ~ i'l<11 ~Rt"' ~

'4<: ~ ~ ~ ~ 6"1<1<1 '4<: <j<$Cfidf

~liT
!nrif

5+10+5

aft<" 1::<4'41<1 m

: 3oo

9f4q; !n"rf {f;ff

( 1.). a+a 'b +a'b'c+a 'b' c 'd + ..... .


(ii) x'y'z + yz + xz

1 "'U/t<l>

;aq<i'l Cli<4>,
a:i "~ '*1
' <iT e <<'~ 4il 1'>1 ~ :.

(<n) bt'Rif a

3r.!'f- '<l'lf II

(iii) "J,Jlll ;sfl"fl<l<'i't q;r

8.

lfUkt

rrif fitiT '4/l.{i/ I


!n"rf Bi?'/1 1 3ik 5 i! H"fl<f ~ I

iii 1<lil !n<if if it


'j"f<i>{ /Ct;'f/1'
!n"rf
'?""
q;71
itq;+fit
9 (i}q; <ifOS
ffirr !n<i'i it> YO<: rflfll r:; I

l:ffit ~~

c{;)fllr:;

<NT

fit

cit

;jq~ffi 31/<t>iJ' q;r 'q4'1

d'1Chl HUIS2 cfJfJtr:; I

9dl<hl 3i'k eihwi'f it>


3fRT4T

;r <ififJT TfZIT

9oqf<:td

aM ~. ~ crq;

'if I

-.nit !nrif it> ajq; +1Rl"1 ff I


Note : English version of the Instructions is printed on
the front cover of this question paper.

___ ...

.-.....

--

-:-:-~-,-=--~:---1

C. S. {ivlain} Exam : 2011

C. _Jc)

Serial No. (

(c-D TN-L -NUA )

~--=-=-='--'

I.

.l\IATHEMATICS.
Pap er-I
(Time Allowe d." Three Hours )

'IT E9'lT ~I

fPfl' ~ql-d< "W

'IF[-rr :f

(a) Let A be a non-singular, n

[Maxi mum Marks : 300J

n square matrix.

I A I2. In Hence show


X

5
7

, X= y

0 6

10

l(n-I)

1 0 -1
(b) Let A= 3 4

B= 6

Solve the system of equations given by


AX= B
Using the above, also solve the system of
equations AT X = B where AT denotes the
transpose of matrix A.
. d
(C) FIll

lim

x2y

(x,y)-+(0,0)

x3 +y3

10
if it exists.

10

(d) Let f be a function defined on IR such that


f(O) = -3 and f'(x} < 5 for all values ofx in IR.
How large can f(2) possibly be ?
10

~ : ~itm 'Ff

Show that A . (adj A) =


that I adj (adj A) I I A

INSTRUCTIONS
Each question is printed both in Hindi and in Englis h
Answers must be written in the.medium specified in
the Admission Certificate issued to you, which must
be stated clearly on the cover of the answer-book
in the space providedfor the purpose. No marks will
.be given for the answers writte n in a medium other
than that specified in the Admission Certificate.
Candidates should attempt Question Nos. I and 5
which are compulsory, and any three ofthe remaining

questionsselecting at least one question from each


Section.
.The number . of marks carried by each question is
indicated at the end of the question.
.
Assum e suitable data if considered necessary and
indicate the same clearly.
Symbols/notations carry their usual meanings, unless
otherwise indicated
'EJ:l'T'1

SEC.TIO~-.A

(e) Find the equations of the straight line through


the point (3, 1, 2) to intersect the straight line

* filgfl Y"'

x+4

y + 1 = 2(z - 2)

and parallel to the plane 4x + y + 5z

0.

10

(Contd.)
www.examrace.com

www.examrace.com

-(f)

\OIV S-"P

I.

the sphere

('F) <flf.llc; ~ A ~ cgnhiJ0~1


n X ncnf 3!10<Ltl ~ I
<;Qif~to: ~ A . (ad j A) = I A I . I .
3!<1((4 <;~lf~tc; ~
n
I adj (adj A) I = I A l(n-IJ 2
I0

(&) t'f)f~o;

I 0 -1
x
2
A= 3 4
5 , X= y , B= 6
0 6
7
z
5

3 q {) 'Ri 'l>T

iiW 111M

Show that the equation of the sphere which touches


4(x 2 + y 2 + z 2) + I Ox - 25y - 2z = 0

at the point (I, 2, -2) and passes through the


point (-1, 0, 0) is

x2 +
2,

(a) (i)

q;-@: $((,

<F
t\

IDU ~ ~B'i'fi{UI ~- ~ l!('[.


iJ\lf.llc; I ~ mq
AT X= 8 ~>ft'fi<OI ~<tit m'f <F\f
.l\l( ~AT
3Jic'{tl A <F qftqo' 'l>T '<Ild'i
'li@ i I
10
(IT) lll< lll 'li)f.llc;

lim

(x, y)--> (0, 0)

x y

3
3
X +y

, ~ 'til; 31fW,q

Tf Wt W I

<F

.. ,\be

the eigenvalues of an x
n square matrix A with corresponding eigen

(ii) VerifY the Cayley-Hamilton theorem for the matrix


I

M' i I

~>ft<H"I 'll< lll q\)f.llc;,

x + 4 = y + I = 2(z - 2)
'l>T 9Rl~{<;'i ~ iF fffi( ~ (3,
I, 2) iF
Wt 3i'h: ~llCIM 4x + y + 5z = 0 iF ~lli<1<

WI"

ofr<r

Wt I

(b) (i)

0 -1

A= 2
1
3 -5

am:
.r-qq
10
~

10

10

10

('<f) ~ ~

Let A. 1, A.2,

= 0.

as that of A. Also find the relation between


the eigen vectors of B and. eigen vectors of A.

(tl) <fi~l( f ~ 'flM 'i, IR 'R


4ft""llf':l<1, ~ fit;
IR Tf X <F ~ "1T'fi "if; fffi(
f(O) = -3
f'(x ) < 5 I f(2) 'l>T flt>CI'i I <f9T
Wt
~

+ z2 + 2x- 6y + I

"vectors XI, x2, .. , xn" lfB is a matrix similar


to A show that the eigen values <if B are same

AX =B

0
I

II

Using this, show that A is non-singular and


findA- 1.
10
Show that the subspaces of 1R3 spanned by
two sets of vectors {(!, 1, -1), (1, 0~ I)}
and {(!, 2, -3), (5, 2, I)} are identical. Also
find the dimension of this subspace.
I0

10
(Contd.)

(Contd.)
www.examrace.com

www.examrace.com

(ii)

ml'l'P

(U) Gffi~ f<l; it~

'fiT a l{t <t> , " 1: ;;fT ~~


(I, 2, -2) '11\ >i'h1'!>
4(x 2 + 1 + i) + !Ox - 25y - 2z
= 0
'fiT ~ <t>{dl
~ <-1, o, o) <F m

:!>'" ,(1,

2.

m om:

m.

('!') (i)

give n by the matrix

x2 + 1 + i + 2x - 6y + I = 0 tnm
I I0
Ji~c; 1..1' 1..2 , ..., \ "c;<t> n x n <llf 3lJC'W
A iF
31WT'1 m'l , f-;[,<f; ~mr
31Wf'l a~~ 1

xi, x2 , ....... ,xn N I ~ B "c;<t> 311'1il


-$ li\'11''\, m <:~1\~c; ~ B iF 311~'1'1 l1T-i

A=

(c) (i)

mA

A -$
;f; "l\1"1(. ~I ~ W< tl B ;f;
311~'1'1 afil~Ti 3TR A ;f; 311~1'1 aR!1U
;f; ~ ~
3\1~'1'1

1JT'iT

T '1 11')>''1 4\~ c; I

(ii)

A= 2

I0

0 -I
I

3 -5

;f; ft:rc; <fiff- ~ fB <'" c '1

Wfrr

1il C't! If<ld

4'! ~ <:; I

~li\<t>l ~~'liM <pffl ~ <;Qif'!c; ~


A C~<if>tJUftq
~ 3iT<; A-, 1 'li\1:'1 4\~c; I
10

("f) (i)

<:~lf'!CZ: f<\;

lilRQ\Y ;f;
tti'l {(! , I, -1) ,
(1, 0, l)}aTR {(1 ,2, -3) ,(5 ,2, l)}<
fi 31IT-'!IT
~ 1R3 'lfr dY\11ifGc!li lilcflil'i ~
I ~ m

it

I -3

3
I

-I

0
I -2

10

Show that the vectors (I, I, I), (2, I, 2) and


(I, 2, 3) are linearly independent in R(3).
LetT : IR.(3) ~ IR.(J) be a linear transformation
defined by
T(x, y, z) = (x + 2y + 3z, x + 2y + Sz,
2x + 4y + 6z).
Show that the images of above vectors under
T are linearly dependent. Give the reason
for the same.
I0

311c~il

Find the nullity and a basis of the null spac e


of the linear transformation A : R( 4) ~ R( 4)

(ii)

Let A=

-2

I
I

and C be a non-

3 -1

sing ular matr ix of orde r 3x3. Find the


eige n valu es of the matr ix 8 3 whe re
B = C 1 AC.
10

~ '3Y!iltJfGe: '!>1 fillik11{ '11\i'i


'!>"i~cz: I

10

(Contd.)

(Contd.)
www.examrace.com

www.examrace.com

---" -

_,. ____,

(ii) 311qi!

A=

-2

-3 -I

3.

(a) Evaluate :

(i)

~-q <11 '

aTI\:

'1 fk .

(ii)

q;r 1:l;ifi

~~c; I
I0
<:~1f1<c; fil; RCJJ if ~~~ (I, I, I), (2, I, 2)

(ii) l"i'' ~ c; fil;

A=

3. -I

(a) Three points P, Q, R are taken on the ellipsoid


x2 y2 z2
- + - + - = I so that the lines joinin g P, Q,
a2 b2 c2
R to the origin are mutually perpendicular. Prove
that the plane PQR touches a fixed sphere. 20

Jx3 'FT 1:l;ifi cg(sti'"JD~q 311c'i;i; ~I 311q~


B 3 <f; ,311\PI 'l 1fR %'f' i q:) ~ ct, ;;rif

B= C 1 Acr

(8, 12)

4.

aln:c<F'tft

Jfn x dx.
l

fil>

2 -2

x=2

(b) .Find the points on the sphere x 2 +


+ z2 = 4
that are closest to and farthest from the point
(3, I, -J).
20
(c) Find the volume of the solid that lies under the
paraboloid z = x 2 +
above the xy-plane and
2
inside the cylinder x + y2 = 2x.
20

2x + 4y+ 6z)<f; iiTU ~ T: IR(J) --.JRC3l


1:l;ifi l:R'lifi 4i<1 (01 ~ I
T<fi 31f.tH d4U<ti1 ~QTI <f; YRIR'<il
l:~ifi<1: 3llfu"<1 ~ I ~ ~ <f; if> I(OJ 9 fWl
~~c; 1
10

aft\ (I, 2, 3) ~~'1><1: <'q(1.'1 WI <'fl~c;


fil> T(x, y, z) = (x + 2y + 3z, x + 2y + 5z,

<:~1fl<c;

x;e2

I .

3TTUR '11(');'1

(lr) (i)

x-2
11

<f; "GRT 1ffi {~if> 4i<WI A : RC 4l --. RC 4l

'Ill

!~ f(x), where f(x) =

(b) Show that the cone yz + zx + xy = 0 cuts the


sphere x2 + + z 2 = a2 in two equal circles, and
find their area.
20

10

(Contd.)
www.examrace.com

(Contd.)

www.examrace.com

--=--

_,

X'2 - 4

~~f(x) 'om

(i)

f(x) =

rc

, x=2

x2

J(n x dx.

(ii)

("f) nH<fi x 2 +

+ z2 =

;;IT ~ (3, 1, -I) <F

'tf{

~ ~ 'll<fli ij(lfi:tq;,
'lffi

3Th m

m
1

20

rr) ~

m:r

z =.x2 +

x2 +

4.

"fiT 311l1d'1

if; ;flit, xy-\'Fidl'l if; .-a;<R afrt fflf?is <


= 2x if;
~ if I
20

x2

y2

z2

('P) (fi;r ~ P, Q, R Gltf'i:tiUI 2+2 +2= ! 'tf{


a
b
c
W \l'W ~ ~ ~ fll> P, Q, R '!i't d<(llli ~ ofl$"1
'I'I'O!t ~ 4 <f<l < <f<r 1
ij(l~ CJ> fli; <ill a11

PQR ~ Pilla mH<fi


("f) o:vrf~ fli;
x

W, aiTt

vi'!1

mm

'!i't

yz

~ Hdl WI
zx

xy = 0

20

llTi'I'P

+ ~ = a2 <Ft ~ Hiiil { if; ~ 'if <filed I


'3'1ifil l?\?14il'l

'4t

y2

11\<fli ij(l~q; I

z2

SEC TIO N-B

ij(lfi:tq;, ;;IT 4<<m<l'"

'll<fli

- + - - - = I are inclined to each othe


r at
a2 b2 c2
an angle of 60" if a 2 + b2 = 6c 2 Find also
the
condition for the generators to be perpendi
cular
to each other.
20

(8, 12)

(c) Show tha.t the generators through any


one of
the end s of an equ icon juga te diam eter
of
the principal elliptic section of the hyperbol
oid

xo02

X- '2 '

-~-

20

5~

(a) Obtain the solution .of the ordinary diffe


rential
dy
equation -=( 4x+ y+ l) 2 ,
dx
if y(O)

1.

10
(b) Determine the. orthogonal trajectory of
a family
of curves represented by the polar equation

a(! - cos 9 ),

(r, 9) being the plane pola r coordinates of


any
point.
I0
(c) The velocity of a train increases from
0 to v at
a constant acceleration f , then remains cons
tant
1
for an interval and again decreases to 0 at a cons
tant
retardation f If the total distance described
is x,
2
find the total time taken.
I0

(Contd.)

(Contd.)
www.examrace.com

www.examrace.com

_,_.,_.

(d) A projectile aimed at a mark which is in the


(TT) <;~If~~

f<F

horizontal plane through the point of projection,

x2
z2
31ffi9{'1Hll"' - + - - - = I if; ~
a2
b2
c2

falls x meter short of it when'the angle of projection


is a and goes y meter beyond when the angle of

qft~g<; if; ~ A<:i:r+i1 Cl:!ffi if; 1m\' if


-ij- fll;tiT ~ if; ~ -ij-' "''1 'fi ~ ~ 1:[\ 60 if;
<Frur. 'R 311.'1d ~ W ~ a 2 + b 2 = 6c 2 1 ~

M'[tf lll

W1

tr

"*

"1'1'1>1.

~ ~ 1:[\ <'for

A I<');A <tl~ ~ I

10

(e)

<F mr
f.mf<"

10

(r, 9)

M '4T

(TT) ~ 1:HII"?"i <t>T

crn>-'Pff <F <1i~<li >rlfftr-tf2l <t>T


"'

q('j~c;.

"ffifi ilipd I

~ 'if; li1A<l<1 ~ f.1'4~1i<li

i1<r

W,

319f'tctdf ~'~<"I r,

Jli1* '!TG ~

319ll:ctdf <AT WID

W 3ih: fiR:

<F

W1

10

W<. o -ij-

:id \IH

if; fffi;

319f't'ldf

1'f<R f 2

<F W< 'fill~-~ o~~ w1 ~ ~ ~


'lFf ~ x llT, dT 'lFf fWn W1!r A I<');A q\)~ ~ I
10

determine :
(i)

a(1 - cos 9)
ctiRt~

and b =sin t i - cos t j

(\<f) ~ 1'ftifi{UI
=

For two vectors a and b given respectively by

a=5t 2 i +t] -t 3k:

('fi) f.'i~'llfiftcld mW{U I dl'l'fiH 1;110\1ifi{UI 'fiT 1?R m'<f


q\)~~ :

If the velocity of projection is

of projection.

20

dy =(4x +y+1 ) 2 ,
dx
~ y(O) = 1

~.

assumed same in all cases, find the correct angle

~ Gm

<f!VS - \or

5.

projection is

and (ii)
(f)

idt (aG)
idt {axb ).

10

If u and v are two scalar fields and f is a vector


'field, such that
uf=gradv,
find the value of

-fcurl -f

10

(Contd.)

(Contd.)
www.examrace.com

www.examrace.com

C'il

~ ~ f"l~11oi 'liT~. orr,.~

<F

<F <~R
ll1 fi'r-;r <P"l <'1 <'I it m, ~ ,.~. ~ ,.~ <FruT
a iKTI w t~<r f"l~11oi ~ x lfln ~ PI '11 i aih:
~ 'J~ <FiuT ~
i, (!<f f"l~lloi 'liT m ql{
y 41n 3TTit "'T PI {<'11 i 1 ~ tMt '"H'1(1l. it ,.~
<f; ~ 'liT <mt 'WI" ft1<rr "111!' o) 'J~ 'fiT tnlT
~

mm

<FfuT '11<');'"1

0fl

$1 f\;1 ~ I

10

a=5t 2 i+d -t 3k
aih: ii =sin ti- cost ]
<F lmT ~ G't e f<l ~Tr

6.

(a) Obtain Clairaut's form of the differential equa


tion
(

x dy -y) ( y dy +x) =a2 dy.


dx
dx
dx

Also find its general solution.

(b) Obtain the general solution of the second


order
ordinary differential equation
y "-2 y 1 + 2 y :-

ifi'1 ~ 1:

aih: ii <F fm!;

15

+ eX

COS X,

where dashes denote derivatives w.r. to x.

15

(c) Using the method of variation of parameters,


solve

(i)

am:

dt

(ii) -d (-axb-) .
dt

(U) ~ u

am:

v G) ~

anr m, w
u

oT

the second order differential equation

(a ii)

anr mam:

10

15

f ~ ef<l~l

(d) Use Lapl ace tran sfor m meth od to solv


e the
following initial value problem :

9 '!i 1{ f.'l;

=gra d v,

-f curl -f

'fiT 'WI" '11(');'1 <f1f\;l~ I

10

d 2x
dx
dx
--2 -+ x= e 1 ,x(0 )=2 and
dt 2
dt
.
dt

t=0

=-1 .

IS

(Cont d.)

(Contd.)
www.examrace.com

www.examrace.com

7.

6.

('Fi) 3Ji'l'!;<.'l ~ 4) ifi { oI

(a) A mass of 560 kg. moving with a velocity of


240 m/sec strikes a fixed target and is brought to
I

rest in I OO sec, Find the impulse of the blow on

x dy
dy + x) =a 2 dy
( dx - y)'( y dx
.
dx.

<F <ffi U ~

the

~ ~

<F\futc(l w:! tft ~ c~lqifi


iR'I' ~ ~ lil\f"l <t\fut q; I
IS

(b) A ladder of weight W rests with one end against


a smooth vertical wall and the.other end rests on
a smooth floor. If the inclination of the ladder to
" the horizon is 60, find the horizontal force that
must be applied to the lower end to prevent the
ladder from slipping down.
20

y" - 2y'' + 2y = x + ex .cos x,

{lf) u;glri'liiM'\'1 f.lf<lri'!ij"Tl{Uiifjy

'<tT fiml:

'Pf ~ lil\'1 ~ ~ ffi:t\l~

0<)

f.l'"'lffi~a .~

fffi!:.

l'lll('ll~. ~qi<.i<:

d 2x

.dx

lfM

~'lflll ~ iR'I'

WU

q;r

ro

dt

<F

~Willl'l <t\futq; :
,~dx

~.-2-+x=e 1,x(0 )=2" 11\-

dt

dt

target and assuming the resistance to be


~niform throughout the time ta.ken by the body
in coming to rest, find the distance through which
it pene:rates.'
20

=-1.
t=O

(c) (i)

After a ball has been falling under gravity


for 5 seconds it passes through a pane of
glass and loses half its velocity. If it now
reaches the ground in I second, find the height
of glass above the ground.
I0

(ii) A particle of mass m moves on straight line


under an attractive force mn2x towards a
point 0 on the line, where x is the distance
from 0. If x = a and

dx
dt
=u

find x(t) for any time t > 0.

when t = 0,
I0

IS

(Contd.)

(Contd.)
www.examrace.com

www.examrace.com

---

IJ

- --

,3U.,

8.
7.

ml!f

('F) 240 rnlsec -if; ij.y -if;

lff<l'"li"', 560 kg

~!!Rl ' ~ ~ PI ~II '1 ~

UfT C'Oi'HI<t'l'

'1ft ~

w 3Th: I ~0

sec ~ Fcl{ili1'1f'.IT 'R ~ ;;mft W1 f.1~11~ 'R


3! I"'I d -if; 3114 lf qiT '11 '");'l q:) f':i t:; afh: 'Q;m
~

fiffi

"* mr Fcl {1'"11'1f'-TT M> ~ ~ rn <rrR mt

W1 ~ 9 Rl {1 !1'

'!iT

t:;'li M I'"I A If.1 t:; I '11 '");'l ~ ~ t:;


fil; fii>a ;f) ~ M> qw ~ !" Rl 'il-!1"'1 if> U0 WI 20

(B)

'iffi'

~~'fiT~

<Ft

~ fZ<!;r

W,

"'<I fii> ~~ {I

w1 ~ ~ <Ft fuRl"'

fttu Rwi>1

Jmr

&t'1W{ <fl'1t{

~ 'li 'J ~ 'R fZ<!;r

3!1'"1Rl 60 i't, m- lil'tli


~ ~ <:; fil; ~ "* f.1 il i1 ftrt 'liT f<h@ '!>{ m urr'l
~ {1 'li 'J -if; lffi; fii> d 9 ~ <[('[ '!iT ('fl II '"II 311.'1 ~ ~ 'li
'R

lWrri20

(lr) (i)

~ 1TG -if; !!'"'1 -if; ar~ 5 ~fs M> l'lmt


~ -if; mG qw ~ 'liiillfl<"'''i ~ ~ !!"' uft i
3Th: 3TTU 'i)-lr 'liT 7fqr ~ i I ~ 3T'f qw
'R 1 ~fs ~ ~ ;;mft w
<t>iillfl<1it> '1ft

Wi

(ii)

m,

~ <hili{ lil'");'l ~~t:;l

~dOf'll'"l m 'fiT ~

~ ~

1Tfi1
X

'iJii
10

'FU\, ~ ~ 0 '1ft

mr 'R mn2x 3\i'PIS\T

'P{dl

W,

= a 3Th:

dx
dt

W1 t > 0 -if;

~ X ~

=u

'

Uf'1' t

lffi; x(t)

am

-if; ~
0 ~I <:rfG"

m fiRlT

<[('[

'

lil'");li '!>1~<:; I

. --

-~

--u .

(a) Examine whether the vectors Vu,


Vv and Vw are
coplanar, where u, v and w are the scalar
functions
defined by :
u= x+ y+ z,
v= x2 +y +z 2
and w = yz + zx

+ xy.

15

(b) If ii = 4yl + x} + 2zk , calculate the


double integral

fi (v xu) ds

over _the hemisphere given by

z ;?: 0.
15
(c) If r be the pos itio n vector of a
point, find the
value(s) of n for which the vector

r"

is (i) irrotational, (ii) solenoidal.


15
(d) Verify Gauss' Divergence Theorem
for the vector
v=x 2i+y 2}+z 2k
taken over the cube
O< x,y ,z< l.

15

,ft
10

(Contd.)
(Contd.)
www.examrace.com

www.examrace.com

--

8.

~)

lU

JL. !W

'lfT!RUI

~~q; fll; 'fliT ld~~l


ldl"l (\J ill ~. ;;rnT u, v 3ih
: w

Vu, Vv

aiT<:

Vw

Serial No.

[~. . _ _ _ ___.)

u= x+ y+ z,
v = x2 +

+ z

llfUJ1 ,
2

~-'l"lf-1

<f; lm T ~ 3\~QI 4i('Fi


~
aiT<: w = yz + zx + xy.
(Iff) ~

( C- DT N- L- NU A]

u= 4yl + x}+ 2z k,

m ~:

15

~ulfih

: 300 )

fl'li<t>H

Jf (vx u)ds

!AriJ'I> !l'F f f&rcf/

3i'IT

<ft:iT if fJ'lT g I
":3"f1< "31ft 1TT!W1 if fffiEr ;;rr2f il7f&
Q, Pftl 'fil
3r.fl<'l 3Tl'Fi> !Wlr-rm if foi;lrr 1JllT g_ 3i'IT
W 'I:fTUl1r
'fil "f'riSC 3<'fi<'l ";:Jm"-~1'1>
p!'f-'J'SO 'IT ;;f{if;ff
AFe'"l f'llR 'IT f'r;iJr UfPIT ilT!fll{ I m-rm rrr
3tffi/&o
t/W IR <ir 31klR<tn 3RT f$ifT 71fV
l11 if fffiEt 7Tf!: ":3"f1<
rrr m 3f<t; 'Tiff filiTi/1
m 11'47 1 3i'IT 5 31f.14J2f ~I omfT !Wif if ft
!lrilif>
@1 5 ft 'fi11-ft-'fi11 "Q'I>"
m 1 '1 if> < Pthff FfFr !l ~q'f
<ir "3m cf/A !{ 1
ufiiif> m if; ~ Pi 41 3f<t; !l'F
f if; Jtrr if ~ rrrz: t 1
"llfii 3174~4'1> it, fit 3lJ&iffl 31i'l>il" w "iTIF
7 <tilRI!{
ff21T >:!'1<t>) f.)ffoc <fi!Jil{
1
!lffl'l>/fi<f:o !Nf?m ~ if 93!iffl ~ 3R/PJT
P!R'oc ~ 1

!Wif <fr

x2 + ; +

<F rnr
(lT) ~

("llT-11)

"!ffirru

a2,

"'IT.

z>0

qf{ iji(' l'j

<t>1~q; 1

15

~ ~ "'liT ~fd fl~QI


n <f; lW f
qi't 'llti;'"i <'{(1fZ;!q;, fZ;!Id~ ~
ld~~l

m. m

r" r
(i) ~. (ii) Gf t:'l ~~~4 ~
I

15

('l") Q'lltfd

0: Sx ,y ,z <1

'fi't 3\fft ;f)'l' C\ fll;lJ;

~ ld ~ QI

v= x 2 i+ y 2 )+ z 2 k
<f; ~ li\J<'l 3\G<'l (DI

vi't<r 'fi't

fl~"llfq(l <t>1fZ;!q; I

15

No te: English version of the Instruc


tions is pri nte d on the
fro nt cover of this question paper.
www.examrace.com

www.examrace.com

--

-~

"

C. S. (Main) Exam : 2011


Serial No.

('--_C--=-"""J~)

'

I.

MATHEMATI CS
Paper-II
[Time Allowed: Three Hours J

SECTION-~A

[ C-DTN-L-NU B)

(a) Sho w that the set

(Maximum Marks : 300 J

= { fl,

f2, f3, f4, fs, f6.}

of six tran sfor mat ions on the set of


Com plex
numbers defi ned by

INSTRUCTIONS
Each question is printed both in Hindi and in English
Answers must be written in the medium specified in
the Admission Certificate issued to .you, which must
be stated clearly on the cover of the answer-book
in the space provided for the purpose. No marks will
be givenfor.the answers written in a medium other
than that specified in the Admission Certificate.

f ( z ) - -I - and f (z) = (z- I)


z
5
6
(I- z)
is a non-abelian group of order 6 w.r.t. com
position
of mappings.
I2

Candidates should attempt Question Nos. I and 5


which are compulsory, and any three ofthe remaining
qu(!stions selecting at least one question from each
Section.

(b) Let S = (0, I] and. f be defined


by f(x) -

where 0 < x < I (in JR). Is f uniformly con


tinuous
on S ? Just ify you r answer.
I2
(c) If f(z) = u + iv is an ana lyti c
func tion of

Assume suitable data if considered necessary and


indicate the same clearly.
Symbols and notations carry usual meaning, unless
otherwise indicated.

z = x + iy and u- v =

All questions carry equal marks.


EZTR ~ : 3i:J;m' ~ ~ "'4/rit (
'RGW#i

r;r.pf- q;;r

fij GH

Y"'

.
su b~ect
to t h e con d".
ltlon, f

[2J
www.examrace.com

eY- cos x +sin x


cos hy- cosx , find f(z)

(1t) = T
J-i.
2

12

(Contd.)
www.examrace.com

(d) Solve by Simplex method, the following


LP Problem :
I.

Maximize, Z

('!>) ~ <#)fOil( ~ ~~ (1\0!1i3il' <f;


\1'j;~i1!l 'R

f/z)

= z,

f2(z)

=I

Constraints,

- z

5x 1 + 3x2
3x 1 + 5x 2 < 15

5x,+2x2<1 0

x 1, x2 > 0
(e) (i)

12

Prove that a group of Prime order is abelian.


6

(ii)

B~
G=

"0": Yid {Olr 'liT \1'j;~i1!l

{ fl'

ffNe'f

~ ~I

15

6 ~ 'liT~ 3R,-illi~\'\1

(0,

I]~ f~ ~

'

"B

f(x) = _!_
X

~ ~ 0 < x < I (IR if) I 'f'!T f S


'!\ l(ifi(1'll'id:

tier<! ~ ? 3T'f.t ~

'liT 3flfil~!l 'ldli<~ I

,._ ,,._
~

~~t

u- v=

eY -co sx+ sin x


cos hy- cosx ,

nx(l - x)", x

z - a

12

f'(a)

= 1

"'~~u

[0, 1]

< R, prove that for 0 < r < R,

rP(S)e-; 9 d9, where P(S) is the real

1tf 0

part of:f(a + re;e).

Examine the uniform convergence of {f (x)} on


n
[0, 1].
15
(c) If the function f(z) is analytic and one valued in

(lT) ~ f(z) = u + iv z = X + iy 'liT


~ ~~i'l~l<'fi
~M~

(b) Let f (x)


n

12

~S=

(a) Give an example of a group Gin which every proper


subgroup is cyclic but the group itself is not cyclic.

f 2, f , J , f , f }
3 4 5 6

9R1fil'l01-(1.~l"l'i <f;

(\'i) l1RT

2.

How many generators are there of the cyclic


group (G , )of order 8 ?
6

15

(d) Find the shortest distance from the origin (0, 0) to


the hyperbola
x 2 + 8xy +
= 225
15

7l

(Contd.)

(Contd.)
www.examrace.com

www.examrace.com

llJ

Jl~

3.
('1)

f.'l '"'i R1 ~ ct t ~ 'li 9l !11 '1

<:P:l f11 :

functions defined on the closed interval [0, 1].


Prove that (F, +, ) is a Commutative Ring with

Ma xim ize, Z = 5x + 3x
1
2
Con stra ints , 3x + 5x < 15
1
2

unity with respect to addition and multiplication


of functions defined pointwise as below :

5x +2 x <1 0
1
2

(f + g) (x) = f(x) + g(x) }

x 1, x ~ 0
2

(&)

'lit 1:!;'1>!l'f f<lfu ~ ~ <j(lf'ilq; I


( i) fi1.1J: $\ ~ t:; f<l> 3NT'itr ~
mctTil

2.

and (f . g) (x) = f(x) . g(x)

12
i'!iT "t:;'li

"Ti

where f, g

3i I~ Jl

('!>) "t:;'li ~ G i'!iT di< IH 0 1 <flf'ilc; ~i;ii


'\
iii611l

mctT i 9("\l:

ili61l01

'ffi'lT

(x

<F fu1:!:

x + 8xy +

7r

[JJ

4.

15

15

)'

15

13

1-z 2

'

with centre z = 1.

15

(a) Let a and b be dements of a group, with a2 = e,


b6 = e and ab = b4 a.
Find the order of ab, and express its inverse
in each of the forms ambn and bma".
20

= 225

C!'!> <'I :J><I '1 ~ mG iC1 f'il t:; I

dx
2

f(z) =

i1.

0 ?

(d) Find the Laurent series for the function

f'(a ) = - I " P(9)e- d9,


mo

""~llT fir> P(9 ) f(a + rei 8) 'fiT <mflfil'l


i 'WT
('1) ~(if<lns; (0, 0) ~ 3lRI9 {q(' j[j

(c) Evaluate by Contour integration,

am~ 11R

< R

nx
, O<x<1
1+n 2 x 2

What happens at x

15

I z- a I< R it fil~(:lfG!q;
i , <IT ft1.a <j(1 fZ;J c; f.l; 0 < r

15

cannot be differentiated term-by-term at x = 0.

15

("f) "'1T'il f<l> f (x) = nx( l - x)"


, x E [0, 1).
n
[0, 1]'1 ( {f (x)} ~ t:;'liW"ll'i ~
<t't ~ $\f'jjq; I
0

(1f) ~ 'liF R f(z)

F.

f.(x)=

dfil d '3 9W~il

~I

[0, 1]

terms

'l1!T

X E

(b) Show that the series for which the sum of first n

8 ~ ~ ii<t>"ill ~ (G , ) ~ ~ct'
"l "''i' li ~
~ ?

(ii)

(a) Let F be the set of all real valued. continuous

I5

(Contd.)
(Contd.)
www.examrace.com

www.examrace.com

3.

('n) 1iRT filiF <fG 3ii1 <I M ( 0, l]'rPrfD:rrfqcnr~ <1 I~ f4 'P


1fR timf 'liM ;oj) 'liT +1 :J> nP1 ~ I fir,g:. <F\ f';t ~ fil;
(F,+,):
(f +g) (x) = f(x) + g(x)} x
aih:. (f. g) (x) = f(x) . g(x)
~fil; f,gEF

(<IT)

m
'liT

X ;t

S'(x) =-2x :L;

<Wr 3ih: TR
S!i 'l f4 f.1 ~ 11 "1ffi<T t?rcrr ~ 1

O<x<l

0 'R "tRW: 3[=q<p=<'f='i


0 'R 'fliT llTcrr ~ ?

"'ftff fil>m

('T) ~ z

5.

(a) Solve the PDE


(D2 - 0' 2 + D + 3D'- 2)

15

az

az

(x + 2z) ax + (4zx - y) fJy = 2x2 + y


12

3ih:

'

ab *r ~ mer <F\f';t~, 3ih: ~ ~Rkil'l <N


20
~ ambn aft< bman if <'1iffi <flf';t~ I
(Contd.)

(c) Calculate

J1+xx

10

15

('n) 1iRT fil; a 3ih: b 1:('!> ~ #;, a 2 = e, b6 = e


ab = b4a ;f; @1 3TqJ:fq WI

[2J

= e<x- y)- x 2y

12

-z
;f; 1m; C'i'R1 -~ mer <Fl f';t ~

(b) Solve the PDE

f.1 'PI fi1 ~ I


= I <n<'1 lfiM'i

f(z)

20

SEC TION -B

15

fcx2 _:3)'13
'liT 1fR

20

"'T +1 Slid I ~ I

Minimize, Z = 6x + 4x
1
2
Constraints, 2x + x > 1
1
2
3x I + 4x.2> I 5
x 1,x >0
2

4'11 Ilfi M
,

,fora llx.

(c) Write down the dual of the following LP problem


and hence solve it by graphical method :

;f;

<F!f';t~ fil; il;BT ~ 'liT f';twF 1m; 'Jql'[ n "'lGT


nx
l+n 2 x 2

2 2
"=' n 2 (l+n x)

(lf) 'PC:"< +l'li'PM'i ~

4.

derivative

15

fn(x)=
X =

@:[ 1:('!>

Show that if

(O, I]

qfr <m ~: ~ lfiM'!l" ;f;

miff 1:('!> ;f;

(~)

(upto 3 places of decimal) by

dividing the range into 8 equal parts by Simp son's


I

3 rd Rule.

12
(Contd.)

www.examrace.com

www.examrace.com

S(x) =

(d) (i) Compute (3205)


to the base 8.
10
(ii) Let A be an arbitrary
but fixed Boolean
algebra with operations A, v
and ' and the
zero and the unit element den
oted by 0 and
1 respectively. Let x, y, z, .....
... be elements
of A.
If x, y E A be such that x
i\ y = 0 and
x v y --: I then prove that y
= x'.
12
(e) Let a be the radius of the
base of a right circular
cone of height h and mass M.
Find the moment
of inertia of that right circular
cone about a line
through the vertex perpendic
ular to the axis.

LJn+
, nx

2 ,

n ..

ciT

\1\i if> I 31 q if> eN

S '(x) = -2x

nl

(1T)

2 2
n (l+n x)

"'4'1 x if; fi1c; I

20

f.1 ~1 R1 f1 d '{f1 ifi \il Wl1 \i 'l f!ll 'PT mft R1 ~ C( 3ih:
3Rf: lJl'l\'t fimr it "\i if> I ~ f.1 ifi I f<1 C( :
Minimize, Z = 6x + 4x
1
2
Constraints, 2x + x > I
1
2
3x +4x >15
1
2
.xl' x _>0

20

12
~-~
5.

6.

('t>) 31i~l't> <1\<l't>\1 'il'i~I<HUI (PDE)


(D2 - D' 2 + D + 3D' - 2) z = e<x- Yl- x 2y
'fit ~ <FJ fi;! C( I
12
~) 31i~l't> 31<l't>C1 \i'iJ<HUI (PDE)

az

az

(x + 2z) 8x + (4zx - y) 8y

'fit

12

(1f) q <I \i qi'r 8 M 1'"1.

mriT if

~ 't> {if;

l"f1 P'B 1 if;

10

R1 i:ll {

f-111 'l

it

touching z = x 3 + J
plane x + y + I = 0.
(b) Solve

Jl+x
dx 'PT <;~I 'l \11=1 if; 3 ~ <l'!i
12
(Cont d.)
www.examrace.com

= 6x + 2 and
Ox.'
along its section by the
20

8y

satisfying the boundary condit


ions
u(O, y) = 0, u(x, 0) = 0, u(x
, b)

au

Ox. (a, y)

=0

3 1tY
T sin -;- .

qf{if>C1 1 <t)fijj C( I

a'z

a'u, +a'u
-, --0 , O< x< a,O <y :< ;b
Ox

2x + y

~ <t)fijj C( I

(a) Find the surface satisfying

20

(Contd.)

www.examrace.com

(t:r) (i)

(3205)

10

'fiT 3Tl"UR 8 ~ ~ if\1~"( I

(ii) "l1AT fcl> A, mM 'il A, v all\ '~ W!T all\


1lFf a!1T acwFt> 3J'!I[g i!l'l~l: <WR'Ia 0 3ih: 1
~. 1:('P f4~g q{-\j; f.'P:Id ~ <\l"11<'1<1~1 WI
"l1AT fcl> x, y, z, ........ A of; 3J'!I[g WI

~
X

X,

y E A ~

v y = 1,

aT

i?f fcl>

X 1\

~ if()~"(

fil> y

= 0 3ih:

= x'

I 12

fil> a & ill { h all\ ~ clj 'll1 M orrc't 1:('P "(1"1'qvft<r ~ of; amm: '1ft E'i ""'~I i 1 an:=r ~ "(1"1'q M

7.

(c) Obtain temperature distribution y(x, t) in a uniform


bar of unit length whose one end is kept at I 0C
and the other end is insulated. Also it is given
that y(x, 0) = I - x, 0 < x < I.
20
(a) A solid of revolution is formed by rotating about
the x-axis, the area betw een the x-axis, the line
x = 0 and x = I and a curve through the points
wiih the following co-ordinates :

(6-) "l1AT

~ ~ ~ 1:('P

t<m

~ ~-~ ~ (1'""1~t1lll ~

'fiT "19rCI ~ m<1 'l>l~l( I

6.

a'z

('t>") -

ax'

12

6x + 2 'fiT

+ y + 1 -' 0 ID\l ~ qft~~<; of; "@1- "@1


z = x3 + l <Fi- ~ ~ ~ GI\%WM <Fi- m<1

00

25

50
9896 9589

75

9089 8415

Find the volume of the solid.


20
(b) Find the logic circuit that represents the following
Boolean function. Find also an equivalent simp ler
circuit :

<Fl~l( I

20

(\9) qf{fl11JI 9fc1GIm

u(O, y) = 0, u(){, 0) = 0, u(x, b) = 0

au
3 ny
-(a y)= Ts inax '
a

'

'fiT 91M1 ~ ~

a'u +a'u- =0,


ax 2 ay 2
<Fi- iof

<t>l~ "( I

0 < x <a, 0 < y < b


20
(Contd.)

www.examrace.com

f(x, y, z)
I

20
(c) Draw a flow chart for Lagrange.'s interpolation
formula.
20
(Contd.)
www.examrace.com

(lT) '11'1'li ~ ~ ~ l(<t>ld'111 W'll'lil

if,

RifFt>l ~

i 3\'r:

~ fmr 3i"'11DUl
filCI(OI y(x, t) "M! <fiiR\l( I <!tr ~ ~ S3!T
10C 'R "00

7.

i,

fmr

Cllq'111

i fit

y(x, 0) = I -. x, 0 < x < I.


20
('li) x-3fll1;
X= 0 3ih: X= I 3ih: f.1~1ffi~C
I
f.1<fqli'li1. CfTi9" ~3IT ~ im ~ om; <F
<ft<r ~
<0 x-3fll1 <F 1lfct
'fi(~ "l('fi qftil ili'l 111 ltRl

Tf'1

i.1'1d~ ~ :

(\cr)

00

25

50

75

9896

9589

908 9

8415

f(x, y, z)

(lT) Muic\j oi<J<fm

WI <F

.__ - - - - ,_

(a) The ends of a heavy


: rod of length 2a are rig
idly
attached to two light rin
gs which can respectively
slide on the thin smoo
th fixed horizontal and
vertical wires Ox and oy
The rod starts at an
angle a to the horizon
with an angular velocity
~ [3 g( l-s in a) /2 a] an d
ni ov es do wn wa rd s.
Show that it will strike
the horizontal wire at the
end of time

30

111 ltfil 'liT "'IW11 m<f 'l>1R\l( I


20
C!'1f ~11
'!>1R\l( -;;IT fit f.1"'1ffiRCICI ~
tm'R 'fi( f.1 fClCI 'fi@ ~ I "l('fi ~ ld (\1CI
{ qf{q<.f
'4't m ~~l( :
X

S.

-~-

(b) An infinite ro w of
equidistant rectilinear vo
rtices
are at a distance a apart
. The vortices are of the
same numerical strength
K but they are alternately
of opposite signs. Find the
Complex function that
determines the velocity
potential and the stream
function.
30

~ "l('fi m

20
ti.Rl'1 '4lRl l( 1

20

(Contd.)

(Contd.)
www.examrace.com

www.examrace.com

8.

('fi) C'ikll{ 2a

<FI ~ '>ITtT ~ ~ fut ~


~ \"9<-Ml.
it ~<pa1 it . ~ ;;IT f<l> lli'
1~,, ~ m it f.'i<la
~ 3ITt 3>!:Rl"m miT
ox 3ITt Oy <n: ftli\1
0 \1 i.fi ell
~I~ '!i)c\)'l Wr ~[3g(l
-sina)/2a] it~
it -<N"ur a 'R ~'i i.fi {cfl ~
3ffi: 31ln :J>,4) "1 C1 cfl ~ I
fu;j; if>~f';j c; f<l; <il.' ~
-

2~ a /(3g) Jog [tan (; - ~)cot ; ]

<f; 3Rf

it

<m: it

C: i.fi (I t;>fl I

(<?'!) ~ a 'R 3TMJT31Wf '11<1{1\'11 "hcf1~


"t;i.fi

aA'ct" ~ ~

~ ~ 9 \g)

'if

1 ~ '11'1 eZC'll
c'1i.fi

_______,]

Serial No. (..____

llfUI<:l

:wiT <t
... ~fir <FI
m>ruf K

t;i.fi i<Hct : f4 9 t1ct ~


;f; ~ I tl fl:q~
mer ~f-ile; ;;IT inT fi'Mq 3ITt
urn LfiM'i -<N
f.'ruffta i.fi M ~ 1
'hM "l

30

>rFT-'P-'r-11

9 rlJ 'li Tfff ~

30

[ C-DTN-L-NUB]

3ih 3litoft ciPi'f it

it ffRit

f9'fT

~a1fq;

: JOO J

~I

rnf& q fJtff'lil
3ih W 11Tm
"if>( f'I1IZ d<"fk"l 'ffl?:-jhl'li
T!'-~ -en- 3lftrr
f.lfifoc <WPr -en- fitlH ;;rr;rr "11/&q 1 "Sf<m-q;r -en- afffifittd
111!44 <t iJiRlRiffl 3RT fiRfT Wm4" it ffRit m; 'ffl?:
"'R" 'f# 3/"f; '7iJf fiM "Uf7f# I
'ffl?: "31ft 11Tm

3TTr:rif> !f<m- 'ffl

d o.fi ('I

Tfff ~

if

1lf'15

it

3ih 5 iJif.141i.f ~I

'PT- if-'PT ~

'3f"T":?t

fiRlr 7J<rT ~'

:wif it if

9rlJ'Ii

:; 'i 'li < fit;rif)' ifFr :wiT

<f; 'ffl?: <flfiit q I

iff ril d 'ljiffl


f.lR"rsc: <t)fiitq 1

;:#!: 3114~'1'1'
aiJq;)

9ifi'Pl 3th
Of 'NIT 7J<rT

tr'lft

~{?;(1'!).

iff I

"9V"'il' <f;

iJil'liil.

9"1tfld

"if>( 'llPf

3M

<t)fiit~

~. Ufif ff'l'

rre:rr

3R[2JT

3/"f; 6 41 'i ~ I

Note : English version of the Instructions is printed on the


front cover of this question paper.

www.examrace.com

www.examrace.com

r:-':'.

.._.\'i ~.\~"'l~

.It-

l2Section-A

F-DTN-M-NUIA
1.

MATHEMATICS

(a)

Allowed : Three Hours

1
y

Paper-1

I Time

Define a function f of two real variables


in the xy-plane by
3

x cos-+ y cos-

IMaximum Marks

300

for x, y" 0

0 , otherwise

INSTRUCTIONS
Each question is printed both in Hindi and in English.
Answers must be written in the medium specified in the
Admission Certificate issued to you, which must be stated
clearly on the cover of the answer-book in the space
provided for the purpose. No marks will be given for the
answers written in a medium other than that specified in
the Admission Certificate.
Candidates should attempt Question Nos. 1 and 5, which

Check the continuity and differentiability off at (0, 0).


(b)

Let p and q be positive real numbers


1

such that - + - = 1. Show that for real


numbers a, b 2: 0
12

are compulsory, and any three of the remaining questions

selecting at least one question from each Section.


The number of marks carried by each question is indicated
at the end of the question.
Assume suitable data if considered necessary and indicate
the same clearly.
Symbols/ notations carry their usual meanings, unless

(c)

Prove
or
statement :

(d)

the

following
12

Let T: 1R 3 --> IR 3

be the linear trans-

formation defined by
T(a, 13, y) = (a + 2~- 3y, 2a + 5~- 4y, a + 4~ + y)

Find a basis and the dimension of the


image of T and the kernel of T.

F-DTN-M-NUIA/15
www.examrace.com

disprove

If B = {11, b 2 , b 3 , b 4 , b 5 } is a basis
for IR 5 and V is a two-dimensional
subspace of IR 5 , then V has a basis
made of just two members of B.

othenuise indicated.

Important Note : Whenever a question is being attempted,


all its parts/ sub-parts must be attempted contiguously. This
means that before moving on to the next question to be
attempted, candidates must finish attempting all parts/
sub-parts of the previous question attempted. This is to be
strictly followed.
Pages left blank in the answer-book are to be clearly struck
out in ink. Any answers that follow pages left blank may not
be given credit.

12

12

2
www.examrace.com

(e)

Prove that two of the straight lines


represented by the equation
x3 +bx2y+cxy2 +y3 =0

will be at right angles, if b+c = -2.

2.

(a)

(i)

o, :;p:rm
IDn ~ ~I (0, 0) '11:
:>Jqifii'Hl~dl
(13)

f'\;

a, b;, 0

~I ~

12

f'\; <'IIR1fil<t>

f.1"" R1 ful<1

'li't

12

3N<'IT "3B"<tiT ~

~:

'1"fu

!R

311un:

<It

3x 1 + 6x 2 + 13x 3 +4x 4 + 14x 5

=0

12
(b)

if;~ B = {q, b 2 , b 3 , b 4 , b 5 ) 11;<ti

m3i'n:

m1

Consider

<liT,
oil B if; ~ <iT ~ "il

(<!) I!R ~ T: !R 3 -> !R 3 11;<ti ~

the

linear

mapping

f(x, y) = (3x + 4y, 2x- 5y)

Find the matrix A relative to the


basis ((1, 0), (0, 1)) and the matrix B
relative to the basis {(1, 2), (2, 3)).

<i'YHHOI ~,

oil

12

(ii) If A is

T(a, [l, y) =(a+ 2[l- 3y, 2a + 5[l- 4y, a +4fl + y)

IDn ~ ~I T <liT
11;<ti 311tm:

(i)

f: 'J/.2 __, 'J/.2 by

v 11;<ti ~- R-zWr ~ mIRs

V "<til 11;<t> 311un:

'f.l1 );311

x 1 +2x 2 +3x 3 -2x 4 +4x 5 =0


2x1 +4x 2 +8x 3 +x 4 +9x 5 =0

m~

(ii) Find the dimension and a basis for


the space W of all solutions of
the following homogeneous system
using matrix notation :
12

B=Ji'\

if;~ ab 5 aP + bq ~I
p

(IJ)

3i'\1:

q i);<f\ ~ q IR1 fil if> <iwm(

3i'\1:

_I_+ _I_=
p

<li\ ~~I

I!R ~ p

f if;

Let V be the vector space of all


2 x 2 matrices over the field of
real numbers. Let W be the set
consisting of all matrices with zero
determinant. Is Wa subspace ofV?
Justify your answer.

12

31ft! 3i'\1: T if;

F-DTN-M-NUIA/ 15

prove that

~ "<til

3i'\1: fum Wi! ~ I

12

a characteristic root of
non-singular matrix A, then

P.T.O.

www.examrace.com

I.
root of Adj A.
F-DTN-M-NUIA/15

is a characteristic
8

4
www.examrace.com

(s)

fir.& ~ f'l;
x3
jffi]

>O!~i'h(UI

(c)

H=[ 2-~i i

+bx2y+cxy2 +y3 =0

fuo;f'rn ~

t&J31'\ i\ i't <it

l ufi

101 ~i'h

'R

M\,

"i"Rb+c=-2ill

Let

2+

ij

1- i

2
1+i

12

be a Hermitian matrix. Find a nonsingular matrix P such that D = pT H P

2.

(q;)

(i) l!R ~

<'llf<1~1fi ~31'\

it; 1$

B""lfl 2 X 2 ~ <JiT Bfur B"'!fu i I l!R


~ w ~ BT\fUrq; ~ B"-.fl ~
"<'ITMl ~ i I <I'll W '('!i ~ i V <Jil?
3l'f.t "3n\ iii '11\l il <T<i; ~ I
(ii)

is diagonal.

'R

3.
8

Find the points of local extrema and


saddle points of the function f of two
variables defmed by

f(x, y)

f.<'"" R-1 fuJ <1 Bl1QT(j ~ iii B"-.fl ~ <liT


B"'!fu w it; ~ ~ ~ q;J ~
~~fcrmam'('!i=Wi!~:

(a)

(b)

12

x 1 +2x 2 +3x 3 -2x 4 +4x 5 =0

x 3 +y 3

63(x + y) + l2xy

20

Define a sequence sn of real numbers


by
sn =

2x1 +4x 2 +8x 3 +x 4 +9x 5 =0


3x1 +6x 2 + !3x 3 +4x 4 + 14x 5 = 0

20

(log(n +i)-log n)

n+l

i=l

Does lim sn exist? If so, compute the


n-->-

("&)

(i) f(x, y) = (3x + 4y, 2x- Sy)


SlfiiA>"I
31TtlT(

am

f: JR

->IR 2 1R

{(!, 0), (0, I)}

:mom

i't

'l:fujq;

jffi]

f<i<m

~I

w:<!f.'~ ~ A

{(!, 2), (2, 3)}

i't

"'f<: ).

"':F\IiA"A~

3lf>:rml\lfUTq; 'i"'
'('!i 3lf':rml\lfUTq;
F -DTN-M-NUIA/15

(c)

~BWi\~1
(ii)

value of this limit and justify your


answer.
20

Jo

12

converges.

20

'('!i

'fT

m. -m m ~ f'l; I~~
'i"f i Adj A

Find all the real values of p and q


SO
that the integral rt xP (log !.)q dx

'fT I

4.
8

P.T.O.

www.examrace.com

(a)

Compute the volume of


enclosed
between
the
2
2
2
2
x + y = 9 and x + z = 9.

F-DTN-M-NUIA/15

the solid
surfaces
20

6
www.examrace.com

('1)

(b)

l!H~

A variable plane is parallel to the plane


X y Z
-+-+-=0
a
b c

_,

+l
21-

2 -i

1+ i

H=

'J

and meets the axes in A, B, C


respectively. Prove that the circle ABC
lies on the cone

1% ~ ~ ~I 1% ~ OIJ>illiqoA4 ~ P

'fi11!RI ~ f<f; D = pT H P fuq,ujf ~ I

3.

(q;) f(x, y) = x 3 + y 3
qft~ ~

~31'\

<Rt

il; ~

(c)

-rnT

63(x + y) + 12xy

yz(~+~) +zx(~+~) +xy(~+~) =0

20

il; ~ <wl

11T'i\ il;

3fu: '"lT"T ~31'\ 'fi11!RI ~ I

20

Show that the locus of a point from


which the three mutually perpendicular
tangent lines can be drawn to the
paraboloid x 2 + y 2 + 2z = 0 is

x 2 +y 2 +4z=1
(<!!)

Sn

i; (log(n +i)-log n)

-rnT

<i&n:i\ ij;

lim sn "" ~ ~7

>:!R il:m t <It ~

~joo

llH ""

mm il;

~ ~ 3fu: ~ ~ il; 'lPJ it

~~I

('1)

3fu:

Seetion-B

'lftmqj ~ I 'l'!T

<fit

Sn

5.

"*

(a)

<nf'l; ~ f

Jo

xP

11T'i\ 'li1

11R1 ~.

(log l)q dx ~'lit I

(b)
20

(q;)

'[!iT x 2

+y2

= 9 3fu: x 2

1Jfu1;& iffi il; 3WRR


F-DTN-M-NUIA/15

Find the orthogonal trajectories of the


family of curves x 2 + y 2 = ax.

12

12

(c)

4.

Solve

dy = _ _ _2.:._xy-"-,e,--lx_f_Y_I-----;;dx
y2 (1 + elxf y) 2 ) + 2x2 elx/ y) 2

20

q i1> <M "tf<1 W'"h

20

qi{<1W'f;

n+l

i=l

20

'f;l

+ z2

= 9 il> <IR

~ ~I

Using Laplace transforms, solve the


initial value problem

y" +2y' + y = e-t, y(O) = -1, y'(O) = 1

12

20

I PT.O.
www.examrace.com

F-DTN-M-NUlA/15

8
www.examrace.com

(d)

A particle moves with an acceleration

~+1{+~=0
a b c
if;~~

31'R ~'

A, B,

Cit 31W it firer<rr


towards the origin. If it starts from rest
at a distance a from the origin, find its
velocity when its distance from the

~I~~ Ff; '['!ABC,~

yz(~c +~)
+=(~a +~)c +xy(~b +~laJ = 0
b
'RWm~l

('l)

Ff; ll:'li

"'T

itit

~,

<fR

'lWR

it

('!1'<!

12

IS~.

20

x 2 + y 2 + 2z = 0 <f'li

clRl

ongm

q{<kl4"1

(e)

~-mnt

If

~ ~, '1>1 ~ x 2 + y 2 +4z = 1

~I

20

-+

-+

2--t

a2

-+

B =2zi +yj -x k

--t

find the value of--(A xB) at (1, 0, -2).


12

axay

12

6,

(a)

Show that the differential equation


(2XYlog y) dx + (x 2 + y 2 ..Jy2.:!1 dy = 0

(1:9) OfSii-~ x 2 + y 2 =
"jll(l

('l)

ax if; <1kl'hl'A4

is not exact. Find an integrating factor


and hence, the solution of the equation. 20

M "'h1

~I

12

~ ~ "" ~

i.Ri\ ~ ~

(b)

l1R

Find the general solution


equation y"'- y" = 12x 2 + 6x.

of

the
20

WW!T

y"+2y'+y=e- 1 , y(0)=-1, y'(0)=1


q;'\~~1

F-DTN-M-NUIA/ 15

(c)

12

P.T.O.

www.examrace.com

Solve the ordinary differential equation


x(x- 1)y"- (2x- 1)y' + 2y = x 2 (2x- 3)

F-DTN-M-NUIA/15

20

10
www.examrace.com

7.

{a)

i 'If<: "'!\(
RHIA wm it 3<(111 it 'if\ a it '{!"' <R, <it 3<(111 it

~ m~ 3<(111

_<"'

*t 3ffi lJfu

<R(I1

<t\ 'if\'<\~~ 'fil m<l ~I

12

A heavy ring of mass m, slides on a


smooth vertical rod and is attached to a
light string which passes over a small
pulley distant a from the rod and has a
mass M (> m) fastened to its other end.
Show that if the ring be dropped from a
point in the rod in the same horizontal
plane as the pulley, it will descend a
.
2Mma b,
.
d !Stance
e,ore commg
to rest.
M 2 -m 2

(&) 'If<:

{b)

A::::

x yz7 -2xz

+xz

B =2z7 +yJ -x 2 k
<it (1'

0, -2)

a2

'<""(

-+

----t

--(A

dxily

B) 'liT llT'f m<'f

~I

("'-!>)

~ \141'-f>\01

(2xylogy) dx+(x

'ltff

'lm(!"''

\1~1'-f>(OI

II

y '- y'

= 12x + 6x

'liT 01IN'Ii j(<'f

20

~I

F -DTN-M-NUIA/1 5

(a}

is the coefficient of friction.

(PTO
www.examrace.com

20

Derive the Frenet-Serret formulae.


Define the curvature and torsion for a
space curve. Compute them for the
space curve
X= t,

20

11

where
8.

2
x(x -1)y"- (2x- l)y' + 2y = x (2x- 3)

'fil j(<'f

20

~logl1+7l
20

m<'!~l

the equilibrium is stable,

The end links of a uniform chain slide


along a fixed rough horizontal rod. Prove
that the ratio of the maximum span to
the length of the chain is

m<'f ~ 3fR

m<'f ~I
2

~) dy =0

i1 \181'-!>('H ~

~ @'1'-f>(OI 'liT j(<'f


"

+y

!!. > J5- 1,

whatever be the weight of the particle.

12

fili

A heavy hemispherical shell of radius a


has a particle attached to a point on the
rim, and rests with the curved surface
m contact with a rough sphere of
radius bat the highest point. Prove that
if

{c)

6.

20

y = t2 ,

Show that the curvature and torsion are


equal for this curve.
F-DTN-M-NUIA/ 15

12
www.examrace.com

20

7.

(iii) ~ m '1>1 ~

mt\

'!Wl, ~ ~ ~m

(b)

lJ'fuT ~ ~ 3fu ~ ~ Q.m ~ it>: ,r


~ s:m ~ "11 ws ,J <;ft a <r>: ~ ffii\ f'lG\ <r>: ,r
:pmft ~ 3fu ~ ~ ful: '!>: ~ ~
M (> m) ~ s"ll ~ I ~ fif; 'lf<! 'lWl '!iJ ~
il ~ il:B ~ ,J fi'm K<n "11'1 "11 f'lG\ it ~
~ il m, -m RwAI'I>?ll (fi'fi ~ ,r ~ ~

Verify Green's theorem in the plane for

~ <r>:

"~\'!'!

'>ft

2 M ma

M2

(fi'fi

-;fRI TcR 3lTl('lT I

fc[(xy + y2) dx + x2dyJ


where Cis the closed curve of the region
bounded by y; x and y ; x 2 .
20
-Jo

(c)

-Jo

---)o

JJ(vxFJ r:ds

20

-m2

-Jo

If F; y i +(x-2xz)j -xyk, evaluate

s
where S is the surface of the sphere
2
2
2
2
x + y + z ; a above the xy-plane.
20

(<9) ~ a it ~ mt\ ~ ~ it ftl! <r>: ~

s:m

~ '!>: ~ q;uJ ~
~ 'lfR ~ ~. ~ b
it~~~itm<~~~ <r>:~il
31'i-l '@; 'Pl '!>: k'l>T
~ I ~ ~ fif; <rf<!

s:m

!!. > -JS - 1 m, -m


a

q;uJ

<!>!

>m '<lT%

m,

"11 >fi

\11141'1f>ll ~ tllMtl

('1)

~ Q,iiiAAH ~ (*r)

20

it ful: <fu

~ ~

~~~~it m<l-m<l m'l><ft ~I~


<fu ~ '!>:

~ fif; :m~ ~ '1>1 *r

~ ~-tlog[ 1+

7]

~ 1-l '1"fur ~

~I
8.

(iii)

20

jF.R -ifu: ~ ~ ~I 3TTilim


q'jfi{ff

'lfu ~ <fu 'lfWr<rT

it fu\(
fu\(

'lfR ~ 'Rfill: ~ I

F-DTN-M-NUIA/15

13

it fu\(

~ I 3TTilim '@;

T-l'liT ~ ~I ~

q'jfi{ff

'@;

fif; ;B

'@;

it
20

I P.T.O.
www.examrace.com

F-DTN-M-NUIA/15

14

www.examrace.com

'.
(1:9) fci[xy + y 2 ) dx + x 2 dy] iii

:!lim 'f>1 ~ ~.
3ft< y

(lll

x2

"[RT

fu1J. ~ il rtR if;

~ c~

qjf;

~. y

F-DTN-M-NUIA

rl'IJ 'f>TI

20

<rR; = y7 + tx- 2xzJ]- xyk, m


fftvxh;:; ds
s
'f>1
x2

~
+ y2 + z2

~.

= a2 'f>1 xy-~

~I

i\

~
'3>'n: 'f>1 '[!!
20

rrriFf; Wof ~

JJk Jiihft <fi;if it fJTlf ~I


w:if ~ JU< :Jift 'fTUf11 it iff& ;;rf.t ~ ~ Jmit; Jl'ffl- 'Til
it fit<Tr 1"f'H ~. 3Jk w 'fTUf11 'liT
~ JU<-~ ~ 'J&- '[II 'I<
Jifitn f.Wl! """"' 'I< fit<Tr "fRT
Jl'ffl-'W 'I< ~ 1f/!2Pl ~
:J7I'ilftm JP< fiRfl 'fTUf11 it iff& ~ JU< 'I< <it{ 3Ft; 'ff.ff fin~ I

.I
=

***

JJk 5 ~ fJ I 'IT'ft w:if it ii rrriFf; &"6 ii --ii-J?'n >Wf "' ~ "ifl;r w:if ~ JU< ~I
rrriFf; Wof ~ /ffr; fWrr 3Ft; Wof ~ 3iif it fiir: ~ {f I
>Wf

"d&:n

>&

it,

<it ~ ~

~I

rrrflq;;m '!Rfffir aro it ~ ff,

'liT TflR ~ if'-lT ~

f.Wl!

f.Wl! g1
~ f#w : '-W ~ ~ fit; Of<! >it fiRft >Wf 'liT JU< 1.' ~ eft, i7<f
m w-r ~ rr>ft 'lfl'if/"J'l-'lfl'if ~ JU< <m<-<m< ~~ W"ifiT wJ 7.W ~fit;
JPTit w-r 'liT JU< ffmR ~ fffr; 3Wt ~ ii 'f! ~ w-r ~ rr>ft
'lfT1if/"J'l- 'lfT1if ~ JU< WITfl it "f1li: I W <m'f 'liT ~ ii 3lJml1f ~I
JU< -~ it &Tf!t ~ S'f '[if <it ~ ii P<1! ~ ii 'liTC ~I &Tf!t
-.gZ S'f '[if ~ ~ iff& S'f "3ilif ~ 3Ft; '1 fiir: "f1lf:. ~ it WRfT ~I

F-DTN-M-NUIA/ 15

15

JS3-1100

www.examrace.com

Note : English version of the Instructions is printed


on the front cover of this question paper.
www.examrace.com

CS(M) EXAM2012

I F-DTN-M-NUIB

Sl. No.

Sedion 'A'
1. (a) How many elements of order 2 are there in the
group of order 16 generated by a and b such
that the order of a is 8, the order of b is 2 and

MATHEMATICS
Paper II
jnme Allowed: Three HoursjjMaximum Marks:

3001

-l_ -1

bab -a .

12

INSTRUCTIONS
Each question is primed both in Hindi and in English.
Answers must be written in the medium specified in the
Admission Certificate issued to you, which must be
stated clearly on the cover of the answer-book in the
space provided for the purpose. No marks will be given
for the answers written in a medium other than that
specified in the Admission Certificate.
Candidates should attempt Question Nos. 1 and 5 which
are compulsory and any three of the remaining questions selecting at least one question from each Section.
Assume suitable data if considered necessary and
indicate the same clearly.
Symbols and notations carry usual meaning, unless
otherwise indicated.
All questions carry equal marks.
A graph sheet is attached to this question paper for use
by the candidate. The graph sheet is to be carefully
detached from the question paper and securely fastened
to the answer-book.
Important : Whenever a Question is being attempted,
all its parts/sub-parts must be attempted contiguously.
This means that before moving on to the next
Question to be attempted, candidates must finish
attempting all parts/sub-parts of the previous
Question attempted. This is to be strictly followed.
Pages left blank in the answer-book are to be clearly
struck out in ink. Any answers that follow pages left
blank may not be given credit.

am!"~: ~if q;r ~ <!'Wil< ~ ~'f~'T51" it; mffl rpT

<rr

<'1T

~I
www.examrace.com

(b) Let
1
I'f X<--,

0,

f,.(x)=

n+1

7r 'f
Sin-,1
- -1E x E -1,

0,

n+1

'f x> -
1
n

Show that fn(x) converges to a continuous


function but not uniformly.
12

(c) Show that the function defined by

is not analytic at the origin though it satisfies


Cauchy-Riemann equations at the origin. 12

F-DTN-M-NUIB

(Contd.)
www.examrace.com

1. ( q;) a

am b ~

2~

ijJU

;;rf.m q;)g- 16 ~ ~ if q;)g-

m<r mA- ~ ~. ~ fifi a <fit q;)g- 8 ~.

<fit q;)g- 2 ~am bab 1 = ii 1 ~ 1

0,

I
~x<-.
n+l

.n..c.
fn ()
x = sm-, 'll'<
x

o.

12

I
n+l

.;;

(d) For each hour per day that Ashok studies


mathematics, it yields him 10 marks and for
each hour that he studies physics, it yields him
5 marks. He can study at most 14 hours a day
and he must get at least 40 marks in each.
Determine graphically how many hours a day
he should study mathematics and physics each,
in order to maximize his marks ?
12
(e) Show that the series

~ (~)n n6

n=l

I!"+

is conver-

gent.

I
x.;; -.

I
~X>-
n

Gll~ fit; f.<x) mrcr q;wr q;]- arfm:!fur


~ ((<tiij"'ii1 ~I

mm- ~'

12

2. (a) How many conjugacy classes does the permutation group s5 of permutations 5 numbers
have ? Write down one element in each class
(preferably in terms of cycles).
15

12
(b.) Let

t(x

v) = j(x2++yy):' if (x, Y) * (0,

'.

at

0)

1 , if(x,

y)=(O, o).

at

Show that - and -a exist at (0, 0) though


ax
y
t(x, y) is not continuous at (0, 0).
15

~ ID'U~ q;wr ~ ~m~~ ~.


~ ~ ~ ~ <mft-~ Wil<ii<i1" <!if~
<miT~ I

F-DTN-M-NUIB

12

(Conld.)

www.examrace.com

(c) Use Cauchy integral formula to evaluate


3o

Jc _e_
(z +I)4

dz, where c is the circle Iz I= 2.


15

F-DTN-M-NUIB

(Contd.)
www.examrace.com

('E1) ~i'tq; ~ 9ktt'G'1 ~ ~ llfUrcr q:;r m

~
'R, ~ I 0 ojq;'f oft 91fu ~ %aW ~
~ ~ 3fVI<fif ~ ~ trOZ ~ ~ ~
5 ojq;'f oft 91fu ~ %1 ~ g t=Gii 3lfu<n ~
arfuq; 14 trOZ 3lb"WR ~ ~ %aiR ~ ~
~ if q;11 ~ q;11 40 ajq; !mf 'fK'IT 3!W.f!!"Q; ~ I
llii'14d: ~ ~ f<l; 3!'f.r ojq;'f q:;r
arru'l>dlil'f>(OI ~ ~ ~' ~ 9ktf'G'1 mA"

rn

l:f1lZ llfUrcr am: mA" l:f1lZ ~ q:;r m

'fK'IT

.I

( S:)

~~ f<l; '-Mt

(~ ~

I (' _!!__)
n +I

11

11=!

arflmro ~ I

12

"lSliT ~~if)

15

( l!f) l!T'f ~

f(x, y) =

~~2

ax

q:;r

15
(b) Let f(x) be differentiable on
f(l)

such that

Jj 2 (x}dx"' I. Prove that

=/(0) =0 and

0
I

f xf(x)f'(.t}dx = -l

15

(i) I <lzl<3 (ii)


(iv)

4 dz, if>T't<4i'l>'1 ~~~<mih1l11'1><1

)~

z+ I z +3

lzl>3

(iii)

lzl<l

) in

O<jz+Ij<2
15

(d) Evaluate by contour integration


2n

[0, I]

15

'!illfm~tfll11<1 ~. ~c'f'flzl=2 ~I
F-DTN-M-NUIB

z.

Laurent series valid for

co. O> "<R 3!ffiRq ~ lRTfii

f(x,y)~~~(O,O)'RI

f (z + 1)

3. (a) Is the ideal generated by 2 and X in the polynomial ring z [X] of polynomials in a single
variable X with coefficients in the ring of integers
a principal ideal ? Justify your answer.

(c) Expand the function /(z) = (

'fR (x, y) ot (0, 0)


I , 'fR (x, y) = (0, 0).

:;:

~~ f<l; at am: aat


e3z

12

2. (q;)5~~~~~~ S5 ~mA"
~<mtmt~ 7 ~<mit~ <WI"~

(lf)

(d) Find the minimum distance of the line given


by the planes 3x + 4y + 5z 7 and x - z 9
from the origin, by the method of Lagrange's
multipliers.
15

15

(Contd.)

www.examrace.com

d8

l=J

l-2acos8+ a
0

F-DTN-M-NUffi

2 .a<l

15

(Contd.)
www.examrace.com

('f) Md<'ll 3x + 4y + 5z = 7 aiR x- z = 9 ~ iWT ;m

mlliT, ~~~~f<mr~iWT. ~~
~;.Umopr~l

15

3. (q;) ~ Z ~ <l<illf if ~ ~ l% ~ ~X if
~ ~ ~ <l<illf Z

[X] if 2 3lR X ~ iWT


<!f.ro 'J"I"IIil<'li illiT ~ 'JOI"tiil<'li tmfT ~ ? 3!'f.t
~ ~ <m if crt 9tWf ~ I
15
(19) lWf~f<t;J(x) atqq;i1.,14 ~ (0, I]

"fl:,

~fit;

f(l) =/(0) = 0 3lR f ! 2 (x) dx =I. fu:ir~f<t;


.
0
I

f xf(x)f'(x) dx = -!

15

4. (a) Describe the maximal ideals in the ring of


Gaussian integers z[i]={a+bila, be

20
(b) Give an example of a function f(x), that is

I/( I

not Riemann integrable but


x) is Riemann
integrable. Justify.
20
(c) By the method of Vogel, determine an initial
basic feasible solution for the following transportation problem :
Products P 1, P2 , P3 and P4 have to be sent to
destinations D 1, D 2 and D 3. The cost of sending product P; to destinations Dj is C ij' where
the matrix

[cij] =

(ll)

'lii1'f

/(z) (z+l)(z+3) 'fiT,

(i) l<lzl<3 (ii) lzl>3 (iii) O<lz+ll<2

Iz I< I

(iv)

~ ~ ~.

<'tRt itoiT if

~I

('f)

9lJR

15

2n

I=

f 1- 2acos8 + a , a
d(J

<I

15

F-DTN-M-NUIB

10 0 15 5]
7 3 6 15 .
0

ll

!3

The total requirements of destinations D 1 D 2


and D3 are given by 45, 45, 95 respectively
and the availability of the products P 1, P2, P 3
and P4 are respectively 25, 35, 55 and 70.
20
Section 'B'

5. (a) Solve the partial differential equation


(D-2D')(D-D') 2 z=ex+Y.

<i ijJijqj(1'1 ~ iWT !J><'<liq;., ~

(Contd.)
www.examrace.com

.z}.

12

(b) Use Newton-Raphson method to find the real


root of the equation 3x = cos x + l correct to

four decimal places.


F-DTN-M-NUIB

12
(Contd.)
www.examrace.com

4. (q;) 'lkmi'l ~

(c) Provide a compu ter algorithm to solve an

Z[iJ= {a+bi [a, bE

'JOI"'i<'lf.t<i'i

z} it

"f<'f11

if ~

q;r quf.1 ~I

20

Q:m 3<.;1~<"1 ~ ~.
m ~ fl'il'h<'t<flll "1 ~ lf(x)l d'1R

(13) q;wr f(x) <'liT ~

1 I 3m ~

fl'il'h<'t<fl'l

6~;\1$,'( I

20

(11") f.?tl'<lf.tfi'.~a qf<<'l~'l WlBlT it fuc;, ~ <fu fmr


it ID'U' affi:fi:lq; ~ mv:r Q\1" q;r f.ruRur
~:
~ P1, P2, P3 afR P4 'lit~ D1, 0 2 afR
D3 (!'<'~'; ~ ~ I '3C'lTG P; 'lit ~ Dj (!'<'~'; ~

<fu <'l1lm Cu ~. ~ ~

(d) Obtain the equations governing the motion of


a spherical pendulum.
12
(e) A rigid sphere of radius a is placed in a stream
of fluid whose velocity in the undisturbed state
is V. Determine the velocity of the fluid at any
point of the disturbed stream.
12

6. (a) Solve

5:
3 6 15.
0 11 9 13

[cu]=l7

~ Dl' D2 afR D3 <fu 'f\1" 3ll'fll~ ~T:


45, 45, 95 ~. afR ~ P 1, P2, P3 31\<: P4 <fu
20
~ ~T: 25, 35, 55 31\<: 70 ~ I

2
(D-2D ')(D- D') z =ex+.v Q\1" ~I

12

(13) f111i'l><ol 3x = cos x + I <'liT, '<'IT< ~lll"<1"'r

BWIT

<'!If(!

M'h 'rf ~ ~ it fuc(,

tmr q;r ~W'il\1

F-DTN-M-NUIB

~I

partial

differential

equation
~v=k
(b) A string of length I is fixed at its ends. The
string from the mid-point is pulled up to a
height k and then released from rest. Find the
deflection y(x, t) of the vibrating string. 20
(c) Solve the following system of simultaneous
equations,
using Gauss-Seidel
iterative
method :

3x+2 0y-z= -18


20x+ y-2z = 17
2x - 3y + 20z = 25.

1!is '13'
arif.lrq; ~ fl'll'h< 0 1

(!'<'~'; ~'

the

po o 15

5. ( q;)

ordinary differential equation dy = f(x, y) in


dx
the interval [a, b J for n number of discrete
points, where the initial value is y(a) = a,
using Euler's method.
12

'"jpl12

(Contd.)

www.examrace.com

7. (a) Find

~~

x:
y:

at x

=01

01
09975

20

from the following data:

02

03

04

09900

09776

09604

20
F-DTN-M-NUIB

10

(Contd.)
www.examrace.com

(tr)

.....
dv
f(x. y) ~ ~
dx
c!iT n Wol!T it fol{(, ~
[a, b] if,
Flfu ~ ~Rl'liH ~ ~. ~ q;f.r lf; ~ ~
#<

m'ji'(UI

~ Wft<l;{'l'f - =

ammr

~ ~~
I
y(a) = a

)IGR

~. ~ ~ lWf
12

(ll) ~ ('fforq; <fir l1fu ~ ~ ~ ~

12
<1>1 9TH~ I
(s-) ~ alt>~~~<fi'r ~~cmrcftmu
fl4ii'fi<Ui'f

6.

if m ~ ~. furifiT ~r-u amm if im v ~ 1


~ (ROf <fir ~ mu ~ f<nffi ~ 'R im <liT
12
f.ruRur ~~ I
<"'> f.tk!!'HK~a armfi'fi 3f'ICMf l:flili'fi<"' <fit ~

20
~ : px + qy = 3z
~I
fun 'R <im
( lir) .r.rr{ l ~ ~ <m 3f'f.t
~ ll'1>1f ~ ~ ~ k CICfi ~ ~ l1iRT
~ ~
~~I aik ~<riG M<l'lii1f41
~ ~ 1 ;:j:;qlWf aR 'liT ~m y(x, r) ~
20
~I

sarr

oo

(tr) ~-~:t:!~'J:i'li'fi f<m.r <liT ~J\&11<'1 ~


~. ~

. lt> f.ikllBR4<1 <ffl <fiT

~:
3x+20y -z=-18
20x+ y-2z = 17
2x - 3y + 20z = 25.
7. (Cfi)

20

Mklf'HMd ~ ~ :~ it X= IH ~ ~ :
X:

y:

F-DTN-M-NUIB

01
09975

(b) The edge r = a of a circular plate is kept


at temperature f( 8). The plate is insulated so
that there is no loss of heat from either surface.
Find the temperature distribution in .steady
state.
20

02
09900

II

03
09776

04
09604
20
(Contd.)

www.examrace.com

(c) In a certain examination, a candidate has to


appear for one major and two minor subjects.
The rules for declaration of results are: marks
for major are denoted by M 1 and for minors by
M2 and M3. If the candidate obtains 7S% and
above marks in each of the three subjects, the
candidate is declared to have passed the
examination in first class with distinction. If
the candidate obtains 60% and above marks in
each of the three subjects, the candidate is
declared to have passed the examination in
first class. If the candidate obtains SO% or
above in major, 40% or above in el!ch of the
two minors and an average of SO% or above in
all the three subjects put together, the
candidate is declared to have passed the
examination in second class. All those
candidates, who have obtained SO% and above
in major and 40% or above in minor, are
declared to have passed the examination. If the
candidate obtains less than SO% in major or
less than 40% in any one of the two minors,
the candidate is declared to have failed in the
examinations. Draw a flow chart to declare the
results for the above.
20

F-DTN-M-NUIB

12

(Contd.)
www.examrace.com

(1!1") ~ 'f<ll'hl< ~ ~ ~ r =a~ <ffCf f(IJ) '~':

oo -:mm ~ 1~ d'>lll11<1m ~ ~ Fcnm ..n- 'fll ~


~ <fi ~ "if ~ I Bll'ft 3f'W1T ~ <IT'f ~
~~I

8. (a) A pendulum consists of a rod of length 2a and


mass m; to one end of which a spherical bob of
radius a/3 and mass 15 m is attached. Find the
moment of inertia of the pendulum :

20
(i) about an axis through the other end of the

m"N ~ ~. 3A1I~Gtl< ~~~aft<: Fit


~ mm ~ ~ ~ mm- ~ 1 qf< 1111i <fr

(if) ~

rod and at right angles to the rod.

15

futJ; afq; M I ~ "[RT


M 2 oft<: M 3 ~ "[RT ~

~ ~ f.r:r:r ~ ~ ~ ~

aft<: ~ ~ futJ; afq;

~ ;;rffi ~ 1 ~ ~ 3A1i<;Gtl~ <fr;if


75% l:fT ~ d'>'l': afq; 9"ffi ~ ~
''m'S

FoR ftlwm..,

3A11 i;Gj 1~ ~ <fr;if


~ ~.

~ ~

mm ~

60%

mm ~
m~

-:mm ~ 1
aft<: d'>'l':

afq;

-a=:fi<;Gtl~ ~ ~ ~ 50% l:fT

d'>'l':, ~ ~ ~ 40% l:fT d'>'l': afq; 9"ffi

(Given : The centre of mass of the


pendulum is a/12 above the centre of the
sphere.]
15

m3A1i<;q I<~ 'g>:n1 ~ ~ mr' ~

-:mm ~ I ~

(ii) about a parallel axis through the centre of


mass of the pendulum.

Wif ~.

(b) Show that l/J =x f(r) is a possible form for the


velocity potential for an incompressible fluid
motion. If the fluid velocity q ~ 0 as r ~co,
find the surfaces of constant speed.
30

aft<: f!'l1l mm ~ 9"fl'liq:J <fi offi:r<1" ~ 5o% l:fT d'>'l':

m~

~ ~ ~ mr'
~ ~ 'iiT<1T ~I 3'1 frlll '3Afi<;Gtl(l. ~. ;;jT
afq; 9"ffi

Wif

~ ~ 50% l:fT d'>'l': aft<: ~ ~ 40% l:fT d'>'l':


afq; grn ~ ~. 'qrn' ~ ~ -:mm ~I~
3A1i<;Gt I< ~ ~ ~ 5o%
40%

B- l:fT Fcnm ..n- ~ ~

B- <m afq; 9"ffi Wif ~ m'311 3A1.1@~

~ ~ qf<o1111i.

<fi

'i'Jwr' mMcr ~ 'iiT<1T ~I

~~~fut:;~~'Gtli~l

F-DTN-M-NUJB

13

20

(Contd.)

www.examrace.com

F-DTN-M-NUIB

14

www.examrace.com

am !iCQ11'1 m ~
<fiiT S3lT ~ I ~ 1:(CI'i fm: IR ~ a/3 am
I:CQ"' I'1 15 m CfiT 1:(CI'i 1'til <tl I< OlG<flif ~ S3lT t I

8. ( q;) 1:(CI'i offi;rq; 2a ~ <lit 1:(CI'i ~

~ CfiT ~ 3fT'i{Vf

o)

+ffii"" ~ :

~ ll> ~ rn=t ll> ci'Rr ~ ~ <l'ffi

ll> a11<i'lo1 am ll> ~ 1


(ii) ~

am ~
15

ll> I:CQ"'I'1-~ ll> ci'Rr ~~am

ll>~l
[lffi: ~ CfiT I:CQ111'1-~ ~ ll; ~

~ a/12 ~ ~ I]

15

(<9) ~~f<f; 1/J=x f(r) l:(CI'i"lfi'ii:s>l ~1lffllj;iffl'


~lj;~l:(CI'i~~~ ~~~ r~oo, ~
~ im ~ o ~.
<>~4f<'lcif <:'R!R ll> q;)-

-q

+fl' ~ I

I F-DTN-M-NUIB I

30

~~T
~ 'FI"if ~ 3/k afit::;ft ifT;ff if WIT ~ I
'FI.n' it; :Tff{" ~ "fWP'T if fi!r&- ~ ~ ~

.I

3fTTft rnr-'f'Jf if RP:H lfl/T ~. 3/k w "fWP'T q;r <'TI! ~


:Tff{". ~ it; p;r-% 'K oiFcno f.iffr! "ffT'f 'K RP:H ;jf"('ifT
rnr-'f'Jf 'K JRifi}w >m?N it; 3ffcrfun 3PI< fit;ifr
"fWP'T if fi!r&- >rr; :Tff{" 'K <iff aicn '1if fTrifir I
'FI"if .wrr I 3/k 5 3ff.jqpf &' I ~ IFI.n' if it ~ 1?fU$ it
<li11-it-<li11 ~ 'FI"if ~ ~ rWr 'FI;ff it; :Tff{" ~ I
zm- ~"l{<f; efT fit J'1'Jffi ~ q;r "'flFf" <li'tf;ro: fM ~
f.iffr! ~I
~ 3/k ~
lfliT efT I

it; ~ ar<f &',

'Jf<T crq; 3fRP:IT "if

-.nit IFI.n' it; aicn IF1T"if ff I

w 'FI"if 'fJf it; <TN JJ4i~qf<)' it; JCPim it; fi'rv: m'1i ~~ ffi;rg

~ I m'1i ~~ q;'f ~q;- 'FI"if 'fJf it 3f1'f1T <R: :Tff{"


<TN ~ <Rtf
if 1
~ :m<fi/7<Efi ~ fc'n ..,.,.
fii;;ft- ~.., 11iT ;;>'ffi" ~ ~ if, 1111"
'3ff ~'f ~ mit IWif/'3fT -~WiT ~ ;;>'ffi" 'fiN- 'fiN ~ I
'fR11iT 3l'>f ~ ~ Fc;; af7Tiit '!l~'f 11iT ;;>'ffi" fitiR ~
arrit
~~..,~mit 1Wff/'3'T-1Wff ~ ;;>'ffi"

rfifm t

m- rri

omr

fmr

~it;;rfl(l ~<mr11iT~it~~l
if 1ffT1'ft ;# ~ '[if q;'f mit if <'TI! IPl it q;rc
if I 1ffT"i'ft ~ 'FI.n' it; om- fi!r&- ~ ~ it; aicn "if ~
omf, i!Jrr efT ~ ~ I
'

:Tff{"

F-DTN-M-NUIB

rfifm
fF

Note : English version of the Instructions is printed on


the front cover of this question paper.

15
www.examrace.com

www.examrace.com

41

fO I a (!n';ftf?T I)

f44'JIRI/ DETACH ABLE

MATHEMATICS (Paper I)

1.

~~-.=IT~~~:

l.(a)

~ tffi:n ~ CfiT ~'ifli(YJ ~ ~' ~

A=[~ ~1 ~ 1

~aicn: 250
Maximum Marks : 250

Time Allowed: Three Hours

3 2

~~~~Ai~T
~ R:c<..dli\1 ~ ~- ~ ~ ~ f.i"i~n
mo ~;:ff ~ ~ ~ it W il7:IT ~ atR ~ ~
\3uflGCII<

cit ~

.q- ~ ~ Ullii'tclCfl ~ ~ 1

x+3 y+z =I0

amrr if m-crr ~ ~ I

2x-y +7z= 21

~ ~<ff ~ \3m ~ ~ I

WFf l ~ 5 ~~' ~if~ "ffi<:f <til~~~~~ ~~~~~~~ I


~rr;at~T ~ 3lCfi \m ~ ~ ~ ~ ~ 1
\3'm ~ lfTC7:llf if~~~

\ill- ti lflfth&ic if~~ I \ffiCfiT ~ ~<1-~-~ (QCA) ~ ii


FrufRcT ~ tR ~ ' CWTI ~ ~ I ~ lfTh<:{lf ~ M'?f ~if ~ ~ tR cnW afcfi ~

l.(b)

l.(c)
l.(d)

Rm~l

. ~ ~ 'CR, ~ ~ +rr<r ~. \3"U CfiT ~ -~: ~<:r ~I

m, ~ ~ ~R 3ff1l cffi: tR ~ m+lFlf ~ CfQ'1 ~ ~ I


~~;:if ~ SllB IR41 if FAT~ I ~rr ~lcfi ~ if fci:;<:rr ~. at mfiAT ~ ~
~ ~ CfliZ
~ ll<IT mI~ mffi tim liT af~T ~ ~ 'j)RaCfll if~ ll<IT t ~ ~: ~<:r CfiTC
~I
~~~~

l.(e)

2.(a)( i)

QUESTION PAPER SPECIFIC INSTRUCTIONS


Please read each of the following instruction s carefully befon: attempting questions.
There are EIGHT questions divided into two SECTIONS and printed both in HINDI
and in
ENGLISH.
Candidate has to attempt FIVE questions in all.
Question No. 1 and 5 are compulsory and out of the remaining, THREE are to be attempted
choosing at least ONE from each section.
The number of marks carried by a question/pa rt is indic~ted against it.
Answers must be written in the medium authorized in the Admission certificate which
must
be stated clearly on the cover of this Question-cu m-Answer (QCA) booklet in the
space
provided. No marks will be given for answers written in medium other than the authorized
one.
Assume suitable data, if considered necessary, and indicate the same clearly.
Unless and otherwise indicated, symbols and notations carry their usual standard meaning.
Attempts of questions shall be counted in chronological order. Unless struck off, attempt
of a
question shall be counted even if attempted partly. Any page or portion of the page left
blank
in the answer book must be clearly struck off.

.:

-d

CJ)TgfdJIJi ~~! ~~tiiilCh{UJ)'iti~


~~~

a-brl-m-nbua

3x + 2y - z = 4

10
~~Cfll~A31R~~A* ~~~
fci131~AA* 3-lRA*A ~~
1=ff.1cmaf4Cf1~ I aM~~fci1~(AA*)=
~(A*A) I
10

JO[~iCfi'1 ~ f~(2x sin~- cos~)dx 1

10

"Q;CFi ~ ~ CfiT ttmCfl:COI ~ cfilMQ


) ~ ~ (0, 1, 1) ~ (2, 0, - 1) ~if 4J4Rdl ~

3TR~~c-1, 1, -2), (3,-2,4)tit~


cm;ITm~wricR~ 1~~mooam:
WRA" ~ ~ ci?t ~ ~:fhrr cFi1 Ni Q) 1
1o
~ ~ S ~ O!ffif ~ 3lll:R -~ ~ fffiT
lR ~ (0, 1, 0), (3, -5, 2) ~I ~ ~
tiJilCfi(OI CfiT ~ ~' Mti<:fll ~
S CfiT ~ 5x- 2y + 4z + 7 = 0 ~ fWt 9 fd'O)
~
~ffi~~if~ I
10
~ fci1 Pn 3ffCl'Cfii~T n lR ~ ~ ~ C4 ltafctC
h ~ cf."T ~T ~<if mRm CfKffi
~
aft< fcF. T: P2 ~ P3 Pl~RiR<ld IDU~ 'Q)Cfl~ Zqia<:ot
~:

T(p(x )) = f:p (t)dt,


P2 ~1"\ P3 ~Sfi+f~T: {1, x, x 2 }

p(x)E P2

3ft< {1, x,

I+x 2 , l+x 3 } 3fTmU~~i:f TCfiT ~

~~~~~~ rcfil~~~m
+n~1

10

2.(a)( ii) ~ fcfl V "Q)q) n-~ ~T


~ ~ alR T: V ~ V ~ ~&iJiOftli ~
tfCfli(Cfi ~ I
~ f3 ={Xt, Xz, .. ., Xn} arrtTKm VCfiT, aT~
~~ {3'= {7X1, TX2 , , TXn}
V CfiT 3TTUR ~ I

m
8

2.(b)( i)

~ fcfi

A=

r: ~2 ~ l~
J
I

(J)

w(>' !)

(1)2

~'l>T'f'lli\'f~ !'!RAt. A2. A, fflfcrcr~

~A 2 ~~1WITCJ)T, m~~fci11A.ti+IA.21+1A.31
a-brl-m-nbua

~ 9.

2.(b)(ii) Pt~RtR.cta ~~~<il'+rf<ilM~:


1

2 3

A=

2.(c)(i)

5 8 12
8 12 17

5
5 8 12 17 23
8 12 17 23 30

. .. ,

Xn

m,

5.

~ 9"~'11 ctll ~ ~

S.(a)

y~llWR~xCl1T, ~mfctl~~ dy HkiRlR<ld ~~~


cos(x + y) + sin(x + y) I x
B'~~ I

WRf 3lW1'=f~T~ at,~~~ nxn ~

~T Xk CfiT
011clliion<:f mar~ I
2.(c)(ii) ~~ fcfl c;3 it ~T XI = (1, 1+i, i), Xz = (i, -i, 1- i)
Clttt~FctCfl ~ ~ ~ lR tR<~Cfia: ~ ~, ~ ~
~~I

.. ,

iln ~I~

~ = a sin ne~

S.(b)

S.(c)

~ cfiT ~ ~ O!fCI~R CfK 00 y = 10 - 2x

yz

S.(d)

aW Gl-cf~'d - +- = 1 ~
q.. ~

~~~ciilM~I
3.(b)

x2

20

S.(e)

3.(c)

JJ xy dA

x(t)=ti + -1 +-

4.(a)
4.(b)

4.(c)

y2

3Tmrlf~~~~\ilT~~ I
15
~~iwCfiT~411~~4t crsfi'~~if~xz + yz + 2ax+ 2by =O,z =O~affi:~~WR
~ (0, 0, c) ~G{Tq~ ~~I~~ y 0 ~ IDU ~~ CfiT 4~~ ~ aillfdiCfiR

t) j+
A

-1 - t-

Jk

~ 0~ ~ I

~ ~ ~ 1~e1 'I:

~ fcl'"<HUJ
2

6.( C)

10

tt+ilCfi(OJ
2

x2
6.(d)

d
d
-t
+ x_l'_ + y = lnx sin(lnx) CfiT
dx
dx

tll+tlf'l4

~~~I

IS

~~Kif
t

=otR x = o~ dx =o~ are:fr.:r ~ ~snCfl<Ot


dt

{ D + n 2 )x =a sin(nt +a), D 2

15

~ ~ p ~ p'

if
7.(a)

!:

CflT ~ ~ fcrfu CfiT ~~+tt&l

~I

a-brl-m-nbua

~~~

~' ~ a, n ~ a HllC1iCfl ~I
15
25 kg a~llH CB~ChUT 09 m ~~~~RR~~~%, Mt!Cfil ~fi:RT
~ @:R ~ ~ m~ '.f ~ ~ I q;ur cit~ 8 m~. ~ tTm ~: g~ fcfim "fRRT
~I

\1fGf ~ (i) ~, (ii) ~w ~~aiR~' ~ Cf)Uf ~ ~ ~ <ffiT


if m

10

10

fcrf'U '[Kf ~ tl+ilCfl\UJ

d2
--?
+ a y =sec ax~~ chlMI:!; I
dx

3W

6.(b)

m, mtnfur~ ~ ~M Pt~RtRcta W:R 'fCf l:Rmm :

x2 + y 2 + z2 + 2ax + 2by = 0
2ax+2by+ cz=0.
~ qf<ctctf ~~'(c{ ~ ~ G) ~~"<c<l ~ ~ ~.

10

~ tl+ilCfl\OJ <it~~
(5.x3 + 12x2 + 6y2)dx + 6xydy = 0.

= 2x + 6 $ IDU

ror

6.(a)

15

= x - 1 atk ~(I~Stl('ll

qf.<Gt4:
~I
15
~~fcfl~2(x2 + y2 +z2) = 3r2lRfcfimm~~~x2 + y2 +z2 = r20CficfR'

~~~q{;sfl('l,

CfiT 'i~iCfl'1 ~' ~ D, ccT y

~snCfl\OJ 9T8" ~I

m ~ 00 OPQ if "fR(13fTcrcf rrfu (S.H.M.) cg ~ ~ I ~ ~ P 3fR Q tR ~~

-7

xy3

10
q)f

cgcrr ~ 8 kg CfiT ~ <Sf(>'!" 20 kg ~ ~ CflT ~ ~ aW ~ CflT lffi'"Cfl<'f


~~~ I
~ affi: ~ ~ ~ 'tf!fur ~ ~ ch\PIQ) I
10
~~ fcfl C@l

fry (0, 0) 6ffi:Jyx (0, 0) ~ ~ ~ qf'(Cf\(1'1 ~

) x+y2 , (x, y)=F(O, 0)


!(x, Y{ 0 , (x, y)=(O, 0).
fxy am /yx cfi (0, 0) IR frl{ct<dl lR ~~I

~ C@)-~ ~ ('j~Cf)lOnll ~

~~' ~O~~~T :x 31'Ry~31'R~ ParRQ~~+t"m-~~~v~ I


~~~Cfil~~~ I
10
~~~Cl1T3fTCTR~if 4 ~affi:~if 3 ~~I ~~~Cf111f

am: x3 = (0, 1-2i, 2- l)

'[Kf

3fK y ~ ~ ~u ~ ~' ~ M- 31C!Cfl(1'Jl /~

(r, e)~~~~Ticn~

C, ~ kciT~

mrm ~ ~ 1:R tR.ctCfla:

3.(a)

dx

~fcfl A~ ~tfifcl ~~, fui<t~~3TWRifrf il 1, il ,


2
X1, X2 ,

' '

~~

a-brl-m-nbua

c::tlT ~

7 .(b)

7.(c )

~ Q)Ch~~

H ~ f~ ~ ~ 45o ~ ciTu
r l:R 00 ~I~~ fflu ~ 3itci
TCR
~lRRcnr~ afR~fuu~
lR~~ I~ J.LafRJ.L'~T : ~afR
~~
Gf
t:
q
am "ffhtT 3fR: ~ ~ ~ ~ 'Cftfur ~ ~ ~:,
en-~~~ tW CfiT ~
cfu
d{q i
KCl~Cfil~ ~ fW; ~<rcfi <'lf"idJi
~ *1 ~

~ ~ cfiT ~ AB, BC, CD


, DE, EF

re-u~~~~~~~~fcfl
~~GM

1.(d)

l .(e)

~I

aTK

15
FA,~~~ CfiT ~ W
~' ~

"lJIIT~ 1 ~ ~ ~W:r
ffi~~

~ ~ ~ AB ~DE ~ lim
~~~~
S.(a )

S.(b )

8.(c )

V 2 (rn )"CfiT qRCfi&FI

2.(a)(i)

. ~ I miT if CFfTCf ~ cf})P!l!) I

eN Ml!) am: r ~ n ~ ~ ~ \31fCf)f ~

Find the matrix of T with respect to the bases {1, x, x2 } and {1, x, 1 + x 2, 1 + x 3}
of P 2 and P 3 respectivel y. Also, find the null space of T.
10
2.(a)(ii) Let V be ann-dimensional vector space and T: V ~ V be an invertible linear
operator. If fJ = {X1, X2 , , Xn} is a basis ofV, showthat{J' = {TX , TX , , TXn}
1
2
is also a basis of V.
8

1o

JJ(a 2 x2 +b 2 y2 +c2 z 2 f 21 dS CfiTt;[f.>lliCfl'1


CfiT ~ ~ I a, b

8.(d)

00 f!Ji1Cfl<1
~' ~

1.
l .(a)

chlMQ), ~ S <n*a~Nt ax2 + by2 + 2


cz = 1
aTtt c ~ ti'11~JiCfl Rllct iCfl ~ I

2.(b )(i)

15

Ic(- y dx+ x dy- z 3dz) CfiT


Jff.>lliCfl'1 ~ ~ ~

Let

fitCfl CfiT ~ Ol:fct~R

z = 1 CfiT Slkt:.,g~.., I

Answer all the questions :


Fin d the inverse of the mat

rix:

A).
10

J~ (2x sin-} - cos }) dx.

10

2 3
3 5

12

A=

by using elementary row ope


rations. Hen ce solve the sys
tem of line ar equations
x+ 3y+ z= 10
2x- y+ 1z = 21
3x+ 2y- z= 4
10
Let A be a square matrix
and A* be its adjoint, show
that
the
eigenvalues of
matrices AA and A*A are real.
Further show that trace (AA
*) = trace (A*
Evaluate

where ro("' I) is a cube root of unity. If A1, A 2 , A denote


3

8 12 17
5 8 12 17 23
8 12 17 23 30

1 3 1]
[3 2 -17

l.(c )

m2

A= 2 -1

l.(b)

!]

the eigenvalues of A 2 , show that IA- 11+ I.A. 2l + IA-31 ~ 9.


2.(b)(ii) Find the rank of the matrix

15

SECTION 'A '

A=[~ ~2
1 m

c ~ mf&t"s{ x2 + y2 =1 3fR ~X+ y +

p (x)E P 2 .

~ (x, y , z) cfu~~' n ~ Rllc


tiCfl ~am: V 2 ~ ~Cfli{Cfl
~I
10
~T if ~ CfSfl RkiHR.ctct
~T ttmCfl{O( ~ GRT ~
~ 1 = t 2i + 2tj - t 3k
~ t = +1 ~ t = - 1 ~ ~
CfSfl ~ roftmarr $ "if:q
cilvr CfiT frra

~~6jqft{OI ~CfiT ~~~


1(1 ~~' ~-~~1Cfl<1

Let Pn denote the vector space of all real polynomials of degree atmost n and
T : P2 ~ P3 be a linear transformation given by
T(p(x)) = ( p (t)dt,

15
~ ~ I r ~ ~ fctim

Rur c:hl M\.!. 1

Find the equation of the plane which passes through the points (0, 1, 1) and
(2, 0, - 1), and is parallel to the line joining the points (- 1, 1, - 2), (3, - 2, 4).
Find also the distance between the line and the plane.
10
A sphere S has points (0, I , 0), (3, - 5, 2) at opposite ends of a diameter. Find
the equation of the sphere having the intersection of the sphere S with the plane
5x - 2y + 4z + 7 0 as a great circle.
10

a-brl-m-nbua

2.(c)(i)

II

Let A be a Hermetian matrix having all distinct eigenvalues A.b A. , . .. , An. If


2
X1, X2 , . , Xn are corresponding eigenvectors then show that the n x n matrix C
whose kth column consists of the vector Xk is non singular.
8
2.(c)(ii) Show that the vectors X 1 =(1 , 1+i, i), X2 =(i. - i. 1- i) and X =(0. 1-2i, 2- i) in 3
C
3
are linearly independent over the field of real numbers but are linearly dependent
over the field of complex numbers.
8
3.(a)

Using Lagrange's multiplier method, find the shortest distance between the line
x2

y = 10 - 2x and the ellipse a-brl-m-nbua

y2

+-

9
6

=1.

20

3.(b)

Co mp ute fxy (0, 0) and


fyx (0, 0) for the functio
n
.xy3
!( x, y ) -- x + y 2 , (x, y); t {0, 0)
{
0, (x, y)= (O ,O) .

4.( b)

6.(b)

Also, discuss the continu


ity of fxy and /yx at (0,
0).
Evaluate fJ xy dA , where
15
D is the region bounded
by the line y = x - 1 and
D
the
parabola y 2 = 2x + 6.

3.(c)

4.( a)

6.(a)

15
Show that three mutually
perpendicular tangent line
s can be drawn to the sph
x 2 + y 2 + z2 = r2 from any
ere
point on the sphere 2(x2
+ y 2 + z 2) = 3r2.
A cone has for its gui
15
ding curve the circle 2
2
x + y + 2ax + 2by = 0, z = 0
passes through a fixed poi
and
nt (0, 0, c). If the section
of the cone by the plane
is a rectangular hyperb
y 0
ola, prove that the vertex
lies on the fixed circle
xz + y 2 + z2 + 2ax +
2by = 0
2ax + 2by + cz 0.
A variable generator me
15
ets two generators of the
system through the extrem
B and B of the minor
ities
axis of the principal elli
ptic section of the hyp
erboloid
x2
y2
2c 2
+
z
= 1 in P and P. Pro ve
2a
2b
that BP B P = a 2 + c2
.
20

6.(c)

dx
Find the general solution of the equation

6.(d)

x 2 d ; + x dy + y = lnx sin(lnx).
dx
dx
By using Laplace transform method, solve the differential equation

( D 2 + n2 )x =a sin(nt +a), D 2

4.(c )

x = 0 and :

7.(a)

t
j

Answer all the questions


:

5.(a)

y is a function of x,
such that the differential
coefficient :
cos(x + y) + sin(x + y). Fin
d

derivative/ differential.

5.(b)
S.(c)

5.(d)

5.(e)

7.(b)

SECTION 'B '

5.

out a relation between x

7.(c)

is equal to

and y, which is free from

any
10
Obtain the equation of
the orthogonal trajectory
of the family of cur
represented by rn = a sin
ves
ne, (r, 0) being the plane
polar coordinates.
A body is performing S.H
10
.M. in a straight line OP
Q. Its velocity is zero
P fu"1d Q whose distan
at points
ces from 0 are x an dy
respectively and its vel
the mid-point between
ocity is va t
P and Q. Find the tim
e of one complete oscilla
tion. 10
Th e bas e of an incline
d plane is 4 metres in
length and the height is
A force of 8 kg acting
3
metres .
parallel to the plane wil
l jus t prevent a weight
from sliding down. Fin
of 20 kg
d the coefficient of fric
tion between the plane
weight.
and the
Show that the curve
10

S.(b)

~x(t )= ti +

S.(d)

2
t Jk lies in a plane.

-1 +-t) j + 1 - t
A

'

10

Solve the differential equation


(5.i3 + 12x2 + 6y2 )dx + 6xydy = 0.
10
Using the method of variation of parameters, solve the differential equation
d2
--{ + a 2y = sec ax.
10

S.(a)

S.(c)

t =

:r:

subject to the initial conditions

0, in which a, n and a are constants.

15

A particle of mass 25 kg hangs at the end of a string, 09 m long, the other end
of which is attached to a fixed point. The particle is projected horizontally with
a velocity 8 m/sec. Find the velocity of the particle and tension in the string when
the string is (i) horizontal (ii) vertically upward.
20
A uniform ladder rests at an angle of 45 with the horizontal with its upper
extremity against a rough vertical wall and its lower extremity on the ground. If
J.l and J.l' are the coefficients of limiting friction between the ladder and the
ground and wall respectively, then find the minimum horizontal force required to
move the lower end of the ladder towards the wall.
15
Six equal rods AB, BC, CD, DE, EF and FA are each of weight Wand are freely
jointed at their extremities so as to form a hexagon; the rod AB is fixed in a
horizontal position and the middle points of AB and DE are joined by a string.
Find the tension in the string.
15
Calculate V 2 (rn) and find its expression in terms of rand n, r being the distance
of any point (x, y, z) from the origin, n being a constant and V2 being the Laplace
operator.
10
A curve in space is defined by the vector equation 1 = t 2i + 2tJ- t 3k. Determine
the angle between the tangents to this curve at the points t = +1 and t == -1. 10
By using Divergence Theorem of Gauss, evaluate the surface integral

fJ (a 2 x 2 + b2 y 2 + c 2 z 2

f 2 dS, where S is the surface of th~ ellipsoid


1

ail+ by2 + cz 2 =1, a, b and c being all

positive constants.

15

Use Stokes' theorem to evaluate the line integral Jc(-y3dx+ x 3dy- z3dz), where
C is the intersection of the cylinder x2 + y 2 = 1 and the plane x + y + z = 1. 15

a-brl-m-nbua
a-brl-m-nbua

= 0, at

15

IA-BRL-M-NBUB I
liNIH

(11{Z{ :

SECTION A

I MATIIEMATICS

~-~

fr?.Tffm

~A

II I Paper II

rft;;r eR

~Jicn : 250

Time allowed: Three Hours

Maximum Marks : 250

~-la ~ ~ fufm ~
~ ~ CfiT TTR #

'f1 -pi f:;dP,f(sm ~~cit t4Mlj\Cf4'! ~:


w# J110 JTR ~ ;W c) "l!fUS1 # fcrmfJJ?r ~ rrm fRt 3/k ~ c);ii # W ~ 1
r:rftm2/f oil !Jif? r:ifw w;ff ~ 3rt~ ~ ~ 1
JTR mr 1 3/k 5 31f.tcrr4 ~ rrm ~ # 8 JRitq; (5j7lg 8 cnrr-if-Cfill 'f!C1i JlR :flct>< fcn::tl rfR w;ff ~ Yff( ~I
~ JTR/'J!J7T ~ 3f<li ~ "8J"lR ~ 1fli ~ I
w;ff ~ Trfr Yfft 1!1~ # ffr& \if"R ~ ~ ~ ;mrr#; JT<ffl- TT:l 1/ fcMr 1J71T ~ 3fk W" lll~W Cfi7 PlOC ytj"&
JrR- ~- Yff( (CR{._. dt. '([.) !f{itrCfi7 #> !{&- i[Od f[( 3ifcf;rr ~ ?=21R r:p: frhm \ifR7 ~ I 3ftiff&rr llJUI7l ~ 3T#rfm; 3FZr
#!iift 1!TF.Zf11 # ffRi 1fli Yff( W CfiTI .Jicn -.:rtf frrWt I
~ ~ ~ cit ~ 3lfcr;if Cfi7 ~ ~, am :r.:rci't Rfffoc cfitfJrr< 1
vw 'ffCfi ~ -:r ~ ~ rrm :tl&:;!ctdl ;rqffrrr 1TRCfi 31gff # ~ ~ 1
w;ff ~ :J"ff<l cfit 7fURT fthJfljHI< cfit ~ I ~ Cfi7CT ;:rtf ~ rrl JlF1 ~ Yff( cfit 7fURT rift ~ ~ ~ Yff( ~: ~ 1J71T
P I Yff(-gfitrCfi7 # ~ ~T 3fT '[tf "llT ~ 3ffl oil fW ~ 8 Cfi7CT ~ ~ I
.

Question Paper Specific Instructions


Please read each of the following instructions carefully before attempting questions :
There are EIGHT questions divided in two SECTIONS and printed both in HINDI and in
ENGLISH.
Candidate has to attempt FIVE questions in all.
Questions no. 1 and 5 are compulsory and out of the remaining, THREE are to be attempted
choosing at least ONE from each section.
The number of marks carried by a question I part is indicated against it.
Answers must be written in the medium authorized in the Admission Certificate which must be
stated clearly on the cover of this Question-cum-Answer (QCA) Booklet in the space provided. No

(c)

11R ~ f(x)

=
x2

=--+2 ~ x<O
2

CRn f, [- 1, 2] 3'jo:('l(l(1 it~ fll""llet'><:1""1l4 ~ ?

~
(d)

feh g' (x) = f(x)

m? ~ ~

Cf;T

acfl ~

~ cf))~q: ~ ~ b ea + 1 < 1, ~ a

(f2ff

zne-a-bez ~~~.q n ~~~I

marks will be given for answers written in a medium other than the authorized one.

Assume suitable data, if considered necessary, and indicate the same clearly.
Unless and otherwise indicated, symbols and notations carry their usual standard meaning.
Attempts of questions shall be counted in chronological order. Unless struck off, attempt of a
question shall be counted even if attempted partly. Any page or portion of the page left blank in the
answer book must be clearly struck off.
A-BRL-M-NBUB

(e)

~cna41Cfl\ 0 1 c:f)l~c; z = 2x1 + 3x - 5x


2
3

~ fcn

A-BRL-M-NBUB

'

x 1 + x 2 + x3

=7

CRff ? CRn ~

~ g

"Cf)T 3'lff'dfCI

;:f))~ q: I

b 'tHklOJCf>

3fu:

CII'Rtfc!et>

"ffi ~

(a)

Show that the set of matrices S ={[:

- :)

a, b E

R}

(d)

is a field

what is the inverse of

=(

(a)

S defined

x2

(c)

Let f(x)

Q2.

(a)

and

2x1 -

R~f<:1fuld !*>~=t:!~l ~

(b)

Xj_?:

Q3.

(a)

(b )

13

liR ~ J ={a+ bi I a, bE Z} Jl13fflll ~ ~ ~ (C ~ JqqC1<!f) ~ I


J f?!l=i R1 ful d it ~ ent.:r -m ~ : ~fcR:rcil ll ~ (>rRr) , ~ :!01\ill Cl cll s1iR
(>rRr), ~ :!OI"'!@sOi ~ (>rRr) ? ~ ~ ~ ~ ~ 4>1Mt!> I
liR ~ RC = [0, 1] "CR

tNT

CII'R1 flCf> lO! HiRf>d

~~
. (f + g) x = f(x) + g(x)

(f~

llR

~ M, R <tt ~ ~ :!OI\l11Cici1 ~ ? ~ ~ ~

(1 2 3 4 5) (6 7).

(c)

mrt

~ M = { r e Rc I~i) ~ o }.

llR ~ f(x, y)

3lf'EfCf)(fl1
(d )

= y2

n s

< n + 1

~ n ~ ~'

m.[x] = n

2
( [x] + 3) dx.

-1

A-BRL-M-NBU B

~_-m-en: f(x, y) ~

mRrn

Cf){ffi

I Cf<!fT ~ f(x)

.q "fu:rR fllOfiCf>('Fflll ~ ? ~ ~' oT '810f$ll~l!> Cf<IT

mqftCflk=td ~
3

3l~ ~ d q ~ ?

(c)

dCfi ~ ~

+ 4xy + 3x + x + 1 ~ I
2

liT-1 ~ [x ] C::tl'k'!f"ClCfl ~ x CfiT ~ mJT

[-1, 2]

...

B"ffif ~ ~ ~' ~ f.:n:;r

Bf9t><!! l3"!l ~

~ ~ ? CFif ? ~ ~ ~ 31W1cT CflT ~


~ ~ 31W1cT ctt ~ ~'licf
?
3<;1 ~{01 ~ I 8 10 # ~ ~ ~ ~ ~

A -BRL-M-NBUB

is uniformly convergent but not

Show that every open subset of R is a countable union of disjoint open


14
intervals.

10

0.

<:hlfl"-41 ~ ~

1 8 7 3 10 5 4 2 6

810

n +x

(fg) x = f(x) g(x).

10,

1 2 3 4 5 6 7 8 9 10
(

(- l)n - 1
- 2- ,
-

(d)

subject to

810 ~

absolutely for all real values ofx.

z = 2x 1 + 3x 2 - 5x3
x 1 + x 2 + x3 = 7
5x2 + x 3

10

(1 2 3 4 5) (6 7).

Show that the series

ifx~O

1
Prove that if b ea + < 1 where a and bare positive and real, then the
10
function zn e- a - b ez has n zeroes in the unit circle.

Maximize

J and

(c)

CXl

--+2 ifx<O
2
Is fRiemann integrable in the interval [-1, 2]? Why? Does there exist a
10
function g such that g'(x) = f(x)? Justify your answer.

(e)

:efU!M"'!

What is the maximal possible order of an element in 8 10 ? Why? Give an


example of such an element. How many elements will there be in 8 10 of
13
th at order?

-x2

(d)

fcr 3"j~{lcli ~ JIOI;f)<!J

(b)

10

Give an example of an infinite group in which every element has finite


10
order.
-+4
2

What are the orders of the following permutations in 8 10 ?

1 8 7 3 10 5 4 2 6

bJ

set of real numbers and Cis the set of complex numbers.)


(b)

fcr 3qft~j'i:i!<!f

[1 2 3 4 5 6 7 8 9 10

- a . Show that f is an isomorphi sm. (Here R is the

a
b

~~

by f(a + ib)

? Consider the map f: C

fco R

~I

under the usual binary operation s of matrix addition and matrix


multiplica tion. What are the additive and multiplica tive identities and

(1 -1]

3l~ ~

= [x) 2

+ 3,

I ~ "lffl ~"ll"11Cfl<.1=flll

(a)

Let J = {a + bi I a, b

Z} be the ring of Gaussian integers (subring of C).


IT

Which of the following is J : Euclidean domain, principal ideal domain,

I=

15

unique factorization domai.n? Justify your answer.

8 de

0
Cf)f

(b)

f sin

~ RCfllf&IQO I

Let RC =ring of all real valued continuous functions on [0, 1], under the
operations

(c)

4._-i<1CflCfi{OI ~

= 5x 1 -

4x2 + 6x3 - 8x4

~~q

(f +g) x = f(x) + g(x)

x 1 + 2x2 - 2x3 + 4x4 :-::; 40

= f(x) g(x).

(fg) x

2x1 - x 2 + x 3 + 2x4 :-::; 8


4x1 - 2x2 + x3 - x 4 :-::; 10

15

IsM a maximal ideal ofR? Justify your answer.

. (a)

(c)

Let f(x, y) = y 2 + 4xy + 3x2 + x 3 + 1. At what points will f(x, y) have a

Let [x] denote the integer part of the real number x, i.e., if n
where n is an integer, then [x] = n. Is the function

15

Machines

10

maximum or minimum ?

(d)

Solve the minimum time assignment problem :

Mz M3

M4

Jl 3

12

14

J2 7

12

Ml

x<n +1

Jobs

f(x) = [x] 2 + 3

Riemann integrable in [- 1, 2] ? If not, explain why. If it is integrable,

J3 5

11

10

12

J4 6

14

11

compute

J ([x] 2 + 3) dx.

10
(b)

-1

Using Cauchy's residue theorem, evaluate the integral


IT

15

I=Jsin ede
Q4.

(a)

rl{PldJ:l ~

f.mcR ~ Cfi) ~ chl fG1 Q\ :


(c)
~

subject to the constraints

Ml Mz M3
5
Jl 3 12

M4
14

x 1 + 2x2 - 2x3 + 4x4 :-::; 40

Jz 7

12

2x 1 - x 2 + x3 + 2x4 :-::; 8

J3 5

11

10

12

4x 1 - 2x2 + x 3 - x 4 :-::; 10

14

11

J4 6
A-BRL-M-NBUB

Minimize z = 5x1 - 4x2 + 6x3 - 8x4

20

xi?: 0

A-BRL-M-NBUB

~B

In an examination, the number of students who obtained marks between


certain limits were given in the following table :

(c)

SECTIONB
Q5.

(a)

=y

f(x) + x g(y) it ~ 1:fl~ f Cf~ g ~

fp:f)Cf){Uf

-q

4;"1+0JI;:fl d

PlWila mm~
~#~~~ :
30-40

3icfi

m;ff~~

Oy2

~ ~

Prove that the necessary and sufficient condition that the vortex lines
may be at right angles to the stream lines are

(d)

m m;ft ~

~ >rrn ~ ~ ~

u, v, w = J.l (Ckp , c3q>,


ax Oy

40-50 50-60 60 - 70 70-80

Four solid spheres A, B, C and D, each of mass m and radius a, are


placed with their centres on the four corners of a square of side b.
Calculate the moment of inertia of the system about a diagonal of the
10
square.

(a)

~ chl~q:

~45<f21150~~~~ I

~ ~

~ ~311 CfiT 'tffiT

fcfi

~ ~'tl Pl~Rif@a ~

u , v, w

~ ~
(e)

(b)

~l ( ap, o<p,
Ox

\fiffi ~ x,

crt ~ ~ m

fll01Chio1 lR ~ CfiT ~ICI~4Ch <f211

Q6.

(D2 + DD'- 6D' 2 ) z = x2 sin (x + y)

:
cy

y, z, t

tn: ~

ax

-Q:m ~ ~

(b)
1

"SfCf)"R "{@

fcrcnuf ~ ~ ~ cnr ~

~ D o2TT D' ~ ~ ~ ~

ap)
oz

-=em: im ~A, B, C <f211 D, ~


~

(a)

crm <p,

@311 -ft

fcf> m

qn:coR-ta

rtl~v:

~~
~I

~2fr -ij fcruq

Reduce the equation

to its canonical form when x * y.

A-BRL-M-NBUB

"f1{

irtt

~ ~ ~ x

=0

x2

+; = 1,

o2TT x

= 1 it :rm=rr

= l t >ffi'll -q

BrRr

~ I ~ ~ ~ ~ Wr A . x (l - x) WU "Cf>Pf.1 ~ ~
fcom ~ t TR ~ fW it fcnm ~ x TR irtJ q:)f

(f)

~~cfi)Mq: I

Solve

(a)

2
2
2
z
z
z
+ (x+y)-- + x y2
2
Oy
Oy
()x
()x

= C(3z + 1), (C ~ f?~ ~)


<iiMCf>d: Slfd-eJ;; Cf){d1 ~ o2TT "it Cfl

W; fenm ~

Form a partial differential equation by eliminating the arbitrary


10
functions f and g from z =y f(x) + x g(y).

~.
ay

chl~q: "it~

~ ~dll(ctCh ~

(c)

o2TT

z(x + y)

cnT ~ 'PfT b
cnR"T tn: m1 crt

CfiT ~&1lOIH m <f211 ~a,


1Tm ~

10

(e)

~ 3m ~<1J~H ~ CfiT $kll011fl ~ ~' &f m;ft ~ ~ "ffiCf ~ ~

(d)

o<pJ
az

where J.l and <pare functions ofx, y, z, t .

31

35

51

42

31

31

35

51

42

Using Newton forward interpolation formula, fmd the number of


10
students whose marks lie between 45 and 50.

=0

cfi)f\i!v, ~ X* y.

~ -qft~ ~'
R~R1f@a

+X-

Ox Oy

Ox2

(Ch'"ilRCh<'i ~)
(c)

a2 z

a2z + (X + y) - a2 zy-

31

No. ofStudents

fp:f)Cf){Ul ~ I

(b )

30 - 40 1 40 - 50 50 - 60 60 - 70 70 - 80

Marks

~ ~ ~ ~

fcl0'14'"1

2
CD2 + DD' - 6D'2 ) z = x sin (x + y)

=0

where D and D' denote -

ax

10
A -BRL-M -NBUB

::

a
.
ay

and -

15

(b)

Find the surface which intersects the surfaces of the system


z(x + y)

(b)

= C(3z + 1), (C being a constant)

orthogonally and which passes through the circle x2 + y2


(c)

= 1, z = 1.

15

A tightly stretched string with fixed end points x = 0 and x = l IS


initially at rest in equilibrium position. If it is set vibrating by giving
each point a velocity A. . x (l - x), find the displacement of the string at

any distance x from one end at any timet.

Q7.

(a)

~ ~, eps
(b)

WR ~

~c--lnf{~

y(O)

20

~ SJ''HI ~"d Xo ~ ~ ~ ~ f(x) = 0

~ - ~ fufu ~

cnT

~ ~ ~ ~

~ gHI~R1lll efa

fcl<t>fBd ~'

f.:rmftf ~8J ~ C12IT ~ f '(x) in fu"Q: f.:rmfur W;r ~~ ~

mate
Use E uler's method with step size h = 015 to compute the approxi
value
initial
the
from
places
al
decim
value of y(06), correct up to five
problem
y' = x (y + x) - 2

(c)

following
The velocity of a train which starts from rest is given in the
r.
km/hou
in
is
velocity
nd
a
table. The time is in minu tes

16

288

10

15

=2

512

464

40

12
320

14
176

16

18

20

32

by using
E stimate approxi mately the total distance r un in 30 minutes

~ liR fll'iflll

15

compos ite Simpso n's .!_ rule.

y' = x (y + x) - 2
y(O)

=2

~ ~ ~~ll'iflCI f~ ~
3WfT1=J
(c)

fcm11

(Rtr ~) h
~ ~

-mi'l:f

"fltt,

y(06) CfiT flblCh( lfR" qf{f;hfe;ld ~

= 015 ~ ~ ferftr Cfi1 ~ ~


~ ~ {fll 11:$) CfiT ~ Hq

"fiRUft -q

if>

~~

QS.

(a)

~ l ~ ~ ~
Pt<:1~(1 ~

3fn:

-B ~ di~h:n:

("l lOJ (1 ~ ~ ~ CR WI ~

~ CflK1 ~7t ~ ~ n2 = ( 3 ~)

~ Tfm ~ I ~

fifcqo (f~ A ~
I ~ fc!1 Sl("lllOJHI

~ AB (1~ BC, B tn: lOJ~oA<fd

7.

~-q~C12IT~ k~-q~ I

(a)

16 288

40 464

10

12

14

16

18

20

512

320

176

32

Develop an algorithm for Newton - Raphson method to solve f(x)

(b)

('f{ff ,

x -31&1 ~ t HklOJCf>

TR 3'1 ClCf>l !tl

a.bf

"Cf>1 tRdT

-it fcl1

qfHf1lOll ~ (f~ ~ ~ ~ (0, a) "Yl: ~ met m ~ 3fn: (0, b) TR m


(f~ ~ =;ffUII r JO!Cf) ~ "Yl: ~ lTGff ~ 31-Rf tR ~ ~ ~ ~
2
2
(a- b ) t ~ p ~ CflT t:f.1fCJ ~
fc!1 qf<fflt:tl tR qftolls:fl ~ np m (a
{2ab + b)}

=0

starting with initial iterate x0, n be the number of iterations allowed,


eps be the prescribed relative error and delta be the prescribed lower
bound for f'(x).
20

(c)

~ ~

ffrcn

dl

m~ K ~ n ("JU;f{@l<!l ~' ~ ~ ~ a ~ ~ ~ ~nl<hl{

~ ~ \iloR~ii (~ ) ~ ~ ~ ("lt:tf?l(1 ~~ ~ ~ ~'


2 3

eft ~ ~ ~

~ 1"ffu ~ I ~ ~ fcnm
a
~ ~ ~ ~ 3fu: LJ>Cfl~~l'"l ~ B ~ (nSn-1)
K

tfi ~ "Yl: ~ :wrn ~


A-BRL-M-NBUB

fun

9
A-BRL-M -NBUB

10

(a)

IC-D~-N-OBUA I

Two equal rods AB and BC, each of length l, smoothly jointed at B, are
suspended from A and oscillate in a vertical plane through A. Show that
2
the periods of normal oscillations are rr where n 2 =(3 ~J g .
15
n
~7
l

1fUH'1

I MATHEMATICS

~-~I/Paperl

(b)

If fluid fills the region of space on the positive side of the x-axis, which is

f.tr!fftcr

a rigid boundary and if there be a source m at the point (0, a) and an


equal sink at (0, b) and if the pressure on the negative side be the same
as the pressure at infinity, show that the resultant pressure on the
rrpm 2 (a- b) 2
boundary is
where pis the density of the fluid.
15

<f11l{ :

rfl;r rR

~-ta~~f<rim~

PlfT J1R1 'fiT~# # f{!i f.l..,fRfrstrr.rrrW. <it un'l#<h fTi:


~ 3/TO W'f t orT Cff '8fUST ff ~ f rlWT fr.;{t 3ft< .HM <ft# ff fJV f

If n rectilinear vortices of the same strength K are symmetrically


arranged as generators of a circular cylinder of radius a in an infinite
liquid, prove that the vortices will move round the cylinder uniformly in
.

time

Srr a

(n -1) K

. Find the velocity at any point of the liquid.

'

rrtt/ffT2ff <6T 1 ~ Jff'if ~ 'Jff< # f I


<&rT 1 3ft< 5 .3Tf.#rrlf f rlWT 00 ff ftJ[fi/q; ?9'15 it q;ll- it- q;q 1!'5 W'f ~ fii;;:tT rfR Jff'if ~ 'Jff< ~ I
J[fitq; W'f/lffTT ~ 3i<n :m<f; <fT1R ~ '71{ f I
Jff'if ~ 'Jff< J8t 1Tf&lll1 ff fffli orR ~ ~ ~ JfTf1i/; J!iW- Q;f ff fWn 1J'1T ~, 3ft< $</' 1Tf&lll1 'fir pre ~
w.r-~-'Jff< (if'i.#t.f[.) ~ ~ !J&-'1'"0 rn: Ji!i1m ~ PIR' rn: fWn <iiT'IT ~ 1 Jf@ftstrr 1Tff2ll1 ~ J1fctftm aP!
f.Wt 1Tf&lll1 ff fffli '71{ 'Jff< 'If q;J{ 3i<i; ;rtf fTrc# I

20

,'

I
I

W'f

2 3

Maximum Marks: 250

Time allowed : Three Hours

{2ab (a+ b)}

(c)

~aiq;:250

;I
'l

.!

~~it, rit :7'1Jm ~ w 'iflR ~ rrwr ~ ~ ~ 1

rrwr ~Jvqi'IJ ~ 11R<f; 31WT ff Jl7jm t 1


Jff'if ~ ;rrrif ''t 1JURT i6111j81l ''t ~ I ~ 'fiT?J ;rtf 'if, rft W'f ~ 'Jff< ''t 1JURT ''t ~ ~ <rtf 'Jff< 3ifrn:
~ 1 w.r -~-'Jff< ~ ff 1STfft ~ s:m '[fJ 'l/1 :m<f; mr <6T Pn! ~ it 'fiT?J <iiT'IT ~ 1
;Jf<f (fq;

,.

A-BRL-M-NBUB

11

Jf@ftstrl "

tt,

~ 1J'1T

Question Paper Specific Instructions


Please read each of the following instructions carefully before attempting questions:
There are EIGHT questions divided in TWO SECTIONS and printed both in HINDI and in
ENGLISH.
Candidate has to attempt FIVE questions in all.
Qu!!stions no. 1 and 5 are compulsory and out of the remaining, THREE are to be attempted

choosing at least ONE from each section.


The number of marks carried by a question I part is indicated against it.
Answers must be written in the medium authorized in the Admission Certificate which must be
stated clearly on the cover of this Question-cum-Answer (QCA) Booklet in the space provided. No
marks will be given for answers written in a medium other than the authorized one.
Assume suitable data, if considered r;ecessary, and indicate the same clearly.
Unless and otherwise indicated, symbols and notations carry their usual standard meaning.
Attempts of questions shall be counted in chronological order. Unless struck off, attempt of a
question shall be counted even if attempted partly. Any page or portion of the page left blank in the
Question-cum-Answer Booklet must be clearly struck off
C-DRN-N-OBUA

1
www.examrace.com

''

. . l

..

. (d)

~A

~t_<""!'iCfi"l ct1~l( :
1

SECTION A

f
o

loge (1 + x) dx
1 + x2

1ii

Evaluate :

10X5=50

Answer all the questions :


(a)

~~

R3 -ij ~

~ww:,

. ~ xy ('tlidel ~ ~

'it fc!> v ~ w if; SIRt'0<:: <fiT ~ t

w~

(1, 2, 3) ~ ~ (1, -1, 1) if; ~

fc!> v

Q2.

-3

-1

-2

<t't CfiTR; ~ ~Nll(

ftr.&

llT-1 ~

~ yz + zx + XY

=0

fct> V 31tt wf.l1::;{

Jqeqf2<li ~ R4 c6t

q;)

=0
10

= OJ 31tt

W = {(a, b, c, d) : a = d, b = 2c}.

l
.

(i) V, (ii) W, (iii) V n W q;r ~ 3mrn:

31tt fclfdR m<f ct)~l(

4
Let V and Wbe the following subspaces of R :

'
!

V = {(a, b, c, d): b- 2c + d =OJ and

-3

-1

-2

~Nll(

(a)

V = {(a, b, c, d) : b- 2c + d

Using elementary row or column operations, find the rank of the matrix

(c)

=0

Examine whether the plane x + y + z =0 cuts the cone yz + zx + XY


\
in perpendicular lines.

Find one vector in R3 which generates the intersection of V and W,


where V is the xy plane and W is the space generated by the vectors
(1, 2, 3) and (1, -1, 1).
10

1 +x 2 ..

(e) . l:ltt~ ct1~l( fct; .~ eHkt X + y + z


eqchluft<l (&of) B9Tan "ij CfiR(Il ~ 1

11m~(~)~ I

loge (1 + x) dx

W = {(a, b, c, d) : a = d, b = 2c}.

10

Find a basis and the dimension of(i) V, (ii) W, (iii) V n W.

(b)

(i)

A. om J.1 ~ llT-1 ~ ct1~l( mf.l; e41Cfi<Ot x + y + z

x + 2y + A.z

= J.1 q;r

(1) ~ ~ ~

15

= 6, x + 2y + 3z = 10,

(2) ~ ~ ~

(3) 3iqfl:fild

~~I

fc!> ex cos x + 1 =0 if; GT

Investigate the values of A. and J.1 so that the equations x + y + z =6,


x + 2y + 3z = 10, x + 2y + A.z = J.1 have (1) no solution, (2) a unique
1'0.
solution, (3) an infinite number of solutions.

qJ~fcl<t> ~if; <fTq ex sin x + 1 = 0

<fiT~ qJfafcl<t> ~ ~ ~ I

Prove that between two real roots of ex cos x + 1 = 0, a real root of


ex sin x + 1 = 0 lies ..
10
C-DRN-N-OBUA

C-DRN-N-OBUA

2
www.examrace.com

3
www.examrace.com

(ii)

~ A= [~

:]

~ ~dif>"' ~

~ ~ ~- ~filcr?:Oi m ~C'<IIfCI\1 cfilNll!; 3fu: ~

cfilNll{

I m21

tt,

A5 - 4A4 - 7A3

11A2

W\1 f.l~ftl\1 ~ ~ ~ <f>lNll!> I

- A- 10

[1

Verify Cayley - Hamilton theorem for the matrix A =

ff

(i)

dx dy

I
By using the transformation x + y = u, y = uv, evaluate the integral
JJ {xy (1 - x- y)J 112 dx dy taken over the area enclosed by the straight
lines x = 0, y = 0 and x + y = 1.
15
(a)

(b)

ax + by2 + cz 2

=1 d"4T lx + my+ nz =0 ~ WU ~
~
~<IT f.thl\1"1 11R ~ <f>lN111> I qfl:uuq ~ '>'<llfilol4 0<11&11 <f>lN111> I
Find the maximum or minimum.values of x 2 + y 2 + z2 subject to the
conditions ax2 + by2 + cz 2 = 1 and lx + my + nz = 0. Interpret the result
geometrically.
20
x 2 + y 2 + z2

-2
(c)

(b)

(i)

11R

cllNl!!> fcl;

~I

("'@-~)

~ -&MI ~ . ~' ";;iT

/_

.
.

P2(X2 +y2
. a
b

CfiT ;;r;R <fi{(fi

+~J=(lx+my +nz)2
c

15

,jjfdq{C:li:?l'l"' ~ @\11.1 z = 0 .~ ~ ~ {ld1:Jl'l

A =

~ ~ (a cos 6, b sin 6, 0)

.q

x + y = 1,. z = 0 ~

a 2 b2

~ <:) "''"1'Cf> 'Wf3TI ~ ~iflq:;(ol Wi'! 4il~l!; I


'&US

Find the equations of the two .generating lines through any point

~ "Cfll ~ 4il Nl!i> I


-2
2 -3

(a cos

Let A=

the hyperboloid by the plane z = 0.

"IICf><"NI

SIRI-m<;"' ~3TI ~

~J = (lx +my+ nz)2

p2(X2 + y2 +
a
b
c

il>~'l

Show that the lines drawn from the origin parallel to the normals to the
.central conicoid ax2 + bl' + cz 2 = 1, at its points of intersection withthe
plane lx + my + nz = p generate the cone

(c)

'

1 - 6 . Find the eigen values of A and the

-1 -2
0
corresponding eigen vectors.

fu:& <f>lNl!i> fcl; ~ ~ ~. ~ ll"Rl CfiT ~&l11R 1 ~ ~

:2

e, b sin e, 0), of the principal elliptic section

+ ~2 = 1, z = 0, of
15

8
I

Prove that the eigen values of a unitary matrix have absolute


value 1.
C-DRN-N-OBUA

2 -3

-1 -2

(ii)

~ fcf>. ~

~ ~ ~41"'11< ~' ~
,~-

4
www.examrace.com

C-DRN-N-OBUA

'
10

ax2 + by2 + cz 2 = 1 ~ ~4.\11.1 lx + my + nz = p ~ ~

~ ~ ~ ~ ~ ~ ~ ~ ~ cfilN1o; it fcl; a~ ~ ~ ~
~3lT~ I
Find the height of the cylinder of maximum volume that can be
inscribed in a sphere of radius a.
15
2

ax 2 + bl' + cz 2 + 2ux + 2vy + 2wz + d = 0 'l!;Cfi


u2 v2 w2
~ f.'l~f?l11 Cf;<OT t ~ - + - + - ,; d
ot 1

a
b
c

Prove that the equation ax2 + bl' + cz 2 + 2ux + 2vy + 2wz + d = 0,

..

'1: ..

ftr.& 4i'l~t!; fcf> ~ifl<f><OI

. . u2 v2 w2 . .
represents a cone if - + - + - = d.
a
b
.c

'"(<i"'liCflOJ

cfilNll{

Q3.

~ ~3TI ~ f.'l~:tli<f> Wi'f 4i'l~l!; ~

10

~ X+ y = u, y = uv CfiT >r<i'trr en@~' ~"'ICfl('l


{xy {1- X- y)}
CfiT ~ '&nt X = 0, y = 0 d"4T X + y = 1 ~ WU ~ l$r 'R

1' + z2 - 4x + 2y = 4

1'

(ii)

112

~ x2 +

~ ~4iR1, ~4\11.1 2x- y + 2z =.1 ~ ~4ict< ~ I


Find the co-ordinates ofthe points on the sphere
x2 +
+ z2 - 4x + 2y = 4, the tangent planes at which are parallel
to the plane 2x - y + 2z = L
10

I~

hence find its inverse. Also, find the matrix represented by


A5 -4A4 -7A3 + 11A2 -A-10 I.
(c)

Q4. . (a)

'': ~

www.examrace.com

\>I

(d)

SECTION B.

Q5.

10X5=50

[y + x rx 2 + y 2)] dx + [y rx 2 + y2) - x] dy = 0
~ ~ wil<t><UI ~ f<fi rx2 + y2), (x2 + y2) Cf>T ~ ~ ~

~~~

fliflCfl{UI

~~~~

1
X

+y 2

Wll<tl('l"''

(e)

~~

Q6.

(b)

crsn ~

(a)

<fi'l~o: ~ ~

y2)

= (x2 +

f<fi ~ q:;r 'lWT ~

31~

rJ2.

wu CfiRJ s_m t

10

..

-J

(b)

1"ffa'

(S.H.M.), ~ 0 'R <t>l('li<t> (<iiiCict<tll('l) T, 3ll<ll11

:rmn t

cos-

(~J ~

(c)

~~

s 27!.

10

ctit f<l'fu ~ ~ ~ ~ :

i.
I
'I

10

chlNlll; :
>

~~

20

,...

'lC"<lict>"l <hlN1ll; :

..

-. ~:

x 2 + + z2 - 2ax - 2ay = 0,
z-Md<.1 ~ ~ B~ 7jfRfT

mm t afu:. m

+ y = 2a, ~ f<f; (2a, 0, 0)

B~

'

Evaluate by Stokes' theorem

A particle is performing a simple harmonic motion (S.H.M.) of period T


about a centre 0 with amplitude a and it passes through a point P,

J(y dx + z dy + x dz)
r

where OP = b in the direction OP. Prove that the time which elapses

C-DRN-N-OBUA

~ r CISii

10

1l

'i

(y dx + Z dy + X dz) ..

before it returns toP is T cos-1 (b).

+ t sin t "j ' 0 s t

~ ~if1Cf>{UI ~

~ f<fi OP = b, OP<I?r
a~ m2l ~ "Cf){ " ~ ~ <rQ ~~Pit
~ it 1 #r.& ~ f<fi ~ s_m ~ 'i1<f <rQ P tn: crr:m ~
,

CISfi(IT.

Solve the differential equation:


d3
d2
d
.

x3
~ +3x 2 --f+x_]_+8y=65cos(logex).
dx
dxdx.

10

bisected at the point of tangency.

d3
d2
d
x3
~ + 3x 2 ; + x_]_ +By= 65 cos (loge x)
dx
dx
dx
.

Find the curve for which the part of the tangent cut-off by the axes is

~ CfiUT ~ ~ 3l1CI(f

~ ~ fil'CI{UI

~'R~Qll

(c)

~ 1:R

d
.
__]_ - 5y = sin x
dx
Solve by the method of variation of parameters:
. dy .
.
- -5y=smx
dx

1
is an integrating factor for it. Hence
2 +y 2

solve this differential equation for rx2 +

'41

Give its magnitude also.

rl is an arbitrary function of (x2 + y2), is not an exact


X

"

CISii r(t) = t cost i + t sin t "j, 0 s t s 27t ~ M


~ ~ I 7mCf>l qjl:qlol 'lfi ~ I
.
.

r (t) = t cost "i

JustifY that a differential equation of the form:


[y + X rx2 + y2)] dx + [y frx 2 + y 2) -X] dy = 0,

differential equation and

Find the curvature vector at any point of the curve

~ ~ ~ fl'"il<t>{UI q:;) rx2 + y2) = (x2 + y2)2 ~ ~ ~ <fi'l~l!> I

where rx2 +

Two equal uniform rods AB and AC; each of length l, ani freely jointed
at A and rest on a smooth fixed vertical circle of radius r. If 26 is the
angle between the rods, then find the relation between l, r and 6, by
using the principle of virtual work.
10

"3ffir 6~(1 ~ll> f<f;


<l?r

. )

w-n

~ dffi: ~ :
~
Answer all the questions :
(a)

c:1 ~ Q}'I>~'IH zy; ABam AC, ~ l~, A 1:R ~~ B ~~am~


r~~ ~ ~m ~ 1:R ~ ~ 1 ~ ~ ~ ~ cnTu! zetor ct>@1a
CfiT1f ~ ~
B l, ram 9~
mi'l <f>lf~ll; I

6
www.examrace.com

where r is the curve given by x 2 + ~ + z2 - 2ax - 2ay = 0, x + y = 2a;


starting from (2a, 0, 0) and then going below the z-plane.
20
C-DRN-N-OBUA

www.examrace.com
. ' -~ ., ~-~ j

Q7.

(a)

f.I1001R1RsH1 ~ ~FnCf><"l ~

(b)

<hlNil!> :

am)

x d y -2(x+l) dy +(x+2)y=(x-2)e 2x

dx 2

~ex~

(!Of

x-318;1

~BCfl.l rcn:u1 ($ x-318J

~ B4id( ~
3rq(

J B ~~ ~

clit

I CfiUT

~ I ..

projected parallel to the axis of x with velocity

~,
~
d~
- -2(x+l)-+(x+2)y=(x-2)e
x
dx
.
dx 2

J.

Find the

parametric equation of the path of the particle. Here ll is a constant.

when ex is a solution to its corresponding homogeneous differential


15
equation.

(c)

it ~~ -ij 'll>Cf> ~ ~ it 'll>Cf> ~ 3'1 fcl d1o:<~


S::&l q 1"1 m cnr
(~ ~ <rrffi) z ~<liT itft it~ ~311 t "Cf)). 'll>Cf> ~ 3lT'Cffif WIT~
~ it fi.n '3W!iT im 2..Jgi ~ ~ ~ 1 q:;ur cnr im am ~ mt ~it ~
<fGf ~~~~it~~~ chlNlll; I

wr ~ (~ tlcrH) ABCDE Gf('I'OR 'llrtT l{Cf){'i14H ~ q:;) "'i'$Cf>< ORl<IT


~311 t it'$ A it ~ "@ ~ am BC ~ DE ~ ~ ~3'11 q:;) ~ ~ 'll>Cf>
~ ~ ~ ~ ~ 3i1=R ~ -ij ~ ~ I ~ ~ -ij Slfdiill.1 ~ chlNll!; I

15

~ liR wlf'll

'll>Cf> q:;ur

d y + y = 8 e-2t sin t,
dt 2
q;l ~l'k'IIB-{<\qia< ~

>r<ftrT B ~

y(O) = 0, y'(O) = 0
<ti1~l( 1

Solve the initial value problem

A particle of mass m, hanging vertically from a fixed point by a light. .


inextensible cord of length l, is struck by a horizontal blow which
imparts to it a velocity 2..Jgi. Find the velocity and height of the particle
15
from the level of its initial position when the cord becomes slack.
(c)

~ (f~ ~. y =a t

S,3ll

A particle is acted on by a force parallel to the axis of y whose


2
acceleration (always towards the axis of x) is llY- and when y = a, it is

~ ~4]Cf)(OI <nT 'll>Cfi ~ ~ I

Solve the following differential equation :

(b)

llY-2

om ~

~ '1'~ Cf>T Sll'qf<1Cfl Bif1Cfl(OI ~ <fi1~l( I ~ 111%

dx

Wro ~14EIId

1% CfiUT 'R: y-318J ~ ~ 1%

d Y + y = 8 e-2 t sin t,
dt 2

y(O) = 0, y'(O) = 0
20

by using Laplace-transform.

'll>Cf>

A regular pentagon ABCDE, formed of equal heavy uniform bars jointed


together, is suspended from the joint A; and is maintained in form by a
light rod joining the middle points of BC and DE. Find the stress in this
20
rod.
QS.

(a)

~ ~>flCf>(OI M(x, y) dx + N(x, y) dy = 0 ~ ~ ~ ~ ~ chlNll!; 'dlfcl;


~ ~141Cf>l.1"1 TJ'Cf>, (x + y) <nT ~it I ~ ~ -ij MICf>l.1"1 TTCf> ~ itTTT ?
+ xy) dy =0 <nT ~141Cf>l.1"1 TTCf> ~
~ ~ ~10-aCf)(UI (x2 + XY) dx +
chlM!!> am~ chlM!!> 1

(r

Find the sufficient condition for the differential equation


M(x, y) dx + N(x, y) dy = 0 to have an integrating factor as a function of
(x + y). What will be the integrating factor in that case? Hence find the.
integrating factor for the differential equation
(x2 + xy) dx + (y2 + xy) dy = 0,

15

and solve it.


C-DRN-N-OBUA

www.examrace.com

C-DRN-N-OBUA

www.examrace.com

'@T1S-A / SECTION-A

C-DRN-N-OBUB

TTfUrn

(~-q;f-11)
1.

l~,(fR~I

3Tf~ ;;lq;: 250

~ '!

li!iHI~'ii'l<l t, ~ f<f; xz ;< 0. ~ f<f; G

<1'!1 N ~ G 'f;f WlPI

m, ~ om ~l.:l<ki) ~ lfl'!ih 3l~ ij ~ ~I

"S!'lTffi <t't TJ1lRT Sii41j\1R <t't O!l11,'ft I 3lifu'h ~ i\ ~ ~ ~ if; T<R q;] >:ft l!P!! ~ O!l11,'ft ~ ;m "ihll1
mI m -m: -T<R ~ ij 1:l!T<'!\ ~ ~ q;j{ '[13 3l21'1! '[13 if; 'lWT q;j ~: "ihR: ~ I

(;Wfur) ~ ~?

3l'f.t

<f'fi

'f;f

aE

JR} OfiT ~ 'li\<!1 ~I

m:wr ~I
~

xz ;< 0. Show that G is a


Let G be the set of all real 2 x 2 matrices [ x yz ] where
.
0

group under matrix multiplication. Let N denote the subset { [

~ ~ ] : a E lR}
10

Is N a normal subgroup of G? Justify your answer.

(b)

~ if;
Tf'll

'gf f.l "'1 R1 fi'3 d f.fuil q;j <f!"!T mq~ I;ft 'i.<fili ~ )

*' ..,

(iih

~ G B'ft 2 x 2 ~ <11\"<lf<lili ~ [ ~

~ ~ ~ 3M'Pfu 1l;i!i ~ ~ll!R ~ N 34lit!fil~ {[ ~ ~]:

G1 1911&1 il ! 3nO m ~ ~ ~ oi\ fRt "{('i 3i"!hft <if.:ii il ~ ~ 1


3Aft~<m q;] f "'fu ~ if; T<R ~ ~ 1
m "ffi9>n 1 3ffi 5 3Tf.r<n<f ~ om 'IT<fi't ~ il i\ ~ @16 i\ 'hl! -it -'hll ~ m ~ <tR ~ if; T<R ~ 1
~ m /'lT'T if; ful; f.!<rn ;;lq; ~ "fiTI'R ~ ~ ~I
~ iii T<R "3<ft ~ lflt:!lll il R1'@ 'li'R ~ ~ 3lT'li!i m -<!;! il fif;<n Tf'll t
lflt:!lll "ih1 <'11!
~ m -m:-T<R (<l'i_o ~ 0 ~0) ~ if; ~ 1R AAl! "fm'! 1'f( fif;<n ;;rRT - I ~ lflt:!lll if; 3lfMu; 3PI
fif;B\ lflt:!lll ij R1'@ ~ T<R 1R q;j{ ;;lq; 1'tff f4W\ I
~ ~ m, m~ ~ "ih1 ~ ~ "~ ~ AAl! ~~
01'!

liR

~-q;r~tft~~
(T<R ~ if;

(a)

31-ffi ~

'!

~ ~ "" '<ftl\lUI ~I

dx

x 2 (1 +e-x)

Test the convergence of the improper integral

.
dx
x 2 (1 +e-x)

10

MATHEMATICS (PAPER-II)

I Time Allowed

: Three Hours

IMaximum Marks : 250 I

(c)

~ ~ f<f; '<K'R f(z) = u

QUESTION PAPER SPECIFIC INSTRUCTIONS

f(z)

= x3

(Please read each of the following instructions carefully before attempting questions)
There are EIGHT questions divided in two Sections and printed both in HINDI and
in ENGLISH.
Candidate has to attempt FNE questions in all.
Question Nos. 1 and 5 are compulsory and out of the remaining, THREE are to be attempted
choosing at least ONE question from each Section.
The number of marks carried by a question/part is indicated against it.
Answers must be written in the medium authorized in the Admission Certificate which must
be stated clearly on the cover of this Question-cum-Answer (QCA) Booklet in the space
provided. No marks will be given for answers written in medium other than the authorized
one.
Assume suitable data, if considered necessary, and indicate the same clearly.
Unless otherwise indicated, symbols and notations carry their usual standard meanings.
Attempts of questions shall be counted in chronological order. Unless struck off, attempt of a
question shall be counted even if attempted partly. Any page or portion of the page left blank
in the Question-cum-Answer Booklet must be clearly struck off.

IP.T.O.
www.examrace.com

+ iv, ~
f(O)

=0

u 0; f(O)

=0

(1 + i)- y3 (1- i), z


x2 +y2

;< 0;

Prove that the function f(z) = u + iv, where


f(z)

= x3 (1 +~)- y:(1- il,


X

+y

satisfies Cauchy-Riemann equations at the origin, but the derivative off at


10
z = 0 does not exist.

(d)

'P<'R

f (z) =

1
z 2 (z -1)

'f;f

=0 <fliT z =1 ~

~-ii'r<f

Expand in Laurent series the function f(z)

[-f !\.0C-~-11)1JI !I ili/39

"Rm >.l;uf\ i! >mR

~I

1
z 2 (z -1)

about z

=0

and z

=1.

10

2
www.examrace.com

(e)

rcJ

~ (J!T1li't<l) f<;fu ~ 1lRl ~ ~ :

f.l10t~f&a

'l'ft<m .wwn ?ti ful; f i <tt

fufu ?ti IDU 3Tiffi:r'!l mmftq; mu~ m'l ~


'1ft ~ ~ :

flrn'lll't

~ 1 Wl't ~ m'! om ~ 'l'ft;m

~~Z=6x 1 +5x 2

t'!fl1(j

mH!i
2x1 + x 2 :S 16
x 1 + x 2 :S 11
X 1 + 2x 2 ~ 6
5x1 + 6x 2 :S 90
x 1 ,x 2 ~0

11

D2

D3

14

16

15

q
0:1
03

10

D4

...

Find the initial basic feasible solution to the following transportation problem
by Vogel's approximation method. Also, fmd its optimal solution and the
minimum transportation cost :

Solve graphically :

Destinations

Maximize Z = 6x1 + 5x 2
subject to
2x1 +x 2 :516
XI + x 2 :S 11
X 1 + 2x 2 ~ 6
5x1 + 6x 2 :S 90
x 1 , x 2 ~0

Origins

10

3.

(a)

11

D2

D3

D4

14

02

16

03

Demand

10

15

Supply

~ fof; ~ {a+bro:ro 3 =1}, ~a <111.1l b qlf<l~'!l ~

20

<I'T"I'!A

<WT om~ iii

3Mf1f<r

1% !$! ~ I
Show that the set {a+ bro:ro 3 = 1}, where a and bare real numbers, is a field
with respect to usual addition and multiplication.

(b)

2.

(a)

~ Ff; Z 7 1:%

a:tf t I

0<1 Z 7

ii ([5] + [6])- 1

(1'41

(-[4Jr 1

mo

J~

f(x) d.x 'fi1

<i'll<li<'H

J~

15

~. ~

f(X) =

Integrate

~I

Show that Z 7 is a field. Then find ([5] + [6])- and (-[4])- 1 in Z 7 .


(b)

15

2x sin l. -cos l.
X

XE

]0,1]

x=O

Obtain

af

(O, O) and

ax ay

af

, f(x, y} =

f(x) d.x, where


f(x)=

(O, O) for the function

ay ax

{ xy(3x2 -2y2)
x2 0+ y2

(x, y)
'

'

I
I
2 xsm~-cosx,

XE

]0, I]

x=O

15

I P.T.O.
www.examrace.com

a2 f .

a2 f

ax ay

ay ax

* (0, 0)

(x, y) = (0, 0)

Also, discuss the continuity of - - and - - at (0, 0).

15

www.examrace.com

lS1'11S-B / SECTION-S

5. (a) ; 3lifuq; ~
Evaluate the integral r"

Jo (

4. (a)

dS
1
1 + 2 cos 8

)2

using residues.

20

Prove that the set Q (../5) = {a+

Ws: a, be Q)

<'f!li;;! ~ <t1 ~ i\;

IDU

'fiT ~ l1R

x 2 + y 2 + z2
2

Find the minimum value of x + y + z

mo

<roil f<!i

xyz =

a3.

subject to the condition xyz = a

~m ~ i\; r;:m f.ltotRtf@ct ~ ~ ~ i\; ~ ~ ffl

by

~it

'ID<J f

dx

Jo 1 + x2

Of;!

wili!i\'H
1

15

mo

(fifi

~~I

SDD' + 2D' 2 ) z = 24 (y- x).

~. 'I@ 'W<\ <~if.\ it~~-~

10

fufu

Apply Newton-Raphson method to determine a root of the equation


cosx -xex = 0 correct up to four decimal places.

(c) , ~

the method of Lagrange multipliers.

Of;!, "''T\

Of;!~~~

15

(c)

~li'li!i<"l cosx -xex = 0

is a commutative ring with

identity.

(b)

Solve the partial differential equation (2D

(b)

m ~ f<!; ~ Q (-,/5) ={a+ Ws: a, be IQ} ~. ~ Sllf<lf.lih! ~ tl

Wflifi(OI (2D 2 -SDD' +2D' 2 ) z=24(y-x) q:;]

Use five subintervals to integrate f

<~if.\ it~ '1fq


dx

Jo 1 + x 2

34id<l<11

10

Of;!~ ~I

using trapezoidal rule.

10

~AND~ OR ~W<~ q;f<\~~~ z=xy+uvit~~wf;-qftqll!

(d)

<!~~~

~~ Z=30x 1 +24x 2

Use only AND and OR logic gates to construct a logic circuit for the Boolean
10
expression z = xy + uv.

;roilf<!;
5x1 +4x 2 S200
x 1 $32

x 2 S40
x 1,x 2 ;:;o

(e)
'

Find all optimal solutions of the following linear programming problem by the
simplex method :

it wflq:;<oi'l Of;! ~ q;f<\ ~ 31m<'~ <'i'tm <t TJftr Of;! ~<fi'li<"l 'W<! ~ 1
Find the equation of motion of a compound pendulum using Hamilton's
equations.
10

Maximize Z = 30x 1 + 24x 2


subject to
5x1 + 4x 2 $200
X 1 $32
x 2 S40

x 1 ,x 2

[-i!l}l 'i-?i-l!l')!ill1l/39

;:;o

20

[P.T.O.
www.examrace.com

15

6
www.examrace.com

{b)

'lrnl-~ ::fl(I~Ri

fo!R!

i\; IDU !itftih(UI ~

s. I''aJI,

2xi- x 2 = 7
-xi +2x 2 -x 3 =1
- x 2 + 2x 3 = 1
'liT ~ ~

~ ~
'""""'"
(i)

a2u =a2u
-,

(X, 0) = 0,

au
(!'z') X( , 0)

(<f'R ::fl(I~Ri<li ~)I

0<x<1, t>O,

ax 2

at 2

0
2

=X,

~ X

:5:1

~X~

at
(iii) u (o, t) = u (1, t) = o, <~oft t ~

Solve the system of equations

a2 u = -a2 u
Solve -

2xi- x 2 = 7
- xi + 2x 2 - x 3 = 1
- x 2 + 2x 3 = 1

at 2

using Gauss-Seidel iteration method (Perform three iterations).

15

(i)

u (x, 0)

(!'z')

au
X( , 0) =X 2 ,

= 0,

at

'

0 < x < 1, t > 0, given that

ax 2

A-.~>
IG<tl ~ foti

~X~

(iii) u(O, t) = u(1, t) = 0, for all t

(c)

~ dy

x = 0 8 'R y 'liT l!R -m<1 "hf.t i\; fBI;,

dx

= x + y, y(O 4) = 0 41

(b)

"1goxhlR i\; ~-~

~'liT ~~I qTT~ h=02~1

(c)

Use Runge-Kutta formula of fourth order to find the value of y at x = 0 8,


where dy=Jx+y, y(04)=041. Take the step length h=02.
dx

7.

(a)

ll."h '1i;qlJR m'l

3ffi li'Tif'f<l; f<la)tj

"'<!\ X 31'\1: y ~
~ foti X + xy = X.
For any Boolean variables x and y, show that x + xy = x.

fotim oft <@\<!

Gl 31-ffi <fl1'i<R

15

~ ~ ~, 1% ~ ~ <lm ~ ~ ~. (~) ~ ~

m,

~-<it<m fl>flifi(OI ~~I

20

Find Navier-Stokes equation for a steady laminar flow of a viscous


incompressible fluid between two infinite parallel plates.
20

***

(~ = 1l, ~ fB't, a2 u = a2 u ) 'liT f<la)tj -m<~ ~. ~ ~ WI 1iJ:<I it


2
2

at

15

ax

f(x) = k(sin x- sin 2x)

till

Find the deflection of a vibrating string


corresponding to zero initial velocity and initial deflection
f(x) = k (sin x- sin 2x)

fbJ firrr<R i\;

15

mm

ll,"hf.t<ll! i\; fBI; ll,"h ll'llO-'lli ~ 1


Draw a flowchart for Simpson's one-third rule.

(c)

c:<1

WI

foMq

cj>

= !.log[(x +a): + y ] i\;


2
(x- a) + y 2

~ ll'llO-mmt -m<1 ~I
2

Given the velocity potential

<t>

Y:], determine the streamlines.

= !.log[(x + a) +
2
(x- aj 2 + y

15

20

[P.T.O.
www.examrace.com

588-550
www.examrace.com

Вам также может понравиться